为什么美国中小学生学的数学比我们简单,美国人却还能做出超级牛的东西?

by , at 23 April 2022, tags : 数学 美国 教育 孩子 中国 点击纠错 点击删除
使用CN2/CN2GIA顶级线路,支持Shadowsocks/V2ray科学上网,支持支付宝付款,每月仅需 5 美元
## 加入品葱精选 Telegram Channel ##

知乎用户 Ming Ali​​ 发表

我当年在土澳备考流体力学。。。。。学渣的我复习到半夜三点。。。。。出国以前我的认知就是这些白人快乐教育长大的,肯定就是水货,现实是半夜两三点了图书馆还满满当当复习的人,还有堆在大厅垃圾箱夸张的咖啡纸杯。

我实在受不了了就要走,正在收拾包,一个金发妹子走过来微笑着看着我,我当时有点迷惑。。。。因为我非常有自知之明,然后金发妹子指着我的练习册,说,这道题她会。。。然后她坐下来一阵秀操作。。。

人家好像完全没看出来我已经熬不动了,说她还会一种解法。。。。。

我只好微笑着看她秀操作秀到了半夜四点。。。。。

后来听说妹子 phd 去了斯坦福。。。

国内我也见过不少学霸。。。但大多数都是那种;" 我也不会唉。。。。。我也没好好复习 “这种。。。虽然最后考试成绩出来,你恨不得掐死你这种。

但是土澳遇到的学霸,是” 我给你秀一下操作,让你见识见识我多吊 “。

这种人,人家是真心爱自己的专业,那就不一样了,就像有的学霸,学完数学分析,考完试再不会碰一下,有些人,集齐了吉米多维奇的全套数学习题集,还嫌题库太小,前者也许在大一考试能考赢后者,但是后来的差距就越来越大。

反正给我感觉,虽然西方教育容易出水货,但是,给牛逼的人蹦跶的天空会更高,你可以夜夜 party,当然你爱学习,图书馆二十四小时都是开的,计算机机房也是二十四小时开的,好多教室定点都会有 tutor 答疑解惑的,你也可以发邮件骚扰教授。

有一个认知鸿沟就是,国内所谓的数学好,指的是十八世纪以前或者十七世纪以前的数学知识掌握的好,但是其实近二三百年的科学数学发展才叫日新月异,前者好比马车,后者已经是高铁,就像小学算术学再好,在中学看来,只是无聊的炫技。而且我们还不鼓励炫技,炫技炫完了,还要补充一句:唉?好奇怪我就是胡写的咋还写对了?” 老师还会善意提醒; 别太飘,还需要好好继续练习。你那个解字快写到密封线上了,要扣分的 "

同样的,外国的学霸炫技炫完了,老师会说,哦哦哦,你好牛逼!想不想学点超纲的?

于是中二的学霸确实比我们发展的好。人家的中二病渐渐转化成了持之以恒的热情,炫技的得瑟转化成了自信。乐于分享。

我们。。。好像对中二的学霸一直是不待见的,你是学霸可以,但你别越界。

所以。。。。。。

38 妇女节送什么礼物?

好多人问我跟那个妹子的后续,嗯。。。。太不了解我了,当然就是她当她的学霸,我当我的学渣了。

知乎用户 Dave 茹兰 发表

私以为

@倪大为

同学引用的答案说的是最正确的。特此赞同一下:

"美国给予不热爱数学的学生最基础的数学教育,而给予热爱数学的天才最高水平的数学教育。"

长久以来,中国人的迷思就是,为何「美国人数学这么差,还能出这么多牛逼科学家?」这个问题的答案已经被答烂了,我综合楼上几位已经提到的「美国学生的数学一点都不简单」,结合自身的经历,系统地给大家科普一下!

首先,先说「美国继续不热爱数学的学生最基础的数学教育」。

美国的每个地区,对于,比如,高中毕业的学生,应该有何种的数学修养,大多是有硬性规定的。比如麻省(Massachusetts ),每年对特定几个年级的学生有统考,这个统考的主要内容之一就是数学,数学不过关的,高中是不允许你毕业的。所以基本上每间正规高中,也都对数学水平有最基本的要求才准毕业。据我所知,这个标准大概在会运算简单的三角函数就可以了。 这个水平,以咱们大天朝的标准来看,确实不算高,而且很多人还都是勉强混过去的。(高中课很松。。。)

这种低要求的直接结果就是,美帝 99% 的学生(99% 这个数字并不夸张,可能更高)的数学都停留在生活勉强可以自理的水平上。

好了,到了说「给热爱数学的天才最高水平的数学教育」的部分了。

以高中为例,对于一些数学比较好的不安分分子,为了安抚他们,不给社会添麻烦,学校(不是每间学校都有这个条件)会提供 Advanced Placement 课程,也就是宅男们喜闻乐见的 AP 课程。以数学为例,高中最高级的 AP 课程,叫做 BC 微积分,2005 年我上高中的时候,课本是下面这个:

我们的任务则是把这本书,cover to cover,学得通通透透。值得注意的是,只要你前面的课程成绩都好,你几年级修这个课是没有限制的,我上学时班上最年轻的同学是一位俄裔美国人,他修此门课程时才上十年级(相当于我们高一),他最后这门课的成绩是 A+ ,在我们学校,意味着每次考试的成绩,都在 95 分以上,实在是学校公害。(这位大哥后来去了 UPenn 学 Material Science,这里就不详细说了。。)

这本书的内容,大家都可以查到,人家高一就学这个,你们各位自诩为学霸的,颤抖了吗?基本上到学 Vector Calculus 之前,学完了这本书,你做微积分已经应已如四则运算般自如。

我作为一名合格的宅男,还选修了 AP 物理,我们的课本是这个:

有兴趣的同学,也可以查查这本书的深度如何。

教材不是统一的,AP 的任课老师可根据自己喜好选择教材。另外老师会推荐一些课外读物,供不安分的宅男们消耗能量。

AP 课修了一整年以后,就可以报考全国的 AP Exam 了。 AP Exam 统考的难度,个人觉得数学比较简单,物理则很难。。。(物理中,力学和电磁学分开考),当然也可能因为我抽象思维能力太差了。。。即便这样,小弟我全考了个 5 分也是没有压力的。(满分 5 分)

说到这就差不多了,中心思想和开始提到的一样,可能论数学物理的平均水平,美国学生确实远远不如中国学生,但是谈到「给天才的教育」时,中国的教育制度,则还停留在解放前。而到了大学,这个差距就越拉越大,到了研究生阶段,没有鄙视国内读研读博的盆友们的意思,但与美国的「高高等」教育相比,真的无法相提并论。国内的院校,资金条件,师资条件和科研硬件条件,都比发达国家相差太远了。

所以说「美国学生学的数学比我们简单」这就是个伪命题,不存在这样的事情。

屁艾斯:

这样的制度,从侧面看,很大一个好处就是,年轻人可以把多余的时间和精力,专注到自己喜欢做的事情上。很多比较聪明的宅男,高中最后一年基本没有什么数理课程了(全提前学完了),学学哲学,历史,艺术,玩玩乐团,体育,对成长都有帮助。前面那位俄裔学霸,还是该地区的长跑冠军,真是令人厌恶。

另外一个好处就是,学霸们可以和学霸们在一起玩一起上课一起耍,不要小看这些宅男们,学霸们聚在一起的能量们是很大的,你没发现,在你们现在打工的地方,老板们年轻时都是学霸吗?

知乎用户 王亚晖 发表

我在美国考过 AP,也给美国中学生补过课。

前面说的差不多了,我稍微补充一下。

中国学生看美国的教材和考卷可能会觉得比较狐疑,因为典型的美国式的课程是,教材里面写了 100% 的内容,按照教材里面写的要比中国的还要多,但是老师只会讲到大概 80%,很多东西只是提一下,之后考试只会考 60%。大部分试卷上题目的内容都不会比课后题难。而中国的正好相反,考试的难度要远远大于教材。

这种教育方式其实会让美国学生分层非常明显,大部分学生只要跟着考试难度学就可以毕业,可能成绩还不错,对于一些好学生来说,完全可以按照教材的最高要求去学习。所以其实看针对高中毕业生的考试,SAT 和 AP 的数学难度完全不像是针对同一个人群的。

SAT 的数学对于中国学生来说会非常容易,而 AP 要困难很多,尤其是 BC 的难度相当高。哪怕对于中国高考高分段的考生来说,BC 也要专门去学才有可能过,因为 BC 的内容对于中国高中来说严重超纲。 BC 的内容就相当于理工科大学一年级的全年微积分课,而美国的顶尖学生里学完 BC 的非常多。

在美国上大学就会发现你周围基本上就两种人,90% 的人都是轻松被你碾压的,剩下 10% 都不比你差的,这两种人的分化非常明显,中间阶层很少,尤其如果在公立大 U 感觉会更加明显,虽然大部分都是混日子的,但是每门课总会有那么几个强到变态的人存在,你觉得你所有都考了满分已经很厉害了,但是总是有人把所有加分也都拿了。你觉得你 4.0 毕业很厉害了,然后你会发现已经有一批人 3 年 2 个学位毕业的,还全是 4.0

所以其实大部分人说美国数学简单都是只看到最普通的那些人了,他们没必要学那么多的数学内容,而对于真的顶尖学生来说,无论是知识储备、学习能力还是刻苦能力都绝对不逊色于中国学生。哪怕你是中国的最顶尖学生,到美国也很难在这群人里凸现出自己来。

这就是美国教育最大的特色,你循规蹈矩的也能正常学下来,你要是特别有天份和能力,那么你也有一条非常顺畅的上升通道,你可以在高中学大学的课程然后去抵大学学分,你可以在大学学一堆专业,你可以提前很久就大学毕业。

我其实并不觉得现在中国的基础教育是有问题的,因为特殊的社会情况决定了很难去改动,中国现在最大的问题一是给顶尖学生的上升空间十分有限,本来竞赛保送和少年班是一个很好的选择,但是这些年被打压的非常厉害;二是中国大学的平均教育质量实在太差,导致了有不少优秀的学生没有办法持续接受教育,只能依赖那些去屈指可数几所好学校的学生创造价值。

知乎用户 史蒂芬周 发表

这个问题,虽然其他答主已经从宏观上解答了,但我还是想从微观方面说一下 “清华” 和“麻省理工” 的差别。

以下我转发的事在 Quora 上《MIT 学生谈清华和 MIT 的差别》的一篇文章。

我不了解斯坦福(因为只是短暂访问过几次那里),不过我对 MIT 和清华都比较熟悉。MIT 是我的母校,而我的朋友有很多都来自清华。说实话,我觉得很难比较这两个学校的优劣。

在校园文化方面,这两个学校有很多本质的差别。而属于清华的很多特点大多并不源于这个学校本身。很多人问我为什么来自清华的创业公司寥寥无几,而来自 MIT 的创业公司又那么多?综合考虑所有因素,我认为其中最大的因素是自由。

清华每天晚上要熄灯。网络连接也被严格的限制着。每栋宿舍楼的入口处都有警卫。在清华,总体上的校园文化是学习最重要。在 Larry Zhang 这个回答中可以看到亚洲人持有一个普遍的观点,即:“MIT 学生的成功有赖于他们的勤奋学习”。这个观点和事实相差万里。

没错,MIT 的学生学习是很勤奋。但是这并不意味这它能培养出优秀的计算机工程师。我们中间的很多人并不是为了学习而来到这里。这些人来到 MIT 是想要过来 hack 我们所能看到的一切东西。在我的本科宿舍里,我们有焊接设备,有木材切割设备,电子仪器,备用的混凝土和木材堆放在走廊。我们很容易就能进入到器材室或者机械五金店,拿到我们所需要的所有设备。而这一切都是为了非学术的私人用途。我们有个叫 Reuse 的邮件列表,你可以在学校里面到处跑去收集各种别的实验室不需要的奇怪设备,包括挂载机柜的服务器和老旧的杜瓦瓶。我们也有一个没有任何端口限制的开放网络,免费的静态 ip 地址,全双工的千兆网络接通到每间宿舍。MIT 鼓励我们使用这些资源来最大程度的满足我们对未来的想象力。在虚拟化技术普及之前,我自己的宿舍里面放了 15 台机器。我对门宿舍的哥们弄到了一个工业级激光器自己做了一个激光切割设备。另外一个哥们在忙着搞一个山寨的 RF 芯片识别设备试图找到地铁和学校的读卡器安全漏洞。还有人在宿舍自己弄了一个二层阁楼,还有人花了 200 美元把一个相机发射到了太空中只是为了证明这并非不可能,还有人做了个 3D 显示器,有人在折腾虚拟现实头盔,有人没事自己改 linux 内核……

这些东西都是什么时候做的?半夜。此刻的清华断掉了他们学生的电。我已经很难想象能有比强行断掉这个国家最聪明的一批学生的电力供应更能阻碍这个国家技术前进的方法了。没有电,你怎么做自己的项目?怎么在项目早期运行自己的服务器?而在 MIT,我们的公司脱胎于学生宿舍,卧室即是数据中心和实验室。

我们为此牺牲了学习的时间吗?当然。我们中的很多人来 MIT 本来就不是为了学习,我们来是为了做东西。

我们为此打破了很多规章制度吗?向来如此!但幸运的是 MIT 从来不会监督我们,没人到处盯梢,没有监视摄像头,没人告诉我们什么该做什么不该做,除非我们真的做了些对人身安全造成危险的事情。这样我们才能把整个校园(不仅仅是宿舍)变成一个高科技恶作剧的试验场,正是这些让我学到了从 Python 到 RF 设计的所有知识。而在清华,他们甚至不让我在里面照相。在每栋大楼的入口出都有一个静默严肃的保安看守。真希望清华的学生们在那里也能愉快的 hack。

这些 MIT 的校友们现在何处?一部分成了主流计算机工程和软件公司的 CEO,一部分正准备从硅谷公司辞职实现自己的下一个伟大创意。计算机工程技能首要并且本质上来自于实践经验,而不是课堂。这正是我们在那么多阴暗的夜晚里面学到的事情。

请不要误解我的话,清华的学生极其聪明,聪明,还是聪明。他们对于创造未来的渴望并不输于我们。我去过那里很多次,我的很多朋友都毕业于清华。真正的问题不在于学生,而在于学校和学校的氛围,传统的亚洲学术评估制度,以及其他一切阻碍了他们放弃学业投身实现梦想和未来的事物。

以下是原文:https://www.quora.com/Who-produces-better-computer-engineers-Tsinghua-University-Stanford-or-MIT

知乎用户 笨熊和傻兔 发表

在美国就是学数学的,对此深有体会……

本科数学物理双专业,研究生物理。美国人数学是真差,平均水平说是烂泥怕不是赞扬他们了 —- 那大学里美国人都抱着什么目的学数学呢?大概以下几类人:

(1)应付必修课,为了毕业不得不上

(2)认为自己需要一定的数学技能,比如读经济的

(3)为了好找工作,学好数理化走遍天下都不怕

(4)真心热爱数学,励志成为数学家

我们学校的数学课分为三个阶段:

第一阶段:必修课 “微积分 1” 或“统计数学”,这是必须上的,不论专业

第二阶段:Minor,副修,上完五节指定课即可拿到。毕业证上会写着拿到了 “数学 Minor” 副修。

第三阶段:Major,正经数学系学位。要达到一系列奇怪要求及一定的课程量(要保证高阶课的占比)才能拿到,对于不喜欢数学的美国人来说,难度非常高

=========================

**第一阶段:可谓鱼龙混杂,1-4 类型的人全都混在一起。**此阶段,我做为一个中国人,傲视群雄都说的算轻了。拿我室友举例,看看班里的平均水准:

写作业碰到类似(a+b)2 的式子,中国学生当然瞬间就能说出 “=a2+2ab+b2”。我直接写下来,室友很费解,说你怎么这么快,肯定不对。于是自己分解成“(a+b)X(a+b)” 后吭哧了 2 分钟,最后得出结论,妈的你算得还真是对的。这还好,毕竟美国人不背公式,然而:

我室友严肃的看着式子,说好像可以因式分解哎,于是又吭哧吭哧 3 分钟,我眼睁睁的看着他把式子又分解回了 “(a+b)X(a+b)”,然后自言自语,哦哦,又分解回去了哎…… 当时我可能是三脸懵逼。关键是,我室友在班里绝对算中上等水平的,你能想象班里都是啥水平了么?

再举一例,有节课要讲矩阵,为了铺垫老师先讲解二元一次方程组,是的就是那个咱们小学就掌握的技能。为了照顾到班里的所有同学,解方程组竟然讲了半个多月,才拿出第一个 2X2 矩阵,你敢信么?

**第一阶段我的状态:上课不如睡觉,睡觉不如不上。**曾当众解开了一个复杂的求导,被尊为 “天才”,同学主动替我打饭,甚至有人趁我上课睡觉在我本上写 “你解题的样子特别帅 —- 你的秘密崇拜者”。妈的这么多年过去我都不知道到底是谁 T_T 你倒是现个身啊

这个阶段中国人经常会聚在一起然后 “群嘲” 美国人,嘲笑他们数学真的是差到小学水平,各种段子满天乱飞

=======================

第二阶段:类型 1 退去,2-4 共同学习。最讨厌数学的人应付完必修课后已经火速撤离。

这个阶段,怎么说呢,大概是终于正式的进入了国内高中和大学早期水平吧。学一些基本的线性代数,空间几何,向量,数学证明。足以让一些人头疼,开始搞不懂作业,听不懂课,因恐慌而四处抱大腿。在这个阶段中国学生仍能轻松制霸,不过偶尔头疼一下而已。

然而让很多美国人无奈的是,中国人大部分都抱团,搞小团体,拒绝和美国人交流,导致美国人想抱大腿抱不上,万般无奈(捂脸)。我一个政治系同学称其为 “强者抱团,弱者游走” 的残酷社会现象。我室友倒是抱紧了我的大腿(是的这个玩意儿撑到了第二阶段)

**第二阶段我的状态:仍能轻松应对,在班里依然是制霸一般的存在,仍是老师心里的万能解题人。**但是因为难度增高,班里的美国数学尖子开始浮现。

=======================

第三阶段:类型 4 在上比武,类型 3 在下叫好。

这个阶段,才是开始真刀真枪上阵的时候,大二下或大三上。类型 2 也走的差不多了,毕竟你要跟学金融的说你们得学实数分析 / 群论环论 / 微分几何,他们可能会跳起来打你。拿个 Minor 就识趣的走人了。

第三阶段是 “登天式” 的难度增加,没有过度。可谓突然变天。一节课人数骤减为 20。浑水摸鱼的同志们在困难模式下纷纷溺水,因此美国数学天才们就全显现了出来。

可怕的是,你会发现这些尖子生,虽然计算能力比你弱,见识比你少,但他们对数学本身以及背后逻辑的理解非常强悍。在他们脑子里,各路数学理论可能已经形成了一定的体系,互相牵连,不再是碎片化的学习理论。

**中国学生前两个阶段仗着国内的数学教育一直在制霸,到这个阶段会纷纷败下阵来,**不再敢轻易吐槽美国人。我也一样,比如复数分析,微分几何,明显感到脑子不够用。所以这阶段基本就是前 20% 在台上争锋,后 80% 在下面看戏并且祈祷自己能过。

高阶课人更少,一节课 8 个人,上课多是讨论和互相出招,8 个里得有 5 个高手级别的,那真是短兵相接,有时看得我心惊胆战的,不得不服。至于剩下的 3 个基本只能是边看天书边保持微笑:)

凭努力和一些运气(主要还是口碑),我勉强在第三阶段保住了制霸的地位。**但是能明显感到,我不是真心学数学,**我只是解题。有几个最尖子的美国人,真能把这么多古怪的理论连成整个体系,这些人也许能成为数学家,我是肯定不行。

至于我室友,那个不要脸的已经知难而退,拿了个 Minor 就开开心心的去学中文了。天天腆着脸跟别人说,我可是有数学背景的人,我心想你拿了个 Minor 拽啥啊你,要点儿脸行不

第三阶段我的状态:作业开始费时,偶尔还要熬个夜。虽然仍是很多人的大腿,但已显出颓势

===========================

好在第三阶段及时结束,毕业了…… 我在大部分人心里仍然保持着霸主的威严,向研究生进发……

===========================

第四阶段:读研,重新认知自我 T_T

虽然研究生学的是物理的一个分支,但是跟学数学是一样的,我发现:

研究院里的美国人好多都是大神啊!!!!而且是真大神,随便挑出一个那都是我要跪拜的,轻则在作业上碾压我们,中则能在对物理的理解上甩我们八条马路,重则那简直就是未来的诺奖候选人啊。再也不敢吐槽这帮人了

一个年级的一半人,我得用崇拜的眼光去看他们(当然其中也有国人)。到这个阶段,国内教育的优势就已经消失殆尽,从小培养的高超计算能力基本没用了,反倒是自己对物理体系的理解开始捉襟见肘。到这个阶段绝大部分国人会沦为 “要抱大腿” 的状态,包括我 T_T

所以美国教育体系就是一个 “大筛子”,经过多次的筛选,坚持不了的早就被筛下去了,最后能留下来的基本都是 “类型 4”,同时是最尖子的学生。他们可以享受美国最高精尖的数学教育。同时为他们内心所向,一定能物尽其用,最大限度的利用这些资源 —- 进而真的在数学领域有所建树,也就回答了题主的 “为什么大众数学烂到原地自爆,但很多 NB 的东西还是人家做出来的”(此理适用于所有理科)

国内不太一样,平均水平确实比美国高了十万八千里。但是最终去读博,享受最高教育资源的人实际上还是 “类型 2-4” 混在一起,读博的理由五花八门:

读完硕士不知道干什么,惯性读博

不想离开象牙塔,不如读博

单纯为了好找工作,混证读博

家长是博士,强迫读博

……

(美国自然也有这些人,但是比例小)

国内真正因志向学数学的人,那是少之又少,当然这里面有很多社会因素在里面。所以各有利弊。国内教育的利处就是平均水平高,老百姓不至于连找钱都费劲,弊端就是真正想学数学的人少,而且还不能分到最大化的资源,所以真正的大数学家就少。

知乎用户 落魄的小爬虫 发表

从首个 IMO 季军谈起(连载中,极品好文,第三段到第六段完美回答了这个问题, 太长了你赞一个然后跳着看吧)

http://blog.renren.com/blog/229607718/970519550

作者 : 付云皓 [他写的,不是我],02,03 两年 IMO 金牌,满分,中国国家队中的战斗机,中学数学竞赛史上最强战力之一

![](data:image/svg+xml;utf8,)

刚刚过去的 IMO,中国史无前例地获得了第三名,也是自 1997 年来近 20 年首次跌出前二。感谢微信等社交软件,相信现在这个新闻已经以火箭的速度传播了。

作为一个与数学竞赛及 IMO 打了多年交道的人,我一直有写点什么东西的冲动,但一直由于懒癌拖拖拖。赶上此时此事,我觉得不能再拖了,写一点感受吧。

首先插个链接:

http://www.imo-official.org

,这是 IMO 官方网站,上面记录了历届 IMO 的数据,只要你会点英语都能看下来。以下所有有关 IMO 的官方数据均出自此处,如有数据错误,请指出。

Part1 曾经的霸主

中国的奥数强不强?是不是梦之队?

五年前你问我,我会很干脆地回答就是梦之队,但现在你问我,我会说,强,但不是梦之队。

圈内普遍认为,中国在 IMO 上大放异彩大约就是 2000-2010 这十年(实际是十一年)。20 世纪 90 年代吧,虽然也是强队,但有一次并列第一,两次第二,一次第六,还有一次未参赛(当然,未参赛不能怪中国队,细节涉及黑历史,略),银牌铜牌还是不少的。

在官网上看 2000-2010 中国的数据,11 次参赛 66 人次,61 金 5 银,9 次第一 2 次第二,绝对亮瞎眼。61 金 5 银什么概念?就是说如果你不是中国队里最弱的两个之一,那么你肯定是金牌,即便是最弱的两个之一,也有大约四分之三的概率拿金牌。

But 你以为这就是全部了?大错特错。

2004 年,某队承认自己前一年 “played something unfair”(明白人都懂不再解释)。

2007 年,中国在集训队淘汰了两个大 BOSS,加之当年题目奇葩(可以搜那年的数据,真心坑),以及主办方越南的双标(甚至对中国队采取了笔误扣分),中国落后俄罗斯 3 分屈居亚军。

把这些数据修正一下,再看看中国队在 2000-2010 的数据,用神队形容也不为过。

不仅如此,官方公布的中国队 9 次第一中,除 2000 年赢第二名 3 分,还有 3 年赢 8-9 分之外,其余 5 年均赢接近 20 分或 20 分以上,最夸张的 2006 年竟然赢第二名 40 分!也就是说除了柳大师之外随便找一个人交白卷,中国还是第一!

这不是虐杀十条街么…………

那么下面一个问题,2000-2010 的中国队到底领先其他队伍多少距离?

一个数据不说明问题,一组数据光看也不说明问题,分析完再说。

每一年情况不同,各队选人就不一样,题也不一样,临场发挥还是不一样,尤其在这种各比各的最后汇总成绩的比赛中,随机性真的很高。但是看看中国队的数据就能发现中国称霸的秘诀:强,而且稳定。

由于各年题目难度不同,我们以金牌分数线 * 6 来作为标准分数,看看每个国家和标准分数的差距(考虑到 2007 年这种神年份的难度被金牌分数线误判,个人将 2007 年的金牌线修正到 26 分参与计算)。

平均实力最强的中、俄、美相关数据如下:

中国队每年总分均超出标准分至少 25 分,超出部分的平均值约为 34,标准差大约为 7。

俄罗斯除 2003 年外均超出标准分,超出部分平均值为 14,标准差大约为 12。

美国在标准分上下波动,超出部分平均值为 3.5,标准差大约为 10。

如此可以看出,中国队超出的成绩平均值不但远高于俄罗斯和美国,标准差也小。以正态分布模型来计算,中国输给俄罗斯的概率也就是百分之六、七,输给美国的概率不超过百分之三,至于其它国家?那都是小概率事件。

中国队为何如此所向披靡?且听下回分解。(TBC)

![](data:image/svg+xml;utf8,)

Part2 取胜的关键

中国队有如此稳定的成绩,秘诀是什么?

中国队没有常青树类型的人物,像 Halls of Fame 里的 Raid Barton, Christian Reiher, Iurie Boreico, Lisa Sauermann, Alex Song 这样挥挥手 N 块金牌,荣誉一大堆的选手,中国一个都没有。事实上,还没有任何一名选手代表中国队参赛 3 次(但有人进过 3 次国家队)。

道理也挺简单的,中国在 2000 年之前就有了系统的选拔,而且 2010 年之前冬令营只有 100 多人,也就是说你必须在联赛中进入全省最前的几名才能进冬令营,更别说集训队只有 30 人左右,随便一处失误都有可能葬送了你。一个初中生要想冲进国家队,简直是难如登天。2010 年之后的问题,后面再讲。

那么,中国队要怎样才能拿第一?

这个问题,我问过奥数界的泰山北斗裘宗沪老师,时间是 2002 年我入选国家队之后。

(顺带一提,2002 年的 IMO 中国国家队,不管是我们自己还是老师们,都觉得是一支烂队。裘老师当时给我们下的目标是 3 金 2 银 1 铜,总分前三。)

裘老师当时回答了三点:

1:1,2,4,5 四个题不丢分或少丢分;

2:3,6 题对选手的胃口;

3:第五,第六名比其它国家的第五,第六名强。

第一条对应稳定,第二条对应运气,第三条则对应短板原则。

![](data:image/svg+xml;utf8,)

在比赛场上,我们完美贯彻了这三点。当年的俄罗斯是一支强队,加上 2,5 不难,我们在 1,2,4,5 虽然基本没丢分,但并不处于优势。但是,运气站在了我们这一边,比赛中出现了第 3 题这样的要通过代数辅助的数论题(这正是我们的强项),我们在这一题中拿到了全场最高的 24 分,与此同时,我们的五六名(各 30 分)强于俄罗斯的五六名(各 29 分),终于使我们有惊无险地拿下了总分第一。

纵观这 11 年的数据,基本都符合第一点,第二点么需要仔细翻题目,不过很多数论与代数结合的 3,6,我们都做的不差。关于第三点则有很多经典的例子,例如 2004 年四个满分没有一个是中国的,但中国队的最低分 34 仍处于并列第 28 名,总分第一(顺带一提,总分第二的美国队同样没有满分,最低分 31 仍然不低,第三名俄罗斯虽然有两个满分,但有两个人分数太低,无力回天)。与此相似的还有 2009 年,日本异军突起,在史上第二难的蚱蜢题里拿到了不可思议的 19 分(此题参赛 565 人只有 25 人得分,所有人加在一起还没得到 100 分!!),但一个 23 分使他们最终还是败在了前五题铁板一块的中国队手下。可以看到,2009 年若将中国的韦教主换成一个 35 分,中国还是会赢,但若将日本队最后的 23 分换成一个金牌分数,中国队就危险了(这里稍微透露一下,在稳居第一,且没有处于金银牌分界线的选手的前提下,队伍一般在协调时不会死皮赖脸地争分,所以事实上中国赢日本不止 9 分)。这也是短板原则所在——毕竟一个满分也就比 35 分多 7 分,而一个低分可能拖累远不止 7 分。

道理都懂,为啥其它国家不能效仿呢?

第一条其实处于前列的国家都在效仿,美国和俄罗斯在 1245 题发挥也不错。第二条纯属运气,无法操控。我认为,数次帮助中国队夺冠,其他队伍难以效仿的,是第三条。

美国队前任领队冯祖鸣老师自小在中国长大,与中国队交流也较多。有几次问过他当年美国国家队的学生水平如何,他经常会说:“今年只有 4 个学生能稳做 4 道题,很悬” 什么的。

“稳做 4 道题”即基本能完成 1245 的水平,也是我们所谓 “高手” 的水平。稳做 4 道题实际意味着学生成绩的期望至少是 5 道题,如果六个学生都是这个水平,那么总成绩期望将至少是 210 分以上。在 2000-2010 年中的大部分年份,中国队所有队员均能保持这个水平。

相对来说,其它国家的队员就难以达到这样的水平了,总会有那么几个差的。即便是 2007 年惜败俄罗斯的时候,中国的最后两名分数也高于俄罗斯最后两名的分数。

这件事在一定程度上也是无可奈何的。尽管训练和选拔机制比中国完善的早,但是毕竟基数有差距,从一两千万高中生里选拔 6 个高手总比两三百万高中生里选拔 6 个高手容易吧。

中国数竞人就这样维持着自己的优势。但是,在 2010 年之后(或者说事实上是从 2010 年左右开始),IMO 这个比赛从题目本身到对手到中国队自己,都产生了一些变化。

(网上应该有很多谈论中国自身问题的,于是我们把这一部分向后放一放。)

IMO 的游戏规则从很早以前就没怎么变过,那为什么题目本身会有变化呢?且听下回分解。(TBC)

![](data:image/svg+xml;utf8,)

Part3 规则的变化

说到题目的变化,首先就得说说 IMO 的题目是怎么选出来的了。

IMO 的题目来自于全世界,比赛当年 3-4 月,各个国家或地区被允许以领队的名义向主办国家(或地区)发送题目,一般来说每个国家最多发 6 道题,都是严格保密的新题,且自认为优质的题目。这些题目一般有 100 多道,称为 Longlist。主办国家(或地区)在收到题目并整理后,组建一支选题委员会(Problem Selection Committee),选题委员会的工作是在这些题目中选出真正优质且全新的题目并加以分类,一般来说初等代数、组合数学、平面几何、初等数论四个领域各 6-9 道题,总计约 30 道题左右,并将每个领域的题目按由易到难的顺序排序。被选出来的这大约 30 道题,称为预选题(Shortlist)。在 IMO 考试前几天,各国的领队及观察员 A 提前集中,拿到这些题目并进行品评,然后投票选出 6 道题作为考试题,同时要求每天的 3 道题分属不同领域,且每个领域至少有 1 道题。在投票过程中,每个队伍(领队 + 观察员 A)有一票。

从 Longlist 到 Shortlist 的过程虽然也挺有意思的,但是与主题无关,这里略去。主要来看从 Shortlist 到比赛题的过程。

在这个规则下,强队所需要关心的不太多,像中国重点关心也就两点。

第一,25 是不是难度恰好,能够让自己的学生基本全拿分,别人的学生拿不全,从而拉开差距;

第二:36 是不是能碰上一个代数题或者是用代数较多的数论 / 组合题。

对于 2000-2010 年的很多年份,甚至这些都不需要,所以中国队基本不太 Care,最多控制一下题目难度顺序别错了。

但是对于一些小国家那可就不一样了。

如果你是一个小国家的领队,那你该怎么办?

其实这个问题从更早就开始了,小国家怎么训练和选拔学生?

什么?你不知道?那回过头看看现在那些 “偏弱” 的,要冲联赛一等奖的学生怎么准备二试吧。

没错,就是狂刷平面几何,最多带点简单数论和三元对称不等式什么的。

国外的天不一定比国内的蓝,很多高手觉得无聊的中国快餐式奥数选手培训流程,被很多小国家一直在模仿,且从未被超越。

那么,当你领着 6 个基本只会做几何(而且还做不了太难的几何题)的选手进入 IMO 的时候,你想要考试出什么题?

首先,你得选几个你的学生能拿分的题。

于是乎……

我们要几何!我们要最简单的几何!我们要 G1 和 G2!(即 Shortlist 中最容易的两个几何,G 即为 Geometry 的简称,后面的 A,C,N 同理)

于是,很多年份中的 G1 和 G2 全被抽中,G2 会被放到 2,5 之一,而它的难度往往连联赛几何题难度还不如!

但是没办法啊, 更 DT 的是小国家一出来就是二三十个,你强国也就一票,只能保留意见。

![](data:image/svg+xml;utf8,)

当然作为小国领队,弄到两个容易的几何还不够,因为你知道强队终究会靠 2 和 5 中的另一个,以及 36 与你的学生拉开差距。

怎么办?怎么办?

于是乎……

KAO!老子豁出去了!

14 不是还有一个位置吗?我们投最简单的!A1!N1!咱们六个学生好歹也是练过的,总能蒙一两个出来吧。

36 题我们投最难的!我们要 A8!C8!N8!你们不是牛 13 吗!难死你们!

25 还剩一个?没辙了。算了,我们泱泱小国,送你们了,让你们赢去。

…………

………… 呵呵

当然,组委会不会任由这些人乱来,领队一般也不会做得太过分,不过最终结果经常是有两个简单的几何,另有一个超容易的题,然后有一个超难的题。

2000-2010 年的大部分年份都有这个特点,除 2005 年的 36 都很容易之外,其余年份都有一个或两个题很难(神年份 2007 年的 36 都超级难,但我认为 6 是领队们误判题目难度所致,因为 6 看解答并不困难,与此相关的内容后面有详述。)

这个困境总得解决啊,不然 IMO 成套路了玩个毛啊。

于是,我们的 Problem Selection Committee 在 2011 年玩了把大的。

相信当小国的领队们拿到 2011 年的 Shortlist 时,他们的脸上一定是一副 “囧” 的表情。

看看 G1,让你证明两圆相交,我们学生没训练过啊,逗我玩?

看看 G2,让你证明几何恒等式,我们学生没训练过啊,逗我玩?

看看 G3,一个四边形………… 卧槽这是 G3 吗,怎么那么复杂,我图都画不出来,逗我玩?

看看 G4,……,卧槽居然到 G4 了!

怎么办?怎么办?

看来没有一个几何能让自己学生拿到分了。

KAO!老子豁出去了!我们投 G8!

还可以投一个几何,但是貌似选前几个哪一个都会吃大亏…… 哎哎哎,组合题里居然有一个组合几何!还是 C3!太好了!我们投 C3!

结果……C3 和 G8,惨不忍睹。

(另有一说法,C3 的题目叙述有风车,与主办方荷兰很配,于是 Problem Selection Committee 设了这个局,使得 C3 能够入选,另外当年投票环节也出了一些小问题。)

组委会发现这样也不行啊,只换来一场闹剧,于是在 2012 年调整一年之后,开始出现了一个新的潜规则:1245 四道题里必须每个领域一道题。2013 年到 2016 年的四年均遵循了这样的规则。

在 2013-2016 四年中,除 2015 年外,其余三年的 25 都属于正常难度(2015 的第 5 题偏难了,可能是因为泰国搞了大飞机,在第一天考试结束时把原来第二天的题目误当成第一天的题目发给了考场外的副领队们,结果又重新投了一遍题目的缘故)。容易看出,新规则下更考验学生四个领域的平衡性,有一个领域有短板就很容易拿不到四个题。

而另外两个难题呢?很不幸,它们仍然被掌控着。平面几何和组合更容易有那种很难,但同时很漂亮,解答还能看的难题,所以 2013-2016 的 36 无一例外是几何 + 组合(个人认为今年第 3 题实际上应该算几何或者组合,数论的东西用的很少,都是平凡的)的搭配。

IMO 的规则在慢慢变化,但光靠这点还不足以让中国以往拥有的巨大优势土崩瓦解。那么,美国队迎头赶上,连续两次获得团体第一还有什么原因呢?这就得说说美国队的训练和选拔方式了。且听下回分解。(TBC)

![](data:image/svg+xml;utf8,)

Part4 真实的美国

“美国人数学很差。”

这是 99% 中国人的感觉。

真实情况呢?

美国的基础教育中,数学的强度确实远弱于中国,但是美国的基础教育可是下午两三点就放学了,课后的辅导班层出不穷,顶尖的孩子学习数学的强度和深度绝对不弱于中国同层次的孩子。

“美国队拿 IMO 冠军全靠中国小孩,美国队 6 个人全是中国人或者华裔。”

这纯属无良的,想拉风的媒体的报导,如果你连这都信,那我写的东西就不用再看了。

真实情况呢?

自己去搜搜美国队小孩的名字,一般来说 2-3 个中国人或华裔就封顶了。

最近也爆出中国小孩 “转会” 到美国的风波,但是我们冷静下来想想,能 “转会” 的,要么应当是原来在中国考不进国家队的孩子,要么是由于政策或制度原因才去的其它国家吧。

![](data:image/svg+xml;utf8,)

2009 年在德国,我们的学生和加拿大队的关系很好,加拿大队的 Danny Shi 和 Robin Cheng 等人便是在小学或初中时从中国转学到加拿大读书的孩子,他们也坦然地说自己在中国考不进国家队。最后中国队最低分 35 分,Danny 考了 34 分,Robin 考了 29 分(金牌分数线 32 分)。如果靠这些孩子能打败中国队,那不是更说明我们的训练和选拔机制不如人家么?

关于后一点,如果是从小呆在美国长大的,那教育与中国无关,如果半路出去的,去搜搜小山智丽吧(中国真的出现这个情况,只能说明我们的体制还有待完善之处)。

回到正题。先来看看美国队是怎么选拔和训练的吧。

美国中学生想参加 IMO,第一步要参加的比赛叫 AMC,AMC 分 8,10,12 三个年级,同时考试,试题有 25 道,限时 75 分钟,每道题都是 5 选 1 的选择题,答对得 6 分,答错得 0 分,不答得 1.5 分(这是鼓励不要乱蒙答案)。

AMC 的难度很低,即便是最难的 AMC12,前 5 题都是中国小学口算难度,6-10 题基本上中国高考选择都不会出,前 17-18 道题对于稍有中学竞赛经历的学生都不算题目,真正有点棘手的问题也都出现在 22 题以后。

AMC12 里得到 120 分以上,或名列所有参赛选手的前 5%,AMC10 里得到 135 分以上,或名列所有参赛选手的前 1%,即可参加下一阶段的比赛,称作 AIME。对于学过竞赛的学生来讲,这都不叫事。(此处数据是往年的,可能有些不准确)

AIME 是一个时间为 3 小时的比赛,15 道填空题,每道题的答案必然是 000-999 之间的数字,为此题目的问法有时会很怪(自己去看几道就明白了)。AIME 的难度开始增加,大约相当于中国联赛一试填空和解答之间的难度,并且计算量更甚,最后一两题将在此基础上更困难。将 AMC 和 AIME 的成绩综合起来,名列前茅的选手可以参加下一阶段的比赛,称作 USAMO(即对应中国的冬令营)。一般来说,AIME 做对 10-11 个题,加上 AMC 的底分足够让你进入 USAMO。不过在中国随便找一个省(不是最强的那种),可能只有几十人能达到这个标准。

AMC 和 AIME 的形式看似不太好,但是它却有一个无可比拟的优点,那就是阅卷容易。由于全部题目都是选择填空,而且填空是可以填答题卡的,所以阅卷成本和错误率都几乎为零。这样,可以保证一个真正优秀的数学竞赛生能够稳进 USAMO。

USAMO 参赛人数众多,一说有三四百人,跟现在的中国冬令营规模相同。USAMO 只考 6 个题,选拔几十人成为美国的集训队。到此为止好像和中国的制度没啥不同。

说到集训队,亮点终于来了!USAMO 是四月考,集训队再推后根本没有时间组队或办签证,但是老美早安排好了,他们的集训队是为下一年的国家队准备的!!

老美的集训队选出 8-10 个 “候补队员”,他们将与当年的国家队一同训练,当年 IMO 结束后,这些人将与国家队里还想继续做的人一起成为下一届的重点培养对象。从当年 IMO 结束到大概第二年 USAMO 结束这段时间,他们将进行大量集训,并四处参加比赛或自己来模拟测试,最后由领队综合各次的成绩选出 6 个最优秀的队员组成美国国家队并持续训练到下一届 IMO 开始。

这样的制度有两个优点,第一,真正优秀的学生只要在 USAMO 发挥好(或者说别发挥差)就可以了,后面都是大量集训,完全能考出真实成绩,也能看出一个学生的优点和缺点;第二,选拔之后的训练,强度也远超其它国家。相比老美的一年左右的训练时间,中国队不到四个月的训练时间显得相形见绌。(当然这套东西是不能搬到中国的,后面细讲)若说缺点也有一个,就是要求你高二必须考出来,高三去考 USAMO 是进不了国家队的(另一说是如果超级优秀可以考虑)。

IMO 说到底还是个比赛,取得好成绩的方法从本质上来讲和大多数体育比赛并没有什么不同。这里转一段别人的文章。我第一次看到这段话是在微信的 “Lens 杂志” 上看到的,深以为然。

“心理学家 Ericsson 的研究发现:决定伟大水平和一般水平的关键因素,既不是天赋,也不是经验,而是‘刻意练习’的程度。刻意练习是指为了提高绩效而被刻意设计出来的练习,它要求一个人离开自己的熟练和舒适区域,不断地依据方法去练习或提高。比如足球爱好者只不过是享受踢球的过程,普通的足球运动员只不过是例行惯事地训练和参加比赛,而顶尖的足球运动员却不断地发现现有能力的不足,并且不断以自己不舒服的方式挑战并练习高难度的动作。”

换句话说,如果你的水平去 IMO 能平均做 4 个题,而你还盯着 IMO1245 难度的题做,那就是在舒适区,提高不会显著,你要做的应当是挑战 36 难度的题目。如果你的几何好,组合差,那么继续做几何题不会给你太大的帮助,你应该尝试着做你原来很怕的组合题。如果你已经是 IMO 保 5 题争 6 题的水平,你应该做的是继续挑战更难的题目。

老美在这一点上做得很好,因为他们在 2010 年前后引进了一个新的玩意儿——ELMO。

![](data:image/svg+xml;utf8,)

ELMO 是美国当年的国家队队员(Sophomores)出题来虐待新进来的候补队员(Freshmen)的一个练习,它还有 Shortlist。在我看来 ELMO 是一个 “反人类” 的东西,它的 Shortlist 远难于 IMO 的 Shortlist,如果一个中国学生能够做出 ELMO Shortlist 里 60% 以上的题目,那么他绝对可以在中国的数学竞赛里横着走。

ELMO 和它的 Shortlist 就是地地道道的刻意练习了,正如中国举重队上台举 120 公斤练习举 140 公斤一样,到了国家代表队这个层面,思考超出你目前水平的题目会对你的水平大有帮助。与此同时,将要出征 IMO 的 Sophomores 还有一次命题的练习机会,对解题亦有不小的帮助。

前面说过,美国顶端优生的数量比不过中国,但是美国的选拔制度却将这些人基本都筛了进来,而魔鬼般的训练体系则使他们比原来更厉害。即便是 4 个 “高手” 和 2 个低一层次的 “准高手”,经过长时间刻意练习后,也有和中国 6 个“高手” 一战的实力,更不用说运气好的时候能凑齐 6 个 “高手” 的情况了。而这,也是美国能连续两届拿到 IMO 团体总分第一名的一个重要原因。

美国的训练和选拔机制有些 “反人类”,但它仍然长期存在并且得到了学生的认可,这又是为什么呢?且听下回分解。(TBC)

Part5 环境的优势

本节涉及大量主观观点,不喜请屏蔽。

上一节最后提到了 “刻意练习”,我们在进入正题之前纠正两个相关的不当想法。

想法 1:在数学竞赛上 “刻意练习” 就是浪费时间浪费生命,就是“黄赌毒”,应该严厉打击。

上一节也说了,像 IMO 这种世界级的比赛,刻意练习是必须的。马上就是里约奥运会了,想想有多少参加奥运会想拿牌的选手没刻意练习过?恐怕一个也没有。若是练跑步打球游泳的人刻意练习是应该的,练数学竞赛的人刻意练习就是黄赌毒,那不就成了双标?

啊对,你说竞技体育有表演性质,体育运动员是兼职演员,所以得刻意练习。那我给你找个没表演性质的,或者外行无法欣赏的东西——围棋。照这个道理,那个李世石,别刻意练习了,浪费生命;那个柯洁,别刻意练习了,浪费生命;那个阿法狗,别………… 算了你生命无限,随便练随便练…………

抛开这些不谈,回到数学本身,别忘了想学好数学,多做难题也是必须的,不然吉米多维奇是干啥的?连数学本身都需要一定的刻意练习,就别说数学竞赛了。

想法 2:既然 “刻意练习” 能出成绩,我不管是什么水平,就 “刻意练习” 好了。

练习要和能力匹配。在竞技游戏中,你的能力(或者称之为等级)越高,刻意练习的绝对收益可能不变甚至更小,但是相对收益更大。

苏炳添一百米原来跑 10 秒 2,练习 N 年跑 9 秒 99,从亚洲级跑进世界级。

你一百米原来跑 15 秒,练习 N 年跑 14 秒,从街道级跑进…… 还是街道级。

明白了吗?

只有顶级的选手,才配得上大量的刻意练习来提高能力。以高联为例,一个省上万人参加高联,只有五六十个省一,如果你自问数学水平排不到省里前一两百,请把更多的时间花在高考或者其他事情上,把竞赛当作一种爱好,把高联当成一次自我检测或者一次娱乐。

这里第一次强调,如果学竞赛只是为了拿奖,那么感觉不适合或达不到拿奖的水平请立即退出。我不希望再看到家长哭着问:“为什么我家孩子学竞赛学了那么多年,连个奖都考不到?” 高中竞赛不是小学生过家家,它是残酷的淘汰赛,有的孩子连快餐式培训流程都接受不了,还想要拿奖,最终就是费钱费时间,还只学到了一些皮毛(以后也用不着的东西)。

回到正题。美国小孩为什么认可 “反人类” 的重复训练?我认为原因有三点,其一是有兴趣,其二是有保障,其三则是荣誉感。

![](data:image/svg+xml;utf8,)

美国的选拔制度,使得真正的天才选手不需花费太多时间在 AMC 和 AIME 的备考上,可以直接跳到备考 USAMO,这样前期基本不需要刷题。前期做得适度,后期就更容易坚持兴趣。而且老美的尖子生也少,真正的天才不需太多训练就能考出来,而达到一定高度之后再去刻意练习,会使学生最终的等级更高。在此也奉劝数学竞赛的天才和小天才们,到达顶峰之前少刷点题,多开发思维,多保持兴趣。

当然,哪个国家都不缺乏有兴趣的孩子,那么第二点就是老美独有别人基本学不来的:保障。

哪个国家高中还学竞赛的孩子都会想,我花了那么多时间做这个,甚至到后期就是反复训练,万一失败了,谁来保障我的权益?我去哪读大学?

老美做这个当然容易了,老美那么多名校,USAMO 就三四百人,大不了都收了去。换成其它任何一个国家,恐怕都做不到吧。

另外老美还有一个隐藏的优势,就是老美从 USAMO 开始(或者说从集训队开始)可以认为基本没有考试以外的奖惩。也就是说,进了这个小圈子,大家基本都是名校随便挑,考好考差都没所谓。这样,既可避免学生过于功利化地学习,让不是真心想在 IMO 上有所建树的孩子自动退出,还可以尽量避免不公平竞争(如作弊等)影响最终选拔的公正。

真正有兴趣的孩子,会在这个机制下,如同还没出名的竞技体育运动员一样,心无旁骛地主动刻意练习的。

关于荣誉感,先放上我与一位朋友在 2014 世界杯时探讨出的一个结论。

“在竞技比赛或者任何事情上,一个人或一群人能做出傲人的成绩(排除天分),要么就是他们需要用成绩来改变自己的命运,要么就是他们有着极强的荣誉感和使命感。”

例如,同样是踢足球,南美的运动员就是前者,欧洲的运动员就是后者。两者都不占的,必定踢不好。

不得不承认,老美从国家文化上就特别注重荣誉感和使命感。日本的 IMO 队员可以在头上围 “必胜” 字样的头巾,韩国的领队可以为了一分用尽方法,朝鲜人甚至可以不遵守规则,但他们比起老美来,可都差多了。老美的潜移默化的洗脑,是比前面那些都更加恐怖的存在,这一点想必很多人比我更加有体会,这里就不说了。

老美的选拔制度,高等教育条件和国家文化使得它的 “反人类” 体制可以继续下去。在整体的选拔与训练上,美国人已经领先了一大步。与此同时,中国的情况如何呢?且听下回分解。(TBC)

Part6 拥挤的游戏

上一节讲到了美国现在数学竞赛上顶尖的小孩所处的状态,看起来说了不少老美的好话。我这里声明一下,我绝不是美分。

其实我在前面有不少伏笔了,相信跟我同时代或略早于我的数竞党都应该能猜到了,包括我在说 ELMO 的时候也用了 “引入” 这个词。

事实上,上一节讲到的美国小孩拥有的所有状态,兴趣,保障(高端无奖惩)和荣誉感,那都和我读高中时(或前后)中国小孩拥有的状态几乎一致!

2000 年前后,还有很多参加冬令营的同学在参赛前甚至不知道进了冬令营能保送,冬令营金银牌可以直接进清北(现在的同学别羡慕,那时候金银牌加一起也就六七十人)。

我们考完冬令营之后,郑志明塞给高分的学生一人一张纸,说这张纸就是北大录取通知书,那时候还是有一些人不信他的话。

那个时候学竞赛,真的靠的都是兴趣。

我上高中的时候有个很旧的文曲星,上面存了 2001,2002,2003 年部分国家集训队同学的名字及去向。那个时候大约三分之二的人去北大读数学,大约四分之一的人去清华读计算机或者基科班,剩下的人还有读物理的,出国的凤毛麟角。大家都是清北,高端无奖惩,全凭自愿。

关于荣誉感,那个时候多多少少都是有一些的,可能因为我们读高中的时候更单纯一点吧。包括拿了金牌回来还是能显摆显摆的。顺带提一下,我的两块牌分别是安妮和德仁颁发的,安妮看起来不太容易升级了,我现在就等明仁退位了……

包括到达一定高度后的刻意练习,中国早就有了。那个时候的国家队队员需要每人提供 20 道难题(当然不一定都是原创),然后互相考(所以我前面才用了 “引入”)。2001 年的时候我旁听国家队集训,被 6 个队员加上一堆教练虐了 20 多天之后水平一下子有了质的飞跃。这个 20 道难题有多难呢?我拿了一个我自己提供的比较难的组合题去考 2009 年的国家队,只有一个人做出来,还不是最强的那个。

当然,2003 年到现在也 13 年了,朝代更替,变化诸多。不过呢,除了大的政策之外,剩余的都是慢慢变化的。我个人认为,影响最大的一个变化,或者说根本的原因就是,参与的人变多了。

我们读高中的时候,真没多少人好好做竞赛。我记得那时北京市在高联结束后会组织冬令营培训,我高一的时候正式队员 6 个,旁听生算上我不超过 10 个,就一个小班,在人大附中每周末上一次课。都谁给我们上课呢?有周沛耕,陶晓永这样的老师,有邹瑾,韩嘉睿,袁新意等 IMO 金牌,后来肖梁毕业之后他也来讲过。这种课没多少来听的,只能说明竞赛这东西根本没多少人玩。

其它省份也差不多,好好学竞赛冲冬令营的人最多几十个,其他人大都打酱油。

那个时候有很多人高联只考一试不考二试的,因为考二试要多交十几块,当然这十几块只是顺便省下来的。

那个时候冲冬令营的人都是二试冬令营一起准备,基本不刷一试题的。

2001 年 10 月我参加高联的时候还考砸了,一试一个短轴算了半天填了个短半轴,二试一个组合题没完全写清楚。即便这样,我比北京的分数线还高几十分。很多同时代的同学都有类似的感受(2000 年高联除外,题目过于简单)。

一个省只有几十个人好好玩的年代,这么玩当然可以。但是一个省上千个人玩命准备的时候,这么玩就不行了。其实人家水平比你差远了,但是人家高联一试能刷到接近满分,二试做一几何,直接 200(那个时候一试 150 分,二试 3 个 50 分题),你要是还按老办法玩,没准就直接 OUT 了。

这么多人挤进来,大都是为了好政策。高联一等奖能拥有保送资格,万一冲进冬令营蒙个两三道题没准能进清北,诱惑太大了。

于是,年轻的天才们一年比一年付出更大,一届比一届刷题更狠,只为了在几千个非天才的围追堵截下冲到前几名。

惨不忍睹。

真的惨不忍睹。

不能怪任何刷题的非天才和他们的家长,制度如此。

中国的优质高等教育资源实在是太匮乏了。

如果中国能有 20 所清北,200 所双一流(或 985),可能事情会好办很多。可是中国没有。

2009 年,我第一次参加冬令营阅卷,当我看到一份 30 多分的卷子(冬令营 6 个题,每题 21 分,总分 126 分,当届需 102 分才能进集训队)的主人,联赛是 260 多分的时候,我先是诧异,然后是无奈,最后则是深深的叹息:可能就是因为这个人,一个高手被挡在了冬令营的大门前。

对于高联 - 冬令营 - 集训队这一条线来说,选拔国家队是其中一项重要任务,如果高联就把全国的高手削掉一半,那后面还怎么玩?

所以,高联被迫改制度,经过 2009 年的调整之后,最终改成了现在的 120+180。

即便是这样还不够,伴随着教育部的政策,数学会和奥委会又大幅度增加了冬令营各省的参赛名额。

即便是这样…… 还不够。

负责任地讲,同样在不玩命刷题的前提下,一个高手现在想进冬令营,难度要远大于 13 年前。即使现在是 400 人冬令营,那时候是 100 人。

水涨船高。

有办法解决吗?

现在来看,真没有什么能快速见效的好办法。

不幸中的万幸是,高手毕竟是高手,高手也刷题,谁都挡不住。一阵玩命死刷题之后,大部分高手也能冲进冬令营。但是这样的高手,由于过早地接触重复训练,对后面多多少少会有一些影响。

参赛人数的增加,直接大幅度增大了真正高手一飞冲天的难度。不仅如此,它还通过其它方式间接增大了这个难度和不确定性。究竟参赛人数的增加会产生哪些连锁反应呢?且听下回分解。(TBC)

![](data:image/svg+xml;utf8,)

Part7 疲倦的裁判

参赛人数增加,第一个影响的就是阅卷的困难。

2015 年 9 月,我有幸参加广东省高联的阅卷和复查。

阅卷的时候,14 个大学老师,7 个题目(一试 3 个大题,二试 4 个题),差不多一千份卷子。

初改要一天完成,毕竟大家都是老师,都有教学任务,谁也没太多闲工夫。

初改不计复核,一个人也要面对五六百张卷子。

一天改五六百张卷子的一道题很难吗?

很难。

竞赛不是高考,联赛的解答有的你读懂就要五分钟。

竞赛不是高考,不按标准答案做的人多了去了。像二试第一题这种题目,据不完全统计,有十几种不同的解法。

竞赛不是高考,你必须判断出学生的解答是基本对还是基本错,是有小瑕疵还是有大漏洞。

还有更坑的呢。

我改几何的时候看到一张卷子写由托勒密定理 blablabla,由角元塞瓦定理 blablabla,由笛沙格定理 blablabla,证毕。我当时真有一种想把这个骗分的家伙拉出来的冲动,最后还是放弃了。孩子是无辜的,没准就是被无良老师教的。

我那天算是试出来了。

我一个白天的极限是改八百道题,复核不算,而且中饭和晚饭都是味同嚼蜡。

第二天我还要上课,我晚上躺在床上,进祖坟的心都有了。

我不禁怀念我读书的时候,那时候要是去改卷,可比现在轻松多了。

梦想是美好的,现实是残酷的,因为改卷完了还有复查!复查!

复查的卷子少了一大半,但是复查的题量变成了全部!而且复查的题里大部分都是写了东西的!

坑哥啊!

我觉得吧,如果只是为了钱,有人拿百元大钞砸我我都不干这个活。

也许是因为我自身的经历吧,我每次拿起红笔的时候心里都是很严肃的。学生实力不够或者没发挥好,OUT 了就 OUT 了,但是学生做对了判错了,这是无论如何不可以的。

高联如此,冬令营更甚。现在 400 人的冬令营,想想就头疼。

首先,冬令营的题变难了,解答也变长了,尤其是时间变成 4 个半小时,够写四五篇作文了,想编点什么东西很容易。经常会出现读十分钟还没读懂答案的情况。

其次,冬令营的每一分都很重要。高联的 30 分和 50 分没区别都是三等奖,100 分和 120 分没区别都是二等奖,只要重点照顾一下省一左右或者以上的卷子,就不会出问题。冬令营可不行,各大学校都看这个成绩招人呢。去年要是一个 48,一个 45,前一个可能就是清北,后一个可能就是孤魂野鬼啊。所以冬令营这种比赛,卷子要复查多少遍,没改过卷子的绝对想象不到。

还有,正因为冬令营分数敏感,复核的时候经常要充分讨论,更别说一个题的对错能折腾一两个小时的事儿了……

这里还是要向参加或将要参加高联、冬令营等比赛的同学和家长们说一句:我们知道你们急,但是我们更急;我们知道你们想要公平,但是我们比你们更想比赛公平。

话说回来,十年前改卷儿的,最少待遇是专家级的,现在改卷儿的,真就是搬砖的家伙。

有人不禁要问了,这么多卷子,为什么不多找些人啊。

我觉得道理很简单。

第一,真没太多适合的人。能看懂竞赛卷子,愿意做这件事,还没有直接利益相关的人太少了。

第二,同一个题不能有太多人改,不然很容易出现你改的严,我改的松的情况。理想状态是 3-4 个人一组,而且互相复核,共同讨论。

第三,人多了,各种产生的负担就更大了。

现在来改卷的大都是纯奉献,但真的不确定大家还能坚持多久。

改卷的也是人。

参赛选手多了之后,阅卷的困难是可以预计的。但是,阅卷的困难又进一步地刺激了某些改变。到底是什么改变呢?且听下回分解。(TBC)

Part8 混乱的训练

本节再次涉及大量主观观点,不喜请屏蔽。另外请勿对号入座。

在讲阅卷困难导致的改变之前,我们先从参加人数增多讲起。

参加竞赛的人数增多,另一个直接的影响就是培训教师的缺乏。

一个省几十个人正经搞竞赛,最多一两个班就行了,老师是永远不缺的。尤其像北京这种地儿,IMO 金牌想来讲课都得排队,更别说国集水平的同学了。

但是现在不一样啦,一个省几千人搞高联,怎么也要弄出三五十个班来,而且大家劲头都很足,算下来最少要近百个老师才能带的过来。

一个省能有一百个教高联能教的明白的老师吗?

很负责任的说,凡是叫 “省” 的,都没有。

小学的竞赛,读完高中成绩优良的理科生基本都能讲,真没多少东西。唯一的区别是有的老师讲的真是好,而大多数人只是能讲而已。

初中的竞赛,大学读数学系,接触过竞赛的好好练两年基本都能讲,真没多少东西。唯一的区别同样是有的老师讲的真是棒,而大多数人仅仅是能讲而已。

高中的竞赛,那可就不一样了。

小学的竞赛可能以初中的知识和思想为背景,初中的竞赛可能以高中的知识和思想为背景,这些都好掌握,掌握了这些的老师,至少在讲授时层次会高于学生,或者通俗一点讲,最少可以装装 13。

高联或者高联往上可就不好玩了。平面几何和对称不等式什么的,还能用时间堆,做上五六年最少耍起把式来能挺像样的。但是代数呢,没有分析和方程的功底,以及数感和放缩的技巧很难讲透;数论也差不多,把进位制、同余、完系和阶弄完之后,后面要么是在分析整数的结构,需要懂一些环和域的东西,要么是和组合结合的问题,运气不好冒出来一个小孩拿本解析数论来问你你可就呵呵哒了;组合更是要命,归纳反证极端原理仅仅是初级工具,既有极富初等思想的不妨设、对称、算两次,也有用对应或同构转化问题,还有母函数、线性代数等等高等的知识和技巧,更不用说组合可以和代数、几何、数论中任一领域结合,简直是无边无际。

顺带一提,个人认为代数里真正经典的是多项式,学过复变的都明白。可是多项式这玩意能代数能数论能组合,各种天坑,能讲明白的太少,学生会的也少,最后还反过来导致不敢随便考……

大部分老师,苦练数年,能讲明白一个领域已经十分了不起了。况且现在的状况,大家都是快餐,没有多少老师再愿意去像以前那样花很长时间只为弄明白一块内容,那样还不如去讲初中小学,学生更多,更赚钱。

![](data:image/svg+xml;utf8,)

一个省就零零星星几个能讲明白的老师,更悲催的是没准这老师还是清高型不愿意讲课,或者学术型天天三点一线,一般的学生和家长甚至连面都见不到。

那这几千个学生找谁去辅导?

当然是不靠谱老师了。

但是不靠谱老师怎么镇住学生?

当然有办法了。

个人觉得办法基本就那么几个:讲套路,讲难题,背答案,教骗分。

下面一个一个介绍。

讲套路,事实上是只讲套路,把竞赛难题按照讲高考容易题的方法讲。其实我不反对讲一些套路,一个完全没接触过高中竞赛的孩子也是需要学套路的,但是把套路当唯一思路就不好了。数学怎么说也是锻炼思维的,这样不是在教孩子而是在毁孩子。当然了,如果家长和学生觉得靠这么学能上个好大学,毁毁孩子也无所谓,那我无话可说。特别还有一些老的竞赛难题,在新的竞赛书上有了系统理论之后就成了套路题,学生发现自己能做很难的竞赛题,那感觉就像武林外传里的李大嘴学了降龙十巴掌之后一样。

顺带说一下,个人觉得 2015 年的冬令营在新颖性上做得很好,六个题没一个能套路。不等式一调整就坑了,组合题不能归纳,平几题基本解法动不了,上解几又困难重重,数论题又考了多项式。冬令营的成绩一下子让我们看到了现在的学生做新题时的孱弱。考完之后猜国集分数线,主试委员会的猜测都集中在 72 到 84 分之间,结果…… 不客气地说,如果这套题拿去考 2003 年前后的冬令营,国集的分数线估计会到 81 分左右(那时约 25-30 个国集),也就是说现在的高端战斗力做新题的能力也没比以前有提升。

![](data:image/svg+xml;utf8,)

讲难题,顾名思义就是类似给非顶尖的初中生讲冬令营,给非顶尖的高中生讲普特南这样的做法。数学是循序渐进的过程,违反自然规律的事可是要不得,偶尔讲一个题让孩子了解高度是可以的,但老是这样做就不好了。但是这样家长和学生可以膜拜啊,他们又找回了学习小学竞赛和初中竞赛时的感觉啊,就是这个 feel,倍儿爽!但是这样做的一个很大的问题就是学生根本动不了笔,从而会养成只听课不思考的坏习惯,以后就是见了题都眼熟但是都不会做的节奏。我自己就会说,这个孩子被教 “废” 了。

背答案,这里注意是背答案而不是看答案。看答案是一个优秀的竞赛老师必须具备的技能,毕竟你不可能像学生一样每个题都做。看答案并非只是看答案,而是要根据答案来辅助理解问题,有时能举一反三得出其它解法,即便没有其它解法也不要紧,最少能总结出解答的哪些步骤是关键,再深一点可以总结背后有什么东西,在什么动机下能想到,是不是自然的,如果学生自己做的时候遇到困难,应当怎样给予提示才能收效最佳,等等。背答案那就性质不一样了,脑子里一背,黑板上一抄,然后解释每一步为什么对,这样好像跟直接发答案没什么区别。

教骗分,上船之后必做的事。题不会做怎么办?乱写一阵,所有你知道不知道名字的定理都写上去,总有阅卷老师看不懂的,看不懂他就不敢给你判错。尤其是证明题,没准就蒙到了。证明的题,尤其是几何,条件推一堆,结论推一堆,中间差一步,写上个显然。可是孩子啊,数学是严谨的,不是靠蒙的,这可比学套路什么的严重多了。学数学之前,先要学会做人啊!如果你不知道自己做的对不对,写上去无可厚非,但是你要是知道自己是胡说八道还写,那就是人品问题了。上一节也提到了,现在的竞赛阅卷压力很大,若是放到之前,阅卷老师完全有时间陪你玩,可是现在不行。因此,很多人就钻这个空子,就赌阅卷老师不敢判错,而这又反过来加重了阅卷者的工作量。冬令营级别的阅卷人尚能保证识破这样的忽悠,但联赛级别确实偶有漏网之鱼。记得某年冬令营,某省所谓最强学生,做了 6 个题有 4 个伪证,大抵就是这个原因罢。

竞赛参赛的人数增多导致了一些不靠谱老师的产生,但这还不是最严重的。还有什么更令人头疼的事情呢?且听下回分解。(TBC)

![](data:image/svg+xml;utf8,)

Part9 脆弱的学生【我的批注:10 年在 CMO 发生了大新闻】

先讲点上一节没讲完的东西。

上一节讲到有很多不靠谱的高中竞赛老师,先说一下他们为什么不去好好教小学初中呢?个人认为还是高中竞赛老师的特殊地位。毕竟小学和初中的竞赛老师确实很多,除非教得非常好,否则没有太高的地位。另一方面,大多数家长和孩子还是拿小学竞赛当小升初的敲门砖,拿初中竞赛当进好高中的敲门砖,同时他们也意识到,大多数市里县里的好的初中高中还是可以靠钱或者关系搞定,但是搞定优秀的大学是非常困难的。另外,省会级城市的第一名学校和第五名学校的教学差距,未必比得上优秀大学里哪怕是头批 985 和二批 985 的差距。经过 9 年(甚至超过 9 年)的竞赛长跑之后,大家也都把高中的竞赛当作终点。

回到正题。

高中竞赛的参赛人数大幅度增加,不能赖政策,也不能赖不靠谱老师,初中高中的招生也仅仅是影响小学初中的竞赛。最终的问题,还是出在学生和家长身上。

近些年的高中生,大多是在 95-00 年出生的。这些孩子,从出生开始就一直被捧上天的人绝不占少数。尤其是如果还能学习号称是尖子生才能学的奥数,那简直就是飞机中的战斗机了!

所以,这些孩子全都成了布袋和尚——说不得。

可是事实上呢?

奥数只适合 5% 的孩子学习(注意只是学习!),可是光参与的孩子恐怕就要超过 20%。现在前 20% 的孩子能上啥大学?二本可都不一定(还有部分学生读不到高考的,所以 20% 到了那个时候可就是 25% 甚至 30% 了)。

天分不够,水平上不去,玻璃心倒是养出来了。

小学的时候呢,多学一点,能听懂也行,当个兴趣爱好。可是慢慢长大了,再花大量时间学竞赛,啥都听不懂,可就没有必要啦。

再说了,竞赛这个东西本来就是逐步淘汰的,学不懂就不要再花时间坚持,十年二十年前的家长都懂,为毛现在的家长就都不懂呢?

这种坚持,说好听的叫做破釜沉舟,说不好听的那就是无谓的坚持。

小学到初中是要淘汰一拨人的,初中到高中也是要淘汰一拨人的。我考华杯赛的时候华杯赛决赛的金牌,也有几个高联省一都没拿,这都很正常嘛(没准人家去搞其它竞赛了)。现在更是如此,小学竞赛全是填鸭,考好根本不代表什么,可能只是刷题多。到了高中,你以为你还能只靠刷题?

当然了,这个东西和学校以及校外的培训机构多多少少都有一些关系。学校嘛,总是希望更多的学生参与这些东西,最少述职报告上可以写写嘛;培训机构就更不用说了,多一个学生多一份钱。但是关键,还是在学生和家长身上。

即便是培训机构,只要是正规的,都时常会有考试和分班,跟不上的就学一些简单的呗,这不是很正常吗?竞赛这玩意再怎么说,每年也是要淘汰一部分人的,如果机构招生进来是 10 个班,讲了三年竞赛还是那 10 个班,一个人都没变,也没有人员交换,可以肯定这是个骗钱的机构。

可是这倒好,这种考试和分班制度,却被家长投诉说是歧视,不人道……

初中竞赛,考试分个班,60 人分两个班,结果 50 名左右的都要去快班……

高中竞赛,到高二要停课集训,结果高一的联赛考 40 多分,高二模考从没上过 100 分的孩子都要去停课……

这种事见多了,见怪不怪。

你去劝学生,学生不听。你去劝家长,家长说:“凭什么我家孩子就比别人家孩子差!”

哭笑不得。

很多家长经常挂在嘴边的一句话就是:“我家孩子学得挺好的,就是没考好。”

应试是有可能有高分低能的,但是绝不可能有低分高能的,竞赛就更不可能了。判错了我信,没考好我不信。

这些孩子还听不得批评,当然老师也懒得批评。想想也是,在学校里的时候老师肯定管不住,谁愿意冒那么大险管教你家孩子啊?万一被你打了咋办?打死了咋办?到机构里人家机构就是讲课收钱的,家长和学生就是消费的大爷,更不会惹你不高兴。

这里插一点别的东西。

就业难因为什么?

因为毕业生高估自己。就这一点没别的。

一本学校出来想一月两三万,想没想过每年毕业接近一百万的一本生,大学扩招的十几年就是近两千万人,可是全中国有多少人能到哪怕是一月一万?这个数字远比你想象的要低。

一般的大学里,你在学校是学霸,是风云人物,出了校门啥都不是。但要命的是,你还觉得你是个人物。

清北即便有一些舆论的影响,还是清北。只有在顶尖的大学里,你才能认识到和真正的神犇之间的差距。校内校外一比,才能精确的定位自己。有的事就不是你能做成的,或者做成的概率不超过彩票中头奖的概率,人要认清现实。(当然,如果你认清现实之后还有这个理想,那肯定要无条件支持)

回到正题。学竞赛的学生和他们的家长,高估的人太多了。

最后的结果大概就是这样的。

四千个水平远不够省一的孩子和两百个水平有希望冲省一的孩子(这里包括十个很牛的孩子)一起玩命训练,一起考高联。该省省一有 60 个名额。

对那四千个孩子,由于题目变化,神发挥和阅卷误差等原因,冲到省一的概率就是 1%,最后有 40 个省一。

剩下 200 个孩子瓜分剩下 20 个省一。

结果呢,那 3960 个孩子自然是撞了南墙回头,你以为那 40 个能幸免?金秋营,自招分分钟把他们刷下来。不仅如此,他们还耽误了宝贵的时间。本来裸考能上 985 的,最后可能连 211 都悬。

但是剩下那 200 个就 DT 了。尤其是最好的十个孩子,完全是大坑。没准冒出来个谁谁谁就抢了一个省队的位置。

那 4000 个孩子坚持搞竞赛,得益的是谁?

只有学校和竞赛老师,不解释。

我还是不说那句 4 个词 10 个字母的话了。

非顶尖水平的学生,其实并不适合在高中还继续花费大量时间学习竞赛,但是由于孩子的脆弱及家长的保护心理,他们还是选择了继续。那么,对于真正的顶尖学生,情况又是什么样呢?且听下回分解。(TBC)

![](data:image/svg+xml;utf8,)

Part10 倍增的压力

非顶尖的学生的继续,使得顶尖学生的晋升压力变大。但与此同时,还有更大的压力等着他们。

第一个原因当然是保送制度,以前的话就算是没进省队的省一也能不用高考了,去哪儿另说最少好听啊。另外高二弄个省一之后就可以完全弃综合了,高三考不进去也认了,不会没学上。当然,我也见过两科竞赛省一,发现保送没好学校去,放弃保送再回去高考考到清北的,这种犇这里不讨论。

第二个原因,则来源于学校和背后的培训机构。近年来重点高中为了吸引尖子,打打旗号是必须的,一个集训队都能给学校加不少的分,更别说物以稀为贵的国家队了。据我所知很多学校都打出了明码标价,什么成绩能拿多少钱(话说我咋没赶上这好时候……)。不过别以为这是有比没有好。考高联的时候的高中生最多也就十七八,有个 “赏罚分明” 很容易让他们情绪更加不稳定,容易想多。另外,指导老师也会在后面不断叮嘱不断打气,但这明明就是让学生更加紧张的做法嘛。培训机构就更别说了,对于尖子那是好吃好喝供着,但是谁也不是傻子,你要没考好看人家还理你不。

当然,前两条说到底也没多大影响,而且尤其是冲进国集之后就基本等同于没影响了。真正能影响顶尖高手在国集和 IMO 发挥的压力,在哪儿呢?

舆论。

当然是舆论。

去年 8 月,2015 年的 IMO 国家队包括领队副领队和队员上了鲁豫的节目,相信关注数学竞赛的筒子们都已经看过了。不知道大家看完之后什么感觉。

我当时看完之后,只有一种淡淡的哀伤。

上节目这种事,本来就是容易说多错多。如果只是简单背几句台词还好,现场(录播还好一点,直播更惨)如果说错一句话,那可就是一失足成千古恨啊!

上节目这种事,本来应该只找成年人的,找熊总和秋生哥做个采访就完了。两位领队都是老江湖了,年岁大,经历的又多,肯定能很好地完成任务。

但是去年的 “中国奥数输给美国” 在网上疯传,队员们还能淡定的坐着吗?

全天下都在看着他们,已经没有人能站起来保护他们了,他们必须自己站起来保护他们自己!

可是他们毕竟还是一群孩子啊。

还是一群追逐自己梦想的少年啊。

他们做错了什么?不就是没考第一吗?这和中国运动员没拿到预期的名次就得出来解释一下有什么区别?!

不,或许比这更严重。运动员多少是国家体育总局花钱培养的,是拿纳税人的钱砸出来的,优秀的运动员甚至还能大把大把地赚代言费。可是数学竞赛呢?我可以很明确的说,最多只有国家队队员出国比赛的机票和食宿是报销的(食宿还是主办方提供),其它从培训到参加国内各种比赛的开销,那可都是家长自掏腰包啊!奖金倒不是没有,但哪能和体育竞赛比?

有的人该说了,你这影响力也不如体育竞赛啊。可是正因为如此,不是更不应该责难他们吗?

现在的国家队队员,应对 IMO 这样的比赛时确实有一些弱点(这一点我会在后面详述),但是考差了可不能怪他们。

我觉得,一个国家队队员,哪怕他 IMO 考得再差,只要他一没有在之前的选拔考试中作弊,二没有在 IMO 考场上消极怠工,三没有公开发表不当言论,他本人就不应该承受任何的指责和非议。大家都是人,人家不管有没有弱点,只是训练,考试,训练,考试,最后领队告诉你你进国家队了,再训练,考试…… 人家什么也没做错。如果说没选出最强的人,那么就是我们的选拔体系还有待完善;如果说选出了最强的人,但是状态没调整好,或者弱点没有补上,那么就是我们的国家队训练体制还可以进步。不管怎么说,这可不能怪 IMO 选手吧。

IMO 选手自然承受着很大压力,不过非 IMO 选手也差不多。近几年大家都有这个习惯,说谁谁谁应该能进国家队,结果人家冬令营考砸了,集训队考砸了,就说这人咋这么弱。

这叫什么?这就叫妄议。

其实很正常啊,即便是顶尖的高手,也有可能进不了国家队。冬令营还好,集训队到最后都是毫厘之争。一个高手没进国家队,可能只是因为吃坏了肚子,可能只是因为题目不对胃口,可能只是因为评分标准恰好对他不利,也可能只是因为一个阅卷失误。是考试就有随机性,为何偏偏要忽视它?退一万步说,就算人家是真的弱,那也是人家自己的事,关你啥事?难道你还能开个赌局赌谁进国家队不成?别忘了人家就算再弱也比你强!

说到这,应该很明白了吧。

数竞选手承受了太多的舆论关注,舆论将关注转化成了压力。

但另一方面,他们并没有从中得到任何的利益。

每个人都可以如同喷明星和运动员一样喷他们,可是他们却没有得到如同明星和运动员一样的利益。

说到底,还是政府不重视,有钱人不重视,只有一帮不明真相的围观群众重视,最后造成了数竞这种 “又要马儿跑,又要马儿不吃草” 的尴尬状态。

想要政府和有钱人重视是不太现实的,更何况数竞本来就需要降降温。

所以解决的唯一途径就是,无关人士速速退散。

顶尖的高手,尤其是 IMO 选手确实不应该为他们的失利承担任何责任,毕竟这只是个游戏。但是,他们也确实在应对竞赛时有一些弱点。那么究竟是什么弱点呢?且听下回分解。(TBC)

![](data:image/svg+xml;utf8,)

Part11 高手的弱点

相比以前的国家队员,近年的队员们确实在应对竞赛时有一些弱点。前面已经提过的诸如高估自己,做新题能力差等等就不再说了,影响也不是特别大。

我在这里主要想谈一下我所看到的两个足以影响最终成绩的弱点。

在谈弱点之前,先以个人角度分析 2015 年与 2016 年 IMO 中国队的输赢之处,如有主观臆断请指出。

先来看 2015 年。平心而论,第一天的题对中国是有些不利,中国队也没有发挥到最好,但是造成这个情况的原因并不仅仅是几何。第一题组合几何与第二题数论,从数据上中国队没吃亏,甚至还有点小便宜,但是至少有 2 位同学因为花费大量时间做这两个题而失去了做第三题几何的宝贵时间,但因为几何题的数据太可怕了,一下子亏了接近 20 分,所以一下子认为几何是输掉的重点也很正常。但是实际上几何题应该也就是亏 12 分(接近 2 个人)的节奏,因为每天有 3 道题,还是要综合考虑。即便第一天考成这样,有第二天的第六题这样一个大礼包,中国队也未尝没有翻盘的可能。中国学生的数感是很好的,前面也说过,需要用代数估计和放缩的题目(即便是数论或组合),中国队都会很占便宜。尽管第六题是组合题,但是需要用代数去估计,中国队就考得很好,全场也就十几个基本做出的,中国占了四个,第六题总分全场第一。与此同时,可以看到第五题函数方程是中国的弱项,这个题 30 人全对中国竟然只占两个,而且剩余四人的分数都很低。中国队最终还是输在了第五题上。倒不是说第五题应该更多的人做出来,毕竟没做完的四位同学中有两位第六题是满分,估计花费了一些时间,但是明显应该能多写一点,争取更多的分数。比较中国和美国第五题的小分就可以看出来,最后的最后还是输在小分上(第一天的题做不出来就是做不出来,无话可说)。

厉兵秣马,来年再战,中国队加强了对几何的训练,选拔也更偏重几何。考试题一出来,没有纯几何作 3,6,可能有些筒子就会觉得 IMO 在针对中国人,其实不然。首先,TST 考什么和能选出哪个领域偏强的选手没有完全必然的联系,大家并没有太多互相针对的,中国的 TST 题目毕竟还是中国人自己放到公共网站上的,这样说未免阴谋论了;其次,今年 IMO 还有第五题这样帮助中国队的好题,这样的一个代数正是中国队的强项;第三,IMO 本来就是让人猜不出,完全定死了规矩那就没意思了;第四,其实第三题是几何题而不是数论题,中国队没有把握好而已。

![](data:image/svg+xml;utf8,)

来看一下具体情况,中国队不仅在第五题上是满分,并且由于花费时间少,顺带着第六题也做的很好,即便是三个满分的韩国队在这两题上也被中国队赚了不少。关于美国队,那真是运气太好,可以看到美国第二天三个满分都是中国小孩,这也是无可奈何的事情,即便如此,第二天中国队还是赢美国队 4 分。第一天的第二题应该是中国此次没拿到好名次的关键,有两位同学没做出,相比几乎全对的美韩,这个失误足以葬送领先优势。但是也要通盘考虑,据说有一位同学是在第一题上花了过多时间而没有足够的时间做第二题。看看第一题和第二题,当然不是某些人所臆想的初中题难度(再次强调,答案只是告诉你这个题怎么样可以做出来,不代表难度,需要仔细读答案才能发现里面的困难之处),但也没有很难。第一题想到复数绝对可以一小时以内做出来,这笔买卖很划得来;第二题只要把 6 乘 6 的棋盘为什么没办法排分析清楚很容易猜到证明的方式,它放在冬令营和集训队也只能算一个一般甚至偏容易的题目。

所以个人认为,2015 年第三题是大坑了中国队,第六题又是大帮了中国队;2016 年第二题是小坑了中国队,第五题又是小帮了中国队。两年的题目看下来,对中国队都是公平的。另外虽然韩国今年的三个满分有点猛,但是美国两年的发挥算不上极佳,最多只能算是略微偏好。事实上中国队发挥也都是一般,即便发挥的不尽如人意,也说明中国队像前些年那样巨大的领先优势是肯定没有了。

下面来说说我眼中中国队的弱点。

第一个弱点,就是四个领域不均衡。

当然,每个选手都有强项和弱项,但是当一个领域弱到会大幅度影响拿全 1245 的概率的时候,即基本上做不下来这个领域 25 难度的题目的时候,事情就不好办了。

早在第二节中就提过,基本拿全 1245 是很重要的,尤其是第五、第六名。如果国家队中相对较差的几个人不能保证拿四个题,对整体的分数影响会很大。去年和今年的 1245,中国队做得都差强人意,而且最终都是输在 25 上,本来用来赚分的 25 现在却输分了,不得不引起重视。

第二个弱点,就是不会应对不利的形势。

其实这个情况我在 2009 年就发现了,有不止一个冬令营满分在集训队第三次考试上栽跟头,几乎一分未得,后面状态越来越差,都无缘国家队。在之后的几年里,越来越多的学生在考试中有 “一脚深一脚浅” 的问题。有的同学大赛考得好,却在集训队里出现大失误,尴尬到靠评分标准才幸运地没出局;有的同学参加两次大赛,一次考得很好一次却十分差强人意;有的同学集训队题难的时候考满分,题容易的时候却几乎零分。

相比十年之前,现在的选手这种 “逆境求生” 的本事基本都荒废了。一旦遇到第一个题不顺手,很容易出现重大失误。考场即是战场,做不出 3 个题也要争取做 2 个,做不到 2 个也要争取做 1 个半,这样的信念和决心,还有应对的能力,正在慢慢从选手身上消失。从统计数据上无法看出全部,也许有的同学一直都很顺,但也拥有这样的问题。

选手们有这样的弱点,原因是什么?且听下回分解。(TBC)

Part12 不同的目标

现在的选手和以前的选手比,确实有一些弱点,产生这些弱点的第一个关键因素,就是赛制,尤其是集训队的赛制。

我们考集训队的时候,是 8 次小考 2 次大考,大考分数乘以四。所以在小考阶段,完全不需要发挥得太好(我高三的时候小考好像也就是正式队员的第三或第四),但是不能落下太多。小考有的时候题很难,甚至有时会有一两个讲解答都要讲一小时左右的题,所以要求抗压能力特别强,大家都不求有功但求无过。题容易的时候三个全做不是本事,题难的时候做一个半或两个可就厉害了。而且,小考时大家都在边考边调整状态,以便大考的时候能够爆发,毕竟大考是乘以四的。2002 年就有一位队友,在小考考得十分不理想的情况下,大考比大部分人多做了一道题,成功翻到了前六(名字就不说了,在场的都知道)。考试次数多,才更要求心理素质好,能够逆境求生。

集训队的 10 次考试,完全可以考出一个选手的所有优点和缺点。像我在 2002 年时就是几何的技巧部分做得稀烂(换句话说就是基本只会硬算),于是被要求练习几何题,到上 IMO 考场上的时候几何已经不那么烂了。

现在的集训队可不行,只有 6 次考试,而且大考的比重严重不足,前面很容易拉开差距(在 24 道题里差出 4 道题,可比在 12 道题里差出 2 道题难多了,后者可能就是一次失误的差距)导致后面无法翻盘。并且,只有 18 道题,各人的优缺点也没有特别明显地体现出来。

另外,联赛进冬令营和冬令营进集训队的扩军也是一个问题。扩军确实能保证基本保证最好的选手能进到集训队了,但是 60 人的集训队必然有大量 60 名左右水平,甚至是 100 名左右水平的选手,加上考试次数少,确实更容易出现大反转的情况。毕竟贝叶斯公式摆在那里,没选出最强的 6 个人,这也是很正常的事情。

集训队的赛制缩减,和主办方不堪重负有一定关系,和命题者的缺少也有一定关系,但我认为最大的原因还是教练组并不像十年前或二十年前那样极度渴求一个团体第一了。如同很多中国体育的优势项目一样,从无到有的时候很激动,第二次第三次蝉联的时候很有荣誉感,可是拿多了第一也没意思,总是成为别人眼中的目标也是心好累的(再说 IMO 拿了第一教练组也木有奖金)。中国的数学竞赛已经度过了艰难的开荒年代,已经有了一个相对稳定的平台。相比之下,教练组更愿意看到手下的学生们在未来有好的发展,也不愿意再多多折腾这些已经训练了 N 年的学生,可能在要求刻意练习的时候手也松了一些吧。

另一个关键因素,就是学生自身的目标和定位。

教练组不愿意加强训练强度,有的高手也不想要。第一是现在的高手们也更注重未来的发展,提前自修大学课程的不在少数。另外,现在高手普遍都是很早就进入刻意练习阶段(甚至有很多初中就停课做竞赛的),练了几年估计到最后也练烦了,大家进了集训队进了国家队之后玩上 N 久也是常态,前几年甚至还出现了一届国家队里一多半到大学不想学数学专业的情况。

其实现在的高手条件要比以前好,机会也比以前多,他们也能更加综合地去考虑和做出选择,他们应该是更加幸运的。能够在注重当下的同时也放眼未来,这是一件好事。

中国的高中数学竞赛变成这样,我不敢妄言是前进了还是后退了。但是数学竞赛本来就是培养和选拔对数学有天赋及兴趣的学生,并给他们尽量光明的未来。从这一点来讲,尽管现在联赛级别大家的压力都很大,但是从顶端来看做得还不错,至少不考虑舆论的话还是在进步的。

IMO 只是一个游戏,不要把它太当真。梦碎梦圆,这都不是终点,区区游戏,何分第一第三。

当然,目前我们的高中竞赛还是有几个问题需要解决的。

第一个问题,就是如何避免高手因为要冲出来而过早做太多的重复性训练导致的一系列问题(后继无力或者进入大学之后不喜欢数学,或者因为学习数学竞赛而过早丢失学习其它学科的机会和时间等等)。

第二个问题,则是如何缓解舆论给予高手们的压力。

第三个问题,就是如何在最后的选拔中选出真正执着于参加比赛,水平也高的学生去参加比赛,并保持不错的成绩。

这三个大坑一般的问题很难给出很好的解决方案,我也只能做到抛砖引玉,且听下回分解。(TBC)

![](data:image/svg+xml;utf8,)

Part13 急需的科普

我想到的第一个解决方案就是科普。

原因很简单,很多人不知道奥数是什么。

中国十几亿人,真正知道奥数是什么的,恐怕只能以百万计,甚至可能还到不了百万。

因为不了解,所以攻击,因为不了解,所以诋毁。

人之常情。

我举一个例子。2013 年我看到一个段子,说一个不识字的土豪解鸡兔同笼问题,先让鸡抬起一只脚,兔抬起两只脚(这时候脚的数量减半),然后再让每只动物抬起一只脚(这时脚的数量减少了等同于头的数量的值),这个时候鸡已经一屁股坐地上,兔子是单脚独立,很容易算出兔子的数量,然后再解出鸡的数量。段子的最后说,这让数学老师和奥数老师情何以堪啊。

看了这个段子,感觉怎么样?奥数老师是不是特 S13?这可是连不识字土豪都会的做法。

真相是什么呢?

我 1992 年在华罗庚数学学校学奥数的时候,老师就是这么教的。

另外拜托仔细想想,不识字土豪真会这个?

对了,这个段子就是来黑奥数的。

但是可惜的是,我看了这个段子三四次,仅有一次有一个人的回复说鸡兔同笼就是这么学的。

另外,跑男第二季第二集包贝尔也用了同样的方法,弹幕里也无任何人提到此事。

这个方法的道理是什么呢?不难看出,这就是解二元一次方程组的做法,只不过不用设未知数。

奥数是很有道理的,永远是低段工具解高段问题。

当然,如果你的奥数老师讲鸡兔同笼上来就列方程组,请让你的家长抽他两嘴巴子,然后申请退钱,因为这是一个不讲思维只念答案的庸师。

其实我觉得奥数真的缺很多科普的书,我们的出版社应该多重印一些好的科普书。我读小学初中的时候手头有好几本科普的小品文,多到我自己都从书里建立好了一套初等数学的体系。但是现在学竞赛的学生,哪还有什么科普书啊。

不仅是奥数需要科普,数学本身也需要科普。前段时间看到一个所谓 “小学奥数五星题”,如下。

![](data:image/svg+xml;utf8,)

这个题怎么做呢?其实不要纠结于只用小学内容,你完全可以先算出面积再说。

不难看出,中间的阴影部分和右下角的阴影部分都是好算的,难点在于右上那一块。

建个坐标系好了,左下角是原点,联立直线和圆的方程不难算出对角线和右边圆的靠右上的交点坐标为 (18,9)。然后连圆心,算扇形面积,发现面积里有个反三角函数。

这个时候基本就可以肯定了,这个题肯定不是小学题,小学题再怎么样,答案里绝对不会有反三角函数!

这个题原题啥样?看出来也很容易,其实就是多了左下角一块小的黑色,这样用对称就得到阴影面积等于矩形面积减去两个圆面积之后剩余部分的一半。

结果呢?这个有技巧的好题被刻意地改动,完了之后又成了黑奥数的东西。

其实,只要掌握了高中知识,很容易识破这样的伎俩。

其它的很多问题也是类似的,其实掌握好高中数学知识和高等数学知识的人,绝对不会上当。相反,很多初中小学的奥数教师,如果高等数学没学好或者没学过,是很容易被一骗一个准的。

所以,我这里还是建议所有奥数教师都先学好高等数学再来教奥数,也避免总是问一些秀智商下限的问题。

除了知识,一些理论也是必须要科普的。

初等数学中有很多解决不了的问题。

这个事中国有多少人知道?

可能百分之五都不到。

换一个具体一点的吧。五次或者五次以上的方程是没有通解的。

这个事估计知道的多一点,但应该也不会超过百分之十。

所以咯,奥数黑完全可以随手抛出若干五次六次方程,说奥数就考这个。

不科普,永远不能洗白,你解释了一个还有另一个。

还有,不定方程也是。别忘了费马大定理就是初等不定方程,孪生素数猜想也可以算进去。一个不定方程不能解太正常了。例如,n 允许含有素因子 2,3,5,n+1 允许含有素因子 7,11,13,求 n。这是个骗人的题,即便能做也是狂讨论,没有任何实际意义,但是一个在中小学没有接触过奥数的数学系本科生比一个大学没学过数学的初中小学奥数教师更容易发现事情的真相。

![](data:image/svg+xml;utf8,)

不止圈外要科普,圈内也要科普。有关难题的问题就是一例。

应该有很多人认为出难题就很厉害,也有很多奥数教师以难倒别人为自豪,然后吹嘘。

但是实际上,出难题不是难事。

我自己出过 6 个超级难题,个个堪比 ELMO Shortlist 压轴题,如果哪个高中生能做上其中的一半,绝对是国家队水平之上。而且,每个题我绝对可以在 30 分钟内解释清楚。

可是我基本不公开拿出来,也从不拿来炫耀,因为没有必要。只有当国家队级别的选手做所有题都秒杀,然后又向我要新题的时候,我才会拿出来一个两个让他们去自虐去。

顺带一说,2009 年选拔的时候我拿出来过一个,被其他几个老师否了,因为他们判断那个题只能考考韦教主,其他人肯定全跪,没有任何选拔意义。老师们的原话是,我们又不是选一个人,我们是选六个人。

题不是出难了才有意义,而是难度恰到好处才有意义。从训练的角度来看,是让学生能够得着,开发思维才有意义;从选拔的角度看,要能区分学生,选出好学生来才有意义。

另一个例子是 2015 年的女子赛第四题,这个题其实是一个广泛问题的简单情况,详述这个问题背景的文章发表在中等数学 2016 年第一期。个人觉得原题放到国家队选拔,都不会有什么人做出来。即便是那篇文章,能看懂的估计也寥寥无几。

只有给大众科普了什么是数学,什么是奥数,才不会有那么多的奥数黑存活的空间,奥数选手的压力才会减轻一些。另一方面,弄清奥数是什么之后,会有很多水平不足的学生和家长退出,间接地给真正的高手更多的位置。

科普是必须的,然而还有什么好方法呢?且听下回分解。(TBC)

- 持续更新 -,推荐一个书单收藏:

复旦大学图书馆

初三和高一学生可以刷这套书:

数学奥林匹克小丛书高中卷

安利一下中科大少年班和创新班 [有天赋的小学生和中学生不要错过];MIT 网课:

麻省理工学院公开课:单变量微积分

麻省理工学院公开课:多变量微积分

麻省理工公开课:线性代数

特大安利一下男神的本科笔记:

Evan Chen • Course Notes

他是我最崇拜的人之一

知乎用户 王天元​​ 发表

如果你是笨蛋,那么你不可能变成天才。

如果你是天才,那么你很容易变成美国人。

知乎用户 Cat Chen​ 发表

你要问数学,那我就用数学来解释。大部分人拿着两个复杂数据集进行对比时,都只能抓住其中一个标量进行对比,无法对分布或向量进行对比。例如说,对比中美两国学生的数学能力,那就只能抓住两个均值进行对比,而无视背后众多学生个体产生的数值分布,也不考虑数学能力到底是个标量还是个高纬度向量。

就算我们把数学能力看作标量,这个问题所问的现象也很容易解释得通:美国学生的数学能力是一个低均值高峰值的分布,那些最厉害的发明创作都是由高峰值那一小部分人贡献的。至于低均值,我来讲一个美国统计学界冷笑话:有一个统计学家,在一条平均深度只有 30 厘米的小溪里溺水身亡了。(呵呵…… 真的很冷,也不知道是谁创作的笑话。)

知乎用户 活泼的喵哥​ 发表

平均水平不能体现尖端水平。

美国人在中小学学校里待的时间很短,每天两三点就放学了。课程也很简单。他们有大把的时间发展自己的兴趣。所以如果一个人对数学有兴趣,而且家里条件不错的话,很容易在高中就学习很多高级的数学。我之前在 Berkeley Math Circle 讲课,遇到一个九岁的小朋友,数学思维非常清楚,对新的概念很快就能消化。这些同学到了大学,很快就可以开始科研活动了。

同样的情况也出现在印度。我博士导师是印度人,他跟我说,他一直到大学的数学都很简单,但是一到研究生院,发现同学个个都懂代数几何,相交理论。有条件的家庭,小孩根本不用依靠学校的教育,他们想学数学有大把的资源。

知乎用户 想飞的猪 发表

丘成桐 2012 中国青年报访谈

中国最好的学生与美国最好的学生相比,在学科准备上有一段差距

记者:这些年您一直在做大学生数学竞赛,能否介绍下具体情况?

丘成桐:我们这几年搞了一个 “丘成桐大学生数学竞赛”,第一年考的时候,很多学生水平不行,第二年改进了很多。我们不考刁难的题目,基本上是美国博士资格考试的水平。让我吓一跳的是,有些名校一个学生都没考上。因为这些名校吃老本,考试一下就露出底了。

我们有 50 多位教授参加组织和出题,组织这场竞赛完全是义务劳动,竞赛到目前为止办了两年,虽然得到政府认可,却没有要求政府拨一分钱,全部都是我们自己找的经费。我觉得竞赛的效果很好,因为很多高校知道要调整自己的教学内容了,一考试就知道学生水平比不上人家嘛。

记者:在我们的印象中,中国学生的数学水平是最好的,怎么会比不上人家?

丘成桐:在哈佛大学,某一年有 9 个来自不同国家的学生参加我们数学专业的博士资格考试,满分 160 分,有 7 个学生考分在 130 至 140 分之间,唯独两个中国学生只考了 80 多分。这还是中国最好的学生。

中国学生在数学上的准备比不上人家。清华有个学生跟着我,刚来的时候,觉得博士生资格考试很困难,经过努力,现在成绩不错了。这表示其实中国学生的基本功并没有那么好。举例来说,中国高中不怎么教微积分,为什么不教?因为高考考得少或者不考。然而,微积分是文艺复兴和科技革命以来最伟大的创造,牛顿靠微积分成就了牛顿力学,大部分科学上的成就也都需用到微积分。

经常能听到某些媒体说,美国很多人连加减乘除都不会。美国的高中生可能会有一部分学生的加减乘除没学好,但总不能专找美国最差的学生和中国学生比较,干吗不找他们优秀的学生来进行比较?

所以,之前有报道说中国学生出去留学数学是最牛的,这是片面的。中国最好的学生与美国最好的学生相比,在学科准备上有一段差距。

记者:基础培养特别重要。

丘成桐:我在国内见到不少应用数学家有这样的毛病:基本功夫不够坚实,却大谈交叉学科的重要性。这样做反而把本来应当发展的基础学科也耽误了,正所谓 “画虎不成反类犬”。本世纪的知识突飞猛进,跨学科的知识更是如此。事实上,大部分创新的科学都是在不同学科的融合中擦出火花产生的。

很多人都同意这个看法,但却忘记了一个重要的事情,就是有能力融合不同学科的学者,其能力和知识水平都要跟这些不同学科的专家相当,即使在某方面的知识跟不上,他也要能理解问题的困难所在,能找合适的专家求教。而能满足这些条件的科学家实在不多。

中国有不少学者只注重科学的应用,而不愿意在基础科学上下功夫,这是非常肤浅的。事实上,从工业革命以来科技的每次突破无不源自基础科学的发展。对基础科学认识不够深入,只满足于应用而沾沾自喜,终究是尾随人后、依样画葫芦罢了。年轻人做学问,务必要踏实,将基础学科学好。

中国现在的教育就像从大锅里盛出一点饭,比较平均地分给所有人

记者:如何评价今天中国的学术水平?

丘成桐:大范围来说,科学上第一流的成果不多。第一流就是其他人做不到的,基本思想从头到尾都是我们做的。最近我去看国防科技大学做的 “千万亿次高效能计算机系统”,看了西昌卫星中心的火箭发射,都做得非常杰出,但无论是计算机还是火箭,基本思想都是外国人的贡献,我们还缺少全面性的创新工程。

美国很多第一流的创新成果是别人没有做过的,他们从头开始做,如半导体和生物科技里面的很多科研成果。与之相比,中国要差得远。结晶牛胰岛素是一项很伟大的事业,但这已经是 50 年前的事了,这几十年,中国还没有像结晶牛胰岛素这样的工程。

记者:怎么看待近年来一流成果的稀缺?

丘成桐:教育没有做到位,尤其是从广泛学生里培养出一部分好学生的力度不够。

大范围来讲,虽然我们现在的教育在 “量” 上是不错,每年差不多有 1000 万高中毕业生,高考也比较公平。但是我们并没有意识到要从这里面培养出一小群适合做领导的、做创新性工作的人来。

要做前人没有做过的工作,就非要培养出一批出色的年轻学者。有些科学在刚开始的时候,可能就是几个人、几十个人做出来的,比如半导体,但它影响的可不只几个人、几十个人,而是大多数人,乃至全世界,给国家带来了巨大的财富。

在中国的大学生或者高中生里面,如果有千分之一甚至是万分之一的人能够这样,或者说,只要有 1000 个杰出学生去从事这些事业,就能对整个中国的创新带来不可估量的促进作用。中国现在的教育就像从大锅里盛出一点饭,比较平均地分给所有人。我觉得,应该有体系地培养有创新能力的年轻人,让他们投入到伟大的科技事业中去。

记者:如何培养这样的年轻人,您的建议是什么?

丘成桐:曾经有很长一段时间,很多大牌教授要做研究,不愿意教书,这造成学生遇不到好的教授。哈佛大学数学系的教授全部被要求教大本科,无一例外。最近几年国内教育部门也开始要求大教授教本科,但是还不够。

国内的好文章数量太少了

记者:前段时间,有研究机构发布我国国际论文的数量指标,数据显示我们在全球上升的速度很快,但引用率不高。有观点认为,我们论文把量做足了,质量就会慢慢上去?

丘成桐:这个论点显然是不科学的。现在看来,中国的博士全世界最多,哈佛大学去年毕业 300 个博士,对比这 300 个博士的论文水准,也许中国很多高校的博士论文加起来都不见得比这 300 个博士的论文好。

创造一定要靠 “质” 而不是靠“量”。也许好的杂志会登你的文章,但不代表你的文章就是杂志中最好的。真正的好文章会影响很久的,文章好不好,要等过了十年八年才会显现出来。不要看发表时的状况,要看五年十年后的引用率,且排除自己引用的部分。这样来看,国内的好文章数量太少了。所以我们只顾着把量做足很难有质的提高。

我觉得,只有求真求美,一心一意追求大自然的真理,摒弃形式主义的东西,才可能在做学问上有所提高。

记者:怎么理解求真求美?

丘成桐:我认识一个院士带了个博士后,文章没发表前,发现逻辑错了,这个院士说没事,因为没有人敢说他错了,他不在乎。我看到身边有很多这样的事情,有些媒体宣传的大教授,明明知道某个问题错了,但就是不愿意纠正,真正变成学霸了。

真正想做学问的人,不要只想着做院士、当领导

记者:您描绘了一个很好的景致,但是就像站在湖边,远看时这湖景挺美的,可是一走到近处,才发现这湖水浑浊不堪,大煞风景,怎么面对这种状况?

丘成桐:学风一定要改正。现在中国学术界有造假的风气,真假不分的话学问怎么做啊?学问在真知面前,在所有力量面前都是不变的。伽利略在教皇面前说:“就算你惩罚我地球还是在转动的,地球还是在围绕太阳转动的,你无论怎么惩罚我,还是有那么一个真理存在的。” 可惜现在一些学者不能讲真话不敢讲真话,名人、权威或领导讲了一句话,反而成了 “真理”。

真正想做学问的人,不要只想着做院士、当领导,为了学问而做学问,终究会成功。我常跟年轻的朋友们强调,学问做好了,肯定有出头的时候。遗憾的是很多年轻人不愿努力几年以后才得到赏识,他们只愿意看到眼前的利益。

记者:您有什么建议?

丘成桐:其实只要领导不要急功近利,真正想做一些好的研究,就会有一群人会为做学问而做学问的。

现在做学问的人都是为了某种目的,比如说为了自己的名利,甚至为了学校和国家的声誉,这样做学问不大可能做得特别好。年青人做学问,还是需要有一些是出于对大自然的好奇。

爱因斯坦做相对论的工作是第一次世界大战的时候,他的研究对枪炮和火车都没好处。量子力学做出来的时候,在当时什么用都没有,只是个哲学的观点。到后来我们才认识到它们都是最重要的工作。

因好奇而沉浸到一个地步才能做好学问,中国应当创造条件让一批人为做学问而做学问,应该鼓励这种人。

记者:您在很多演讲中,都说把中国科技发展希望寄托在青年身上,但很多人认为今天中国的年轻人很浮躁,您怎么看?

丘成桐:学生浮躁的主要原因是学校环境不好,社会舆论也没有引导好。无论是学校的教师,还是政府官员,甚至包括媒体,都营造了一种不好的风气。在学风和社会风气的影响下,一些聪明能干的大学生只想着赚钱,不想继续在学问上下功夫。有能力的学生大多数都奔着学金融去了,没有人想留下来做一些基础工作。

国家这么大,如果年轻学生都只想着捞一笔是绝对不行的,还是需要有好好做学问的学生。

美国之所以强大,是因为美国年轻人挣钱归挣钱,但总还有一批年轻人愿意全心全力去做学术,不去想挣钱的事。年轻人学问做好了,总会有出头的时候,做好了学问也不是不能名利双收,只是看青年朋友们耐不耐得住这种寂寞,有没有挑战的精神。

总的来说,我有信心看到大批年轻人带领我们国家科技取得大进步,但国家需要保护他们,让他们发挥他们的长处。

未来 10 年是中国科技发展的最好时机

记者:这些年您一直在中美之间走动,您感受到中国科技进步表现在哪些方面?

丘成桐:中国科技这几年起飞了,是不是真的能飞起来是另外一个问题。近两三年形势大好,也可以说是千载难逢的好机会,中国科技能不能够脱胎换骨,就看这五到十年,因为这期间海外经济遇到大困难,欧美都遇到困难,真正有学问的学者把中国看得很重要。

中国的教育科技环境也有所改善,现在的年轻人愿意花时间读书了,以前觉得只要出国就好,现在也知道在中国也可以做学问了,这是很重要的改变。

但现在中国基本都请华裔的学者,华裔学者跟全世界最好的学者比不见得是最好的。我们有能力请非华裔学者到中国来,我们清华的数学中心成立了才两年,有很多一流的非华裔学者全职来参加了。这些都不是普通的学者,一个密歇根大学的讲座教授全职来中国;堪培拉大学的教授计划每年来九个月,除了教书外,也带研究生;一个荷兰的院士也准备来。还有一大批很好的年轻学者都要来参与。

所以,未来 10 年是中国科技发展的最好时机,中国一定要把握这千载难逢的机会,错过了,下一次再遇到这样的机会,还不知道要等多少年。

当年,陈省身先生和华罗庚先生在极为艰苦的情况下,可以做出世界一流的工作,都是他们年青时候完成的。我相信在政府正确的领导下,我们有能力培养一批年青的学者,做出世界一流的工作。

注:丘成桐, 美国哈佛大学数学与物理系教授, 菲尔兹奖、沃尔夫数学奖得主, 曾证明世界数学难题卡拉比猜想。

[

我的世界观 爱因斯坦 著

京东

¥43.50

去购买​

](https://union-click.jd.com/jdc?e=jdext-1268706278400503808-0&p=AyIGZRprFQEXA1ITUhYyVlgNRQQlW1dCFFlQCxxKQgFHRE5XDVULR0UVARcDUhNSFh1LQglGa2ocFQZXEwxgYVJfDnIhVHd0WTNQEkMOHjdVElsXChMGVRxYJQITBlUfXhYBFAZlK1sQMkRpVRpaFAMTAlYZXyUDIgdRE18WBRAHUBJeEgUiB10fa8OshtLdpIKlp8St%2B87WpdebrIGl3SUyIgRlQA5BS1cbVRhcHAQTAFYdUhADEwNUGVIUCw4GZRhaFQMbAFcZUh1sE2lVE1ISAxMHVB9dJQIiBGVFNRRREgcHGQ4WbEhVEVsCSgZSaVUbXxIFGgRdK1kUAxAF)

知乎用户 何足道 发表

最近两学期我在做 Calculus 的 TA,这门课面对非数学专业(主要是工程和经济),其难度大概相当于国内中等级别的高数,并且内容相差不大。我发现,班上中国学生成绩大多名列前茅,这部分归因于他们中学较好的基础。美国学生的基础并不好,他们不太喜欢背东西,例如至少有一半的同学不能很快写出二次方程的求根公式。另外,相比于证明题而言,美国学生更喜欢计算题,即那种老师教一种算法,然后照着做就行的一类问题。

再说数学专业,在许多顶级学校确实有相当比例的天才是美国人。我的观点是,美国的数学教育并没有培养天才,而是保护天才。其实不只美国教育,世界上任何教育都不是为教育出天才而设计的。区别在于有的教育环境差,许多有天赋的人被扼杀或者被引入歧途,有的教育坏境好,有天赋的人能够有机会充分发挥自己的潜能。

知乎用户 刘翔熙 发表

上面很多中美教育系统的讨论已经很好了,我就想说一句啊,今天的美国中小学生数学比中国简单,不等于 1960-1970 年的美国中小学数学比中国简单,甚至我可以肯定地说,那时候美国中小学数学比中国难多了。 (中国那时候在上山下乡和文革,而美国因为冷战高潮正在拼命搞阿波罗登月计划 1961 年。。。)

科研这东西有正经的滞后性,今天美国做出超级牛 X 东西的,不是接受今天中小学教育的人,而是 1960 年出生那批,看着父母叔叔阿姨登月(阿波罗计划 1961-1972),由二战历练出的精英领导,并被冷战逼的拼命选拔最牛 X 理工科人才,然后送给二战那批来美国的(逃来的,请来的,俘虏来的)逆天科学家用不差钱(真不差钱,在美国和教授关系好的可以问问当年他们读本科生,研究生的钱能过什么生活,那里像今天,一群民工)方式打造的人们。

这里多说一句不差钱是什么程度哈,我老婆博士导师是很有名的一个环境工程教授,因为做的东西非常权威,于是每年州政府要给 N 百万 funding 保证他的研究成果能上庭解决纠纷用。 可是他们实验室一共就 6 个人。。。根本花不完(做数学模型的),于是他一个人撑起了半个系研究生的钱,其他老师没钱又需要学生,那好,每个月去替我们实验室去户外指定地点采样一次(一般是荒山野岭,一去一天,三九三伏下冰雹也都得去不过只有每月一天),你学费我包了。。。 怕大家采样回来做实验累到,专门雇了个为大家刷试管和清理试验仪器的。。。你只管采样和实验就好。

然后他给他实验室的博后开了 9 万美元 / 年的年薪(博士生工资学校定标准,博后工资是老板定的),羡慕死那群刚入职的 AP。。。就这样,那个博后还提出了要求:我想什么时候来就来,想什么时候走就走。 导师就同意了。 然后这哥们一年休 5 个月去飘流。。。但是活一点不丢。 最后顺利成了教授,接管了整个实验室(导师要退休)。

至于我家领导,我媳妇每天下午四点不到就有实验室的同事提醒,该回家给老公做饭了。。。 不说她每天早晨 10 点才到实验室(睡懒觉),但是这样不等于说学业要求不严格。由于我老婆是小转方向(硕士光催化偏化学,博士数学建模),导师硬让她上了足足 3 年课补足理论基础。 (根本不要求出活出文章,平时就在实验室里打下手),而且都是要求最难的课,学的她死去活来的,然后最后帮忙联系跨行业的博士共同研究题目。 她要做实验,需要 25 美元一个的 sensor, 于是找导师求购买两个,导师隔天告诉她,订好了,怕额外耗损,于是直接订了一打。。。我老婆直接跪了。。。此外,他一生学生 (博士)数只有那么几十个,因为他从带博士开始,一次只带一个人,等这个人毕业了,才换下一个。 我老婆经常说,真正工作了才知道导师的要求是多么有道理。 这是典型的 1960-70 年代风格的教授,和现在比如何?

而美国公立学校中小学数学全面放弃治疗外加研究生民工化,恰恰在冷战结束以后。。。可以说,现在美国这批中小学生放到 1960-70 年接受那时候的美国基础教育,估计都得哭死(一些我接触过的教授也持有这种看法), 要知道,中国今天的系统化大纲化教育是从苏联学的,而苏联当年是从英国学的。 美国很长时间内(包括现在的很多私立学校)也使用的这种计划式教育。 (有兴趣的可以看看私立学校 Deerfield Academcy 的教学计划和作业量) 而今天的英美公立学校在做毛线? 培养垃圾么?

至于现在接受教育的这批人是不是还能同样造牛 X 的东西? 我们 2057 年再看看吧。。。

-—————– 回复一些回答的分割线 ——————

Q: 美国自己可以靠吸纳世界人才来维持自己的强势。

A:先不说现在民主党是把难民看的比人才重(留学生平均硕士 2 年 + 博士 6 年 + OPT2 年 + H1b6 年 + 绿卡 N 年 + 入籍程序 1 年 = 17 年才能有国籍,难民来了不久就可以绿卡,然后 2 年公民)共和党不少人根本是反移民(难民不要,人才也不要,相比之下,川普看着还正常点)。就是吸纳人才也等于是输血,而且在整个社会不重视理工科的情况下,吸纳的人才也无法培养好的中层工程师和底层技术工人(在美国做公立大学理工科 TA 的可以尽情吐槽,私立大学的那些人根本不会去做这些位置) ,相当于失去了造血能力。 这对一个国家是很可怕的。

况且一些位置根本不能给外国人才。

正如 Red 同学提到: 实际上(美国)现在不仅需要(本土人才),而且在一些专业上缺口很大。有的是像 Navair, Darpa 这样只招本土 PhD 的地方。我身边最极端的一个例子,没正经发过文章 + 有撤稿记录 + 导师拒发 RA/TA 的美国女生照样拿 Nasa 的 Fellowship 读 PhD。

这是因为涉及可能泄密的情况下,谁都担不起责任,美国已经不是那个欧洲科学家拿来就能用的美国了。 不用说军工 我们所有联邦政府部门工作人员 哪怕是扫地的都得是公民 绿卡都不要 这是基础 往上走归化公民都不要 必须出生国籍。 这样也不够,关键技术领导岗位基本要美国出生的美国人。 军工更是各种排查,包括很多政府项目承包商都得明写,本项目涉及政府机密,只招公民(绿卡都不要)并需要过不同等级的背景审查(有三代内亲属在国外基本过不了)

有兴趣的朋友可以读读我另一篇回答: https://www.zhihu.com/question/62023084/answer/194584073

我再重复一遍我认为的科技领先要素 – 个人认为,保持武器的领先地位需要:

1。 有足够牛 B 的领头人(中国现在缺的就是这个,军工科技这行一个牛领头的关键性有多重要搞过都知道)

2。 有足够大的中层工程师队伍,进行项目进行中的质保,检测,回馈,改装尝试等

3。 有足够质量的,庞大的技工队伍,即使现代开始大规模机器人生产,你也得有能造机器人的人。

4。 有足够多元的科研团队进行不定向的储备。

5。 有足够的资金保证 1-4 的一切。

而美国现在投入科研的钱相较于消费水平已经很低了。 能吸引国际人才无非是靠惯性延续(事实上,欧洲教授除了东欧已经很少移民来美国了),而且吸引的国际人才实际的结果是废了这些人才,使之不能为其他国家所用 – 由于不能涉密,这些人才很多是去做些无关紧要但是能过中产的生活的工作罢了。反正这个水平的人做啥都容易年薪 10 万吧,如果接触不到核心科技,跟不上节奏,过几年就废了。

所以再过 20-30 年,随着中印等国发展,和学者减少,美国科技优势可能会出现垮塌式的衰落 – 主要是后继无人。

顺便说一句,这是 2017 年的美国 IMO(数学奥林匹克)国家队构成:

Ankan Bhattacharya (印度) Zachary Chroman (俄裔)Andrew Gu (华人)Vincent Huang (华人)James Lin (华人) 和 Junyao Peng(华人)构成,六个人队伍四个华人一个印度一个白人。。。

2019 年美国 IMO 国家队构成: 

Vincent Huang(17-19 年三朝元老),Luke Robitaille (法裔), Colin Tang (华人) ,Edward Wan (华人),Brandon Wang (华人), Daniel Zhu (华人).

这是美国队还是中国二队啊。。。

知乎用户 立党 发表

举个简单的例子。

我妹妹所就读的中学,是美国某 9 分学区,也是该州 top 10 的中学。

公立中学当然是免费的,但钱不是从天上掉下来的。他家的房子价值 60 万美元,每年要交 3% 的房产税,折合 1.8 万美元,而这笔钱大部分都是要交给公立学校的。也就是说,如果他们在这栋房子里住一辈子,就要交一辈子的每年 1 万美元给高中、初中和小学当学费。而这个学区大概有几百户家庭,所以高中其实是非常有钱的。

结果就是,这个 9 分的高中,可以用 6 万美元的工资来招聘老师。这在这个农村里虽然不算高,但已经是非常不错又闲适的工作了(毕竟有假期,每天下午 3 点下班)。

这些老师呢,有的来自某州立的计算机系,所以他们就教 PyTorch 啦、ROS 啦、JavaScript、树莓派、Python 给他们学,于是有的人高中就早早地搭起了神经网络,或者更早地就用树莓派 + 各种传感器做一堆小东西出来;

有的毕业于很厉害的数学系物理系,于是就带他们上微积分、统计、物理、算法,考 AP;

还有的老师毕业于被劝退的但很强的生物专业,于是他们甚至有机会去周围顶级的生物和医学中心去做 research,要推荐信。

结论:中产区、富人区的高中资源比很多国内大学资源还要好,于是高中生们用高中三年的时间,相当于读了一个精简版的大学,而且有能力的人可以直接跳到研究生阶段的科研经历。

而为什么会这样呢?归根到底还是因为他们老师工资高,能招来很厉害的人。

知乎用户 三山 发表

你去买一本美国的数学教材,你会发现数学是那么好玩的东西,他们真的想教懂我数学,从数学的诞生开始讲,讲人们如何发现数学规律,如何用和我一样朴素的思路去用数学解决实际问题,又是在什么契机下发现了一些惊人的 bug,从而把数学带向新的篇章,我们在做的题目是为什么,是什么时候发明的,前因后果是什么,有什么用,可以导向什么。

你从小在国内读书,有没有问过 “学数学有什么用” 或者有没有听别人问过?

美国会在孩子小的时候教他们数学有什么用,会带着孩子去走前人走过的路,去模仿,去造轮子,去复刻一次次重大发现,带他们走完数学的历史。

等到了高等数学阶段,一切都是准备好的。他们知道手中的工具意味着什么,知道如何用数学语言去创作。

而我们呢?背乘法表,背公式,背定义,甚至公式定义的教学结构都存在严重的问题,比如初高中断档,更严重的是高等数学和初等数学的断档,数学在学校里不再是一件自己拥有意义的事情,而是智商的度量尺,即使是成功证明了自己智商的人,也大多在最后的创造力环节被淘汰。


有些人,不用把节奏带到什么公知啊,快乐教育啊,教育分层上,淘宝拼多多京东哪家拦着你买书了?但凡在一个领域做了很久的人都会发现,死记硬背能让你快速赶上视野范围内现有的成就,但知其然知其所以然才能让你提出自己的问题,做出自己的发现,也就是所谓的创新。没有创新,谈何成就,万年老二也有做到头的时候,有一天别人都不行了,没得抄了,或者联合起来不让你抄了,抄什么去,现在为什么内卷,为什么不能突破,为什么连每年区区几百万本科生都消化不了,还搁这自嗨呢?


你跟他讲追本溯源,学得完整通透,他通篇只能看到快乐二字,且以此大做文章。他根本不能想象学习原本就是件快乐的事,一心只想他吃过了苦必不能叫别人好受,他学不会的东西必不是方法有错。他渴望摸到捷径,却放不下他廉价的自尊心,当别人告诉他捷径,他又红起了脖子忘记初衷只想着争个高低。

更不用说他嘲笑的美国公立 “放养教育” 对标的底层 “穷人” 家长们其实开着凯美瑞和索纳塔,校园窗明几净硬件齐全支持孩子的各种兴趣。而我们的底层穷人尚在等待社会捐款去盖一间只求遮风避雨的校舍,他们能请得起的老师甚至讲不好一口普通话。他不会做这样的横向对比,他只会拿北上广、长三角、一二三线的公立小学去对比,因为他只见过这些。

他心里没有家国,也弄不好自己的事情。知识全在互联网上,没有对任何人隐瞒。他因为懒得搜索不想去找资源,因为不知学习的用处而只想应付痛苦的考试,因为听过捕风捉影而觉得懂得了举一反三。


看到这么多人把 “数学好” 概括为心算快、考得高,我想这个问题都不言自明了。

知乎用户 匿名用户 发表

我是一个在国内升学高中完成高中学业,但是因为高考失利没能考上北京两校而被迫选择来美国念书的人。我在国内主攻物理竞赛,平时也参加数学竞赛辅导,权当做课外补习。虽然我们高中不是明星高中,但是往年高考成绩还是不错的,一般有十几个能考上北京两校。

以上是我的个人背景。

==================

来美之后,我和很多中国学生一样选择念数学系。我们学校数学系庞大,主要因为应用方向很多:应用数学,计算数学,数学 / 经济,数学 / 精算 等等。许多人念数学系是出于找金融工作 + 好刷 gpa 的考虑,因为大家都认为美国人数学水平很菜,一般课上拿 A 没问题。甚至像我大一的时候还有点小骄傲,认为自己有竞赛背景,对付这里学生就跟砍瓜切菜一样,毕业时拿个 highest honor 不在话下,然后 top grad school 随便申申。。。

现在想起来,这真心算我的黑历史了。

其实,如果我不上 Honor 课,不现在这样大三开始修 grad 课,日子还是应该会很好过的,起码每天能保证 8 小时睡眠,周末抽空能出去玩一玩,不像现在这样,焦虑得有时甚至会大把大把地掉头发,被一些压力巨大的 grad 课活活逼哭,为了能 survive,有时早上 6 点起来看书做题,一边做早饭一边想思路,然后去学校上课,去 office hour,去做 research,晚上在图书馆一直待到 1 点才回家。。。我从大二上一些高级课开始,认识到自己在国内打下的基础实在非常差。国内高中的学习经历的确帮助我吃了一年老本,但是到开始挑战独立思考能力的高级课时完全变成了负担。我甚至非常悔恨,因为这里认识的热爱数学的美国人,很多都在高中时学习了线性代数,代数,分析等等大学的课程,他们对于这些分支了解的宽度与深度都大大超乎我的想象,这也给他们学更深入的东西打下坚实基础。同样在高中时,我除了会刷题,对这些了解完全为 0。为了赶上这帮人,我就只好开始了一年到头累死累活苦学的日子,可以说,这简直比我高三还要累上一倍,而且我就算卖出了 12 分的力气,目前前途还是非常茫然,很担心下年申请会全聚德。

为啥?

======== 接下来我想谈谈这里本科数学生的评价标准 + grad school 的录取 pool 水平 ====

如果只是高中就学了一些大学的数学课程,这些美国人的水平在这里最多只能评价为 “可以学生”。因为向上还有“优秀学生” 和“明星学生”。

“优秀学生”:一般出自 top 10 学校 + CMU(这几年 CMU 本科生水平突飞猛进,作为衡量标准之一的 PUTNAM 竞赛,他们已经连续 N 年进 top5, 而且今年 top500 学生总数他家位列第二,仅次 MIT,只能说他家请来美国队总教练 po-shen loh 来当辅导老师实在太正确了。还有大把钱吸引 USAMO,IMO 的拿牌学生入学),优秀学生 GPA 一般都是 4.0, 而且是上了一众 Honor 课 + grad 课而保持的 4.0。高中时参加过 Intel,USAMO, 甚至 IMO 有过好表现。本科时参加两个以上的 summer REU, 其中有一个是高水平录取竞争非常激烈的 REU:williams college SMALL program; UMD Duluth REU; UMinnesota, combinatorics REU(这些去的 IMO 金牌选手非常多,我也不知道为什么后两者这么 prestigious,因为学校本身很一般,总之是迷)。平均 2-3 篇 publication, 其中一篇杂志发表的水平大致是 journal of number theory 这类分支里的 top journal。在校期间拿了不少奖,putnam 进入 top150 左右,毕业时可以成功申请到 churchill 之类去 oxbridge 的奖学金(这类奖学金每年全国颁总共 30 个人左右包含各个学科,所以拿到也是非常难的,而且多半被 H,Y 垄断)。

“明星学生”:在高中时就是明星了,比方说 Evan O’Dorney, 年少成名,有自己的 wiki page。一般都是 Intel finalists 前三,IMO 金牌,大学的 PUTNAM fellow。这种人一般只在 Harvard,MIT,Princeton 三所学校,因为这三个学校对天才学生的挖角能力非常厉害。一般来讲,大一刚入学,这种人的水平会直接上 Harvard Math 55(这门课是史上著名的变态课,有自己的 wiki page 和课友录,能 survive 下来了都是以后数学研究道路能一条道走到黑的大神神)。或者自己学校的 grad 课,因为他们从初中开始就把本科,甚至一些研究生课都上了(自学 + 到家附近大学蹭课)。summer REU: 除了上面我列的,立志想做 number theory 的同学会去参加 Emory Ken Ono 教授主持的 summer REU, 这个项目是出了名的只招明星学生的项目。一般来说,一个暑假下来,人均能分摊到 2 个 publications。参加者很多大四时就拿到 AMS/MAA 获颁的 Morgan Prize(最佳本科数学生奖,每年全国 1 个),或者 honorable mention(每年 1-2 个),或者女的能拿到 Alice Schafer Prize(最佳女本科数学生奖,每年 1 个)。到毕业时,平均 paper 7 篇,而且大多是 journal of number theory 的水平(我这里只是吧 JNT 做个标杆而已,同适于其他领域,什么 journal of topology 等等)。甚至有一篇发了四大,比如 John Pardon,本科在 P 时发了 Annals,因为他破了一个 50 年未破的 conjecture(PS,这人中学时就把 ArXiv 上的论文天天当杂志看了,爸爸是 duke 数学教授)。

说到 grad school 录取水平。“优秀学生” 一般都可以进 6-10 名的学校,其中 UCLA/UMichigan/Courant 最好进,但是 top5 尤其是 P,H 两所还是很难。如果是明星学生,P,H 无忧。

打了这么多字,楼主应该就明白为什么美国能做出牛逼的数学了吧,这里的 “优秀学生”,“明星学生” 实在是太变态了。。。T—T

知乎用户 徐辰 发表

其实,美国做出超级牛的东西的那些人学的数学并不比中国简单,学的数学比中国简单的那些人也基本上做不出出超级牛的东西。

知乎用户 Yuhang Liu​ 发表

看到那么多人吹美国的基础数学教育也是醉了。。我就说一个事实:美国数学厉害的学生,本科基本都不上 calculus 的,因为 AP 肯定修过了。而且他们肯定会在本科就选 PhD 课并且拿个 A

这表明美国数学厉害的学生都是 ahead of schedule 的(当然中国也是一样)。或者换句话说,数学好的学生根本就不是通过水课教出来的(不好意思美本很多数学课就是水,我当 TA 我有这个发言权)

另外美国数学厉害的学生里面,亚裔其实挺多的,华人和印度裔尤其多。

知乎用户 匿名用户 发表

扒皮时间:

答主谷歌了一下 Yaning Hsu Columbia,发现了这个网站

Research Team

在这个 research group 的网站上,这位答主的 email 地址是 Yaning Hsu <Hsu2@exchange.http://tc.columbia.edu>

敲黑板,划重点,EXCHANGE,是啥意思?访问学者 / 交换生的节奏?(评论区提醒可能是邮件系统的前缀,这一点目前存疑,希望有哥大 tc 的小伙伴可以解释一下)

而且在组里的身份还不是 faculty,而是一个最边缘的 coordinator。我不清楚教育学和心理学研究里 coordinator 具体是干啥的,但是我知道在医学临床研究项目当中,research coordinator 的工作基本就是帮忙协调、招揽参与科研的病人的。

也就是说,是个秘书。

这就很尴尬了,对吧。

8.16 更新: 评论区的小伙伴提到,这位 Ya Ning Hsu 的 LinkedIn 显示她是哥大教育学院的 adjunct assistant professor。

这样就解释了为什么她的信息在 directory 里搜不到。不过 adjunct assistant professor 和所谓哥大教授之间差了多少。。。大家自己掂量吧。

-– 以下为原答案 —

本来并不想写答案反驳的,但是某高票哥大教授实在吹牛吹得太过了,顺便还把我评论给折叠了,那我干脆把评论的内容稍微整合一下写个回答算了。

高票答案请戳 https://www.zhihu.com/question/20953633/answer/211441718

一段一段反驳好了。

当我的孩子还有我的学生进入了 7、8 年级的阶段,我看他们的数学,把我愣住了,我自认数学是满好的,也是在亚洲成长的学生。但当时孩子拿题目来问我,我竟然跟孩子说,「大家退后 3 步让我好好想这一题」。10 分钟后,我居然还想不出来,这是 8 年级的数学,我想怎么可能呢? 我数学那么好怎么可能做不出来?

这段话吓得我差点把饮料喷到电脑上。我姑且认为答主的孩子来问的问题是比较难的,所以我顺手搜了一下美国七八年级小朋友的竞赛题。

这是去年的 AMC-8 试题。Art of Problem Solving

你们造我有多努力吗,我瞪着眼看了老半天都没看出来哪个问题需要让小朋友们后退三步啊!

答主,有的时候,你做不出数学题,还真不怪别人教得高级。

譬如说吧,以现在美国最流行的 Exeter Math,就是美国一个最有名的寄宿学校发明的数学课程,基本上就完全颠覆了以前的想法,没有课本没有公式,学生上课就是一本厚厚的练习本,每一题都是要花一个小时或半个小时才能完成的。

答主我认真的,知乎上美本美高的那么多,谁还没几个上过 Exeter 的朋友?Exeter 的朋友我在大学里交了好几个,自己还辅导过一个。人家上学上课,也是讲知识点讲题做题,只不过因为资源充足体制灵活,可以选择的课程比较多,不是所有人都上同一门课。至于最机灵的那帮跟着冯祖明混的小孩,平时上课就和我们国内的竞赛班没啥区别。而所谓没有课本、学生上课用厚厚的练习本什么的,这到底和我们国内的黄冈题典有什么区别?

但是很多孩子到了高中的时候突飞猛进,因为他们不需要背公式,他们脑子反而非常灵活,他们脑子反而没有被各种公式及规则塞满,而是在找各式各样可能性。

好巧不巧,我男票和大学里的一众好友都是以前搞竞赛并且后来学习数理专业的 ABC。我顺手发短信问了男朋友他当时上中学的时候是不是老师都不会让他们直接套公式。男朋友后来打电话给我强势吐槽了这个问题,并且让我独自在外生活的时候千万别吃错药

哦对了,考虑到答主这动不动后退三步的姿势水平,我还要提醒一句,**不管是在哪个国家,理科学得好的学生都是不会『背』公式的。**正常的套公式的过程,是在理解并且基本内化了公式推导过程的基础上,在看到问题的时候可以迅速反应出来哪个公式是适用的。学数学和其他理科要是学到了要『背』公式的那一步,那估计也已经 GG 了。

我本来以为,到了这个年代,那些鼓吹『美国数学都不刷题不教公式教思维』的毒鸡汤应该都已经死绝了。没想到居然能在知乎上看到那么大一碗,还有那么多赞。我在这里想说一句,想要扎扎实实地学好数学和其他理科,刷题是一个必不可少的过程。美国的教育体系相当 self-driven,那些喜欢数学的孩子在有经济能力的情况下可以获得数不胜数的资源。所以即便体系相对宽松,站在金字塔尖的那堆人刷题量也是不少的。而由于教育资源的丰富,那些真正喜爱数学的孩子可以在学校里提前学高数(不仅是微积分,有些比较强的学校还教线代和实分析),在压力较小的情况下搞竞赛(好大学多,USAMO Qualifier 基本都能有好去处),甚至可以提前接触纯数学的科研。而我们国家的很多超级中学,比如人大附中、上海中学等等,也在朝这个方向发展,想办法给学有余力的同学提供更好的资源了。而体系过于宽松的后果就是那些天赋或者兴趣稍微欠缺一些的孩子基础会打得极为不牢靠,上了大学如果要选相关的课程会非常吃力。我大学里的数学课基本都是和竞赛党一起上的,不熟悉其他课程的情况,但是我在物理课里倒是见到了基础打得不牢靠的后果——大三大四的时候,我为了 pre-med requirement 必须要选基础的物理课,看着身边的同学做作业做得那么吃力我真恨不得替他把作业给做了。好些同学考试的时候 cheat sheet 写满两大张纸(半开卷,允许把记不住的公式什么的写在纸上带进考场参考),公式铺得密密麻麻,真的写起解题过程来还是满头大汗,根本不知道用哪个公式才行。考完后一帮走投无路的同学去问任课老师要怎么提高,任课老师让他们去买这个:

https://www.amazon.com/Problems-Solutions-Introductory-Mechanics-David/dp/1482086921

对,这是一本习题册。哈佛本科的 instructor 告诉我们,理科学不好怎么办?宝贝,老老实实刷题吧。

总结一下就是:美国八年级的数学没有某哥大教授鼓吹得那么难,所谓的高端 Exeter Math 也没有想象中那么高级。足够的习题训练对于任何理科科目的学习来说都是必不可少的。

希望大家别被软文带跑偏了。

知乎用户 匿名用户 发表

-— 06/09/2017 更新 —-

几周前正巧和美国 IMO 领队

@罗博深

讨论过这个问题,所以在这里写一下他的想法和我的一些思考。

关于美国和中国的基础教育,罗博深认为中国的基础教育要更加扎实,并且相比较于美国学生数学水平分布的比较平均。

关于第一条,罗博深认为中国的学生普遍基础扎实,对于课本的掌握程度要远比美国人高。我个人看法是这主要来源于中国老师上课时对同一知识的不断灌输和远高于美国学生的做题数量。就我所知的美国 STEM 教育而言,比如 AP 物理课,老师主要教授的是 “这个公式是什么”,而不是 “这个公式该怎么用”。包括美国的考试题目,一般来说也只是对公式的最基础的应用而已,稍微复杂一点的都不会涉及。

第二条,他认为中国学校的水平分布比较平均。我对此的理解是,在他接触过的学校中,学生在数学方面普遍表现出的程度差距不大。然而罗博深接触过的学校应该以北京和上海的重点学校为主,而对非大城市、甚至乡村的教育所知甚少,所以才会有此言。如果把范围局限在北京上海的重点学校的话,我的确同意他的观点。**在美国的教育体系下,学生水平无论是放在学校与学校之间比较,还是学校内部比较,都明显的呈现出两极分化的趋势。**举例,就美国数学集训队的学校分布和中国数学集训队的学校分布而言,中国数学集训队的名单中传统强校的名字是永远保持着一定比例的;然而美国数学集训队的学校分布则要杂乱很多,基本除了 TJHSST, PEA 和 PA 以外,其他学校的集训队往往只有一个到两个。CMO 和 USAMO 的名单比较同理。背后的理由也很简单:美国的教育体系是一种从高中开始就可以选择 “专业” 的体系,所以一个学校内部不可能所有人都选择数学作为以后的发展方向。由此,两极分化也是很正常的。然而在中国的传统强校中,竞赛往往是很大的侧重,所以实验班的水平比较集中也是可以理解的。

然而罗博深教授同时也觉得,**中国的 STEM 教育总体而言有些固步自封,创新较少,而且没有给学生足够的发展空间。**引用他在聊天中提及的一句话:“Teaching is not about teaching the best stuff, but is about getting people interested to be there.“翻译一下,教育不应该是去考虑如何教最好的内容,而是让学生对所学的知识产生兴趣。(“Teaching shouldn’t be focusing on content only.”) 此处他举了两个例子:

1. 一次他给纽约一些重点高中的老师进行培训,在培训过程中,他拿了一道逻辑推理题给老师们做(他把题目给我们看了一下,大概也就是初中生做的逻辑推理题吧),结果没有一个老师做出。他想借助这个例子说明,老师们自己都没有创新的意识,就更难给教育带来创新了。

2. 他提到了一个学校叫做 Ron Clark Academy。这个学校是一个 innovative 学校的典型:通过一些有趣的教课方式(比如做游戏,跳舞,积极互动等)教授 African American girls 理科知识,最后在 Mathcounts (美国著名初中生数学比赛)中取得极佳的成绩。他给我们看了一个介绍这个学校的视频,youtube 链接:https://www.youtube.com/channel/UCFEx_RwX7lLrwUib0VtfINQ

-— 03/22/2018 更新 —-

以下是一个小小的实验

去年年中与 Po-shen Loh 交流完之后,深受触动。因为他的教育理念不仅仅停留在宏观的批判上,更落实到我们的 everyday class 中 — 与其他老师传统意义上的 “做题 -> 讨论 -> 讲题” 三件套不同(其实在很多中国课堂里,连讨论这一步都没有吧),在他的课堂里,我们每个人都可以 self-paced,选择只做认为对自己有帮助的题,并时时刻刻可以与其他人讨论。Po 很少在黑板上讲课,但他会一直穿插在学生中间,回答每一个学生的 personal questions, 并且是以 hint 的形式给学生提供思路(而不是直接提供答案)。

我和另外一个女生深受这种教学方式的感染,于是决定在我们创立的 math club 里小试牛刀。我们在每一次的 handout 里包含 problems within a large range of difficulties: from AMC 8 to AIME, for example. 这样,每个人都可以找到适合自己的题目,并且通过自己的思考和老师的 hint 获得成长。从此,课堂不再成为少数人的舞台;每个人都得以在这种新的课堂环境下获利,peer pressure 也被大大削弱。在某种程度上,Po 打破了 “课堂” 的原有规则:学生和老师之间的关系变得平等化、私人化,学习成为了一个双向概念(同学向老师学习、老师向同学学习),最大程度上减少了阶级堡垒的形成。


上面是主要从 “教学” 的角度说的。接下来我想回到原题,说说我作为一个学生自己的看法。

首先,题目中的前半句话:

美国中小学生学的数学比我们简单

这句话,结合问题的后半句,有两种理解方式:1. 美国中小学的平均数学教学比我们简单;2. 能 “做出东西” 的那部分人在中小学的数学学习比我们简单。

对于第一种理解,前面的高票回答已经说了很多,此处不去赘述。这里主要说一下第二种理解。

我不同意 “能做出东西的那部分人在中小学的数学学习比我们简单”。

首先,能在数学上 “做出超级厉害的东西” 的人,在高中阶段基本都是竞赛参与者。那么我们基本可以把名单锁定在 USAMO/USAJMO 参与者,对比 CMO 参与者。

那么在美国能进 USAMO 的人,在高中阶段数学学到了什么程度呢?

  1. 基本在 9 年级或者 10 年级学完 AP 微积分。8 年级的相对少一些,但绝不是没有。(希望某些高票答案醒醒 — 在高中阶段学个 AP 微积分简直就是不值一提的事情,是完全无法给这个问题提供任何有用信息的。国内的高中也学微积分啊。)
  2. 在高中毕业前,基本修完多元微积分和线性代数。好奇难度的,我可以给你们看一下课本(我指的不是自学,而是认认真真上过课的):

3. 不少人修到了微分方程和数学分析,还有一些复分析的。但这些相对而言偏少吧… 我感觉主要集中在美国集训队。

在中国进 CMO 的人,在高中阶段数学学到了什么程度呢?

我的大体感觉是,很多人都对微积分,线性代数,微分方程和分析有一定了解,但不一定系统的学过。比如有一次中国集训队训练,据说有些人不知道取三个 vector 的 determinant 是平行六面体的体积。由于中国的竞赛加分政策,很多竞赛生对于大学数学的了解主要停留在高等知识的初等运用上,不知道我的理解是否正确。(当然啦,也有珂神和大一学代数几何这种 dalao OTZ)


最后,说到计算机领域,我随便截几张图,就可以看到中国人的比例(截图取自于 CVPR 2016 open access):

ummmm… 好像不止美国人能做出超级厉害的东西吧?

知乎用户 微微笑笑微微 发表

我不在美国,我在加拿大,据说加拿大数学学的比美国还简单。

我以前也觉得简单,你问小孩,学什么了啊,三四年级了,跟我说,掰手指头。

疫情期间上网课,我天天旁听。

简单,简单个大头鬼。

我家老二,周岁才三岁多,一堆小孩最大的不到五岁,上学前班,数学怎么教呢?从 1 到 20 一周教一个数。比如 15,怎么写怎么读,多少加多少十五,十五减多少是多少,三排五就是十五。也就是说关于 15 的加减乘都学了!!

然后呢,讲就是讲到这个难度,真的很难了。

但是它讲了之后呢,不要求会,爱会不会。这么多小孩,几个会了,会了多少,老师完全不管啊!

考试考啥呢,考 1 到 10,认识就行,都不要求会写。我最不理解就是这个,讲到 20 考到 10 是几个意思?问老师为啥?她说是为了让每个孩子都能做对一些题,这样孩子比较有自信。

我家老大课程也差不多。一年级开始加减乘除,平方三次方概念都开始引入,列竖式也讲,进位退位也讲,爱会不会,就讲一下。要知道,加拿大六岁上一年级,真的已经很难了。

讲完也不做题。比如 10,他讲一堆东西,课后练习是什么呢,是小朋友们画十个棒棒糖!?!或者涂十个颜色,刚才讲的是列竖式进退位啊,就是讲一下…… 真是学会的都是脑子好的。

就酱,有哪个小孩确实是天才那他就理解了,如果他家里有个好爹或者好妈,那推着也学会了,如果都没有,把他就不会,反正也不考也不练习,他自己都不知道自己不会。

而且跟不上也没关系,老师有的是办法夸人,只要是不闹就行,而且规定小孩不能因为成绩差留级,就是一年不上,跟爹妈旅游去了,都不用留级。

另外如果小孩学的特别好,有专门的 talent 班,还是差不多的讲法,会的自然就都会了。

我看了他们的讲法,才彻底明白为啥有的小孩数学特别好,六七岁能跟人讨论开方平方,而那些学习不好的,又都连近视都不近视,身体特好吃嘛嘛香,自信心足足的,就是一点科学常识都不知道。

知乎用户 恶少恶言 发表

很简单,因为小时候数学差的那帮人和发明很多牛逼东西的那帮人是两拨人。你在美国你会看到 100 以内加减乘除算得连滚带爬的人,但我眼睁睁的看到我们大学的旁边有一所私立高中,高中有一条规定,只要有三个学生提出对某门课的兴趣,学校就去请来老师给他们就他们感兴趣的方向开课,而眼睁睁的看着我们学校物理系的教授被请去给当时高中的某几个高中生上量子物理和高等数学的课。

平时跟我们踢球的是大学里化学系的几个博士生,我问那几个原来中科大的孩子为什么读化学系,回答只是容易申请好拿奖学金,但是一同踢球的化学系的美国同学回答我说,因为家里是开药厂的,他从小对这个东西就感兴趣。你说他们两帮人谁更有可能以后造出比较牛的化学产品呢?

如果我们把满分看作 100 分的话,美国有大部分的人 60 分都达不到,但是有少部分的人分布在 95~100 分,而我们中国是通过苦练的方法,使大部分人都到了 75 分。所以我们日常生活中总是拿我们的 75 分和大部分 60 分不到的美国人比,我们会觉得他们数学很差,但真正我们拿不出那么多 95 分以上的人和他们的尖端水平去比拼。大部分时候我们觉得我们数学好不是因为我们聪明,单纯因为我们练的多,且不许用计算器。

走向社会,在美国待久了,你就会发现美国是一个少数精英集体精英,这部分精英把很多东西都发明好了,把制度规则都弄好了,大部分人比较笨,但是这些笨的人会不假思索的去用精英发明的东西,去执行精英制定的规则和制度。这当中且不说还有美国网络的其他国家移民在帮忙添砖加瓦。

这跟我们国家完全是反的,我们国家的那些精英,有的时候不够那么精英,但是底下的老百姓有的时候又过于聪明,从而导致精英做出来的东西制定的制度没有那么先进完善,反而被底下过于聪明的老百姓找到并利用了很多漏洞。

很明显他们社会的状态容易发明一些比较牛的东西,举个例子来说计算机很强大,人人都想用,但是在 windows 发明之前要用 DOS 命令行来操作计算机,但是有些美国人实在太笨了,不可能学会 DOS 语言,所以就有像比尔盖茨这样的聪明人,发明了 windows 这样的东西。

而这个事情要是在中国就绝不可能发明 windows,只可能各大公司把熟练掌握 DOS 甚至考个证书作为招聘条件之一,经过一段时间的勤学苦练,把年轻人个个逼成了 dos 高手。既没有极其聪明的人会去发明更高级的工具,而普通人也足够聪明,可以通过自己的努力,凑合着使用一个不舒服但能凑合的东西。

补充回答…………………………………………

老实说,我真没想到这个题也有这么多抬杠的。

抬杠的点集中在说什么经济基础决定上层建筑,我们的国情不行,不能像美国这么搞。谁说我们的教育制度不行。我们的精英明明比美国多……

在此统一回复一下,首先我也盼望我们的祖国繁荣富强,但眼下美国更能网罗全球英才,更多的掌握先进东西,这也是事实,承认这个差距,我并不觉得丢人。

第二,题目提出了一个问题,我解答这个问题。我说明的是两国的差异,我没有做高下判断,更没有说过中国要学人家美国人做那一套,请不要自己加戏。带这种判断立场的思维的人,麻烦你们提高基本的阅读理解能力。我全文说过中国要学美国那样做吗?

那些说什么经济基础决定上层建筑的人,我当然明白你们想说什么,但这不是本题要问的,就不再展开了,如果你们有兴趣可以另开一提,邀请我来回答。那些回复肤浅没答到点子上,不完全对,的人,我很乐于和你们讨论,有什么观点我洗耳恭听,如果你就回复这几个字的话,恕我只能拉黑你,不欢迎没有观点的单纯抱怨指责。

知乎用户 ELLA SHI 发表

2015. 08. 03 更新

我和大家一样,在国内从小就听过一个说法,美国教育是精英教育,可是来这里学 education 发现根本不是这么回事(这里不讨论大学,因为大部分顶尖名校都是私立的,有自己的运营模式,只讨论 K-12 公立基础教育)。终于,我们教育学院的 professor 解答了我这个疑问,原因如下:

【上个世纪 80 年代,由于美苏冷战,美国的教育重点放在培养理科顶尖人才上。但是现在年代不同了,教育重点是一个叫 No Child Left Behind 的项目。】

算算年份,那会儿公立教育重点培养的理科小孩现在应该 40 多岁 50 多岁,正好事业巅峰期嘛。。。再回过头来看现在的 No Child Left Behind,听名字就知道重点是在帮助哪一部分小孩了。至于理科教育现状,见下文吧。

=========== 正文 ===========

美国麻州 Math Education 专业的来凑个热闹。大家提了很多理论,很多观点,很多数据,我来给大家说说初等教育的现实!同意

@阿成

的说法,美国的理科真的是靠移民了。

我们来一步一步往前倒:

一个顶尖数学家,必然在博士和研究阶段受到了好的教育,有好的科研环境,这点毫无疑问,美国胜。

在成为 PhD 前,一定在本科也有扎实的数学基础,这点中美哪国更强我不清楚,因为没在中国念本科。

在本科选择数学专业之前,首先,得热爱数学,退一步讲,得至少相信自己能学好数学。这里有一点文化差异,中国人会为了一份高薪工作去从事自己的事业,美国这样的很少,他们觉得如果我不热爱就一定做不好。所以,哪怕 math 毕业高薪,也只有热爱数学的美国人才会选择数学专业。这里就开始出现差距了,很多明明有数学天赋,或者至少高中数学成绩不错的学生,已经放弃数学了。

(下面是重点)

再看高中阶段,能不能培养兴趣不去讨论,就说说学生的数学能不能学好。答案是:基本不能。为什么呢?因为他们的数学老师就不懂数学!这个观点大家可能一时不能接受,那就先说点大家熟悉的。OPT STEM,为什么要有 stem?因为缺理科人才啊!美国移民法特别严格,千防万防外国人抢了本地居民的就业机会,万恶的 H1b 配额大家都知道,理科生需求大,但因为法律限制,外国理科高材生并不能补上这个空子,所以 math 专业毕业的美国人不仅有工作,还有高薪工作。高中数学老师呢?在高消费的波士顿地区,第一年年薪 5 万,还是税前,并且薪水都是按年份学历规定死的,干一辈子年薪也过不了 10 万。数学好的都去干好工作了,留下当数学老师的都是什么水平?当然了,美国人会随着兴趣找工作的嘛,有没有可能名校数学高材生就是想当老师呢?不好意思,有个门槛叫 teaching license,要想拿到这个东西,不是通过几个考试就行的,必须要修完 license program,就是一堆课程加上实习,可以理解为再回大学念书。当然了,私立校教书可以没有 license,但工资还是一样低。所以啊,留下来教高中数学的老师,很多自己都不懂数学(还不信的看回答最后附的真实故事),他们教出来的学生,又能多懂数学?高中阶段,又一批未来的数学家放弃数学了。

初中阶段不多吐槽,初中数学的 license 和高中数学的 license 不同,有了高中数学 license 可以直接拿初中的,但是初中的不行,因为初中数学的 license program 中,对数学要求更低。根据经验,高中数学 license 念不下去才会只念初中。

小学阶段,又是一个重点!为什么呢?因为所有课都是一个老师教的啊(体育、音乐这类除外)!这些小学老师大多文科出生,自己数学基础就不怎么好,美国的小学生真的 “数学是语文老师教的” 啊。谁也说不清楚,多少数学有天赋的孩子在小学阶段就对数学有了错误的认识。举个例子,教 odd number、even number 的时候,小学老师让学生记住 24680 结尾是 even,13579 结尾是 odd,而不是解释给学生听 even 是能对半分的意思,这样学生就会有错误的认识:数学就是死记硬背。

优秀数学老师的稀缺,会造成一个恶性循环,社会对理科生的需求不会变,但理科生的数量越来越少,好的理科生就更不会去当数学老师,基础阶段的数学老师就更差,于是学生的数学也越来越差,理科生就越来越少。文科方面的老师也有这么一个循环,不过是良性循环,因为那些 English、History 毕业的真心找不到工作。所以啊,美国数学教育大的趋势就是在走下坡路。

下面说点亲身经历、真实事件吧。

请看题:

我在修 teaching method 的时候,一次模拟教学用这道题作为一个很难的 task,其他同学(就是未来的高中数学老师们)扮演学生,模拟教学的时候他们一个人都不会做这道题,我一开始以为他们故意扮演的,结果 discussion 的时候他们说,真的不会做。。。不会做的原因是,他们不知道内切圆圆心到三边垂直。。。

有很多人提到高中 AP Calculus 难度很高,这是真的,但课程难度高不等于老师教得好!不等于老师教得好!不等于老师教得好!跟 Harvard 一位数学教授讨论过这个问题,教授说,“I have quite a few high school teachers in my summer school class now. you know they have a masters of math education program there.” 我说,“Hmmm, sometimes I feel those high school math teachers don’t really know math.” 然后,

这是我所了解的公立学校的情况,不能一概而论,欢迎补充指正

知乎用户 王某​ 发表

对大部分平民阶层,中国的教育并非是完成成长需要,而是干掉对手成为养蛊中最后的蛊以完成阶层流动。

1 美国基础教育粗浅,只是生源差的公立粗浅,并不是所有都粗浅。这一点中国相对好,中国对于生源差的基础教育通过毛坦厂衡水这样的模式水平比美国高,家庭负担比美国低。

2 美国的教育特点是成长性,中国的教育特点是选拔性。举例,文史哲等学科在基础教育领域扮演的角色,除了通识性的知识输入,不过是提供成长性空间。而理科带有鲜明的竞争性选拔性特点,容易分出结果,但因为壁垒分明,学科外成长性低,导致人们对理科生产生情商的刻板印象。

而中国学生对于数理化的选择,刨除小部分天赋型的热爱,大部分普通水平学生并非是出于满足自我成长需求,而是出于低记忆负担、清晰题目结果(问题结构良好)这种易分辨性的特点,结合基础学科就业不如应用性强的学科,而选择。

美国的成长性教育,以在学生抽象思维虽不发达的情况下,通过高自由度选择保证高兴趣专注获取了大量命题——一个鲜明的特点是美国的小孩仿佛特别能讲,特别小大人,演讲思辨仿佛从小特别强,而中国小孩显得容易沉默。人完全可以在具备抽象思维而不懂得哲学的情况下思考世界上没有相同的两片树叶和人不能一次踏进两条河流,并获得成长性体验。但人不可以通过成长性体验、自我成长需要的满足竞争掉你的对手,干掉和你抢名额的人。

决定以上特色无非是教育资源的多寡,人均教育投入。

简言之,当你的目标是干掉对手,抢夺 top2、985211 的高等教育资源,完成阶级流动,干掉村里的狗蛋二丫,成为那个来到北上广 18 年后喝咖啡的人,你根本不会考虑哪些知识能够让你变得更好——你只会考虑怎么样变成凤凰。

知乎用户 浣熊数学​ 发表

我是一名教数学竞赛的老师,美国教材和国内教材我都教过,我不认为美国中小学数学比中国数学简单,相反,我认为美国基础教育向美国高等教育输出了优质的成果

作为一个教育工作者,我主要从中学数学都学什么来分析下这个问题。

我国一两百年前喜欢自称天朝,如今在数学方面,依然喜欢以天朝自居。总是有着蜜汁自信:“美国、欧洲的数学基础教育都远不如中国,那边的高中生数学水平只相当于我们的初中生。”

有人说:

①中国成绩中等的学生去考 ACT、SAT、SAT Ⅱ ,数学部分都能随便考满分。

②对于美国的数学竞赛,中国学生只需掌握课内知识,就能轻松应对。

但实际的情况是什么样的呢?我们来对比下中国高中数学体系和美国高中数学教学体系。

一:中国高中数学学什么,如何与大学知识衔接的?

我们首先来说中国高中生每天玩命的学、冲刺高考,到底学了哪些方面的技能?概括来说,是函数、几何等几大板块,每个板块各一道大题和一些小题,板块之间相对独立。

中国高中课本侧重于高中知识往 “深” 和“复杂变化,与大学知识结合不多。结果是:**很多中国的学生高中数学不错,但到大学就听不懂、跟不上了。**不信你看看全国多少大学每到考高数时,中国学生处于不理解,硬背公式的状态,继而有人去作弊,或者挂科一大片。

二:美国高中数学学什么,如何与大学知识衔接的?

美国高中及国际学校高中数学主要路线是:高一学习 Pre-calculus ,这部分内容和中国的函数部分类似,快速进入和大学衔接的 Calculus 的学习。

虽然有些板块的知识确实不如中国学生学的那么深、刷那么多题,但把有限的时间分配在更” 远 “的知识上,大家对于一些大学的数学知识,在高中已经有了直观的了解。这对于很多学生在大学的发展,是非常有利的。

三:我们为什么觉得美国数学比较简单?

①拿我们重点考的板块和他们不重点考的板块比较

比如解析几何和立体几何,中国高中的解析几何会研究比较复杂的椭圆、双曲线、抛物线的问题,而在美国数学课内的解析几何中,对这些东西只作简单介绍。有人就以此来断定美国数学简单,但这么比较这是不公平的。毕竟,中国学生也不一定能做出 AP 微积分中的题呀。

②美国是精英教育,对于普通大众的题本来就简单

说实话,无论以哪个角度来看,SAT / ACT 试卷数学部分比高考数学都是要简单的。根本原因是,SAT I 的目标考生就是美国普通水平的考生,在美国这样一个崇尚精英教育的国家,会把给大众的题出的特别简单,给精英的题出的特别难。

但美国高校录取可不止看 SAT / ACT 啊,还有 SAT II, AP 和更难的 AMC / AIME 才会让一个学生 stand out。

而中国有一个统一的高考,最顶尖的学生和最普通的学生都参加这个考试,所以在中学教育层面上,中国没有 SAT 数学这样的简单考试。

如果还以为美国数学教的太简单、以为美国数学不如中国数学,那你就错了。你以为美国就没有数学尖子生吗?

我的一位朋友王之鑫,现在耶鲁读应用物理的博士,引用他的原话:

平时我们觉得谁反应快、智商高的时候看到的通常只是假象,更有可能的原因是别人早期基础好——当我们还在刷《五年高考三年模拟》的时候,有童鞋就已经学完了一遍微积分;当我们当年还在学用计算器的时候,有些小朋友就已经开始学编程。
这种早期基础的差异会在长期的积累中逐步放大,最终形成人与人之间巨大的能力差距。
中国学生在基础教育阶段的超前学习意识是很弱的,如果中学阶段能掌握微积分、C 语言,在清华就算是基础很 “顶尖” 的大神了。可在美国名校,基础碾压可能是另一个级别的。

他在实验室里听到的闲聊都是这样的:

“我家在宾夕法尼亚州西北部,那是一个教育很落后的地方,我第一遍正式学微积分都是高二了。”(相当于中国高一)
“外人总觉得我们高中出了八个诺贝尔奖,一定很牛的样子。其实它就是一个公立高中,很大但质量并不好,比私立学校差很多。”
“我五岁的时候我爸就给我讲核反应堆的运行原理…… 他一直说我动手能力差,现在的实验条件比他读博士那会儿好太多了。”(这位小哥的动手能力已经非常爆表了)

以上只是闲聊,更多的碾压体现在学习和工作中。

毫不夸张的说,很多中国留学生,尤其学习理工科的,都因为没有这种超前学习意识,基础不够好,影响了学习进度。置身于基础优秀的美国大学生中,很多学生很容易产生自卑。语言水平又相当于当地的初中生,想要主动交流效率就更低。

可以说,由于没有超前学习意识,造成的基础薄弱,会产生多米诺骨牌式的负面效应。

而在美国的基础教育体系中,普通的美国学生不会因为数学太难失去信心,高智商的精英美国学生又早早开始打好数学的基础,各类学生因材施教,最终培养出大批人才也就不足为奇了。

四:美国目前不重视理科基础教育?

几篇高赞答案中提到,目前美国基础理科教育很差,中小学在培养垃圾等言论。我反对这个观点,我认为现在美国甚至要比 1957 年更重视理科基础教育。

这就必须要谈谈 STEM 教育的出现。(下面的科普信息参照了我的朋友小萨老师的研究)

1957 年 10 月 4 日,正值美苏冷战。在拜科努尔航天中心,苏联科学家和工程师将一颗名为「史普尼克一号 (Sputnik 1) 」的人造卫星,送入地球上空的近地轨道。举国沸腾!

史普尼克一号是人类历史上第一颗人造地球卫星。苏联的创举刺痛美国——自诩世界最强的美国,居然在航天科技上被苏联人碾压。怎能忍?于是,美国总统艾森豪威尔下令,加速发展航天事业,尽早反超苏联。

想发展航空事业,美国自然需要大量科学 (Science) 、科技 (Technology) 、工程 (Engineering) 、数学 (Mathematics) 人才。伴随着政府的慷慨拨款和大力宣传,越来越多美国年轻人在国家使命的感召下,走入这四大领域的学习、研究、工作。

而这,正是前几年很火的一个概念「STEM 教育」的起源—— STEM 是 Science, Technology, Engineering, Mathematics 这四个单词首字母的合成词。虽然在 1957 年前,美国人也热衷学习这四大领域,但热度从来没有如此高涨;1957 年美国的理工科学习氛围,有点类似我们国家二十年前的一个热潮——学好数理化,走遍天下都不怕

潮起潮落,时间来到 2000 年,在 STEM 相关产业上,美国再次遭遇失去世界霸主的危险。美国许多学者的报告显示,国家正面临严重的 STEM 教育缺失。五年后,又一份报告表明,美国在 STEM 教育的发展上,已落后许多国家。说句题外话,这份报告的作者,也是有记载的
「STEM 教育」这个词组的发明人。这个词组的确朗朗上口,好念又好记。也对,既然是个运动,是个倡议,那就得有个响亮好记的名字,如果起个名字,叫 KDHG 教育之类的,不仅听着就一点也不教育,运动也很难流行开来。

这份报告得到政府的很大重视。感受到危机的政治家们,迅速成立专项小组 (Caucus) ,开始起草提案,推动 STEM 教育的发展。美国政府两次下决心推动 STEM 教育,都是源于危机感。看来,危机感的确是自我改变的强大动力。

在 STEM 教育成为热搜后没多久,美国学术界又开始讨论一种新的教育形式。2011 年,时任罗德岛设计学院 (Rhode Island School of Design) 校长的前田 · 约翰教授 (John Maeda) 发起一场名为 STEM to STEAM 的运动,以推动 STEAM 教育的发展。相比 STEM ,STEAM 教育加入 Arts ,也就是艺术和设计这两大元素。

这场运动号召人们既学习工程和科学,又努力培养自己的审美能力和以人为本的思维能力——审美和以人为本这两大能力,在大众眼中,一直是艺术家和设计师的专利。但约翰教授认为不应该这样。他相信,在未来的世界里,人们将遭遇许多复杂的问题,而要想找到解决方案,需要人们具备跨学科思考的能力和全面的技能库,这其中就包括艺术和设计教育善于培养的想象力、创造力、同理心等。

总的来说,STEAM 教育体系的目标,是为社会培养出越来越多像达芬奇一样,能写、能画、能设计、能科研、能创造的多面手。可能有些读者对达芬奇的认知,仅限于他的画家身份。但实际上,达芬奇创作过的画并不多,大概只有 15 幅左右。对于这 15 幅,非艺术圈的人能马上叫出名字的,通常就只有《最后的晚餐》和《蒙娜丽莎》。顶多再加幅《安吉里之战》。达芬奇的自我定位,以及后世对他的定位,都是一个通才。由于达芬奇和其他类似的通才在文艺复兴时期大批出现,后世就把通才也称为文艺复兴人 (Renaissance Man) 。

中国留学生在美国顶尖大学中,不仅是第 3 点中提到的被基础碾压,更多是被他们强大的表达能力、动手能力等各方面技能碾压。追根究底,其实是因为美国持续地在推行 STEAM 教育。Science 排在第一位,我们能说美国不重视理工科?

这股 STEAM 教育的风已经刮遍全球,当然包含我国,但我们要追赶的又何止五年十年?


2018.5.2 更新:

@LucienF

@supersarah

二位大佬,你们说的都很有道理。我觉得是这样:积分那道题并不难,事实上,基本的换元积分在咱们中国大一课程中也是基础题。可是中国大部分高中生真的不会啊,至少我自己高三的时候,做高考后两道压轴题都很熟练的情况下,对于积分的了解只是:“猜一个函数,求导之后变成现有的函数,猜出的函数就是积分”。于是只会做简单幂函数的积分。

以至于到了大学,学积分还挺吃力的,我大学是建筑系,数学课少,我又对大学数学感兴趣,想自己多学一些,自学时真实地感受到了即使高考数学 150,面对微积分还是挺难理解的。

高考数学如何拿满分?

因为高中导数大题 “套导数公式” 那一步做完之后,就不再用微积分知识了,后面的解法主要靠初等知识的叠加(比如不等式和分类讨论)。可能真的是咱们高中教育,更偏向于往 “综合、变化” 这种 “深” 的地方考,而非往 “进阶、应用” 这种 “远” 的地方引导。

至于另外一些评论,其实**我文章中并没有说中国数学教学模式不好,**用高中的知识点出一些难题,对于培养研究数学的思维有好处,对于高考试卷的区分度也有好处。只是,大部分人需要的数学只是在 “应用数学” 的层面(包括在物理、化学上的应用),对于这些人,数学只是工具,高中具体知识没必要学那么“深”,用些时间学学高中到大学的过渡内容,可能对这些人更实用,有助于他们在大学的发展。

知乎用户 zakufish 发表

不在美帝,本人以澳村的例子来答一下。

澳大利亚国家信息通讯技术委员会(NICTA - National Information Technology Australia)每年会组织一个叫做 Young ICT Explorer Competition 的比赛。在这个比赛里,小朋友们拿出自己做的 project 来比拼,然后争夺各个年级组的第一。

然后本人有幸当了今年活动的志愿者。

喏,这些就是我大澳村(输送到美帝)的未来码农。

这些小朋友厉害到什么程度呢?

一个两年级的小朋友,我不知道用了什么方法,做了一台简易钢琴,只有你人按上去才能出声音,狗啊猫啊棍子啊什么的按上去都没反应。

(抱歉这个照片我忘记拍了)

但是最让我印象深刻的是两个五年级的小朋友。

她们用 C# 实现了对人类心脏的模拟。

这是我当时拍的照片。在最左边那台电脑的左边,是一摞书。什么书呢?

关于人类心脏的医学书籍,以及一本厚厚的离散数学。其中的图算法部分,两个小朋友甚至还做了笔记。之前她们想要模拟血液流动,特意去学了离散的图算法,因为听说计算机里面很多图像都要用到离散的知识。她们跟学校说要看离散,学校里数学系毕业的某老师就把一本崭新的书给她们了。虽然最后发现没什么卵用。

作为上学期离散期末图算法没做出来的人,我那时候觉得这么多年书都白读了。

这是两个小朋友的 github。有谁有兴趣可以去看看。一想起自己空空如也的 github…(~_~;)

这俩厉害的小朋友最后拿了五年级组的冠军 (。・ω・。)(领奖的时候一个孩子身体不舒服所以没来)。

结束的时候,小朋友在大礼堂里听演讲。

就在这时候我问旁边一个也是中国来的小伙伴志愿者: 你说咱们小学时候在干嘛?

“上补习班” 他说,“还有魔兽争霸 3。”

可怕的是,我们的国人至今还没有意识到自己教育不足。

“为什么我们的素学比美帝蓝辣么多, 他们的科学还比我们发达?素血不似一个国家科技实力的体现吗?哼!伐开心!╯﹏╰”

在小弟五年级的时候,班里数学厉害的孩子都在刷英语和奥数题,没人研究过离散图算法和心脏结构。

并不是说学习奥数就是错,应该教全国小学生离散,让他们长大个个能设计算法——我的意思是,如果一个国家的教育不能给一个孩子提供他想要的成长通道,那么无疑是可悲的。

我相信中国有一天也能实现像外国这样的教育——固然有千军万马过独木桥般进一流大学,也能在一个稳定的环境中给孩子学习他想要学的东西。但是估计,我这一代是看不到了。

知乎用户 丁浩​ 发表

恰好不久前在某论坛看到类似的话题,小可自愿当个搬运工,听听某论坛上在美藤校执教多年的国人说说他的看法,对中美教育体系有较为深入的剖析,此人刚好专业方向是数学,在美多年,应该有更多话语权,兼听则明吧。

@Dave Geng

@冷哲

本人对理科教育一直很感兴趣,在河里多次就此话题发帖。很多河友对教育以及中美体系比较 甚感兴趣 观点众多。现在我想简述一下我在美国七年求学执教的一些第一手见闻和感受。一是希望起到抛砖引玉之效, 二是给自己总结一下,为自己回国工作后和人侃教育整理一下材料。

几点说明:

1 我在 21 世纪初在北京某大学 A 学习,之后来美留学读博士(涉及学校 B C),再往后在美国执教(大学 D)。我对国内本科硕士,国外本科博士等阶段的基础科学教育(包括学习和教学)都有一定体会,在我的系列中也会对中美的大学做一些对比。

2 我的专业方向属于纯数学(以及相当程度的理论物理)。我对实验类学科或工科了解很少, 故无法涉及这些方向。但我教过不少数学课(大学里学生最杂的课),这使得我能大范围接触全校各个专业学生并由此感知他们的数学水平。

3 我求学的学校各自代表了中美的最高水平,我执教的也是老牌藤校。一方面我感觉到了一流大学确有过人之处,另一方面我也看到了一些令我大跌眼镜的现象。我感到这两方面的很多东西我似乎没有在中文媒体网站上见到较多的提及 (虽然谈论建立世界一流大学的文章很多很多)。

我是大学 B 的学生(研究生),但我的导师在大学 C 导致我常到 C 校去,故我对两校均有了解。

本科新生数学基础弱于中国同地位大学

B 有美国数一数二的理科生源并有很多顶尖国际生源支援,但我感觉他们的平均数理水平仍不及我在中国的母校 A 的(规模更大的)入校生源的水平(当然 A 的生源在中国也是数一数二的)。我认为这是中国基础教育的一个胜利。 当然 B 的新生水准也是不错的,少有我认为明显配不上 B 的。

题海 / 考试海 / 教辅 / 补课 体系

尽管 B 有很好的生源,他却毫不懈怠,反倒响鼓偏用重锤敲。B 高度重视数学基础课并有自己一套方案:

除了通用的教材,还有两三套自编的教学讲义和 “教辅材料”;教学讲义和“教辅材料” 各有侧重,相互补充,时有更新;大量布置作业习题并对习题精心组织分类;除去正课外,要求学生必须上一周两次的小班习题课(由助教上),而由于有超强的研究生生源 这些助教(在读博士研究生)大都具备独立主讲全课程的能力(相比之下中国的大学 A 里甚至很多博士后都不能做到这一点);由于具有极高的师生比例 以及博士生本科生比例 B 可以为每一门大课组织 庞大的教授 / 助教 / 批改作业者 体系;教授和助教均提供很多课外答疑时间;相关院系,学生宿舍,学生内部 均会组织“学习班”“解题班” 等, 有的学生甚至自己出钱雇家教教自己。

此外 小班习题课上 常有各种小测验(计入最后成绩);全体的大考试 则每学期(12 周)有 3 到 4 次 (相比之下中国的大学 A 通常 16 周内只大考两次)。

开始我还没太在意,后来我突然意识到 B 的这套体系不是很像中国高中盛行的 题海 / 考试海 / 教辅 / 补课 体系么?

那么这套体系运行情况如何呢?至少从出口看,效果很好:学生考试成绩不错,我教高年级课时也感觉学生的低年级课的基础打得好。学生的负担很重吗?那是很显然的。有时我都觉得没有必要搞这么多任务。但可能是学生已成年以及本身上进心强等方面的因素 我没有听说过什么被压出严重心理问题的事(也可能我孤陋寡闻了)。

B 的这套本科基础课教学体系 是 B 在北美诸校中脱颖而出的一大法宝,这一点我在到了大学 D 后有极深体会。

B 是让很多中国大学流口水 很多知识分子常挂嘴边的榜样,可是有多少人知道 B 的基础课教学体系 是这样严酷(和类似中国高中)呢? 我曾试图和来 B“考察高等教育”的中国高校人员 介绍 B 的经验, 可他不感兴趣。 能调动起兴趣的是 各种雕像 Logo 和特色建筑带来的 “人文气息” 和 学生活动招贴,草地休闲等体现的“自由学术氛围”。。。。。。

待续

现在我讲讲 B 和 C 大学的 研究生教育情况。 我只了解数学这样的基础理论科学,工科和实验科学的情况可能不大一样,敬请注意。

研究生学术水平远超中国同地位大学

这当然是来美之前我就预计到的,但来了之后我才知道究竟强到何种程度。

雄厚的学术背景

不少研究生在刚入学时或入学后一年内 就已经掌握了不少较深的理论。两年以后,我见过的国内来作访问的一些 985 学校教授已经无法和我们讨论问题了 因为有太多我们懂而他们不懂的东西。

精神原子弹

一个突出的感受 就是关于人的精神面貌。 这里是我见过的人的精神力爆发性最强的地方。有一批为学术痴狂的学生 时不时处于唯恐自己学得太少太慢的精神状态。 我也是其中之一。在来到这里之前,我觉得自己长期处于一种与周边格格不入的学术化偏执状态,来了之后我觉得自己不孤独了。我甚至很担心我的某些同学会过劳死。

惊人的效率

强大的背景和只争朝夕的精神 造就了惊人的学习效率。和中国的 A 大学做个对比。 比如在 A 大学 五六个好学生 进行任务分解 折腾十二三个星期的讨论班(每周讨论一次) 可以理解清楚某一复杂前沿学术文章。 在那时候我感觉良好得很,因为若把这文章交得我手里 我一个人便能在相同时间内轻松搞定。 而到了在 C 大学里 一个研究生中的猛将 靠单干两三个星期就能拿下。当然我没多久也成了这样的人。 但如果不是 B 和 C 的环境 我恐怕还在沾沾自喜自己鹤立鸡群一个顶五六个 而没有意识到自己的效率还有提高至少五倍的潜力。

强烈的主人翁意识

B C 的不少教授自身便是学界领袖,决定着学术的发展方向。这深深影响了学校的气氛和学生的心态,仿佛自己就处于世界中心 未来也要争做主人翁一般。这固然有自大和可笑的一方面,但也意味着很多学生 面对再难啃的学习上硬骨头 都有 “志在必得”“攻必克” 的自信。而这种自信在中国国内的大学是非常缺乏的:我见过太多国内的聪明孩子不敢去学最难的学术文献。

也有一些平庸者

当然我要承认再好的学校都有一批水平一般的学生。不过他们中的一些在退出学界走向社会后 反而可能成为 比我们这些学究社会影响力更大的人。

人的差距是最大的

我一向不认为 中国大学和世界一流的差距 主要在于体制。首当其冲的问题是 钱比人家少太多。这个问题在近年来得到了较大改善,但也面临 需要持续大投入和一定时间积累的问题。

那么在钱的问题不那么突出以后,什么是主要矛盾呢? 我以为是人的差距。确切地说 是中国未能在本土完成 基础科学一流人才的原始积累。我希望我的见闻能使读者理解 为什么即使中国现在全盘复制美国的大学体制,中国的大学的基础科学研究生教育也是无法和 B C 这样的学校竞争的 — 研究生们的 学术基础 天赋 野心 勇气 甚至勤奋程度都远远拼不过。

那么是不是 中国的基础科学没希望赶上美国了呢?中短期内(比如 15 年内)我觉得是这样的。 但长期看则不然。不过这要等我以后的文章解释了。

待续

毕业后我在大学 D 任教。D 是老牌名校,是美国综合性大学第二梯队的一个典型代表(B C 属第一梯队)。我在 D 大学教过从大一到大四的来自各个院系专业的学生,有的课还是人数颇多的大课。和以前一样 我只能通过数学能力来了解学生。

本科生入学水平远弱于 BC

有些人在我看来需要重修高中乃至初中的一些数学课。个别人连小学数学(如分数的四则运算)都不熟!这绝不是我个人的偏激看法。比如 学校出的关于新生数学能力的诊断性考试题 竟然要考察自然数的四则运算, 分数的四则运算,一元一次方程等(当然后面会考到高中的复数 极限 导数等)。而多数新生在这样简单的考题上竟只能拿到一半分。更可怕的是,他们中很多竟然敢于免修第一门微积分课(直接上第二门)。。。

教学困境

显然,在这样的情况下想让学生知其然也知其所以然已成了几乎不可能的任务。一些刚从象牙塔毕业的青年教师(我是其中一员)对此没有思想准备。我试图保住一条底线:最起码我要解释教科书上写了的东西 让他们在学完后能看懂教科书。但即便这样低的目标 我也根本达不到而以惨败结束。比如我一直耿耿于怀的一件事是:我教的微积分学生 可能几乎没有人明白积分的定义。。。直到第二年 我才从更有经验的人那儿了解到:不要试图准确描述基本概念,也不要花较多时间解释主要思路动机 或多问为什么(因为这对学生来说实在太难了。。。而且会遭到学生的强烈反对)。

那么是不是说 在所谓 “素质 / 启发性 / 创造性 / 实质性” 教学完全走不通后 只有 “填鸭” 教育一条路 让学生会算一些题就行了呢?事实上由于没有大学 B 那样强的(类似于中国高中的)管控体系 我们连 “填鸭” 的能力也没有。

最终的结果是 通常已经有所放水的期末考试 常常是多数人无法及格。而且低年级的劣质情况 积累影响中高年级,甚至导致系里不得不把一些 有多年历史的 中高年级的课程 内容砍掉 1/3 以上 (但之后仍是大多数人考不及格。。。)

我刚到学校时曾与系里一位老教授攀谈,当谈及基础教育失败导致学生数学能力极差且有越来越差之势时 这位曾上过战场的老头竟然控制不住 在我面前哭起来。我在教了好几门课后 开始明白他的痛苦了。

学生有自由但反受耽误

另一件让我印象深的事是:D 没有让我觉得学术上拔尖的本科学生。到了高年级 确有一些功课学得很好 也有心向学的人。我给过他们一些进一步发展的意见指导,但无一例外的发现 他们学过的东西非常非常少 对学科的理解 则基本还停留在新生和科普水准。在了解了他们在本科三四年中的学习经历后 我发现他们基本上都有几个问题:

1 在中低年级并没有完全想清楚自己未来要干什么 因此花了不少时间去体验其他的专业以及很多 “通识课”。大学自由灵活的体制也鼓励他们这么做。

2 以为自己所有的专业课都学得很好就行了。

这样做有啥问题呢? 问题在于他们被真正拔尖的孩子(其他学校的和外国的) 甩开得太多太多了:三四年前大家的起点差不多,而现在别人 “轻舟已过万重山” 高出他们好几个境界去了。 怎么会这样呢?这是因为最拔尖的学生 具有很强的 目的性 (我一定要当科学家),自我指导性(给他足够的书,他自己就能钻研出来) 和 自我激发性 (学得越好就越不满足甚至越焦虑)。

D 大学的好学生没有这么强 但是如果他们从大一开始 就坚持贯彻明确的专业培养计划 他们和最顶尖学生的差距不至于这么大(起码学的专业课会多得多)。而现在 我只能指出 他们的学术基础太薄弱(很多基础专业课没学)并开出长长的课单书单。 但我知道他们没时间了:毕业期申请期快到了,一些孩子也痛苦地意识到:自己看似没有什么问题的学术之路 实际上导致自己已经输在起跑线附近(如果以成为一流学者为目标的话)。

坚持贯彻明确的专业培养计划 这其实是中国大学的办法。中国大学给学生转专业和定专业的自由 远不及美国学校。这当然导致了众所周知的 学生不得不学不喜欢的专业的问题 但也保证了 次好的学生在愿意走此专业道路时 不至于落后先进太多。 这些次好的学生(假设在中国最顶尖学校)的学术竞争力 强于大学 D 的上述好学生(虽然 D 的科研和教授实力目前肯定强于中国最顶尖学校)。

我知道中国一些大学 近年来喜欢搞所谓 “宽基础” 的综合性强的 “XX 班”并赋予这些班的学生广泛的专业选择权。但我也了解到很多这样的尝试产生了不小的负面后果(比如事实上削弱了学生的竞争力),在有些学校甚至导致了公开的矛盾冲突。D 大学和中国的实践 提醒我们 “宽基础”“通识”培养体制存在的弱点:最顶尖学生不会因此更厉害 但次顶尖学生可能会严重落后(并在某个时候遭遇信心重挫)。

待续

上文讲到大学 D 的新生基础太差,导致教学质量很差。这其实对学生的伤害最大。最直接的一个表现就是

数学科学死亡行军(math-science death march)

这是纽约时报一篇长篇文章链接出处中强调的一个词。这篇文章主要讲的是:为什么 有很多爱好科学 有志于学习理工科的孩子 到了大学学习两三年后纷纷转专业?—- 因为理工科实在是太难了(so darn hard), 特别是孩子们无法战胜低年级的数学科学死亡行军(math-science death march)。

But, it turns out, middle and high school students are having most of the fun, building their erector sets and dropping eggs into water to test the first law of motion. The excitement quickly fades as students brush up against the reality of what David E. Goldberg, an emeritus engineering professor, calls “the math-science death march.” Freshmen in college wade through a blizzard of calculus, physics and chemistry in lecture halls with hundreds of other students. And then many wash out.

这篇文章本身没什么大不了,最有价值的其实是后面的 1000 多条读者评论。其中有大量认真撰写的长评论,我强烈推荐英文好的人看一看(顺便说一下,英文主流媒体常有读者评论比正文有意思的情况)。除了一些有些离题的抨击外国人或华尔街抢饭碗外,评论中出现了两派:一派有很多学生,指责大学要负极大责任:如教授高高在上,所学内容过于晦涩,教学方式过于呆板等等;另一派有很多大学教师和老工程师(很多人上来先亮身分:我做 XX / 我教 XX 已经多少年了。。。),这一派认为主要是学生水平太次,不如自己这一代人。我虽然比学生大不了多少, 但我属第二派(屁股决定脑袋?)。大学教学固然可以挑出毛病,但在学生的极薄弱理科基础(尤其是数学)面前都属于小问题了。

我对这件事(数学科学死亡行军) 有直接的体会, 亲见一些有梦想 上进心强 人其实也算聪明的学生 由于搞不定我讲的课(也许还有其他课) 而基本上丧失掉了追求梦想(探索科学或者做工程师)的可能。我观察过一些 常来问问题的学生 发现糟糕的中小学教育 导致他们脑子里的知识点处于一团乱麻的状态(可能比没学过还糟糕)而这种状态又进一步导致 他们有条理的梳理组织思路的能力极为欠缺。即使我问一些他们中学学过的东西 他们往往都会很快暴露出这些问题来。

把问题带到下一个阶段

也许有读者会说 我这个学究要求太高 很多学生准备做应用技术或商业金融 不需要把基础课那么好。但我强调的其实是 即使是只要求不求甚解照虎花猫地使用 不少学生也有不小困难。如果要打一个比方 我觉得可能类似于 要一个只认识 1000 个汉字的人读报纸。以我们输出的学生水平看,很多人要想在实际工作中 (比方说)考虑优化问题 分析处理信号 或者 分析稍复杂的统计数据 势必感到吃力而且缺乏变通能力(这点从他们做应用题就能看出)。

一些对中学教育很愤怒的大学教授 用 “garbage in, garbage out” 来描述这种情况:高中送给我们基础太差的学生 导致应用专业毕业的也是基础很差的学生 而这又进一步影响到工业界。我可以告诉河里一些在工业界工作的河友一个秘密:除非你的工程师来自第一梯队大学(最有名的那六七所)否则你得假设他的成绩可能水分很大 特别是数学基础课(因为数学系通常在学校里较弱势 不敢得罪人)。比如如果他的成绩为 B 这其实可能意味着他实际成绩不及格。。。。

研究生教育不算拔尖

再简要说一下研究生教育。D 大学的教授虽不向 B C 那样强势 但也基本都是功力深厚的悍将。可是 D 的研究生精神面貌 就远远不如 BC 的研究生, 反而更像中国最顶尖学校的研究生:有强烈的主人翁意识的人很少,没有很多精神原子弹,(最令我窝火的毛病)很多人不敢学习自己方向上最难的学术工作。。。

我感到中国国内的最顶尖大学在研究生教育上 和 D 大学这样的学校比 虽然仍然落后 但已经可以望其项背了。比如如果未来几年能够保持现在的吸收海龟的势头,那么在我熟悉的学科(数学)上 十年后达到美国 10-15 名学校的研究生教育水平 是完全有机会的。

第二梯队基础科学教育形势不容乐观

如果连 D 大学这样的学校生源都大成问题,我就不得不怀疑:除了个位数的最好学校外,美国其他的学校的 美国理工科生源质量 都有大问题。我没有专门调查其他学校情况,但我零星接触的情况同我的怀疑是吻合的。

我回想了一下我在国内的高中情况:中国的第一梯队学校和其后的学校理工科生源质量远没有这么恐怖的巨大落差。

不要再猜测学校

顺便说一下,因为我说了很多 D 大的毛病,所以希望河友不要猜测 D 大是哪所学校(其实如果仔细看帖又对北美大学熟悉的话 范围很容易缩小到两三所)以免在网上流传后引起不必要的麻烦。ABC 也不要猜了(虽然可能比猜 D 还容易)。

待续

注意:1 我在这篇里只讲大学的基础科学研究和教育。2 我说的很多是对理论科学的观察,实验科学因为依赖于大量实验设备 分析起来可能会有所不同。不过我认为我讲的 大体上恐怕还是成立的。如其他领域的河友有不同意见 欢迎提出。

美国最顶尖大学在基础理论科学研究和教育上拥有巨大优势

简单的讲 美国拥有的第一流教授和学者 占世界的份额实在是太大了。按我的粗糙印象 这类似于美军在世界军事格局上的地位。其他的表现 比如吸引全世界最好学生从而拥有最好研究生院等 都是建立在这一基础上的。 (虽然我提过美国大学本科生源的严重问题,但最顶尖大学的还不错 而且作为象牙塔里的东西 基础科学的研究拼的是教授和研究生生源(不限于最顶尖学校)。)

不错 随着美国经济地位的下降 它维持这一地位的能力会下降。但是这并非一个突变的过程(假定不爆发世界大战或者苏联式经济社会崩溃)。 除了美国的经济地位在中短期内大概不会暴跌式下滑外,具体到学界 有三个特殊因素是很值得注意的:1 除了中国 美国在经济上没有威胁大的赶超型的竞争者(欧洲日本经济更惨),而目前中国仍未在本土攒出哪怕一个 理科世界一流的大学;2 一个第一流学者成熟后往往有 30 年 “一流水准保质期”,这是很难被政治经济条件改变的(当然做实验的人会面临经费问题)。 这意味着即使现在美国繁殖一流学者能力受到了严重打击 仅凭已有人才也能维持一些时日,更何况这件事还没发生;3 最顶尖的学校本身处在经济社会金字塔顶端 抗经济打击能力很强。

如果你认为中国有在 15 年内基础科学赶上美国的机会 不妨打开美国最强的 10 所大学查看相关领域的 50 岁以下非华人学者的名单。15 年后这些人极有可能仍在美国。然后你再想想中国如何能在 15 年内制造出一个超出这些人的群体来。 我的看法是:即使所有华人立马回国 中国一流学生也不再赴美留学 也未必做得到。原因很简单:时间不够 来不及繁殖那么多国际一流学者。

美国获得第一流地位的 “经验” 对中国借鉴意义不大

那么美国是如何做到这么强的呢?我的看法是

1 美国的人口对任何其他发达工业国都具有压倒优势。从科学圈的角度看 这人口不仅指美国的三亿人 也包括加澳两国和部分英国的。

2 二战导致了人类近现代史上最大规模的科学人才转移。转移目的地是美国。其实虽然美国在 19 世纪末就是第一工业国 直到 20 世纪 30 年代初世界科学中心仍在欧洲 特别是受一战重创的德国法国。然而仅用短短 10 多年 希特勒就重画了世界科学版图。我愿举一个很有代表性的例子。国际数学家大会是 4 年一次的全世界所有分支数学均参与的大会,具有很广的代表性(其他学科没有这么全面的大规模的全行业大集会)。 1932 年国际数学家大会 20 个全会报告只有两个是用英语作的(读者可以由此想想当时美英在国际学术界的地位)。 之后希特勒上台,二战来临, 在 1936 之后就 14 年没开会。 到再下一次(1950)时, 22 个全会报告有 20 个是用英语作的,英语也从此有了国际学界的绝对统治地位。

3 苏联解体导致了人类近现代史上第二大规模的科学人才转移。苏联人才转移对生化可能影响小一些, 但对数理科学来说可是极为壮观:这是从 20 出头的天才大学生到 80 多岁的泰山北斗的全体系的迁移。转移的人少数被欧盟吃了,大部分被美国吞了。

4 在美国的基础科学 于二战后取得西方阵营的独大地位 以及冷战后取得全世界的独大地位后 有能力输出第一流后备人才的国家(如中印等)的一流学生蜂拥而至 更使美国如虎添翼。

其他的因素(比如很多人津津乐道的 “体制”“创新精神”“自由” 等)即使是有利因素 其影响力也和上述因素不在一个量级上。

对中国而言:因素 1 天然具备,在经济赶上来自然就会发挥作用;因素 2 3 可遇而不可求(多半是不可遇);因素 4 属于已经成第一科学强国后的锦上添花,对如何做到第一没什么用。

美国大学的基础科学体制宏观上是一个容易学的顺其自然的体制

虽然美国获得第一流地位的 “经验” 对中国借鉴意义不大,但仍可考察一下 其维持第一流地位的 “经验”。比如说大家都爱谈的 “体制问题”。

在我看来 美国的基础科学体制 在最宏观的层面上 其实很简单:用大把的钱 养大把的 “学霸”(对大学者的昵称),学痴,顶尖学生; 然后让这些人 自己折腾 自己治理 就行了。中国有了大把的钱后也可以学 当然学了以后还有等足够的时间才能见效。

现在人们抱怨的种种中国学界问题 基本都可以用这套体制来化解。缺乏创新精神? “学霸”和顶尖学生就是创新的标杆;学术抄袭腐败?“学霸”,学痴等很少会这样 对此容忍度也很低;心思不在学术上?在中产之后还心思不在学术上的人 会被边缘化;学术上近亲繁殖,派系山头斗争?嘿嘿,“学霸”/ 大学者 基本都是有态度的人 因为学术上总要面临有所为有所不为的问题。为了贯彻自己的学术判断,“学霸”分派系抢话语权是常态。那美国学界为啥没乱套呢?学霸太多 自然地就形成制衡了,派系之间的 “多不管” 地区也很多 为标新立异的人提供了空间。

顺便说一句:我认为真正最难学的是苏联体制,而苏联体制的效率明显高于美国体制(至少在数理科学上是这样的)。 不过 这需要另开一帖才能谈了。

是不是说 中国从美国除了最简单的烧钱 “顺其自然” 外 就没什么可学的了呢?不然。 美国体制的一些具体举措仍有可借鉴之处。

待续

美国体制的一些具体举措有可借鉴之处(举例)

我举例讲一个我觉得值得借鉴的东西:tenure track 体系(终身教职轨道体系)。 研究性大学选择自己认为比较有前途的 做了一段时间博士后的青年学者 将其置于此终身教职轨道上考察数年。 期间较好地保障其研究条件。数年后若发现此人确实不错则委任为终身教授 若觉得没有那么优秀则让其走人(即所谓的 “不升即走”)。这是一种竞争性很强的制度,对最强者很有利(快速上升 有效地避免论资排辈),对次强者有一定风险但也可以强烈激发其动力。

对中国来说 这种制度不仅有利于增强国内学术环境里 一直偏弱的学者的斗志, 而且也是中国在有钱后同美国抢夺一流人才的需要(毕竟绝大多数海外华人科学人才是在美国)。10 年前我就听说过有引进此种制度的传言 但可能因为客观条件所限(钱少?)没有施行。最近几年,几所国内最好大学开始在基础科学院系 逐步开始使用这套 高门槛高风险高回报的制度。 据我所知 这在海外一流青年学者中引起了很大关注 很多人都动心了,不少已决定回国或正在筹划回国。。。对这种 “高门槛高风险高回报” 而且显然对非海龟不利的制度 当然有不少不同意见(科学网前不久曾集中讨论过这话题)但看来上层下了决心要和美国正面直接竞争。

美国理科基础教育质量差而且积重难返

我前面的几篇已描述了 我认为美国理科基础教育质量差的理由 这里就不再多说了。那么有没有办法改变呢?在 D 大学让我大跌眼镜后 我决定做一下调查。当然很快我发现早有受不了了的大学教授作了调查并常年奔走呼号。(Berkeley 的伍鸿熙就是这样的一个热心人, 他收集了大量资料和例证链接出处 教育爱好者可以去看看)

总的说来 我认为明显改善的希望很小。

1 中小学教师素质太弱 本身也是糟糕理科基础教育的受害者

比如伍鸿熙和他的同事调查了很多中小学数学老师,竟然罕有人能解释 类似于 “除以一个分数等于乘以它的倒数” 这类简单事实的原因!(可惜我一时找不到有多高比例教师做不到这些事的原始数据,不过绝对是骇人听闻的高)。最后 作调查的大学教授们发现 中小学数学老师在上中小学时 他们的数学老师就未能给他们解释 这些简单事实的原因。等他们长大了,逐渐见怪不怪,也能顺利做计算, 可是他们始终没有理解为什么!他们虽然上了大学 但往往是大学里学得较吃力的学生 被微积分线性代数折磨得够呛, 而且最重要的是:大学也不会教这些东西呀。

这时我就深刻感觉到了中国这套师范学校体系的重要性。

2 教材和教学标准体系有严重质量问题

看了愤怒的大学教授和一些家长的控诉后,我才了解到原来很多流行的数学教科书 内容有严重漏洞,歧义或错误。感兴趣的河友可以去看上面链接中伍鸿熙找出来的各种奇葩例子。我只半剧透一个。一些美国学生不懂分数的乘法:为啥 2/3 * 1/5=2/15? 就是不懂!甚至还专门造了一个词(fraction phobia– 分数恐惧症)来描述这种不懂的恐惧。 (中国人想不通)这事儿怎麽就能这么难呢?美国的教科书害的。。。

教科书的问题其实是教学大纲和教学指导思想的产物。那么教学大纲和教学指导思想出了啥问题?大致说来美国有两派。第一派由中小学教师和教育学专家组成 主张初等教育要实行 “启发式”“探索式” 教学 多从实际从动手出发 用计算器等代替传统计算教学等;第二派由大学教师和一些家长组成 主张传统教学法 强调记忆和练习 强调严格的书本知识学习。两派争执不下 矛盾公开化 在 90 年代曾酿成著名的“数学战争”(链接出处)。从现在的情况看 第二派在掌握教育主导权上失败了。前述的问题是在第一派的思想下产生的。

3 教育改革阻力极大

那么美国能不能回归一些传统 或者直接向中国之类的体系学习呢?不少人就是这么主张的(当然他们中的很多不说学中国 而说学新加坡之类)。但是这类改革会受到强大的教师工会阻挠。家长总是分散的,大学教授人数少 而且 外国人多 不接地气。 他们是无力和人数众多高度组织的中小学教师争权的。 更不用说 把握相当一部分话语权的教育学专家们往往是第一派。。。。。。

4 经济困难

宏观经济困难导致美国政府捉襟见肘。各利益集团都要争夺缩水的经费蛋糕。在金融军工医疗等巨人面前 公立教育是弱势。。。

5 种族因素

非洲裔和拉美裔的理科成绩弱 在好大学的理工科里所占比例极低。而他们占人口的比例在不断上升。。。

6 教育标准在全国范围内不统一

在联邦制下 地方有很大的自主权。 我询问过我的学生,感觉不同地区的学生学的课的难度 知识点 甚至顺序都会有很多不一样, 情形十分混乱。 更麻烦的是美国人迁移性大。 我听说有很多孩子在搬家后 会出现学习的课程无法衔接的麻烦。我甚至知道有的人在中学里没学过物理(连牛顿三定律都没学过)。。。

7 反智主义的干扰

有至少两类反智主义。一类基于保守宗教思想。 这类思想对知识分子没大的影响力 但有一定群众基础, 有民粹式的政治影响力。其对某些地区出身的人 有洗脑作用。比如曾有学生告诉我 他因为对上帝的信仰和家庭教育因素 长期以来对学习物理和数学有难以克服的感情排斥性。。。

另一类则以 相对主义 多元化 极端环保主义 女性主义等面貌出现 或者表现为 一些受过良好高等教育但理科学得不好的人 质疑学习理科的必要性。这些人虽不如第一种多 但往往更难缠。我可以举一例管中窥豹。纽约时报曾刊登一文链接出处鼓吹 “数学作为必修课 会阻挠我们发现和发展年轻天才”(Making mathematics mandatory prevents us from discovering and developing young talent.)并建议 大多数人不用学中学的代数了。注意,这是中学的代数:是教你用 xyz 代表变量 教你了解多项式 教你解一元一次方程的代数课! 他的提议固然受到了多数读者的抨击 但值得注意的是也有相当一些人支持他。。。

中国的这方面的情况要好一些。 第一类在轮子被镇压后没有什么引起我注意的迹象;第二类呢?嘿嘿 我觉得中国公知的脑残度是不输于美国的。好在他们的几乎全部注意力一直放在 TG 身上。。。

美国的理科基础教育质量差 是美国人自己也意识到了的一大问题 各种抱怨充斥媒体 但始终不见什么好办法。 我在做了一些调查后 也感到问题积重难返 解决希望渺茫。

待续

美国的基础科学研究高度依赖外国人才输入

这是大家都熟知的事实。那么依赖程度有多高呢?最好的办法是看各学科的行业学会的年度报告。我查看了美国数学会的多年报告 发现美国的数学博士中 出生在美国的 所占比例常年维持在四成多。另外在其所统计的美国顶级大学数学系高引用率学者中 外国人占六成。所以在数学界可以说至少半壁江山靠外国人。其他的学科也许会略有不同 但美国的基础科学研究高度依赖外国人才输入 应该是个准确的判断。

美国的这种情况是非常奇特而且前所未有的。历史上曾出现过的其他顶级科学强国(老欧洲三强英法德以及后来的苏联) 在巅峰期 其科学事业都是基本上完全由本族人支撑起来的。

基础科学研究高度依赖外国人才输入 是一把双刃剑。

先说其好处:1 它使美国科学在世界上所占份额 大大超出其经济总量占世界的份额(美国 GDP 约为世界的两成多 但顶级学者显然不只这一比例)。2 它使美国的潜在竞争者追赶美国的难度大增。

但它也有明显的风险:当国家相对衰落到一定程度后 可能会导致科学水平大幅度下滑 难以坚守住一个高位。

我仔细解释一下。

以数理科学为例。外国人才的最大两个来源是 前苏联人(或者稍广一点 – 前华约人) 和中国。苏联(和华约)靠着具有惊人培养人才效率的苏联体制 在冷战结束前夕积累了群星璀璨后浪推前浪的 全年龄段顶尖人才梯队。苏联崩溃后 人才梯队几乎全体系地离开苏联 多数去了美国。此后苏联地区的优质科学研究和教育体系基本崩溃。现如今不仅在一流学术竞争方面基本出局(Perelman 等极个别人例外, 而且 Perelman 是脱离学术体系的异类) 前苏联地区甚至连向美国输出一流后备人才(顶尖大学博士生)的能力都已非常薄弱(我留意过好几年 B 的研究生来源,我想不起一例 B 大学从前苏联地区的大学招研究生的例子而同期 B 从中国招了一大批学生)。简单的说 解体后的苏联不仅金蛋被抢光 会下金蛋的母鸡也被杀死了。

苏联解体 20 年了。未来 15 年内美国从苏联获得的人才战利品将过期而且无法补充。

中国的情况则是最有意思的。自中国对外开放以来 中国理科的最顶尖学生就持续不断的流向美国。这一方面强化了美国的领先地位 另一方面导致中国长期失血 始终难以完成本土一流人才的原始积累。可以说中国近二三十年的状态对美国维持其地位来说是非常理想的:中国基础和本科教育足够强可以大规模长期为美国输入后备人才 但研究生教育和基础研究足够弱以至于无法大规模吸引顶尖人才回流。

然而中国的很多事业都有这样的特点:格外漫长, 挫折失误连连, 长期看不到希望(或者说只能将希望寄托于人多), 眼看着别人有很多 “捷径” 但学不到手, 最后走了一条异常艰苦一个台阶都绕不开的路,然后逐渐收获果实并能望见难以限量之光明前途 但很多人还没意识到最难的阶段已过去。。。

我感觉中国的基础科学研究大概也是这个路子:顶尖理科人才的长期外流 纯粹从国家角度看,在三十年尺度上是吃大亏的事,但从五六十年尺度看却可能是最终盈大利的。中国通过美国的体系在海外顺利完成了一流人才原始积累和建设全年龄段人才梯队。由于中国经济实力的急速提高,美国的令本国人民和精英都始料不及的地位下滑 以及中国青年有了美国可被中国 “取而代之” 的“皇帝轮流做之贼心”,最近几年来已经有 未来 10 年内还会有 成批第一流中国青年人才回流。在我看来 中国基础科学在本土完成一流人才原始积累 就如中国经济总量超过美国一样 是没有什么悬念的事。各种人们抱怨的国内不利因素 只会影响早几年还是晚几年实现的问题。当然从在本土完成一流人才原始积累到基础科学赶上美国还需要一段不短的时间(学术后代繁殖需要时间)。

一个中国顶尖学生 20 多岁到美国学习 30 多岁回国工作 这是一种新模式。六年前(08 美国金融危机前)这样的模式凤毛麟角, 但最近几年呈现滚雪球之势。我周边(包括我)有越来越多这样的人 而且相互影响相互呼应。这和更早时期的顶尖人才的 20 多岁到美国学习 之后在美定居的模式 大不同。 后者对中国也是很有意义的(回国讲学组织会议,保持华人在世界学术中心一席之地等)但对美国的直接贡献更大。而前者则显然对中国贡献远大于对美国的 而且做贡献的能力是在美国获得。如果说美国在一流基础科学人才这个领域从中国抽了 30 年血的话, 现在中国要开始反过来从美国抽血了。美国无法通过刻意压缩从中国招研究生阻止中国抽血:如果它这样做 则不仅会导致生源质量下降 而且会导致顶尖学生不得不在中国深造从而帮助中国更快的进行一流人才的繁殖。更可怕的是 中国学生在 21 世纪以来在美国后备科学人才中占的比例似乎越来越高 近年来更是迅速扩散到本科阶段(我亲身体验似有爆发趋势 不知是否和美国高中生的理科水平进一步下滑有关)。这意味着中国的潜在抽血能力还在增强。。。

美国能不能找到弥补 “苏联空洞” 和“中国抽血”的办法呢? 由于本土理科教育质量积重难返 靠本国人是极为困难的。这件事和美国高度依赖外国人才输入的现状是相互促进的。 这事情有点类似于美国要将已经丢掉的制造业从海外拉回来一样 不靠谱。更麻烦的是,在制造业方面 美国总还可以将希望寄托于新的高科技,而在基础科学方面 按照定义 已经没有更 “高端” 的东西了。

依靠其他外国也不太可行。印度或有在未来向美国输入更多科学人才的能力 但我很难想象他能弥补 “苏联空洞” 加“中国抽血”。欧洲有人才 但只要法德荷比瑞等几个老欧洲核心经济不崩溃 美国无法在现有基础上多捞到多少。 非洲 大中东(以色列除外) 拉美 东南亚等 人口众多 但理科教育体系太弱 输出后备人才到美国一流学校搞基础科学的能力很差。

从更长远角度看 如果美国的经济地位持续下滑(哪怕仅仅相对于中国) 美国的白人人才也存在外流中国的可能。我以前觉得不大可能,但我近年来已经看到了一些国内最有钱学校用高薪招到美国中档藤校正教授水平的白人。顶尖学者由于职业特点 国际主义倾向较重 而且美国和西方已有一代人以上学者是在高度推崇全球化的大学环境中成长的。。。

日本在经济出现 “失去的 20 年后” 基本保住了本土的一流科学人才(当然这也说明了日本的封闭性)。在中美经济力量对比易位后的 20 年内 美国的科学界只怕“欲为日本而不得”。

教育问题显现效应有较长滞后期 美国精英可能准备不足

我以前提过 一个第一流学者成熟后往往有 30 年 “一流水准保质期”。另一方面 一个中国学生可能现在刚到美国来读书 离他回国可能还有 10 年 离他在国内开始有效的繁殖学术后代可能还有 15 年;这意味着虽然他最终属于“从美国抽血” 的人但这事有 10 几年滞后期。

从主流媒体看 美国的精英阶层 对教育问题显现效应有较长滞后期这件事似乎没有足够警觉性 至少没有警觉到认为 就算现在集中资源死保基础科学地位都有可能动手晚了点。不错 有很多指出美国基础教育差的声音, 但对于基础科研和高等教育(研究生教育)舆论似乎还是普遍很乐观 认为这是中国差距甚大美国不必太忧虑的领域。对于美国高度依赖外国人才 很多人单纯觉得这是美国的优势:外国人来说明我们厉害嘛。更有甚者 各种有浓重意识形态色彩的肤浅言论还大有市场 似乎喊几句中共体制压制自由就能推出中国创新赶不上美国。。。

美国例外论

根植于上述许多问题深处的精英思想根源 是 “美国例外论”。我认为 1 美国冷战后的经济地位与其人口比例高度不匹配 2 美国冷战后的基础科学地位与其经济比例不匹配(与其人口比例更是超高度不匹配)。 美国例外论者大概也同意这两点。

我们的分歧在于:我认为从长的历史尺度看 美国的上述地位的获得有很大的偶然性 而且是不可长时间持续的(即使美国没有犯大的错误),更不用说美国还犯了一些大错误;而例外论者认为美国是 “例外的” 从而即使相对地位有所削弱, 也必然是居第一位的领导者,不会被中国取代。很多诚实的例外论者也承认既成或几乎既成事实(比如从中国出口超美到中国制造业超美到中国经济超美),但是他们往往在下一个里程碑处继续持 “例外论”。

结语

上述历史观的差异是很重要的。对很多美国精英来说 美国的基础科学和高等教育 具有中国在值得预测的未来没有机会哪怕接近的地位 从而不仅成为他们自信的源泉 而且可能为其他领域(如制造业)提供美国 “王者回归” 的机会。对我来说 在基础科学和理科高等教育方面 “中国速胜论”和 “中国无法超美论” 都站不住脚 但中国在跨越我一生主要学术期的未来 30-40 年的 “持久战” 颇有胜算。在认识到这一点之后 我就知道我一定要回国:不仅因为我是中国人 也因为我是希望参与到新的世界科学中心形成进程中去的“国际人”。另一方面 我衷心希望美国人能平稳过渡到世界第二 并和欧洲等力量一起实现基础科学不过度集中于一国(中国)。这种长期存在的平衡和竞争力量对中国和人类的基础科学的长远发展都会是有利的,有长久历史经验积累的中国人当能体会。

全文完

原文转载自西西河 changshou,原文地址:美国求学执教的见闻和感受(0)

知乎用户 陈杰 发表

因为人家学的是现代数学,我们学的是古典数学。

计算机图形学其实一点都不难。我们国家的教材比计算机图形学难多了,只是和计算机图形学不兼容而已。

和我小时候(80 后)相比,现在的小学教材已经开始讲坐标系了,虽然火候还不太够。

但是现在初中教材和高中教材一下子又把数学拉回了尺规作图的年代。

所有的中学几何,是完全可以用坐标系,向量这些现代概念彻底重新解释一遍的,而且现代概念可以直接转换为计算机程序,直接转换为生产力。

我有幸早年就体会到,数学书里证明和计算题里常用的一般式,和真正能画出图形的参数方程,完全就是用大脑的两个不同的区域在思考。从初中的直线方程,一元二次方程,高中的圆锥曲线,都是如此。

我高中就发现了,向量,复数,参数方程,这些教材中几乎一笔带过,高考几乎从来不考的内容,实际上才是能真正能经常应用到计算机程序里的内容。

我大学时发现,矩阵,四元数其实根本就不应该放到大学再学,这些工具简单实用,而且和最新的 CPU 指令集兼容。

我们初高中时浪费了很多精力在无用的几何证明题上。它给你引入了一个特定的体系,一个逻辑自洽的系统,一个空中花园,它企图让你在这个花园里做头脑体操,来锻炼逻辑思维。然而,这整个系统只是一个老掉牙的系统,你学会的这些体操知识只是屠龙之技。

我们 80 后在学习鸡兔同笼解方程的时候,美国的同龄人(至少是一部分有条件的同龄人)在学习 BASIC,学习 LOGO 海龟画图,let x=320, let y=240,同样是小学难度的知识,我们在学骑马射箭,他们在学习火药大炮。

现在,2022 年,我们小学生还是在学习鸡兔同笼解方程,人家在学 Python 了。Python 这个语言,把向量、元组、集合之类的现代数学和计算机理念体现在基本语法里了,a=v2(320,240), b=a+(100,100),就好像我们在学骑马射箭,他们已经在学习飞机火箭了。


应该说,我们这边不是学不到 Basic,不是学不到 Python,但是制度不鼓励,而且会惩罚有这些奇思妙想的人。


至于说学习几何证明是用来锻炼逻辑思维,似乎没人否认编程也是同样可以锻炼逻辑思维,而且编程要更加实用一些。


能在初高中普及现代数学当然是最好的,或者退一步,让内卷的中高考不要拖现代数学的后腿也可以。


认为中小学数学的作用是选拔而不是实用的人,就不要参与讨论了。经院哲学,八股取士,要么没落,要么变革。我们希望解放和发展生产力,我们认为学习现代数学可以适应现代的生产力工具,满足现代的生产力需求,你不认可这些,自有认可这些的人会去做。

知乎用户 MichaelHe 发表

高赞答案搬运得不完整,而且答主还被封号了,我再搬运一个完整的(答主自己加的卖萌图就算了)。

客观来说,中国在 IMO 的历史成绩是全球最好的(见下文的 IMO 官网)。第一次参赛的 1985 年,第 32 名;1986 年,第 4 名;1987 年,第 8 名;此后历年中,只有 1996 年是第 6 名,1998 年未参赛,2016 年第 3 名,其他年份(共 27 年)从未跌出过前 2 名。作者写此文是因为 2016 年罕见地跌到第 3 名,到 2017 年就又回到第 2 名了。不过,不得不注意到,2010 年以来,第 2 名及第 3 名的出现频率已经达到了一半的年份,而在 2000-2009 年的 10 年中,只有 2 年是第 2 名。中国的 IMO 实力仍然强大,但有所下降。

这种水平甚至略强于当年的苏联。近年来美国的实力有所增强,多次获得冠亚军。俄罗斯的实力则明显下滑,跌出了前三名的范围。韩国是 2017 年的冠军,近几年来突飞猛进,基本已经进入了前三强。越南 2017 年得了季军。

====================================

从首个 IMO 季军谈起(极品好文,第三段到第六段完美回答了这个问题)

http://blog.renren.com/blog/229607718/970519550

作者 : 付云皓,02,03 两年 IMO 金牌,满分,中国国家队中的战斗机,中学数学竞赛史上最强战力之一

刚刚过去的 IMO(本文写于 2016 年 7 月 16 日),中国史无前例地获得了第三名,也是自 1997 年来近 20 年首次跌出前二。感谢微信等社交软件,相信现在这个新闻已经以火箭的速度传播了。

作为一个与数学竞赛及 IMO 打了多年交道的人,我一直有写点什么东西的冲动,但一直由于懒癌拖拖拖。赶上此时此事,我觉得不能再拖了,写一点感受吧。

首先插个链接:http://www.imo-official.org,这是 IMO 官方网站,上面记录了历届 IMO 的数据,只要你会点英语都能看下来。以下所有有关 IMO 的官方数据均出自此处,如有数据错误,请指出。

====================================

Part 1 曾经的霸主

中国的奥数强不强?是不是梦之队?

五年前你问我,我会很干脆地回答就是梦之队,但现在你问我,我会说,强,但不是梦之队。

圈内普遍认为,中国在 IMO 上大放异彩大约就是 2000-2010 这十年(实际是十一年)。20 世纪 90 年代吧,虽然也是强队,但有一次并列第一,两次第二,一次第六,还有一次未参赛(当然,未参赛不能怪中国队,细节涉及黑历史,略),银牌铜牌还是不少的。

在官网上看 2000-2010 中国的数据,11 次参赛 66 人次,61 金 5 银,9 次第一 2 次第二,绝对亮瞎眼。61 金 5 银什么概念?就是说如果你不是中国队里最弱的两个之一,那么你肯定是金牌,即便是最弱的两个之一,也有大约四分之三的概率拿金牌。

But 你以为这就是全部了?大错特错。

2004 年,某队承认自己前一年 “played something unfair”(明白人都懂不再解释)。

2007 年,中国在集训队淘汰了两个大 BOSS,加之当年题目奇葩(可以搜那年的数据,真心坑),以及主办方越南的双标(甚至对中国队采取了笔误扣分),中国落后俄罗斯 3 分屈居亚军。

把这些数据修正一下,再看看中国队在 2000-2010 的数据,用神队形容也不为过。

不仅如此,官方公布的中国队 9 次第一中,除 2000 年赢第二名 3 分,还有 3 年赢 8-9 分之外,其余 5 年均赢接近 20 分或 20 分以上,最夸张的 2006 年竟然赢第二名 40 分!也就是说除了柳大师之外随便找一个人交白卷,中国还是第一!

这不是虐杀十条街么…………

那么下面一个问题,2000-2010 的中国队到底领先其他队伍多少距离?

一个数据不说明问题,一组数据光看也不说明问题,分析完再说。

**每一年情况不同,各队选人就不一样,题也不一样,临场发挥还是不一样,尤其在这种各比各的最后汇总成绩的比赛中,随机性真的很高。**但是看看中国队的数据就能发现中国称霸的秘诀:强,而且稳定。

由于各年题目难度不同,我们以金牌分数线 * 6 来作为标准分数,看看每个国家和标准分数的差距(考虑到 2007 年这种神年份的难度被金牌分数线误判,个人将 2007 年的金牌线修正到 26 分参与计算)。

平均实力最强的中、俄、美相关数据如下:

中国队每年总分均超出标准分至少 25 分,超出部分的平均值约为 34,标准差大约为 7。

俄罗斯除 2003 年外均超出标准分,超出部分平均值为 14,标准差大约为 12。

美国在标准分上下波动,超出部分平均值为 3.5,标准差大约为 10。

如此可以看出,中国队超出的成绩平均值不但远高于俄罗斯和美国,标准差也小。以正态分布模型来计算,中国输给俄罗斯的概率也就是百分之六、七,输给美国的概率不超过百分之三,至于其它国家?那都是小概率事件。

中国队为何如此所向披靡?且听下回分解。(TBC)

====================================

Part 2 取胜的关键

中国队有如此稳定的成绩,秘诀是什么?

中国队没有常青树类型的人物,像 Halls of Fame 里的 Raid Barton, Christian Reiher, Iurie Boreico, Lisa Sauermann, Alex Song 这样挥挥手 N 块金牌,荣誉一大堆的选手,中国一个都没有。事实上,还没有任何一名选手代表中国队参赛 3 次(但有人进过 3 次国家队)。

道理也挺简单的,中国在 2000 年之前就有了系统的选拔,而且 2010 年之前冬令营只有 100 多人,也就是说你必须在联赛中进入全省最前的几名才能进冬令营,更别说集训队只有 30 人左右,随便一处失误都有可能葬送了你。一个初中生要想冲进国家队,简直是难如登天。2010 年之后的问题,后面再讲。

那么,中国队要怎样才能拿第一?

这个问题,我问过奥数界的泰山北斗裘宗沪老师,时间是 2002 年我入选国家队之后。

(顺带一提,2002 年的 IMO 中国国家队,不管是我们自己还是老师们,都觉得是一支烂队。裘老师当时给我们下的目标是 3 金 2 银 1 铜,总分前三。)

裘老师当时回答了三点:

1:1,2,4,5 四个题不丢分或少丢分;

2:3,6 题对选手的胃口;

3:第五,第六名比其它国家的第五,第六名强。

第一条对应稳定,第二条对应运气,第三条则对应短板原则。

在比赛场上,我们完美贯彻了这三点。当年的俄罗斯是一支强队,加上 2,5 不难,我们在 1,2,4,5 虽然基本没丢分,但并不处于优势。但是,运气站在了我们这一边,比赛中出现了第 3 题这样的要通过代数辅助的数论题(这正是我们的强项),我们在这一题中拿到了全场最高的 24 分,与此同时,我们的五六名(各 30 分)强于俄罗斯的五六名(各 29 分),终于使我们有惊无险地拿下了总分第一。

纵观这 11 年的数据,基本都符合第一点,第二点么需要仔细翻题目,不过很多数论与代数结合的 3,6,我们都做的不差。关于第三点则有很多经典的例子,例如 2004 年四个满分没有一个是中国的,但中国队的最低分 34 仍处于并列第 28 名,总分第一(顺带一提,总分第二的美国队同样没有满分,最低分 31 仍然不低,第三名俄罗斯虽然有两个满分,但有两个人分数太低,无力回天)。与此相似的还有 2009 年,日本异军突起,在史上第二难的蚱蜢题里拿到了不可思议的 19 分(此题参赛 565 人只有 25 人得分,所有人加在一起还没得到 100 分!!),但一个 23 分使他们最终还是败在了前五题铁板一块的中国队手下。可以看到,2009 年若将中国的韦教主换成一个 35 分,中国还是会赢,但若将日本队最后的 23 分换成一个金牌分数,中国队就危险了(这里稍微透露一下,在稳居第一,且没有处于金银牌分界线的选手的前提下,队伍一般在协调时不会死皮赖脸地争分,所以事实上中国赢日本不止 9 分)。这也是短板原则所在——毕竟一个满分也就比 35 分多 7 分,而一个低分可能拖累远不止 7 分。

道理都懂,为啥其它国家不能效仿呢?

第一条其实处于前列的国家都在效仿,美国和俄罗斯在 1245 题发挥也不错。第二条纯属运气,无法操控。我认为,数次帮助中国队夺冠,其他队伍难以效仿的,是第三条。

美国队前任领队冯祖鸣老师自小在中国长大,与中国队交流也较多。有几次问过他当年美国国家队的学生水平如何,他经常会说:“今年只有 4 个学生能稳做 4 道题,很悬” 什么的。

“稳做 4 道题”即基本能完成 1245 的水平,也是我们所谓 “高手” 的水平。稳做 4 道题实际意味着学生成绩的期望至少是 5 道题,如果六个学生都是这个水平,那么总成绩期望将至少是 210 分以上。在 2000-2010 年中的大部分年份,中国队所有队员均能保持这个水平。

相对来说,其它国家的队员就难以达到这样的水平了,总会有那么几个差的。即便是 2007 年惜败俄罗斯的时候,中国的最后两名分数也高于俄罗斯最后两名的分数。

这件事在一定程度上也是无可奈何的。尽管训练和选拔机制比中国完善的早,但是毕竟基数有差距,从一两千万高中生里选拔 6 个高手总比两三百万高中生里选拔 6 个高手容易吧。

**中国数竞人就这样维持着自己的优势。**但是,在 2010 年之后(或者说事实上是从 2010 年左右开始),IMO 这个比赛从题目本身到对手到中国队自己,都产生了一些变化。

(网上应该有很多谈论中国自身问题的,于是我们把这一部分向后放一放。)

IMO 的游戏规则从很早以前就没怎么变过,那为什么题目本身会有变化呢?且听下回分解。(TBC)

====================================

Part 3 规则的变化

说到题目的变化,首先就得说说 IMO 的题目是怎么选出来的了。

IMO 的题目来自于全世界,比赛当年 3-4 月,各个国家或地区被允许以领队的名义向主办国家(或地区)发送题目,**一般来说每个国家最多发 6 道题,都是严格保密的新题,且自认为优质的题目。**这些题目一般有 100 多道,称为 Longlist。主办国家(或地区)在收到题目并整理后,组建一支选题委员会(Problem Selection Committee),**选题委员会的工作是在这些题目中选出真正优质且全新的题目并加以分类,一般来说初等代数、组合数学、平面几何、初等数论四个领域各 6-9 道题,总计约 30 道题左右,并将每个领域的题目按由易到难的顺序排序。**被选出来的这大约 30 道题,称为预选题(Shortlist)。**在 IMO 考试前几天,各国的领队及观察员 A 提前集中,拿到这些题目并进行品评,然后投票选出 6 道题作为考试题,同时要求每天的 3 道题分属不同领域,且每个领域至少有 1 道题。**在投票过程中,每个队伍(领队 + 观察员 A)有一票。

从 Longlist 到 Shortlist 的过程虽然也挺有意思的,但是与主题无关,这里略去。主要来看从 Shortlist 到比赛题的过程。

在这个规则下,强队所需要关心的不太多,像中国重点关心也就两点。

第一,25 是不是难度恰好,能够让自己的学生基本全拿分,别人的学生拿不全,从而拉开差距;

第二:36 是不是能碰上一个代数题或者是用代数较多的数论 / 组合题。

对于 2000-2010 年的很多年份,甚至这些都不需要,所以中国队基本不太 Care,最多控制一下题目难度顺序别错了。

但是对于一些小国家那可就不一样了。

如果你是一个小国家的领队,那你该怎么办?

其实这个问题从更早就开始了,小国家怎么训练和选拔学生?

什么?你不知道?那回过头看看现在那些 “偏弱” 的,要冲联赛一等奖的学生怎么准备二试吧。

没错,就是狂刷平面几何,最多带点简单数论和三元对称不等式什么的。

国外的天不一定比国内的蓝,很多高手觉得无聊的中国快餐式奥数选手培训流程,被很多小国家一直在模仿,且从未被超越。

那么,当你领着 6 个基本只会做几何(而且还做不了太难的几何题)的选手进入 IMO 的时候,你想要考试出什么题?

首先,你得选几个你的学生能拿分的题。

于是乎……

我们要几何!我们要最简单的几何!我们要 G1 和 G2!(即 Shortlist 中最容易的两个几何,G 即为 Geometry 的简称,后面的 A,C,N 同理)

于是,很多年份中的 G1 和 G2 全被抽中,G2 会被放到 2,5 之一,而它的难度往往连联赛几何题难度还不如!但是没办法啊, 更 DT 的是小国家一出来就是二三十个,你强国也就一票,只能保留意见。

当然作为小国领队,弄到两个容易的几何还不够,因为你知道强队终究会靠 2 和 5 中的另一个,以及 36 与你的学生拉开差距。

怎么办?怎么办?

于是乎……

KAO!老子豁出去了!

14 不是还有一个位置吗?我们投最简单的!A1!N1!咱们六个学生好歹也是练过的,总能蒙一两个出来吧。36 题我们投最难的!我们要 A8!C8!N8!你们不是牛 13 吗!难死你们!

25 还剩一个?没辙了。算了,我们泱泱小国,送你们了,让你们赢去。

…………

………… 呵呵

当然,组委会不会任由这些人乱来,领队一般也不会做得太过分,不过最终结果经常是有两个简单的几何,另有一个超容易的题,然后有一个超难的题。

2000-2010 年的大部分年份都有这个特点,除 2005 年的 36 都很容易之外,其余年份都有一个或两个题很难(神年份 2007 年的 36 都超级难,但我认为 6 是领队们误判题目难度所致,因为 6 看解答并不困难,与此相关的内容后面有详述。)

这个困境总得解决啊,不然 IMO 成套路了玩个毛啊。

于是,我们的 Problem Selection Committee 在 2011 年玩了把大的。

相信当小国的领队们拿到 2011 年的 Shortlist 时,他们的脸上一定是一副 “囧” 的表情。

看看 G1,让你证明两圆相交,我们学生没训练过啊,逗我玩?
看看 G2,让你证明几何恒等式,我们学生没训练过啊,逗我玩?
看看 G3,一个四边形………… 卧槽这是 G3 吗,怎么那么复杂,我图都画不出来,逗我玩?
看看 G4,……,卧槽居然到 G4 了!
怎么办?怎么办?
看来没有一个几何能让自己学生拿到分了。
KAO!老子豁出去了!我们投 G8!
还可以投一个几何,但是貌似选前几个哪一个都会吃大亏…… 哎哎哎,组合题里居然有一个组合几何!还是 C3!太好了!我们投 C3!
结果……C3 和 G8,惨不忍睹。

(另有一说法,C3 的题目叙述有风车,与主办方荷兰很配,于是 Problem Selection Committee 设了这个局,使得 C3 能够入选,另外当年投票环节也出了一些小问题。)

组委会发现这样也不行啊,只换来一场闹剧,于是在 2012 年调整一年之后,开始出现了一个新的潜规则:1245 四道题里必须每个领域一道题。2013 年到 2016 年的四年均遵循了这样的规则。

在 2013-2016 四年中,除 2015 年外,其余三年的 25 都属于正常难度(2015 的第 5 题偏难了,可能是因为泰国搞了大飞机,在第一天考试结束时把原来第二天的题目误当成第一天的题目发给了考场外的副领队们,结果又重新投了一遍题目的缘故)。容易看出,新规则下更考验学生四个领域的平衡性,有一个领域有短板就很容易拿不到四个题。

而另外两个难题呢?很不幸,它们仍然被掌控着。平面几何和组合更容易有那种很难,但同时很漂亮,解答还能看的难题,所以 2013-2016 的 36 无一例外是几何 + 组合(个人认为今年第 3 题实际上应该算几何或者组合,数论的东西用的很少,都是平凡的)的搭配。

IMO 的规则在慢慢变化,但光靠这点还不足以让中国以往拥有的巨大优势土崩瓦解。那么,美国队迎头赶上,连续两次获得团体第一还有什么原因呢?这就得说说美国队的训练和选拔方式了。且听下回分解。(TBC)

====================================

Part 4 真实的美国

“美国人数学很差。”

这是 99% 中国人的感觉。

真实情况呢?

美国的基础教育中,数学的强度确实远弱于中国,但是美国的基础教育可是下午两三点就放学了,课后的辅导班层出不穷,顶尖的孩子学习数学的强度和深度绝对不弱于中国同层次的孩子。

“美国队拿 IMO 冠军全靠中国小孩,美国队 6 个人全是中国人或者华裔。”

这纯属无良的,想拉风的媒体的报导,如果你连这都信,那我写的东西就不用再看了。

真实情况呢?

自己去搜搜美国队小孩的名字,一般来说 2-3 个中国人或华裔就封顶了。

最近也爆出中国小孩 “转会” 到美国的风波,但是我们冷静下来想想,能 “转会” 的,要么应当是原来在中国考不进国家队的孩子,要么是由于政策或制度原因才去的其它国家吧。

2009 年在德国,我们的学生和加拿大队的关系很好,加拿大队的 Danny Shi 和 Robin Cheng 等人便是在小学或初中时从中国转学到加拿大读书的孩子,他们也坦然地说自己在中国考不进国家队。最后中国队最低分 35 分,Danny 考了 34 分,Robin 考了 29 分(金牌分数线 32 分)。

如果靠这些孩子能打败中国队,那不是更说明我们的训练和选拔机制不如人家么?

关于后一点,如果是从小呆在美国长大的,那教育与中国无关,如果半路出去的,去搜搜小山智丽吧(中国真的出现这个情况,只能说明我们的体制还有待完善之处)。

回到正题。先来看看美国队是怎么选拔和训练的吧。

美国中学生想参加 IMO,第一步要参加的比赛叫 AMC,AMC 分 8,10,12 三个年级,同时考试,试题有 25 道,限时 75 分钟,每道题都是 5 选 1 的选择题,答对得 6 分,答错得 0 分,不答得 1.5 分(这是鼓励不要乱蒙答案)。

AMC 的难度很低,即便是最难的 AMC12,前 5 题都是中国小学口算难度,6-10 题基本上中国高考选择都不会出,前 17-18 道题对于稍有中学竞赛经历的学生都不算题目,真正有点棘手的问题也都出现在 22 题以后。

AMC12 里得到 120 分以上,或名列所有参赛选手的前 5%,AMC10 里得到 135 分以上,或名列所有参赛选手的前 1%,即可参加下一阶段的比赛,称作 AIME。对于学过竞赛的学生来讲,这都不叫事。(此处数据是往年的,可能有些不准确)

AIME 是一个时间为 3 小时的比赛,15 道填空题,每道题的答案必然是 000-999 之间的数字,为此题目的问法有时会很怪(自己去看几道就明白了)。AIME 的难度开始增加,大约相当于中国联赛一试填空和解答之间的难度,并且计算量更甚,最后一两题将在此基础上更困难。将 AMC 和 AIME 的成绩综合起来,名列前茅的选手可以参加下一阶段的比赛,称作 USAMO(即对应中国的冬令营)。一般来说,AIME 做对 10-11 个题,加上 AMC 的底分足够让你进入 USAMO。不过在中国随便找一个省(不是最强的那种),可能只有几十人能达到这个标准。

AMC 和 AIME 的形式看似不太好,但是它却有一个无可比拟的优点,那就是阅卷容易。由于全部题目都是选择填空,而且填空是可以填答题卡的,所以阅卷成本和错误率都几乎为零。这样,可以保证一个真正优秀的数学竞赛生能够稳进 USAMO。

USAMO 参赛人数众多,一说有三四百人,跟现在的中国冬令营规模相同。USAMO 只考 6 个题,选拔几十人成为美国的集训队。到此为止好像和中国的制度没啥不同。

说到集训队,亮点终于来了!USAMO 是四月考,集训队再推后根本没有时间组队或办签证,但是老美早安排好了,他们的集训队是为下一年的国家队准备的!!

老美的集训队选出 8-10 个 “候补队员”,他们将与当年的国家队一同训练,当年 IMO 结束后,这些人将与国家队里还想继续做的人一起成为下一届的重点培养对象。从当年 IMO 结束到大概第二年 USAMO 结束这段时间,他们将进行大量集训,并四处参加比赛或自己来模拟测试,最后由领队综合各次的成绩选出 6 个最优秀的队员组成美国国家队并持续训练到下一届 IMO 开始。

这样的制度有两个优点,第一,真正优秀的学生只要在 USAMO 发挥好(或者说别发挥差)就可以了,后面都是大量集训,完全能考出真实成绩,也能看出一个学生的优点和缺点;第二,选拔之后的训练,强度也远超其它国家。相比老美的一年左右的训练时间,中国队不到四个月的训练时间显得相形见绌。(当然这套东西是不能搬到中国的,后面细讲)若说缺点也有一个,就是要求你高二必须考出来,高三去考 USAMO 是进不了国家队的(另一说是如果超级优秀可以考虑)。

**IMO 说到底还是个比赛,取得好成绩的方法从本质上来讲和大多数体育比赛并没有什么不同。**这里转一段别人的文章。我第一次看到这段话是在微信的 “Lens 杂志” 上看到的,深以为然。

“心理学家 Ericsson 的研究发现:决定伟大水平和一般水平的关键因素,既不是天赋,也不是经验,而是‘刻意练习’的程度。刻意练习是指为了提高绩效而被刻意设计出来的练习,它要求一个人离开自己的熟练和舒适区域,不断地依据方法去练习或提高。比如足球爱好者只不过是享受踢球的过程,普通的足球运动员只不过是例行惯事地训练和参加比赛,而顶尖的足球运动员却不断地发现现有能力的不足,并且不断以自己不舒服的方式挑战并练习高难度的动作。

换句话说,如果你的水平去 IMO 能平均做 4 个题,而你还盯着 IMO1245 难度的题做,那就是在舒适区,提高不会显著,你要做的应当是挑战 36 难度的题目。如果你的几何好,组合差,那么继续做几何题不会给你太大的帮助,你应该尝试着做你原来很怕的组合题。如果你已经是 IMO 保 5 题争 6 题的水平,你应该做的是继续挑战更难的题目。

老美在这一点上做得很好,因为他们在 2010 年前后引进了一个新的玩意儿——ELMO。

ELMO 是美国当年的国家队队员(Sophomores)出题来虐待新进来的候补队员(Freshmen)的一个练习,它还有 Shortlist。在我看来 ELMO 是一个 “反人类” 的东西,它的 Shortlist 远难于 IMO 的 Shortlist,如果一个中国学生能够做出 ELMO Shortlist 里 60% 以上的题目,那么他绝对可以在中国的数学竞赛里横着走。ELMO 和它的 Shortlist 就是地地道道的刻意练习了,正如中国举重队上台举 120 公斤练习举 140 公斤一样,到了国家代表队这个层面,思考超出你目前水平的题目会对你的水平大有帮助。与此同时,将要出征 IMO 的 Sophomores 还有一次命题的练习机会,对解题亦有不小的帮助。

前面说过,**美国顶端优生的数量比不过中国,但是美国的选拔制度却将这些人基本都筛了进来,而魔鬼般的训练体系则使他们比原来更厉害。**即便是 4 个 “高手” 和 2 个低一层次的 “准高手”,经过长时间刻意练习后,也有和中国 6 个“高手” 一战的实力,更不用说运气好的时候能凑齐 6 个 “高手” 的情况了。而这,也是美国能连续两届拿到 IMO 团体总分第一名的一个重要原因。

美国的训练和选拔机制有些 “反人类”,但它仍然长期存在并且得到了学生的认可,这又是为什么呢?且听下回分解。(TBC)

====================================

Part 5 环境的优势

本节涉及大量主观观点,不喜请屏蔽。

上一节最后提到了 “刻意练习”,我们在进入正题之前纠正两个相关的不当想法。

想法 1:在数学竞赛上 “刻意练习” 就是浪费时间浪费生命,就是“黄赌毒”,应该严厉打击。

上一节也说了,像 IMO 这种世界级的比赛,刻意练习是必须的。马上就是里约奥运会了,想想有多少参加奥运会想拿牌的选手没刻意练习过?恐怕一个也没有。若是练跑步打球游泳的人刻意练习是应该的,练数学竞赛的人刻意练习就是黄赌毒,那不就成了双标?

啊对,你说竞技体育有表演性质,体育运动员是兼职演员,所以得刻意练习。那我给你找个没表演性质的,或者外行无法欣赏的东西——围棋。照这个道理,那个李世石,别刻意练习了,浪费生命;那个柯洁,别刻意练习了,浪费生命;那个阿法狗,别………… 算了你生命无限,随便练随便练…………

抛开这些不谈,回到数学本身,别忘了想学好数学,多做难题也是必须的,不然吉米多维奇是干啥的?连数学本身都需要一定的刻意练习,就别说数学竞赛了

想法 2:既然 “刻意练习” 能出成绩,我不管是什么水平,就 “刻意练习” 好了。

练习要和能力匹配在竞技游戏中,你的能力(或者称之为等级)越高,刻意练习的绝对收益可能不变甚至更小,但是相对收益更大。

苏炳添一百米原来跑 10 秒 2,练习 N 年跑 9 秒 99,从亚洲级跑进世界级。

你一百米原来跑 15 秒,练习 N 年跑 14 秒,从街道级跑进…… 还是街道级。

明白了吗?

**只有顶级的选手,才配得上大量的刻意练习来提高能力。**以高联为例,一个省上万人参加高联,只有五六十个省一,如果你自问数学水平排不到省里前一两百,请把更多的时间花在高考或者其他事情上,把竞赛当作一种爱好,把高联当成一次自我检测或者一次娱乐。

这里第一次强调,如果学竞赛只是为了拿奖,那么感觉不适合或达不到拿奖的水平请立即退出。我不希望再看到家长哭着问:“为什么我家孩子学竞赛学了那么多年,连个奖都考不到?” 高中竞赛不是小学生过家家,它是残酷的淘汰赛,有的孩子连快餐式培训流程都接受不了,还想要拿奖,最终就是费钱费时间,还只学到了一些皮毛(以后也用不着的东西)。

回到正题。美国小孩为什么认可 “反人类” 的重复训练?我认为原因有三点,其一是有兴趣,其二是有保障,其三则是荣誉感。

美国的选拔制度,使得真正的天才选手不需花费太多时间在 AMC 和 AIME 的备考上,可以直接跳到备考 USAMO,这样前期基本不需要刷题。前期做得适度,后期就更容易坚持兴趣。而且老美的尖子生也少,真正的天才不需太多训练就能考出来,而达到一定高度之后再去刻意练习,会使学生最终的等级更高。在此也奉劝数学竞赛的天才和小天才们,到达顶峰之前少刷点题,多开发思维,多保持兴趣。

当然,哪个国家都不缺乏有兴趣的孩子,那么第二点就是老美独有别人基本学不来的:保障。

哪个国家高中还学竞赛的孩子都会想,我花了那么多时间做这个,甚至到后期就是反复训练,万一失败了,谁来保障我的权益?我去哪读大学?

老美做这个当然容易了,老美那么多名校,USAMO 就三四百人,大不了都收了去。换成其它任何一个国家,恐怕都做不到吧。

另外老美还有一个隐藏的优势,就是老美从 USAMO 开始(或者说从集训队开始)可以认为基本没有考试以外的奖惩。也就是说,进了这个小圈子,大家基本都是名校随便挑,考好考差都没所谓。这样,既可避免学生过于功利化地学习,让不是真心想在 IMO 上有所建树的孩子自动退出,还可以尽量避免不公平竞争(如作弊等)影响最终选拔的公正。

真正有兴趣的孩子,会在这个机制下,如同还没出名的竞技体育运动员一样,心无旁骛地主动刻意练习的。

关于荣誉感,先放上我与一位朋友在 2014 世界杯时探讨出的一个结论。

在竞技比赛或者任何事情上,一个人或一群人能做出傲人的成绩(排除天分),要么就是他们需要用成绩来改变自己的命运,要么就是他们有着极强的荣誉感和使命感。

例如,同样是踢足球,南美的运动员就是前者,欧洲的运动员就是后者。两者都不占的,必定踢不好。

不得不承认,老美从国家文化上就特别注重荣誉感和使命感。日本的 IMO 队员可以在头上围 “必胜” 字样的头巾,韩国的领队可以为了一分用尽方法,朝鲜人甚至可以不遵守规则,但他们比起老美来,可都差多了。老美的潜移默化的洗脑,是比前面那些都更加恐怖的存在,这一点想必很多人比我更加有体会,这里就不说了。

老美的选拔制度,高等教育条件和国家文化使得它的 “反人类” 体制可以继续下去。在整体的选拔与训练上,美国人已经领先了一大步。与此同时,中国的情况如何呢?且听下回分解。(TBC)

====================================

Part 6 拥挤的游戏

上一节讲到了美国现在数学竞赛上顶尖的小孩所处的状态,看起来说了不少老美的好话。我这里声明一下,我绝不是美分。

其实我在前面有不少伏笔了,相信跟我同时代或略早于我的数竞党都应该能猜到了,包括我在说 ELMO 的时候也用了 “引入” 这个词。

事实上,上一节讲到的美国小孩拥有的所有状态,兴趣,保障(高端无奖惩)和荣誉感,那都和我读高中时(或前后)中国小孩拥有的状态几乎一致!

2000 年前后,还有很多参加冬令营的同学在参赛前甚至不知道进了冬令营能保送,冬令营金银牌可以直接进清北(现在的同学别羡慕,那时候金银牌加一起也就六七十人)。

我们考完冬令营之后,郑志明塞给高分的学生一人一张纸,说这张纸就是北大录取通知书,那时候还是有一些人不信他的话。

那个时候学竞赛,真的靠的都是兴趣。

我上高中的时候有个很旧的文曲星,上面存了 2001,2002,2003 年部分国家集训队同学的名字及去向。那个时候大约三分之二的人去北大读数学,大约四分之一的人去清华读计算机或者基科班,剩下的人还有读物理的,出国的凤毛麟角。大家都是清北,高端无奖惩,全凭自愿。

关于荣誉感,那个时候多多少少都是有一些的,可能因为我们读高中的时候更单纯一点吧。包括拿了金牌回来还是能显摆显摆的。顺带提一下,我的两块牌分别是安妮和德仁颁发的,安妮看起来不太容易升级了,我现在就等明仁退位了……

包括到达一定高度后的刻意练习,中国早就有了。那个时候的国家队队员需要每人提供 20 道难题(当然不一定都是原创),然后互相考(所以我前面才用了 “引入”)。2001 年的时候我旁听国家队集训,被 6 个队员加上一堆教练虐了 20 多天之后水平一下子有了质的飞跃。这个 20 道难题有多难呢?我拿了一个我自己提供的比较难的组合题去考 2009 年的国家队,只有一个人做出来,还不是最强的那个。

当然,2003 年到现在也 13 年了,朝代更替,变化诸多。不过呢,除了大的政策之外,剩余的都是慢慢变化的。我个人认为,影响最大的一个变化,或者说根本的原因就是,参与的人变多了。

我们读高中的时候,真没多少人好好做竞赛。我记得那时北京市在高联结束后会组织冬令营培训,我高一的时候正式队员 6 个,旁听生算上我不超过 10 个,就一个小班,在人大附中每周末上一次课。都谁给我们上课呢?有周沛耕,陶晓永这样的老师,有邹瑾,韩嘉睿,袁新意等 IMO 金牌,后来肖梁毕业之后他也来讲过。这种课没多少来听的,只能说明竞赛这东西根本没多少人玩。

其它省份也差不多,好好学竞赛冲冬令营的人最多几十个,其他人大都打酱油。

那个时候有很多人高联只考一试不考二试的,因为考二试要多交十几块,当然这十几块只是顺便省下来的。

那个时候冲冬令营的人都是二试冬令营一起准备,基本不刷一试题的。

2001 年 10 月我参加高联的时候还考砸了,一试一个短轴算了半天填了个短半轴,二试一个组合题没完全写清楚。即便这样,我比北京的分数线还高几十分。很多同时代的同学都有类似的感受(2000 年高联除外,题目过于简单)。

一个省只有几十个人好好玩的年代,这么玩当然可以。但是一个省上千个人玩命准备的时候,这么玩就不行了。其实人家水平比你差远了,但是人家高联一试能刷到接近满分,二试做一几何,直接 200(那个时候一试 150 分,二试 3 个 50 分题),你要是还按老办法玩,没准就直接 OUT 了。

这么多人挤进来,大都是为了好政策。高联一等奖能拥有保送资格,万一冲进冬令营蒙个两三道题没准能进清北,诱惑太大了。

于是,年轻的天才们一年比一年付出更大,一届比一届刷题更狠,只为了在几千个非天才的围追堵截下冲到前几名。

惨不忍睹。

真的惨不忍睹。

不能怪任何刷题的非天才和他们的家长,制度如此。

中国的优质高等教育资源实在是太匮乏了。

如果中国能有 20 所清北,200 所双一流(或 985),可能事情会好办很多。可是中国没有。

2009 年,我第一次参加冬令营阅卷,当我看到一份 30 多分的卷子(冬令营 6 个题,每题 21 分,总分 126 分,当届需 102 分才能进集训队)的主人,联赛是 260 多分的时候,我先是诧异,然后是无奈,最后则是深深的叹息:可能就是因为这个人,一个高手被挡在了冬令营的大门前。

对于高联 - 冬令营 - 集训队这一条线来说,选拔国家队是其中一项重要任务,如果高联就把全国的高手削掉一半,那后面还怎么玩?

所以,高联被迫改制度,经过 2009 年的调整之后,最终改成了现在的 120+180。

即便是这样还不够,伴随着教育部的政策,数学会和奥委会又大幅度增加了冬令营各省的参赛名额。

即便是这样…… 还不够。

负责任地讲,同样在不玩命刷题的前提下,一个高手现在想进冬令营,难度要远大于 13 年前。即使现在是 400 人冬令营,那时候是 100 人。

水涨船高。

有办法解决吗?

现在来看,真没有什么能快速见效的好办法。

不幸中的万幸是,高手毕竟是高手,高手也刷题,谁都挡不住。一阵玩命死刷题之后,大部分高手也能冲进冬令营。但是这样的高手,由于过早地接触重复训练,对后面多多少少会有一些影响。

参赛人数的增加,直接大幅度增大了真正高手一飞冲天的难度。不仅如此,它还通过其它方式间接增大了这个难度和不确定性。究竟参赛人数的增加会产生哪些连锁反应呢?且听下回分解。(TBC)

====================================

Part 7 疲倦的裁判

参赛人数增加,第一个影响的就是阅卷的困难。

2015 年 9 月,我有幸参加广东省高联的阅卷和复查。

阅卷的时候,14 个大学老师,7 个题目(一试 3 个大题,二试 4 个题),差不多一千份卷子。

初改要一天完成,毕竟大家都是老师,都有教学任务,谁也没太多闲工夫。

初改不计复核,一个人也要面对五六百张卷子。

一天改五六百张卷子的一道题很难吗?

很难。

竞赛不是高考,联赛的解答有的你读懂就要五分钟。

竞赛不是高考,不按标准答案做的人多了去了。像二试第一题这种题目,据不完全统计,有十几种不同的解法。

竞赛不是高考,你必须判断出学生的解答是基本对还是基本错,是有小瑕疵还是有大漏洞。

还有更坑的呢。

我改几何的时候看到一张卷子写由托勒密定理 blablabla,由角元塞瓦定理 blablabla,由笛沙格定理 blablabla,证毕。我当时真有一种想把这个骗分的家伙拉出来的冲动,最后还是放弃了。孩子是无辜的,没准就是被无良老师教的。

我那天算是试出来了。

我一个白天的极限是改八百道题,复核不算,而且中饭和晚饭都是味同嚼蜡。

第二天我还要上课,我晚上躺在床上,进祖坟的心都有了。

我不禁怀念我读书的时候,那时候要是去改卷,可比现在轻松多了。

梦想是美好的,现实是残酷的,因为改卷完了还有复查!复查!

复查的卷子少了一大半,但是复查的题量变成了全部!而且复查的题里大部分都是写了东西的!

坑哥啊!

我觉得吧,如果只是为了钱,有人拿百元大钞砸我我都不干这个活。

也许是因为我自身的经历吧,我每次拿起红笔的时候心里都是很严肃的。学生实力不够或者没发挥好,OUT 了就 OUT 了,但是学生做对了判错了,这是无论如何不可以的。

高联如此,冬令营更甚。现在 400 人的冬令营,想想就头疼。

首先,冬令营的题变难了,解答也变长了,尤其是时间变成 4 个半小时,够写四五篇作文了,想编点什么东西很容易。经常会出现读十分钟还没读懂答案的情况。

其次,冬令营的每一分都很重要。高联的 30 分和 50 分没区别都是三等奖,100 分和 120 分没区别都是二等奖,只要重点照顾一下省一左右或者以上的卷子,就不会出问题。冬令营可不行,各大学校都看这个成绩招人呢。去年要是一个 48,一个 45,前一个可能就是清北,后一个可能就是孤魂野鬼啊。所以冬令营这种比赛,卷子要复查多少遍,没改过卷子的绝对想象不到。

还有,正因为冬令营分数敏感,复核的时候经常要充分讨论,更别说一个题的对错能折腾一两个小时的事儿了……

这里还是要向参加或将要参加高联、冬令营等比赛的同学和家长们说一句:我们知道你们急,但是我们更急;我们知道你们想要公平,但是我们比你们更想比赛公平。

话说回来,十年前改卷儿的,最少待遇是专家级的,现在改卷儿的,真就是搬砖的家伙。

有人不禁要问了,这么多卷子,为什么不多找些人啊。

我觉得道理很简单。

第一,真没太多适合的人。能看懂竞赛卷子,愿意做这件事,还没有直接利益相关的人太少了。

第二,同一个题不能有太多人改,不然很容易出现你改的严,我改的松的情况。理想状态是 3-4 个人一组,而且互相复核,共同讨论。

第三,人多了,各种产生的负担就更大了。

现在来改卷的大都是纯奉献,但真的不确定大家还能坚持多久。

改卷的也是人。

参赛选手多了之后,阅卷的困难是可以预计的。但是,阅卷的困难又进一步地刺激了某些改变。到底是什么改变呢?且听下回分解。(TBC)

====================================

Part 8 混乱的训练

本节再次涉及大量主观观点,不喜请屏蔽。另外请勿对号入座。

在讲阅卷困难导致的改变之前,我们先从参加人数增多讲起。

参加竞赛的人数增多,另一个直接的影响就是培训教师的缺乏。

一个省几十个人正经搞竞赛,最多一两个班就行了,老师是永远不缺的。尤其像北京这种地儿,IMO 金牌想来讲课都得排队,更别说国集水平的同学了。

但是现在不一样啦,一个省几千人搞高联,怎么也要弄出三五十个班来,而且大家劲头都很足,算下来最少要近百个老师才能带的过来。

一个省能有一百个教高联能教的明白的老师吗?

很负责任的说,凡是叫 “省” 的,都没有。

小学的竞赛,读完高中成绩优良的理科生基本都能讲,真没多少东西。唯一的区别是有的老师讲的真是好,而大多数人只是能讲而已。

初中的竞赛,大学读数学系,接触过竞赛的好好练两年基本都能讲,真没多少东西。唯一的区别同样是有的老师讲的真是棒,而大多数人仅仅是能讲而已。

高中的竞赛,那可就不一样了。

小学的竞赛可能以初中的知识和思想为背景,初中的竞赛可能以高中的知识和思想为背景,这些都好掌握,掌握了这些的老师,至少在讲授时层次会高于学生,或者通俗一点讲,最少可以装装 13。

高联或者高联往上可就不好玩了。平面几何和对称不等式什么的,还能用时间堆,做上五六年最少耍起把式来能挺像样的。但是代数呢,没有分析和方程的功底,以及数感和放缩的技巧很难讲透;数论也差不多,把进位制、同余、完系和阶弄完之后,后面要么是在分析整数的结构,需要懂一些环和域的东西,要么是和组合结合的问题,运气不好冒出来一个小孩拿本解析数论来问你你可就呵呵哒了;组合更是要命,归纳反证极端原理仅仅是初级工具,既有极富初等思想的不妨设、对称、算两次,也有用对应或同构转化问题,还有母函数、线性代数等等高等的知识和技巧,更不用说组合可以和代数、几何、数论中任一领域结合,简直是无边无际。

顺带一提,个人认为代数里真正经典的是多项式,学过复变的都明白。可是多项式这玩意能代数能数论能组合,各种天坑,能讲明白的太少,学生会的也少,最后还反过来导致不敢随便考……

大部分老师,苦练数年,能讲明白一个领域已经十分了不起了。况且现在的状况,大家都是快餐,没有多少老师再愿意去像以前那样花很长时间只为弄明白一块内容,那样还不如去讲初中小学,学生更多,更赚钱。

一个省就零零星星几个能讲明白的老师,更悲催的是没准这老师还是清高型不愿意讲课,或者学术型天天三点一线,一般的学生和家长甚至连面都见不到

那这几千个学生找谁去辅导?

当然是不靠谱老师了。

但是不靠谱老师怎么镇住学生?

当然有办法了。

个人觉得办法基本就那么几个:讲套路,讲难题,背答案,教骗分。

下面一个一个介绍。

讲套路,事实上是只讲套路,把竞赛难题按照讲高考容易题的方法讲。其实我不反对讲一些套路,一个完全没接触过高中竞赛的孩子也是需要学套路的,但是把套路当唯一思路就不好了。数学怎么说也是锻炼思维的,这样不是在教孩子而是在毁孩子。当然了,如果家长和学生觉得靠这么学能上个好大学,毁毁孩子也无所谓,那我无话可说。特别还有一些老的竞赛难题,在新的竞赛书上有了系统理论之后就成了套路题,学生发现自己能做很难的竞赛题,那感觉就像武林外传里的李大嘴学了降龙十巴掌之后一样。

顺带说一下,个人觉得 2015 年的冬令营在新颖性上做得很好,六个题没一个能套路。不等式一调整就坑了,组合题不能归纳,平几题基本解法动不了,上解几又困难重重,数论题又考了多项式。冬令营的成绩一下子让我们看到了现在的学生做新题时的孱弱。考完之后猜国集分数线,主试委员会的猜测都集中在 72 到 84 分之间,结果…… 不客气地说,如果这套题拿去考 2003 年前后的冬令营,国集的分数线估计会到 81 分左右(那时约 25-30 个国集),也就是说现在的高端战斗力做新题的能力也没比以前有提升。

讲难题,顾名思义就是类似给非顶尖的初中生讲冬令营,给非顶尖的高中生讲普特南这样的做法。数学是循序渐进的过程,违反自然规律的事可是要不得,偶尔讲一个题让孩子了解高度是可以的,但老是这样做就不好了。但是这样家长和学生可以膜拜啊,他们又找回了学习小学竞赛和初中竞赛时的感觉啊,就是这个 feel,倍儿爽!但是这样做的一个很大的问题就是学生根本动不了笔,从而会养成只听课不思考的坏习惯,以后就是见了题都眼熟但是都不会做的节奏。我自己就会说,这个孩子被教 “废” 了。

背答案,这里注意是背答案而不是看答案。看答案是一个优秀的竞赛老师必须具备的技能,毕竟你不可能像学生一样每个题都做。看答案并非只是看答案,而是要根据答案来辅助理解问题,有时能举一反三得出其它解法,即便没有其它解法也不要紧,最少能总结出解答的哪些步骤是关键,再深一点可以总结背后有什么东西,在什么动机下能想到,是不是自然的,如果学生自己做的时候遇到困难,应当怎样给予提示才能收效最佳,等等。背答案那就性质不一样了,脑子里一背,黑板上一抄,然后解释每一步为什么对,这样好像跟直接发答案没什么区别。

教骗分,上船之后必做的事。题不会做怎么办?乱写一阵,所有你知道不知道名字的定理都写上去,总有阅卷老师看不懂的,看不懂他就不敢给你判错。尤其是证明题,没准就蒙到了。证明的题,尤其是几何,条件推一堆,结论推一堆,中间差一步,写上个显然。可是孩子啊,数学是严谨的,不是靠蒙的,这可比学套路什么的严重多了。学数学之前,先要学会做人啊!如果你不知道自己做的对不对,写上去无可厚非,但是你要是知道自己是胡说八道还写,那就是人品问题了。上一节也提到了,现在的竞赛阅卷压力很大,若是放到之前,阅卷老师完全有时间陪你玩,可是现在不行。因此,很多人就钻这个空子,就赌阅卷老师不敢判错,而这又反过来加重了阅卷者的工作量。冬令营级别的阅卷人尚能保证识破这样的忽悠,但联赛级别确实偶有漏网之鱼。记得某年冬令营,某省所谓最强学生,做了 6 个题有 4 个伪证,大抵就是这个原因罢。

竞赛参赛的人数增多导致了一些不靠谱老师的产生,但这还不是最严重的。还有什么更令人头疼的事情呢?且听下回分解。(TBC)

====================================

Part 9 脆弱的学生

先讲点上一节没讲完的东西。

上一节讲到有很多不靠谱的高中竞赛老师,先说一下他们为什么不去好好教小学初中呢?个人认为还是高中竞赛老师的特殊地位。毕竟小学和初中的竞赛老师确实很多,除非教得非常好,否则没有太高的地位。另一方面,大多数家长和孩子还是拿小学竞赛当小升初的敲门砖,拿初中竞赛当进好高中的敲门砖,同时他们也意识到,大多数市里县里的好的初中高中还是可以靠钱或者关系搞定,但是搞定优秀的大学是非常困难的。另外,省会级城市的第一名学校和第五名学校的教学差距,未必比得上优秀大学里哪怕是头批 985 和二批 985 的差距。经过 9 年(甚至超过 9 年)的竞赛长跑之后,大家也都把高中的竞赛当作终点。

回到正题。

高中竞赛的参赛人数大幅度增加,不能赖政策,也不能赖不靠谱老师,初中高中的招生也仅仅是影响小学初中的竞赛。最终的问题,还是出在学生和家长身上。

近些年的高中生,大多是在 95-00 年出生的。这些孩子,从出生开始就一直被捧上天的人绝不占少数。尤其是如果还能学习号称是尖子生才能学的奥数,那简直就是飞机中的战斗机了!

所以,这些孩子全都成了布袋和尚——说不得。

可是事实上呢?

奥数只适合 5% 的孩子学习(注意只是学习!),可是光参与的孩子恐怕就要超过 20%。现在前 20% 的孩子能上啥大学?二本可都不一定(还有部分学生读不到高考的,所以 20% 到了那个时候可就是 25% 甚至 30% 了)。

天分不够,水平上不去,玻璃心倒是养出来了。

小学的时候呢,多学一点,能听懂也行,当个兴趣爱好。可是慢慢长大了,再花大量时间学竞赛,啥都听不懂,可就没有必要啦。

再说了,竞赛这个东西本来就是逐步淘汰的,学不懂就不要再花时间坚持,十年二十年前的家长都懂,为毛现在的家长就都不懂呢?

这种坚持,说好听的叫做破釜沉舟,说不好听的那就是无谓的坚持。

小学到初中是要淘汰一拨人的,初中到高中也是要淘汰一拨人的。我考华杯赛的时候华杯赛决赛的金牌,也有几个高联省一都没拿,这都很正常嘛(没准人家去搞其它竞赛了)。现在更是如此,**小学竞赛全是填鸭,考好根本不代表什么,可能只是刷题多。**到了高中,你以为你还能只靠刷题?

当然了,这个东西和学校以及校外的培训机构多多少少都有一些关系。学校嘛,总是希望更多的学生参与这些东西,最少述职报告上可以写写嘛;培训机构就更不用说了,多一个学生多一份钱。但是关键,还是在学生和家长身上。

即便是培训机构,只要是正规的,都时常会有考试和分班,跟不上的就学一些简单的呗,这不是很正常吗?竞赛这玩意再怎么说,每年也是要淘汰一部分人的,如果机构招生进来是 10 个班,讲了三年竞赛还是那 10 个班,一个人都没变,也没有人员交换,可以肯定这是个骗钱的机构。

可是这倒好,这种考试和分班制度,却被家长投诉说是歧视,不人道……

初中竞赛,考试分个班,60 人分两个班,结果 50 名左右的都要去快班……

高中竞赛,到高二要停课集训,结果高一的联赛考 40 多分,高二模考从没上过 100 分的孩子都要去停课……

这种事见多了,见怪不怪。

你去劝学生,学生不听。你去劝家长,家长说:“凭什么我家孩子就比别人家孩子差!”

哭笑不得。

很多家长经常挂在嘴边的一句话就是:“我家孩子学得挺好的,就是没考好。”

应试是有可能有高分低能的,但是绝不可能有低分高能的,竞赛就更不可能了。判错了我信,没考好我不信。

这些孩子还听不得批评,当然老师也懒得批评。想想也是,在学校里的时候老师肯定管不住,谁愿意冒那么大险管教你家孩子啊?万一被你打了咋办?打死了咋办?到机构里人家机构就是讲课收钱的,家长和学生就是消费的大爷,更不会惹你不高兴。

这里插一点别的东西。

就业难因为什么?

因为毕业生高估自己。就这一点没别的。

一本学校出来想一月两三万,想没想过每年毕业接近一百万的一本生,大学扩招的十几年就是近两千万人,可是全中国有多少人能到哪怕是一月一万?这个数字远比你想象的要低。

一般的大学里,你在学校是学霸,是风云人物,出了校门啥都不是。但要命的是,你还觉得你是个人物。

清北即便有一些舆论的影响,还是清北。只有在顶尖的大学里,你才能认识到和真正的神犇之间的差距。校内校外一比,才能精确的定位自己。有的事就不是你能做成的,或者做成的概率不超过彩票中头奖的概率,人要认清现实。(当然,如果你认清现实之后还有这个理想,那肯定要无条件支持)

回到正题。学竞赛的学生和他们的家长,高估的人太多了。

最后的结果大概就是这样的。

四千个水平远不够省一的孩子和两百个水平有希望冲省一的孩子(这里包括十个很牛的孩子)一起玩命训练,一起考高联。该省省一有 60 个名额。

对那四千个孩子,由于题目变化,神发挥和阅卷误差等原因,冲到省一的概率就是 1%,最后有 40 个省一。

剩下 200 个孩子瓜分剩下 20 个省一。

结果呢,那 3960 个孩子自然是撞了南墙回头,你以为那 40 个能幸免?金秋营,自招分分钟把他们刷下来。不仅如此,他们还耽误了宝贵的时间。本来裸考能上 985 的,最后可能连 211 都悬。

但是剩下那 200 个就 DT 了。尤其是最好的十个孩子,完全是大坑。没准冒出来个谁谁谁就抢了一个省队的位置。

那 4000 个孩子坚持搞竞赛,得益的是谁?

只有学校和竞赛老师,不解释。

我还是不说那句 4 个词 10 个字母的话了。

非顶尖水平的学生,其实并不适合在高中还继续花费大量时间学习竞赛,但是由于孩子的脆弱及家长的保护心理,他们还是选择了继续。那么,对于真正的顶尖学生,情况又是什么样呢?且听下回分解。(TBC)

====================================

Part 10 倍增的压力

非顶尖的学生的继续,使得顶尖学生的晋升压力变大。但与此同时,还有更大的压力等着他们。

第一个原因当然是保送制度,以前的话就算是没进省队的省一也能不用高考了,去哪儿另说最少好听啊。另外高二弄个省一之后就可以完全弃综合了,高三考不进去也认了,不会没学上。当然,我也见过两科竞赛省一,发现保送没好学校去,放弃保送再回去高考考到清北的,这种犇这里不讨论。

第二个原因,则来源于学校和背后的培训机构。近年来重点高中为了吸引尖子,打打旗号是必须的,一个集训队都能给学校加不少的分,更别说物以稀为贵的国家队了。据我所知很多学校都打出了明码标价,什么成绩能拿多少钱(话说我咋没赶上这好时候……)。不过别以为这是有比没有好。考高联的时候的高中生最多也就十七八,有个 “赏罚分明” 很容易让他们情绪更加不稳定,容易想多。另外,指导老师也会在后面不断叮嘱不断打气,但这明明就是让学生更加紧张的做法嘛。培训机构就更别说了,对于尖子那是好吃好喝供着,但是谁也不是傻子,你要没考好看人家还理你不。

当然,前两条说到底也没多大影响,而且尤其是冲进国集之后就基本等同于没影响了。真正能影响顶尖高手在国集和 IMO 发挥的压力,在哪儿呢?

舆论。

当然是舆论。

去年 8 月,2015 年的 IMO 国家队包括领队副领队和队员上了鲁豫的节目,相信关注数学竞赛的筒子们都已经看过了。不知道大家看完之后什么感觉。

我当时看完之后,只有一种淡淡的哀伤。

上节目这种事,本来就是容易说多错多。如果只是简单背几句台词还好,现场(录播还好一点,直播更惨)如果说错一句话,那可就是一失足成千古恨啊!

上节目这种事,本来应该只找成年人的,找熊总和秋生哥做个采访就完了。两位领队都是老江湖了,年岁大,经历的又多,肯定能很好地完成任务。

但是去年的 “中国奥数输给美国” 在网上疯传,队员们还能淡定的坐着吗?

全天下都在看着他们,已经没有人能站起来保护他们了,他们必须自己站起来保护他们自己!

可是他们毕竟还是一群孩子啊。

还是一群追逐自己梦想的少年啊。

他们做错了什么?不就是没考第一吗?这和中国运动员没拿到预期的名次就得出来解释一下有什么区别?!

不,或许比这更严重。运动员多少是国家体育总局花钱培养的,是拿纳税人的钱砸出来的,优秀的运动员甚至还能大把大把地赚代言费。可是数学竞赛呢?我可以很明确的说,最多只有国家队队员出国比赛的机票和食宿是报销的(食宿还是主办方提供),其它从培训到参加国内各种比赛的开销,那可都是家长自掏腰包啊!奖金倒不是没有,但哪能和体育竞赛比?

有的人该说了,你这影响力也不如体育竞赛啊。可是正因为如此,不是更不应该责难他们吗?

现在的国家队队员,应对 IMO 这样的比赛时确实有一些弱点(这一点我会在后面详述),但是考差了可不能怪他们。

我觉得,一个国家队队员,哪怕他 IMO 考得再差,只要他一没有在之前的选拔考试中作弊,二没有在 IMO 考场上消极怠工,三没有公开发表不当言论,他本人就不应该承受任何的指责和非议。大家都是人,人家不管有没有弱点,只是训练,考试,训练,考试,最后领队告诉你你进国家队了,再训练,考试…… 人家什么也没做错。如果说没选出最强的人,那么就是我们的选拔体系还有待完善;如果说选出了最强的人,但是状态没调整好,或者弱点没有补上,那么就是我们的国家队训练体制还可以进步。不管怎么说,这可不能怪 IMO 选手吧。

IMO 选手自然承受着很大压力,不过非 IMO 选手也差不多。近几年大家都有这个习惯,说谁谁谁应该能进国家队,结果人家冬令营考砸了,集训队考砸了,就说这人咋这么弱。

这叫什么?这就叫妄议。

其实很正常啊,即便是顶尖的高手,也有可能进不了国家队。冬令营还好,集训队到最后都是毫厘之争。一个高手没进国家队,可能只是因为吃坏了肚子,可能只是因为题目不对胃口,可能只是因为评分标准恰好对他不利,也可能只是因为一个阅卷失误。是考试就有随机性,为何偏偏要忽视它?退一万步说,就算人家是真的弱,那也是人家自己的事,关你啥事?难道你还能开个赌局赌谁进国家队不成?别忘了人家就算再弱也比你强!

说到这,应该很明白了吧。

数竞选手承受了太多的舆论关注,舆论将关注转化成了压力。

但另一方面,他们并没有从中得到任何的利益。

每个人都可以如同喷明星和运动员一样喷他们,可是他们却没有得到如同明星和运动员一样的利益。

说到底,还是政府不重视,有钱人不重视,只有一帮不明真相的围观群众重视,最后造成了数竞这种 “又要马儿跑,又要马儿不吃草” 的尴尬状态。

想要政府和有钱人重视是不太现实的,更何况数竞本来就需要降降温。

所以解决的唯一途径就是,无关人士速速退散。

顶尖的高手,尤其是 IMO 选手确实不应该为他们的失利承担任何责任,毕竟这只是个游戏。但是,他们也确实在应对竞赛时有一些弱点。那么究竟是什么弱点呢?且听下回分解。(TBC)

====================================

Part 11 高手的弱点

相比以前的国家队员,近年的队员们确实在应对竞赛时有一些弱点。前面已经提过的诸如高估自己,做新题能力差等等就不再说了,影响也不是特别大。

我在这里主要想谈一下我所看到的两个足以影响最终成绩的弱点。

在谈弱点之前,先以个人角度分析 2015 年与 2016 年 IMO 中国队的输赢之处,如有主观臆断请指出。

先来看 2015 年。平心而论,第一天的题对中国是有些不利,中国队也没有发挥到最好,但是造成这个情况的原因并不仅仅是几何。第一题组合几何与第二题数论,从数据上中国队没吃亏,甚至还有点小便宜,但是至少有 2 位同学因为花费大量时间做这两个题而失去了做第三题几何的宝贵时间,但因为几何题的数据太可怕了,一下子亏了接近 20 分,所以一下子认为几何是输掉的重点也很正常。但是实际上几何题应该也就是亏 12 分(接近 2 个人)的节奏,因为每天有 3 道题,还是要综合考虑。即便第一天考成这样,有第二天的第六题这样一个大礼包,中国队也未尝没有翻盘的可能。中国学生的数感是很好的,前面也说过,需要用代数估计和放缩的题目(即便是数论或组合),中国队都会很占便宜。尽管第六题是组合题,但是需要用代数去估计,中国队就考得很好,全场也就十几个基本做出的,中国占了四个,第六题总分全场第一。与此同时,可以看到第五题函数方程是中国的弱项,这个题 30 人全对中国竟然只占两个,而且剩余四人的分数都很低。中国队最终还是输在了第五题上。倒不是说第五题应该更多的人做出来,毕竟没做完的四位同学中有两位第六题是满分,估计花费了一些时间,但是明显应该能多写一点,争取更多的分数。比较中国和美国第五题的小分就可以看出来,最后的最后还是输在小分上(第一天的题做不出来就是做不出来,无话可说)。

厉兵秣马,来年再战,中国队加强了对几何的训练,选拔也更偏重几何。考试题一出来,没有纯几何作 3,6,可能有些筒子就会觉得 IMO 在针对中国人,其实不然。首先,TST 考什么和能选出哪个领域偏强的选手没有完全必然的联系,大家并没有太多互相针对的,中国的 TST 题目毕竟还是中国人自己放到公共网站上的,这样说未免阴谋论了;其次,今年 IMO 还有第五题这样帮助中国队的好题,这样的一个代数正是中国队的强项;第三,IMO 本来就是让人猜不出,完全定死了规矩那就没意思了;第四,其实第三题是几何题而不是数论题,中国队没有把握好而已。

来看一下具体情况,中国队不仅在第五题上是满分,并且由于花费时间少,顺带着第六题也做的很好,即便是三个满分的韩国队在这两题上也被中国队赚了不少。关于美国队,那真是运气太好,可以看到美国第二天三个满分都是中国小孩,这也是无可奈何的事情,即便如此,第二天中国队还是赢美国队 4 分。第一天的第二题应该是中国此次没拿到好名次的关键,有两位同学没做出,相比几乎全对的美韩,这个失误足以葬送领先优势。但是也要通盘考虑,据说有一位同学是在第一题上花了过多时间而没有足够的时间做第二题。看看第一题和第二题,当然不是某些人所臆想的初中题难度(再次强调,答案只是告诉你这个题怎么样可以做出来,不代表难度,需要仔细读答案才能发现里面的困难之处),但也没有很难。第一题想到复数绝对可以一小时以内做出来,这笔买卖很划得来;第二题只要把 6 乘 6 的棋盘为什么没办法排分析清楚很容易猜到证明的方式,它放在冬令营和集训队也只能算一个一般甚至偏容易的题目。

所以个人认为,2015 年第三题是大坑了中国队,第六题又是大帮了中国队;2016 年第二题是小坑了中国队,第五题又是小帮了中国队。两年的题目看下来,对中国队都是公平的。另外虽然韩国今年的三个满分有点猛,但是美国两年的发挥算不上极佳,最多只能算是略微偏好。事实上中国队发挥也都是一般,即便发挥的不尽如人意,也说明中国队像前些年那样巨大的领先优势是肯定没有了。

下面来说说我眼中中国队的弱点。

第一个弱点,就是四个领域不均衡。

当然,每个选手都有强项和弱项,但是当一个领域弱到会大幅度影响拿全 1245 的概率的时候,即基本上做不下来这个领域 25 难度的题目的时候,事情就不好办了。

早在第二节中就提过,基本拿全 1245 是很重要的,尤其是第五、第六名。如果国家队中相对较差的几个人不能保证拿四个题,对整体的分数影响会很大。去年和今年的 1245,中国队做得都差强人意,而且最终都是输在 25 上,本来用来赚分的 25 现在却输分了,不得不引起重视。

第二个弱点,就是不会应对不利的形势。

其实这个情况我在 2009 年就发现了,有不止一个冬令营满分在集训队第三次考试上栽跟头,几乎一分未得,后面状态越来越差,都无缘国家队。在之后的几年里,越来越多的学生在考试中有 “一脚深一脚浅” 的问题。有的同学大赛考得好,却在集训队里出现大失误,尴尬到靠评分标准才幸运地没出局;有的同学参加两次大赛,一次考得很好一次却十分差强人意;有的同学集训队题难的时候考满分,题容易的时候却几乎零分。

相比十年之前,现在的选手这种 “逆境求生” 的本事基本都荒废了。一旦遇到第一个题不顺手,很容易出现重大失误。考场即是战场,做不出 3 个题也要争取做 2 个,做不到 2 个也要争取做 1 个半,这样的信念和决心,还有应对的能力,正在慢慢从选手身上消失。从统计数据上无法看出全部,也许有的同学一直都很顺,但也拥有这样的问题。

选手们有这样的弱点,原因是什么?且听下回分解。(TBC)

====================================

Part 12 不同的目标

现在的选手和以前的选手比,确实有一些弱点,产生这些弱点的第一个关键因素,就是赛制,尤其是集训队的赛制。

我们考集训队的时候,是 8 次小考 2 次大考,大考分数乘以四。所以在小考阶段,完全不需要发挥得太好(我高三的时候小考好像也就是正式队员的第三或第四),但是不能落下太多。小考有的时候题很难,甚至有时会有一两个讲解答都要讲一小时左右的题,所以要求抗压能力特别强,大家都不求有功但求无过。题容易的时候三个全做不是本事,题难的时候做一个半或两个可就厉害了。而且,小考时大家都在边考边调整状态,以便大考的时候能够爆发,毕竟大考是乘以四的。2002 年就有一位队友,在小考考得十分不理想的情况下,大考比大部分人多做了一道题,成功翻到了前六(名字就不说了,在场的都知道)。考试次数多,才更要求心理素质好,能够逆境求生。

集训队的 10 次考试,完全可以考出一个选手的所有优点和缺点。像我在 2002 年时就是几何的技巧部分做得稀烂(换句话说就是基本只会硬算),于是被要求练习几何题,到上 IMO 考场上的时候几何已经不那么烂了。

现在的集训队可不行,只有 6 次考试,而且大考的比重严重不足,前面很容易拉开差距(在 24 道题里差出 4 道题,可比在 12 道题里差出 2 道题难多了,后者可能就是一次失误的差距)导致后面无法翻盘。并且,只有 18 道题,各人的优缺点也没有特别明显地体现出来。

另外,联赛进冬令营和冬令营进集训队的扩军也是一个问题。扩军确实能保证基本保证最好的选手能进到集训队了,但是 60 人的集训队必然有大量 60 名左右水平,甚至是 100 名左右水平的选手,加上考试次数少,确实更容易出现大反转的情况。毕竟贝叶斯公式摆在那里,没选出最强的 6 个人,这也是很正常的事情。

集训队的赛制缩减,和主办方不堪重负有一定关系,和命题者的缺少也有一定关系,但我认为最大的原因还是教练组并不像十年前或二十年前那样极度渴求一个团体第一了。如同很多中国体育的优势项目一样,从无到有的时候很激动,第二次第三次蝉联的时候很有荣誉感,可是拿多了第一也没意思,总是成为别人眼中的目标也是心好累的(再说 IMO 拿了第一教练组也木有奖金)。中国的数学竞赛已经度过了艰难的开荒年代,已经有了一个相对稳定的平台。相比之下,教练组更愿意看到手下的学生们在未来有好的发展,也不愿意再多多折腾这些已经训练了 N 年的学生,可能在要求刻意练习的时候手也松了一些吧。

另一个关键因素,就是学生自身的目标和定位。

教练组不愿意加强训练强度,有的高手也不想要。第一是现在的高手们也更注重未来的发展,提前自修大学课程的不在少数。另外,现在高手普遍都是很早就进入刻意练习阶段(甚至有很多初中就停课做竞赛的),练了几年估计到最后也练烦了,大家进了集训队进了国家队之后玩上 N 久也是常态,前几年甚至还出现了一届国家队里一多半到大学不想学数学专业的情况。

其实现在的高手条件要比以前好,机会也比以前多,他们也能更加综合地去考虑和做出选择,他们应该是更加幸运的。能够在注重当下的同时也放眼未来,这是一件好事。

中国的高中数学竞赛变成这样,我不敢妄言是前进了还是后退了。但是**数学竞赛本来就是培养和选拔对数学有天赋及兴趣的学生,并给他们尽量光明的未来。**从这一点来讲,尽管现在联赛级别大家的压力都很大,但是从顶端来看做得还不错,至少不考虑舆论的话还是在进步的。

**IMO 只是一个游戏,不要把它太当真。**梦碎梦圆,这都不是终点,区区游戏,何分第一第三。

当然,目前我们的高中竞赛还是有几个问题需要解决的。

第一个问题,就是如何避免高手因为要冲出来而过早做太多的重复性训练导致的一系列问题(后继无力或者进入大学之后不喜欢数学,或者因为学习数学竞赛而过早丢失学习其它学科的机会和时间等等)。

第二个问题,则是如何缓解舆论给予高手们的压力。

第三个问题,就是如何在最后的选拔中选出真正执着于参加比赛,水平也高的学生去参加比赛,并保持不错的成绩。

这三个大坑一般的问题很难给出很好的解决方案,我也只能做到抛砖引玉,且听下回分解。(TBC)

====================================

Part 13 急需的科普

我想到的第一个解决方案就是科普。

原因很简单,很多人不知道奥数是什么。

中国十几亿人,真正知道奥数是什么的,恐怕只能以百万计,甚至可能还到不了百万。

因为不了解,所以攻击,因为不了解,所以诋毁。

人之常情。

我举一个例子。2013 年我看到一个段子,说一个不识字的土豪解鸡兔同笼问题,先让鸡抬起一只脚,兔抬起两只脚(这时候脚的数量减半),然后再让每只动物抬起一只脚(这时脚的数量减少了等同于头的数量的值),这个时候鸡已经一屁股坐地上,兔子是单脚独立,很容易算出兔子的数量,然后再解出鸡的数量。段子的最后说,这让数学老师和奥数老师情何以堪啊。

看了这个段子,感觉怎么样?奥数老师是不是特 S13?这可是连不识字土豪都会的做法。

真相是什么呢?

我 1992 年在华罗庚数学学校学奥数的时候,老师就是这么教的。

另外拜托仔细想想,不识字土豪真会这个?

对了,这个段子就是来黑奥数的。

但是可惜的是,我看了这个段子三四次,仅有一次有一个人的回复说鸡兔同笼就是这么学的。

另外,跑男第二季第二集包贝尔也用了同样的方法,弹幕里也无任何人提到此事。

这个方法的道理是什么呢?不难看出,这就是解二元一次方程组的做法,只不过不用设未知数。

奥数是很有道理的,永远是低段工具解高段问题。

当然,如果你的奥数老师讲鸡兔同笼上来就列方程组,请让你的家长抽他两嘴巴子,然后申请退钱,因为这是一个不讲思维只念答案的庸师。

其实我觉得奥数真的缺很多科普的书,我们的出版社应该多重印一些好的科普书。我读小学初中的时候手头有好几本科普的小品文,多到我自己都从书里建立好了一套初等数学的体系。但是现在学竞赛的学生,哪还有什么科普书啊。

不仅是**奥数需要科普,数学本身也需要科普。**前段时间看到一个所谓 “小学奥数五星题”,如下。

这个题怎么做呢?其实不要纠结于只用小学内容,你完全可以先算出面积再说。

不难看出,中间的阴影部分和右下角的阴影部分都是好算的,难点在于右上那一块。

建个坐标系好了,左下角是原点,联立直线和圆的方程不难算出对角线和右边圆的靠右上的交点坐标为 (18,9)。然后连圆心,算扇形面积,发现面积里有个反三角函数。

这个时候基本就可以肯定了,这个题肯定不是小学题,小学题再怎么样,答案里绝对不会有反三角函数!

这个题原题啥样?看出来也很容易,其实就是多了左下角一块小的黑色,这样用对称就得到阴影面积等于矩形面积减去两个圆面积之后剩余部分的一半。

结果呢?这个有技巧的好题被刻意地改动,完了之后又成了黑奥数的东西。

其实,只要掌握了高中知识,很容易识破这样的伎俩。

其它的很多问题也是类似的,其实掌握好高中数学知识和高等数学知识的人,绝对不会上当。相反,很多初中小学的奥数教师,如果高等数学没学好或者没学过,是很容易被一骗一个准的。

所以,我这里还是建议所有奥数教师都先学好高等数学再来教奥数,也避免总是问一些秀智商下限的问题。

除了知识,一些理论也是必须要科普的。

初等数学中有很多解决不了的问题。

这个事中国有多少人知道?

可能百分之五都不到。

换一个具体一点的吧。五次或者五次以上的方程是没有通解的。

这个事估计知道的多一点,但应该也不会超过百分之十。

所以咯,奥数黑完全可以随手抛出若干五次六次方程,说奥数就考这个。

不科普,永远不能洗白,你解释了一个还有另一个。

还有,不定方程也是。别忘了费马大定理就是初等不定方程,孪生素数猜想也可以算进去。一个不定方程不能解太正常了。例如,n 允许含有素因子 2,3,5,n+1 允许含有素因子 7,11,13,求 n。这是个骗人的题,即便能做也是狂讨论,没有任何实际意义,但是一个在中小学没有接触过奥数的数学系本科生比一个大学没学过数学的初中小学奥数教师更容易发现事情的真相。

不止圈外要科普,圈内也要科普。有关难题的问题就是一例。

应该有很多人认为出难题就很厉害,也有很多奥数教师以难倒别人为自豪,然后吹嘘。

但是实际上,出难题不是难事。

我自己出过 6 个超级难题,个个堪比 ELMO Shortlist 压轴题,如果哪个高中生能做上其中的一半,绝对是国家队水平之上。而且,每个题我绝对可以在 30 分钟内解释清楚。

可是我基本不公开拿出来,也从不拿来炫耀,因为没有必要。只有当国家队级别的选手做所有题都秒杀,然后又向我要新题的时候,我才会拿出来一个两个让他们去自虐去。

顺带一说,2009 年选拔的时候我拿出来过一个,被其他几个老师否了,因为他们判断那个题只能考考韦教主,其他人肯定全跪,没有任何选拔意义。老师们的原话是,我们又不是选一个人,我们是选六个人。

题不是出难了才有意义,而是难度恰到好处才有意义。从训练的角度来看,是让学生能够得着,开发思维才有意义;从选拔的角度看,要能区分学生,选出好学生来才有意义。

另一个例子是 2015 年的女子赛第四题,这个题其实是一个广泛问题的简单情况,详述这个问题背景的文章发表在中等数学 2016 年第一期。个人觉得原题放到国家队选拔,都不会有什么人做出来。即便是那篇文章,能看懂的估计也寥寥无几。

只有给大众科普了什么是数学,什么是奥数,才不会有那么多的奥数黑存活的空间,奥数选手的压力才会减轻一些。另一方面,弄清奥数是什么之后,会有很多水平不足的学生和家长退出,间接地给真正的高手更多的位置。

科普是必须的,然而还有什么好方法呢?且听下回分解。(TBC)

====================================

Part 14 体育的启示

这一段是我在去年 8 月就构思过的东西了。

其实纵观中国各大体育项目,谁的成绩好,谁的成绩不好,很容易得出一些东西的。

回到第五节的观点。

“在竞技比赛或者任何事情上,一个人或一群人能做出傲人的成绩(排除天分),要么就是他们需要用成绩来改变自己的命运,要么就是他们有着极强的荣誉感和使命感。”

看看奥运会,其实中国队能露脸的队员基本都有同时兼备这两点。早前的中国队员拿了金牌都是哭,后来有人说哭太假,逐渐也有很多人不哭了。其实说哭太假的人真的该拉出来游个街什么的,人家运动员辛辛苦苦几年十几年,就把在奥运会上奏起国歌当梦想,梦想成真了还不让人哭?

中国的很多项目的队伍,队员被给予的经济条件都不好,但队伍的荣誉感却是满点,这样的队伍很容易出成绩。

去年 8 月的世锦赛,苏炳添成为首个站上世锦赛男子 100 米决赛跑道的中国人,并在数天后与队友莫有雪,谢震业和张培萌合作,第三个冲过男子 4*100 米决赛终点,然后由于美国队犯规而幸运的把铜牌升级成了银牌。这件事和前后发生的一些议论还是让我感受良多。

首先是运动员的知名度。说实话比赛之前我只知道苏炳添和张培萌,毕竟还是 10 秒俱乐部的人,谢震业好像听过一次,莫有雪赛前听都没听过。当然这也和我不是体育迷有关。

然后是经济条件。几个人在成为名将之前从体育圈拿到的都差不多,我后面看了一个跟踪报道,莫有雪出名前在省体训队一个月就挣两千多,世少赛冠军也没什么大笔奖金。

其实我觉得这样也挺好的,给好的训练条件,但不要赢了比赛就拿钱砸。这样运动员到了很高水平之后才会和金钱打上交道。一辈子不让打交道是不对的,太欺负人了,但是还没到世界级或者准世界级水平就开始接触大量金钱,那很可能水平就停到那了。

有些项目,省级市级的选手就能挣很多钱,这不仅成为了阻碍选手进一步训练的阻力,也使很多只想挣钱的人进入到这个圈子,这种项目是铁定出不了成绩的。

另外一个事儿,就是四个人拿了银牌之后我看到的评论。

评论里面北京人和广东人可就吵起来了,一个说苏炳添最牛 13 张培萌是渣,一个说张培萌超越 N 个队伍拿到亚军前面都是拖后腿的,最后上升为地域黑。作为一个长期在广东的北京人,我感觉这些人真是吃饱了撑的,人家苏炳添和张培萌好好的把棒接了,你们在这打起来了。个人认为苏炳添么为人低调踏实,喜欢的人更多一些,张培萌稍微高调一些,但也没说错话,大家都是功臣。但是这种喷子总是存在的,且存在于每个项目。

还有一个事儿也是从体育圈这么多年看出来的。

中国民众特别非常极其重视竞技的成绩和名次。

这个很明显,拿了金牌特别高兴,丢了到手的金牌大家骂的跟卖国似的。

我想过两年,或者没准今年,就有人会指着谁谁谁说你考砸了害中国 IMO 队没拿到团体第一什么的。

所以我觉得,从体育上的启示,我们至少可以做三点。

第一,就是不要上来就给学生太好的回报。不过这个感觉挺难的,小学初中的竞赛还是那么火热,机构和学校都是拿钱招好学生…… 至少高中这块应该好一些,比如初三高一拿省一的孩子完全可以等等再签约嘛,或者签个省队一本什么的,一下子签了一本人家玩儿去了。

第二,还是要增强学生的荣誉感。爱国教育什么的不要嫌弄的太早。这样学生至少进国家队后还有一个努力的方向。我们说 IMO 是一个游戏,还是置身事外说的,学生要是真出工不出力那就惨了。

第三,也是我感觉比较重要的一点,就是要减少关注者的数量。

越多人关注,越多人骂,学生就会越不淡定,这是自然的。另外,关注的人太多,就会自然而然地衍生出资本市场,这对我们维持这个游戏的天然性和公平性是十分不利的。

其实我很希望,我们最后的国家队,甚至是冬令营集训队,只有数学竞赛的参与者们和希望数学竞赛良性发展的人们关注,高手有他们的荣誉,但不需要太多其他的人来肆意评论。

当然,除此之外,在参与者的受众上我们也需要有一些变化,且听下回分解。(TBC)

====================================

Part 15 更广的受众

数学竞赛在产业化之后,难免遇到一个很严峻的问题,那就是受众。

竞赛培训是要花钱的,又要花时间的,大多数人花钱花时间只是为了让孩子上一个好大学。

什么人会这样做呢?

中产阶级。

太穷的人付不起学费,而顶端的人或者有自信做好子女的教育,或者可以把孩子扔出国,或者说不太 care 孩子上什么牌子的大学。

但是,只以中产阶级为主力的事情,必然存在不少问题。

这里引用水木上 ID:NightElf 的一段话:

“穷人家的孩子上不了,在道义上就站不住。

官商子弟上不了,在很多方面就得不到支持。”

这段话虽然是说另一个东西的,但是我觉得对数学竞赛也是一样。

数学竞赛接受批驳的另一个原因,自然是衍生的相关产业——培训,只能带动中产的消费,这是非常不好的一件事。

如同上面所说,我们不能只让中产在这个圈子里玩,需要让两头的人都参与进来。

当然,穷人那一边是我早就想过的,另一边是看了上面的话才有感悟。

数学竞赛要想重获好名声,必须重新接纳穷人家的孩子,为他们想想办法。(我说的是能出一定成绩的穷人家孩子,超神那种早就被盯上了各种喂养,不需要考虑)

毕竟,现在高考改革之后,更加注重能力化,穷人家的孩子并不是不能出头,但是更难出头了。如果这个时候能够让穷人家的孩子在竞赛上获得公平,那竞赛的名声自然会好起来。

前面已经说过,很多没到很高水平的孩子重复训练,但获得好大学门票的几率微乎其微,同时又阻碍了他人。去除这一部分人当然是很重要的。但是,更多的孩子是穷人家来的,他们可能连上大课的学费就交不起。更何况现在的制度下,各个培训机构的竞赛课一个赛一个的贵,让人哭笑不得。

当然,贵也是有道理的,道理还是家长们的 “不选对的只选贵的。”

放着一天三四百的好老师的大课不上,去上一天一千多的庸师的大课,逼着人家良心价涨成黑心价。

一对一也是。曾听说有个家长,朋友的朋友是国家教练组的大师,人家看朋友面子给他就要点辛苦费,结果发现没有隔壁骗人机构的一对一贵,果断学了一天就走了。

再这么下去,穷人家的孩子恐怕连一些基本的想法都难以学到了。

更可怜的是,穷人家的孩子还很难指望学校。我读书那会儿,学校请老师讲课我们不掏钱,但是考试得奖了也不拿钱,其实挺好的。搁现在,估计没法这么玩儿了。

所以我觉得我们需要一些公益的讲座,和一些针对贫困竞赛生的特殊计划。

很多明星都做过慈善,人家一捐一座教学楼,几百万起步。

我们可没那么多钱。

但是我们可以做公益讲座啊。人家是捐教育资源,我们自己就是教育资源,捐自己就行了。

我很希望未来我们的优秀竞赛教师能够多做一些这样的事情。

但是要满足三个条件。

第一,自己先解决生活问题;

第二,讲公益务必更加尽心;

第三,也是最重要的一点,必须在大环境合适的时候再做。(大环境会在后面讲到)

关于针对贫困竞赛生的特殊计划,交给学校好了,相信学校会做的很好的。

数学竞赛要想获得各个方面的支持,就必须让 “上流社会” 也参与其中。

顺带说一句,其实中国 1990 年能办起 IMO 也与这个原因有关,细节略过。

数学竞赛适合 5% 的孩子学习,官商子弟中适合学习数学竞赛的比例可能要比这个更多。虽然有些人并不是特别执着于参加竞赛或者帮助升学,但是别忘了数学竞赛还有开发智力的功效,这一点做好了依然能起到很大的影响力。

但是能够纯粹开发智力和培养兴趣的教师难找。不客气地说,**现在中国市面上的数学竞赛教师,称职的比例已经很低了,**而能脱离应试只开发思维的教师更是凤毛麟角。如果官商子弟碰不上这样的教师,那学了半天觉得没什么用,回头说奥数不好,那也属正常。

所以事实上我们也需要一些这样的教师,真正为纯思维服务的教师

由上面可见,数学竞赛缺少一些愿意做公益的老师,和一些愿意纯开发智力的老师。但可惜的是,在这样的大环境下,这两类老师既是稀缺资源,又不愿意做这些事情。只有大环境好一些,才能引得部分人愿意去这样做。

大环境什么样子才叫好?且听下回分解。(TBC)

====================================

Part 16 更清的环境

现在的大环境并不是很好,竞赛的培训队伍良莠不齐。

刚刚考完了一个不是奥委会主办的高中竞赛。考试一结束,套用赵忠祥老师的说话模板就是:

“竞赛结束了,网络上又到了培训机构和民间教师抢学生的时节。”

所谓 “抢学生”,不是抢着让学生去报名,而是向大家公布哪个获奖学生是自己的学生。

其实这种东西最开始都是学校自己公布自己有几个学生获奖,一般是放到自己家网页上。后来有了培训机构之后,大家为了抢生源争相报道这也无可厚非,但是随着培训业竞争的激烈,大家对于学生的底线越来越模糊的时候,事情就不好玩儿了。

最开始吧,大家都是说长期跟着的叫学生,后来呢变成上过一期课就叫学生(能坚持这个原则的已经不错了),再到后来发展成听过一次课,或者跟老师聊过天,甚至不小心做过老师出的一道题的,都叫学生,这个就过分了。

哎呀,按这个说法,就凭区区在下,怎么说也是带过队的,出过不少题的,那不是带出了三四十国家队员,两三百集训队员?就算只按正经讲课算,我好歹还是在集训队国家队讲过课的,算下来手下也教过十几个国家队员,一百多集训队员。

天啊,真的吗?太神奇了!

另外我觉得呢,宣传学生本身没错,但是能不能在考试之前就告诉大家哪些参赛选手是你带出来的?事实上,宣传自己的人大都是事后诸葛亮。按这种模糊的定义,一次冬令营我 “教” 上一百多学生,怎么着也得有十几个进国集的,完全可以很轻松的宣传(这里呢说一下,如果机构真的只招到了特别牛的学生,结果比如 20 个学生考冬令营 10 个学生进集训队,那还是可以认为是机构的本事,最少还有 10 个集训队水平的人愿意去呢)。更要命的是,考试之后只说出了成绩的,没说没出成绩的,这在误导大众的同时还会让没出成绩的学生觉得自己被抛弃了,对他们又是一次打击。

各个机构和老师大力宣传还会给公众另一个误解。由于好学生总是会被更多家 “抢”,所以粗看起来好学生似乎是去了很多机构学习。事实上呢?可能只是宣传的原因,即便有真正聪明的孩子和家长是去了很多家的,那也更可能是先遇到不靠谱的机构和老师,最多学一期就退出(有可能还学不到一期),直到遇到靠谱的老师,稳定一个或几个地方为止。看起来学了很多地方,但和你去乱学还是不可比拟。

这么多宣传的,事实是什么呢?

真正厉害的机构,只需要实实在在记下有哪些学生在自己这里学了几期课;真正厉害的老师,甚至不需要自己给自己打广告。真正厉害的机构,招生都是限制人数甚至限制报名水平;真正厉害的老师,平时哪会有时间干这干那。

当然啦,宣传么也得有人信,至于谁会信呢我就不说了,大家看了这么多都该知道了。

至于谁是好老师,谁是好机构啊?你问高中生,人家跟你有竞争关系,或者迫于一些压力可能不会说实话,你不会不问高中生?

教师队伍良莠不齐,未来当然是要想办法。对于那些水平不够的老师,该怎么办呢?

水平不够的老师分成两种,一种是还想继续加强自己的,一种是不想继续加强自己只想骗钱的。

对于第一种,其实都还好,我们需要做一些竞赛教练的培训(这点已经做得挺好的了,还有命题研讨会什么的,大家多交流是好事),让他们也融入这个高起点的圈子,继续修习。同时,已经合格的教师也能继续提高,彼此互勉。

对于第二种啊,没什么好说的,只有寄希望于消费者的自然选择能把他们淘汰掉。如果真淘汰不掉那可没办法,即便到最后劣币驱逐良币那也只能赖消费者了。

只有市面上的教师大都是靠谱教师的时候,才能分出一部分人完成上一节所说的工作,数竞从培训上才会更加规范,环境才会更清澈,才会向着更有利于选拔和培养人才的方向发展。

培训是一方面,当然也有让比赛的进程更加完善的理想。且听下回分解。(TBC)

====================================

Part 17 更明的进程

高中的数学竞赛兼有普及,选拔中高端和选拔顶端的功效,为此进程上也可以考虑更加透明一些。

其实完全凭兴趣搞的时候,大家很少纠结成绩,像我高一的时候联赛差一名没进冬令营,也觉得很正常嘛,也不会说要去追求查卷什么的,另外冬令营大家分高分低都是认的(当然现在涉及到保送不保送,降分不降分,也难免),集训队也差不多。现在么利益那么大,学生不急家长急,这也是没有办法的事情,所以现在还是需要有更高的公信度。而透明,则是公信度的一大保证。

说到进程透明,我首先要说的就是考试规则。其实在这个网络时代,我原以为考试规则这个事大家都非常清楚,但是还是有一些不清楚规则的考生。像不知道联赛二试每个题多少分,或者一试考多少分钟这都能发生。冬令营开始的一些规则就有更多人不知道了,例如有且仅有前 30 分钟可以对题目本身提问,主试委员会会给予解答,以及如何查分与申诉的问题。我觉得这些问题都挺好解决的,一个是在网站上公布,另一个在冬令营等比赛的秩序册(参赛须知)上面可以专门用一页来列考试规则,这样也给那些不常上网的同学一个提醒。

下面就是阅卷规则,其实这也是被很多学生与家长所诟病的事情。因为试卷肯定是无法完全公开的,所以很多学生会说自己被黑了什么的。但是我这里要说的是,联赛我不能保证,冬令营这里大家都是想要给学生一个公平,从最开始的改一遍查一遍,然后除了非常清楚的零分和满分之外所有卷子需要再找一个人复查一遍,之后再查,有少数很难判定的卷子都是看上六七遍的。去年冬令营出现了集体惨剧,很多人的预期分数和实际分数能差出两道甚至三道题,也会有很多人质疑,但是前面也说过,冬令营阅卷是严格的,很多混分的都会得不到分。我这里有个想法,如果某位同学真的觉得自己比窦娥还冤,那么他完全可以实名把自己的解答重写一份放到网上,接受大家的检验,而不是空口无凭地说自己被黑了。集训队么有几年没参加只是零零星星供题,不再多说。

联赛确实问题也很大,当然我前面也说了,试卷太多而阅卷人员和时间不足是一个大问题。我个人有一个不成熟的提议,联赛的预赛只考大量的选择填空,然后如同 AMC 一样有底线分和比例(前几天得知 AMC 马上就没有底线分只有比例了)。以 AIME10-11 分水平对应的水平来选人,应该能卡到更少的决赛人数(但是这个说不准,如果真这么搞会不会又有一波准备预赛的培训班),并且没有阅卷负担。决赛还是大题小题结合也好,或者只考大题也好,都应该能减轻一些阅卷压力。

然后就是选拔的机制,其实我们不应该太神化国家队队员(哎呀这个话好像我说还好,换个人说会不会被认为是酸葡萄心理?),毕竟有些人喜欢在这上面多付出,有些人则喜欢干些别的,例如提前系统学习大学课程。前几天和一位同学聊到某一年的成绩,再次说到书写标准和会打逆风球的事情。事实上书写标准和会打逆风球,在水平接近的时候是能够大幅度增加进入国家队的概率的,并且去考 IMO 的话成绩的期望值也会高一些,但是这两个东西在科研里确实没什么用。所以说,即便是最好的赛制和选题,也只能保证选出的是最适合参赛的六个人,而不是最强的或者说数学上最好的六个人。到了集训队之后,学生们完全可以分成 “我强,我稳,我就是玩命要考进国家队” 和“我强,但我不稳,我就是想随便考考,进就进不进也没关系”以及 “我就是打酱油的,我想先学大学课程” 三派。进了国家队的,就得有牺牲精神,从集训队结束到出国比赛就别闲着了,继续刻意训练去;没进国家队的,接受现实,早四个月学习大学课程也挺好。当然,只有老师,家长和学生对国家队不再看得那么高了,这个理想才能真正实现。

最后说说本人对于选拔国家队的细节的一个点子。集训队的成绩是不公布的,可能一些随机的问题例如阅卷偏差会导致某些高手进不了国家队,这种问题还是偶尔会发生的。真的想改变这个情况,最少让学生自己服气,比较好的思路是让学生自己来为自己协调分数。现在美国最后的准国家队里的练习,便是学生自己来给老师解释,这也是模拟 IMO 的协调。某位学长说过,他非常希望我们能够有朝一日也是这样。但是对于 60 个人的集训队,这无疑是一个无法承受的负担,粗略估计最少需要 20 个以上的教练才能完成,即便有这么多教练,彼此缺乏交流和判罚尺度不统一将再次产生不公平。我的点子是,集训队前面的考试可以选出 10-12 个人成为 “准国家队” 并公布名次,其余人打道回府,这 10-12 个人继续换个地方调整状态和考试,后面的考试便可以让学生为自己协调。这样,不仅从考试次数上有了保障(直接增加考试次数会让主办方不堪重负),而且也更加公开公平公正(协调的时候必然互相能看到成绩,但是之前考过的卷子不能查看和改动了,算总分),同时减少了教练的付出(10-12 个学生有 6 个教练负责协调绰绰有余),还能真正选出希望国家争得荣誉的学生(不然随便考考进了国家队的人可没有后悔药吃,这么做他后面可以弃权也不丢脸),并且减少了关注度(60 个人里挑 6 个显然比 10-12 个人里挑 6 个更惹人关注,并且可以换个没有网络的地方啥的),唯一增加的负担就是题目的数量。

啰啰嗦嗦说了三万多字,其实只是想让大家更加了解我们的数学竞赛,更加理性地选择是否参加,如何参加。数学竞赛的本意是选拔和培养对数学有天赋有兴趣的学生,并给他们创造更佳的环境。我想,等到我们的国家队仅仅是一个与金钱无关,与未来关系也不大的荣誉,但同时 IMO 成绩还能稳居世界前列,甚至回到霸主的地位的时候,等到我们的国家队队员,集训队成员,冬令营营员们能够在未来在数学和其它自然科学上大放异彩的时候,我们才会觉得这些年的辛苦没有白费。

正文到这里就结束了。另外有一些前面没有讲完或者没有合适位置插入的科普,会做成两篇番外篇,接着正文写出。这些争取在 6 号之前结束,不耽误大家看里约大冒险。(Fin)

====================================

番外篇 1 尴尬的难题

从今年 IMO 的题目统计数据来看,第三题只有 15 个人基本做对(至少 5 分),因此第三题是一道难题,果真如此吗?

其实不然。

仔细分析一下题目,真的除了吓人之外难度并不是很大,并不属于那种死难死难的题。

这个题几何方面,托勒密定理,海伦公式和 S=abc/4R 知道两个基本就能下来;数论方面则只要知道化成素数幂和 “一颗老鼠屎坏一锅汤” 的结论(即若干个数中恰有一个数所含素因子 p 的幂次最低,则这些数的和所含素因子 p 的幂次等于这个最低的幂次)肯定能做。

看看几年前的题要到什么难度才能得出这样的数据吧:04 年的第三题组合,大坑题,至少没有那么显然;08 年的第六题几何,一个呵呵哒的题,画图画半天;12 年的第三题组合,骗人游戏,慢慢折腾吧;13 年的第六题组合数论,即便猜出要用归纳也不是那么容易的。当然,同时也要重视前面题目的阻击性,可是看起来今年前两题并没有那么狠。

再看看 15 个基本做对的人有 10 个集中在前三强的国家,其实能猜到大概为什么了。

没错,稍微弱一点的队伍,学生做第三第六题的欲望和动机在慢慢变小。

前面我也说过 IMO 投题的过程,大家都喜欢选难题做 36,长此以往,那些并不是很强(5-20 名)的队伍就会想办法应对,应对的办法也很简单,只做 1245,确认无误再稍微做做 36。

别小看 1245,近年来,全部人只做好 1245 在所有年份都可以拿到前十,部分年份可以拿到前五甚至前三。对于队员个体?多少 29,28 的金牌线,金牌根本毫无压力。

有的领队吐槽:“我们的学生已经不想做 36 了。”

所以,今年的 3 不是个难题,仅仅是大多数人不想做而已。

所以,在 IMO 上的 36 需要适当降降难度,多让大家参与。

IMO 如此,国内也如此。

女子西部每天 3 个题可以拿到奖励,冬令营每天 2 个题可以进集训队,集训队每天 2 个题到大考爆发就能进国家队。

长此以往,大家都不会去做难题了。包括训练的时候,应该做 3 个题的时间只用来做前 2 个,全在舒适区,一点刻意训练都没有。

所以我们也需要一届分数线是 105 分左右的女子西部或者冬令营,让大家重新重视难题。

同学们在训练的时候也经常会遇到这个情况,做完容易题,时间到了,于是难题就不做了。其实这样很不好,好的方式是考试结束之后花时间继续想难题(或者吃个饭睡个觉之后继续),而不是白白浪费一道难题的训练机会。

毕竟,到达一定水平之后,做难题才是提升你能力的最快途径。

====================================

番外篇 2 骗人的答案

题目答案是什么?

题目答案不是思路,题目答案不是解释,题目答案只是 “我告诉你这么做能做对这道题”。

这也就是我为什么说,背答案是庸师的表现。

有的老师和同学喜欢用答案的长短来判断题目的难度,这在大部分场合都是有效的,但也有很多反例。我这里举两个例子,先来看下面这道题。

![](data:image/svg+xml;utf8,)

这道题的解答特别长,看起来似乎是一道难题。但实际上呢?解答后面的所有部分都是在证①,由于出现了大量讨论,显得解答很长,但是讨论的技术含量远低于前面下结论的部分。

再来看一道题。

这道题的解答非常言简意赅,只有区区几行字,似乎是一道容易题。但是实际练习中,不会做这道题的同学远远多于不会做上一道题的。

其实来仔细分析一下就明白了。第一道题在尝试的时候很容易发现 “偶奇奇偶奇奇” 是一个永远无法移动到全 0 的状态,于是立即得到所有不小于 4 的偶数都不可以,然后只需要在操作中躲开这个状态(当然还有全偶数的状态,无法控制),剩下的就是一步一步递归就行了,反正最大数是只会减少不会增加的,怎么弄都行。

相反地,第二道题,归纳并不是一个很显然的点子,因为多一个数的时候事情会变得特别复杂。或许能想到最大的数比所有其它数之和都大的时候可以归纳,但是其余情况会很乱。另外 k 的选取也特别有讲究,直接一步到位。在实际练习中,很多学生都尝试使用其它做法,然后说理很难说清楚,总有重复和遗漏。只有在最大的数比所有其它数之和小的时候继续考虑次大的数,才会慢慢向这个归纳靠拢。可以看到,这个问题的解答是思考过程的凝练,直接使用极端原理构造 k,省去了一次递推,后面也基本没有废话,才使得解答如此短。

这就是答案的骗人之处。

因此,我们看解答的时候,要先理解它的做法,然后多问自己几个为什么。

数学竞赛题目的难度不在于解答长短,这里简而言之,是在于点子难以想到或者难以实现(并不完全准确,还有很多其它因素)。

比如,上面两个问题我们都问过为什么了之后,就会发现第二道题里面的归纳是不自然的,然后顺下去发现第二道题要难于第一道题。

对于学生来说,即便看答案,也要在充分思考后再看答案。答案的哪一步你没有想到,你就可以问问自己是为什么没想到,是因为知识不够牢固,工具不够如臂使指,还是因为题目本身就很难想到这一步。

对于老师来说,要求就更高了,老师可是没时间做题的。老师必须在看答案的过程中发现答案中的不自然之处,然后试图解释这种不自然之处的动机,并引导学生做题时向这种不自然之处去努力。

**很多题目,答案的入手点很难想到,这种题目的解答基本只有前几句话是精髓。**例如 2007 年 IMO 第 6 题,看解答只要看第一句话(顺带一提,那年不知道有意无意,4,5 题分别是几何和数论,没有组合,于是 6 还有组合的可能性,而且 6 的二维的情况是一个小学题目,估计会更加坚定考生用组合做的信念)。同样的问题还出现在 2009 年国家队选拔考试第 6 题,那个题怎么取模都没用,必须扔掉较小的数,考虑较大的数,才能得出矛盾。

也有些题目的结论难猜,例如 2010 年 IMO 第 5 题,看答案是一个初中级别的题目,但是那个题问的却是 “是否”!相信如果题目改成证明存在,做出此题的人起码要多出一两倍。2016 年 IMO 第 2 题也与此类似。与此相似,有的时候答案会直接猜到一些极值,但实际上是要做很多尝试才能得到那个值,然后进行论证。

**看了答案之后,需要提炼答案的精髓,而不是全篇照抄。**所谓重剑无锋,大巧不工,**很多时候答案的精髓不在于美轮美奂的变形,不在于高级的定理,而仅仅在于一次 “不妨设”,一次“考虑……” 或者一次“假设……”,**上面第二个例题就是一个例子,后面的连续不等式只是末流的炫技而已,前面的归纳才是精髓。

看答案可以,但要多问问题;看答案没错,但要提炼精髓收为己用;看答案正常,但要理解为什么要这样做。

我真心希望我们未来的数学竞赛书籍的题目都有长长的分析,而不是干巴巴的答案。

当然,对于每个同学和每位老师来说,从答案中不应该仅仅明白这道题怎么做,而是应该看得更远。

知乎用户 阵雨 发表

吴军的《数学之美》这本书中,也讲了教育问题。

这本书 2012 年 6 月第一版的第七章,介绍了语音识别和自然语言处理的专家贾里尼克,这个学者童年长期在街上玩耍,荒废了学业。十几岁来到美国,一边进工厂打工一边学习。

吴军本人 70-80 年代上小学、中学,称花在课本上的时间也不到现在学生的一半。

所以作者观点如下:

一、小学生和中学生其实没必要花那么多时间读书,而他们的社会经验、生活能力以及在那时树立起的志向将帮助他们的一生。 二、中学阶段花很多时间比同伴多读的课程,在大学以后用非常短的时间就能读完,因为在大学阶段,人的理解力要强得多。 三、学习(和教育)是一个人一辈子的过程,很多中学成绩好的亚裔学生进入名校后表现明显不如那些因为兴趣而读书的美国同伴,因为前者不断读书的动力不足。 四、书本内容可早学也可晚学,错过的成长阶段是无法挽回的。(因此,少年班的做法不足取)

在 73 页,作者写道:

现在中国的好学校里,恐怕百分之九十九的孩子在读书上花的时间比我当时要多,更比贾里尼克要多得多,但是这些孩子今天可能有百分之九十九在学术上的建树不如我,更不如贾里尼克。这实在是教育的误区。

(引用处都是原文)

这里面有个很明显的问题。

贾里尼克是麻省理工毕业,任教哈佛大学和康奈尔大学,美国工程院院士;吴军曾是清华大学讲师,霍普金斯大学博士,2000 年 Eurospeech 的最佳论文奖,Google 中日韩文搜索算法的主要设计者。

假设,有一个 1000 人的学校,改变目前的教育方式,能不能出现 1% 类似吴军和贾里尼克的人?

如果可以,那将是 10 人。

放到全国又是怎样的数量级?

这意味着中国人将变成神族。

现实是,我们任意选取一个小学或中学。不论他们学还是玩,不管什么教育方法,肯定都有 99% 的人不能达到贾里尼克和吴军的水平,甚至不足以考取北大、清华。

所以吴军和贾里尼克的经历,没有普遍性,无法指导现实。模仿或借此下结论都不行。

作者提出的四条观点,除第二条,大部分都赞同。

兴趣、志向、健康的成长,多了解社会生活,知行合一等等,很多人多年前都说过了,所有人都知道对。但是到了具体实践上,大家都是一头雾水的。

教育问题极复杂,得出一些正确的结论很难,找到切实可行的方法更难,投入实践则难上加难。

想凭几千个字就解决这其中任何问题,不可能,也做不到。

知乎用户 清浊 发表

也许可以从这篇报道中看出端倪:

丘成桐,哈佛大学终身教授,国际知名数学家。现任香港中文大学博文讲座教授兼数学科学研究所所长、清华大学丘成桐数学科学中心主任。

以目前的本科教育模式,国内不可能培养出一流人才,中国大学生的基础水平,尤其是修养和学风在下降。哈佛毕业生的论文水平比国内有些院士的文章都好,如果不重视学风建设,中国科技至少后退 20 年。

“如果不重视基础教育,以目前的本科教育模式,国内不可能培养出一流人才。”8 月 6 日,数学大师丘成桐在北京接受记者专访时,开门见山道出了他的隐忧。

作为目前华人数学界的领袖人物,丘成桐先生不仅在学术上造诣深厚,而且十分关注国内数学人才的培养。他所在的哈佛大学近年来频繁接触国内大学及大学生,对于两国高等院校的差距,丘先生当然最有发言权。然而,通过和国内学校的直接接触,丘成桐却抛出一个让人意想不到的结论:“这些年,中国大学的基础教育存在很多问题,大学生的基础水平,尤其是修养和学风在下降。”

“有人说中国的基础教育不比美国差。但是在实践中我发现,国内大学的教育水平并非人们想象得那么高,美国的大学教育比中国好得多。研究生教育美国比中国好得更多。”

他举例说明:哈佛大学理学院每年大约招收 20 名念理科的中国留学生。“这些都是中国最好的学生,多数来自北大这些名校。” 但是从学习成绩来看,在和其他国籍学生的比较中,“很难看出这些国内来的学生毕业成绩比来自其他国籍的学生成绩好得多。”

3 年前,丘成桐招收了两个来自北大的留学生。可是这两个学生来到哈佛以后,一个学生连续三次没有通过考试,学校建议这名学生重新再念本科,还是不行,最后只好离开。另一个学生 “考得也不好”,经过努力以后大有改进。

“我每年都会收到很多国内学生的推荐信。后来发现有很多是假的,是学生自己写了推荐信,然后请教授签个名,许多教授很不负责。” 而且,他接触到的很多中国学生十分骄傲,不愿意用功,令他很失望。丘成桐说,10 多年前,来哈佛念书的中国学生至少考试是没有问题的,但是近年来这种情况也发生了变化。“从前中国来的学生至少考试成绩在班级里占到前 1/3 位置,但是这些年顶多排在后 1/3。当然也有好的,但是整体而言,平均水平下降了。一些甚至在国内是奥数比赛第一,但刚进来成绩也不好,经过重新训练才有好转。”

这位经验丰富的数学大师认为:这些现象都表明,是本科出了问题,学生本科没有念好。“做学问的修养等基本问题都没有得到应有的训练,而且还不用功。归根结底是本科教育不扎实。”

丘成桐尖锐地指出:他发现国内高校一些 “怪现象”:一些所谓的“名教授” 不花时间参与本科教学。他以美国哈佛大学为例介绍,哈佛的教授数量比国内大学要少很多,数学系也就是十七八个正式教授。但是数学系每年都要开会一到两次,研究讨论本科生和研究生的学习进度,教授会花很多功夫去培养本科生。比如数学系的本科生,一年级就会根据学生情况划分不同水平,选择最好的学生去重点培养,当本科生毕业的时候,一般就会有 2 到 3 个毕业生的学术论文可以登载在第一流的杂志上。“论文水平比北京大学的教授还好。”丘成桐说。

国内名校的教授又是怎么和本科生打交道的呢?

丘成桐介绍:大概在 1998 年,国内有一个学生申请到哈佛大学念书,申请表上写明自己学问很高,推荐信说他是北京大学最好的学生,丘先生当时看到申请表,想既然学生这么好,来北京时就亲自和他面对面谈。结果在和北京大学 20 多个资深的教授座谈时,这 20 多个教数学的教授没有一个知道这名学生。“我还以为自己搞错了。后来有一名助理教授证实,的确有这个学生。”

“看得出来,最好的学生,20 多个教授居然都没有见过。表示所谓大教授从来不参与本科生教学。” 后来那个学生因为念书思路视野十分狭窄,只是考试成绩好,丘先生并没有接收他。

丘成桐认为,教授不带本科生,并非因为国内师资力量紧张。他介绍,在美国的大学,比如数学系的教授 20 名左右,而北大数学院的教授则大概在 100 多个,是美国的 4 到 5 倍,哈佛学生 6400 多人,北大有学生 1 万多人。如果按照比例,国内应该有条件让教授带本科生,提高教学质量。

为什么那些 “名教授” 没有时间塌实做学问?

丘成桐先生一语道破:“现在名校教授花时间不是在学术上。有些人首先到海外弄好处,捞金钱加荣誉,让外国人来评价自己,一年有 3 个月到 5 个月在国外‘走穴’。不管这个教授有没有能力,评博士点,自然基金会,甚至评某某大学这些活动都有他们的身影,至少 1/3 的时间用在学术交易和拉票。算起来,2/3 是非学术生活。当然不愿意做学问了。”

另一个让丘成桐先生感到困惑的现象是:现在国内有的教授,一个人居然带 30 个研究生。

“这样怎么保证教学质量?即便在哈佛这样的世界名校,一个教授一般带五六个学生已经很多了。国内这种做法是典型的讲量不讲质。质量粗糙,怎么可能搞好研究?

“老师自己的水平都不够,居然带 30 多个学生?真是糟糕!许多学生认为不用念书就可以做博士,博士论文抄抄写写,怎么可以?这样下去很危险。”

前不久在和人大附中学生交流之后,丘成桐还迫切感到中学也存在这样的问题。“目前国内很多城市中学一班有 80 个学生,这么多学生老师怎么能教好?不可想象。” 他说,美国稍微好点的学校一个班一般不过是 20 多个学生。

丘成桐记忆深刻的一件事是:“大概 4 年前,有一个北大博士毕业写信给我,认为自己博士论文很好,希望到哈佛做教授,前后来了 3 次信,后来知道这人是国内某院士的学生,但我发现他的博士论文是至少 70 年前大家就熟悉的结果。他的论文水平也就相当于香港中文大学毕业生的水平。连香港中文大学学士的水平都不够,更不要讲哈佛了。但是对这个学生北大吹得很厉害,说他 21 岁就拿了博士,是个天才。但是我发现他的论文水平很低,所以坚持没有收他。” 让丘成桐意想不到的是,“现在听说北大已经提升这个学生做了教授。这种事让真正做学问的人真的很失望。”

丘成桐认为,不重视本科和研究生教育的直接后果是,形成恶性循环,使整体水平慢慢压下来。“教育是长远的事情,基本的东西没有掌握,就根本没有前途。” 说起这些,他十分着急。

为此,丘成桐在浙江大学成立了数学英才班,就是希望用先进的数学教学方法,比如采用了美国大学一些先进的教学方法,来培养中国学生扎实的基础能力。“选用美国的方法教并不是崇洋媚外。数学教学是没有国界的,我一定用最好的方法教中国学生。”

在近年来和国内学术机构打交道的过程中,更让丘成桐忧心忡忡的是:国内学术风气已经到了必须整治不可的时候。

“学风很糟糕,我呼吁一下,如果不重视这个问题,中国科技的发展至少退后 20 年,如果这些基础教育不重视,问题越积越多,以后会更难。”丘成桐认为,导致教育重量不重质的原因,就是有些高校和主管部门喜欢用 “文革”、“亩产万斤” 的形式作为工作成绩向上汇报,却完全不考虑教育的真正使命。

为了说明整个问题的严重,丘成桐专门讲述了两个他亲身经历的例子。

“在数学界有一个很重要的数学猜想,100 多年没有一个数学家不想解开它,非常重要。一个朋友在我的建议下做了很重要的第一步。1995 年,我感觉时机成熟,可以再沿着已有的方向,做出有史以来第一流的成果,于是拿到中国办了一个讨论班。这个讨论班是开放式的,吸引了很多青年学者。但是后来有些参加的教授自己不想做了,因为他们考虑假如做这个事情,写文章时间就少了,而每一年写论文,创造多少文章才是他们关心的,做这种费力的事情他们觉得划不来。尽管当时有很多年轻的博士后很想念这个东西,这些教授还是不准学生继续做这个研究,怕论文数量不够。最后甚至硬性干预。自己没有学问也没有冲劲,还不允许年轻人做。最后这个本来很有希望的征服猜想的计划不了了之。”

于是,丘成桐又找到广东中山大学数学院朱院长继续做这个研究。朱做出了很好的成绩。“结果又惹恼了当时反对做这个事情的一些人,对朱很不满意。 2002 年在陈省身先生和我倡议召开的世界数学大会上,大会给中国数学机构一些 45 分钟的演讲名额,结果他们自己定的名单全部是北京的,还有一个是上海的。其实就像奥林匹克 100 米比赛,朱的成绩当时是有目共睹的,但就是不容许其他地方有好处。”

还有一件事更让丘成桐耿耿于怀。

“我的一个学生,现在做了院士,刚毕业的时候不错,以后一塌糊涂,错了不愿意改,出错的文章现在还挂在网上,表示他的成绩。

“这个人现在既是国内引进人才,又是院士,基金会什么都管,平时许多人都怕他,他的固定职位在国外,却作为引进人才,一年至少给他 100 万薪水,还不包括经费。但是他在好几个地方任教,有的年轻人学问比他好,但是薪水不到他的 1/20,在海外名校,比如哈佛如果在 9 个月的授课时间里教授兼职是违法的,必须全职,但是这个人却兼了很多职。这个人做学生时还是不错,现在学问只是二流,他在国外拿的奖都是在我的帮助和指导下取得的,国内高校引进人才,给他大笔资金,可他全世界走,来一个月到三个月不等,行踪不定。

“而且严重的是,把我十几年前的文章,基本上改头换面,又据为己有,这种学风无疑误导很多学生,因为看到不用花太多时间就可以做院士。我批评他,这个人还说我荒唐可笑。真是不道德。”

丘成桐介绍,当年他花了很多功夫培养这个学生,“连我的儿子也没有下这么大功夫。太太都说我过分。” 毕业那几年,哈佛一位名教授告诉丘成桐,这个学生抄袭他的论文,出于保护年轻学生的目的,丘成桐并没有深究。结果愈演愈烈。

丘先生说,这个人不光彩的记录还有很多,比如有年轻学者的文章,他看到不错,就要求把自己的名字放在真正作者的前面,如果不从,在国内就恐吓说,以后不给你研究经费,在国外则恐吓说写很差的介绍信,让作者找不到工作,导致很多搞学问的人很怕他。

“听说国内很多人怕他。学风坏到了这个地步,作为这个人的老师,我再不讲话,其他学者就不敢做事了。我有义务来澄清。”

还是这个人,在国际杂志上的一篇论文出了错,当时恰好有国内两个访问学者到丘成桐那里去,丘先生让他们解决这个错误,两个礼拜时间,进展很好,但是当丘先生说这是那个人的文章出现的问题时,“两个访问学者吓了一跳,再也不敢做了,害怕那个人打击他。你看,现在的学风有多糟糕!”

我们应该学习陈省身先生的学风

学风方面,身为数学大师陈省身的高徒,至今丘成桐仍然对恩师的学风赞不绝口,认为那才是做学问者应该遵循的方向。

“陈先生做学问也出过错,海外几个学者提出来错误,也没有办法,错了就承认。”

丘成桐介绍说,上世纪 30 年代,陈省身等人大学毕业到海外求学,学成后毅然全部回国,没有在海外留下,但是拿来了世界最先进的学问,当时国内许多大学互相交流,学风好得不得了。

尽管当时条件很艰苦,陈省身的数学讲义都是手写和油印,就在那种条件下,陈省身几乎看完了大部头的数学巨著。最终作出了划时代的贡献,包括华罗庚先生等人,共同训练出中国第一批数学高等人才。

就是在这些数学开拓者的努力下,当时在 1946、1947 年期间,中国第一批数学家出来了,即便是现在看也是世界第一流的。新中国成立以后,在华罗庚先生回国以后,陆启铿、谷超豪、王元、杨乐、张广厚、陈景润、潘承洞,这些人才相继脱颖而出。丘成桐认为,在 “文革” 以前,基本上中国数学已经接近世界一流。但是后来大跃进、反右,直至 “文革” 等历次政治运动,把这些全部破坏了。

丘先生对恩师至今念念不忘。尤其是前辈大师那种学贯中西的气质,现今已经很难看到。而丘成桐本人就具有深厚的古文功底,身边常常带一本《史记》,早年写就的一篇古文曾被人们至今记得。其中,“无奈华夏虽众长城未修,天地虽宽,瑕疵难容,终究德不如欧美,力不逮乎日苏。根之腐亦,枝叶不荣,叶之枯亦,根基何养?” 更是振聋发聩。

丘成桐认为,现在中国改革开放,生活水平提高了,应该有理由在学术上作出更大的成绩。

英国数学家约翰 · 拉慈称丘成桐是 “华人数学界的领袖”。在陈省身等一代大师离世之后,重任已经落在丘成桐这代人的身上。

“领袖的工作就是确定方向。我生平立志只做好两件事情。第一,作出一等的数学研究,千古留名;第二,为中国数学教育服务,帮助中国成为数学强国。” 丘成桐先生说。

“迫切的问题,是提高质量。学风要彻底改掉。”他认为,目前和国际优秀大学比较,国内仍然存在较大差距,需要迎头赶上。丘成桐饶有意味地说,“首先,中国学生素质完全可以比得上哈佛的学生。假如有好的导师和好的风气,是可以有所作为的。”但是他着重强调了 “假如” 两个字。

知乎用户 PseudoCat 发表

个人观点,不喜勿喷。

谁说在美国做出牛逼东西的科研人员是在美国培养的?美国牛逼的地方在于全世界最牛逼的学生挤破了脑袋去美国读研读博读博后,然后挤破了脑袋排队拿绿卡,最后挤破了脑袋在美国搞出顶尖的科研成果可以成为美国公民。所以说事实就是,其他国家培养好了的本国最优秀的本科生和高中生,美国再从中挑出适合他们需要的人才。所以很多人说清华北大已成为美国大学的预科,我们高考到头来都是为了美国的科研发展筛选人才。改革开放以来,中国的人才流失是很严重的。不过我们不算最倒霉的。

二战期间以及结束后,欧洲一大批最优秀的科研人员移民美国,从此科学的中心从欧洲变成了美国。例如德国航空领域差不多所有人都跑到美国去了,这帮人直接促成了阿波罗计划的成功。可以搜搜土星五号火箭设计者的履历,都是德国培养的。冷战结束后,历史的一幕又得到重演。前苏联一大帮科学家们又集体被美国高校高薪聘请。据说美国 90 年代以后的美国本科力学教程就是前苏联人编写的。整整一个系列,从理论力学到固体力学。那时还流行把前苏联的期刊翻译成英文,结果老美发现曾经许多认为是自己最先创立的理论多年前前苏联就有人发表了,多么坑爹的感觉。

所以真不是美国教育有多牛逼,关键是过去一百年美国几乎把全世界最牛的科研人才通过各种途径都搞到美国来。有这些大牛带领学术界,你说能不屌吗?

-——/ 分割线 /——–

关于很多人问的为什么美国能持续吸引各国优秀人才去,我就在这统一回答了。

美国的学生是幸福的,尤其是理工科。他们的教材都是历史大牛们编写的,用的还是英文,他们自己的母语。同时他们的教授也都是历史上大牛们的子弟,而且还有许多来自世界各地最优秀的教授们。这样优越的环境是国内无法比拟的。在这样的环境下,人才会得到充分的发展。当然更别提美国自身相较于国内更公平和合理的科研体制。这些因素都跟美国的小学中学教育关系不大,更别提半数以上的理工科教授都不是在美国接受的基础教育。

前人种树,后人乘凉。而且还是许多世界最顶尖人才种的全世界最大的树。这样的优越的科研体制和科研环境现在还在不断吸引着全世界最优秀的人才。而国内的科研水平和军工科技不如美国是有历史原因也有现实原因。科技人才奖惩措施不合理造成积极性不高,科研人员待遇低下造成没人愿意埋头苦干从事基础研究和基础工艺研究,社会流行的拜金主义也严重腐蚀科研环境。从这个角度看,咱们国家能取得现在的科技发展实属不易,也值得称赞。只是目前还没看到能短时间赶超美国的希望。

至于历史上美国为什么能吸引欧洲和前苏联的顶尖人才,除了二战和冷战的原因,也跟美国自身地理位置,政治制度的优越性有关,更跟美国自身的包容性文化有关。至少目前看来,这些因素至今也发挥着作用。

答主所在的学校的工科专业中,有超过一半以上的教授不是在美国接受的基础教育,基本都是来美读的本科和博士。国家构成也很复杂,有中国,以色列,印度,日本,伊朗,英国,甚至黎巴嫩和智利。所以从这点就能看出美国对全世界人才的吸引也是全方位无死角的。各种小国的顶尖人才都来到美国了,更何况中国这种人才大国。

所以真不是美国基础教育多牛逼,而是美国从一百年前就不断的吸引全世界最顶尖的人才。他们留在美国,也为美国创造了最优越的环境来不断吸引人才,形成了一个良性循环。起码目前看来,这个循环还在起作用。

知乎用户 匿名用户 发表

举例失败。这些都是"工程",重点在于经验积累和团队合作,而不在于技术是否够尖端。

更确切的例子可以是"为什么他们出了那么多数学成果?""为什么开发持那么多优秀算法?"等。如果是这些问题,我的答案是:

"美国给予不热爱数学的学生最基础的数学教育,而给予热爱数学的天才最高水平的数学教育。"而你只看到了前者,不奇怪。毕竟,芸芸众生大多只是普通人。

知乎用户 Mass​ 发表

我觉得美国的整个科研和基础教育体制其实比较适合工程和应用学科,不那么适合搞纯数学和理论物理的。你提数学有点马屁拍在马腿上的意思。

美国的教育一直到本科基本上都是面向社会的通识教育,专业知识只是其中的一方面。所以我们会看到很多在美国读数学本科的小孩乱七八糟的课学了一大堆,其实真正的数学水平拿到中国也就是大二下学期比较好的学生的程度。对于应用学科来说这么做是没什么问题的,18 岁高中毕业确实还不是做人生选择的时候,等到本科毕业有志于科研的学生去读 PhD,利用五年的时间学习本专业的前沿知识然后开始自己的研究生涯。但是纯数学有一点特殊性就是需要的积累时间特别长,发文章的周期也特别长,如果本科阶段学得不是很超前的话,PhD 五年往往是不够的。就像前面高票答案说的一样,美国有很多其实还比较聪明但是不幸没有爬到金字塔顶尖的学生,在二流学校四年读下来,痛苦地发现他们的知识积累已经完全不容许他们选择纯数学了。

然后美国的科研体制也是对工科和实验科学更有优势。首先就是有钱,可以砸钱全球挖人,而且只要混出头了研究经费往往是很充足的;其次在美国招兵买马没有任何体制上的障碍,只要能拿到足够的钱可以立马招来一大帮搬砖干活的学生和博后来实现自己的想法,不用熬年头评博导什么的;最后竞争足够激烈,从 PhD 毕业以后整个职业生涯不管是找工作还是拿 funding 基本都是零和游戏,人人都是刺刀见红的状态。对身在其中的千年老博后来说这是很残酷的,但是确实对整体发展有好处。这三点对纯数来说好像有用的只有有钱挖人这一条。数学打人海战术是没用的,再怎么有钱,招来一帮博后天天打卡上班,每周组会报告进度,甚至搞末位淘汰每年开掉一个,几个人给同样的题目鼓励组内竞争,该证不出来的猜想照样证不出来。funding 的压力也是一样,对工科确实有用,但对于搞纯数的来说拿一个 NSF 三年就要有东西拿去交差,说不定还是副作用。

至于为什么美国仍然是第一流的数学大国,主要还是能砸钱挖人吧,看看菲尔兹奖名单就知道多少别国培养的优秀人才被挖到美国。另外美国自己也是有精英教育的,普通公立学校里面学那些 “很简单的数学” 的和日后在数学界混的风生水起的根本就不是同一拨人。美国和中国不一样,好的学生不是经过层层选拔以后集中起来批量培养的,而是分散在普通学生中间,数量不多但确实很厉害。好的高中里面里面稍微差不多的学生都学了一堆 AP 课程,至少和别的国家的同龄人是在同一条起跑线上的,然后真正顶尖的高中生会到大学里面选课,和大学生一起上课一起考试。出现在我课上的高中生高中毕业时候的知识积累和数学上的成熟度我觉得大概相当于美国排名 30 左右的学校里面数学系本科毕业生前 1/3 的水平,可以随便碾压任何 math education 专业(毕业以后去教高中数学)的本科生。来美国读 PhD 的很多中国学生都会有这种感觉,看看自己系里的美国研究生和美国教授,再看看自己 TA 班上的本科生,感觉完全就不是同一拨人。

总的来讲对于搞科研,特别是理论学科的科研来讲,美国缺少一个把最优秀的学生选拔出来集中培养的地方。就算 MIT、哈佛、普林斯顿,其实本科也都是面向社会的,混个本科学位都不是那么难。结果很多天资不错也有兴趣的学生因为没有这种来自同龄人的压力自己就放松了。欧洲我一点都不了解不敢乱发表评论,但中国我觉得基本上是反过来,从小就是零和游戏,不断地把最好的学生选出来放在一起让他们互相竞争。这样一届学生里面能出几个特别厉害的,但是很多稍微差那么一点没爬到塔尖上的学生到处被 “大神” 们碾压,然后就废了。

知乎用户 233​ 发表

因为大学可以开火箭啊,一学期从初中数学飞跃到数学分析不成问题。

如果不需要做高考那意义不大的圆锥曲线题、导数题,每天四小时一周就能学完高中数学好吧……

何况从集合论、关系、映射、自然数、群环域、多项式、有理数、极限、实数走上来的高等数学根本不需要半点高中数学知识,最多解个一元二次方程(然而这是初中的)。

至于高中 AP 课程,考试据说挺水的,但掌握的内容已经差不多够用了,于是省去开火箭之苦。

知乎用户 匿名用户 发表

我在美国某小镇居住。

去年某一天,邻居很开心地分享,他上小学的儿子已确定进天才班了——“天才班”,这个名字是不是很熟悉?

整个小镇,这一届,选了小十个孩子参加前期培训,后来就这一个孩子进了。够不够严?

之后,这孩子的学习难度和强度都会加大,会有天才班的老师团队帮助他进步。

这孩子啊,刚上公立小学没多久数学天赋就体现出来了。当时他的数学老师是位五十多岁的老教师。据老教师说,二三十年前,他们那时候的北美教育,并不是快乐教育,而是类似中国现在的严肃严格的方式。老教师找到孩子家长,希望能给孩子布置额外的家庭作业——多作作业和练习,熟悉不?

幸好遇到了负责任的老教师,督促着孩子一直在进步,才能有现在的好结果。

美国中小学数学简单吗?

挺简单的啊,学生学得开心,老师教得轻松。普通人就一直简单快乐的活下去吧。

小部分有天赋的同龄人,人家早就被特殊照顾了。

知乎用户 肥肥​ 发表

亲身经历… 澳洲新南威尔士大学,电气工程专业,科目:高等控制原理。

有一回老师布置了一道步进控制的题。我用 matlab 写了一个小时代码做出来了。同班的一个白人小哥手写了半个小时做出来了。

我个人观点是西方教育分的很清楚,高级人才与应用人才在初高中就分开了。精英去念大学(university)其他人去念专科(college 也有很牛逼的专科,不杠)或者技校(tafe)。这些念大学一般家境都比较好一点,将来或者从事白领或者进一步深造搞科研。跟国内最大的区别的专科或者技校毕业的人也很吃香,单说收入甚至比一般的大学生还要高。

就澳洲而言,水管工上门修一次水管的价格在 400 刀左右。研究生毕业起薪是 5w,也就是修一百多根水管。您要是一天一根…

人家发达国家大部分人随便干点什么都能养活自己。小学识个字,会数数就完了。按最低时薪算,也够一个月的花销。有更高追求的自然会钻研更难的东西,没有的也不影响过日子。国内情况就不一样了。高校扩招是因为真的缺人。专业院校根本没资源发展。当年全国可能都没几个会用电脑的,怎么可能办清华同方?我觉得当初的方针是先培养大量技术人员,不需要你多牛逼,基础的东西能用就行。所以培养出来的都是带点专业性的万用人才。先把人才缺口补上,发展起来再说。

现在已经不太缺这样的人了(才已经去掉了)。所以出现了大学生不值钱,4000 块只能雇一个大学生之类的说法。我就聊两句我知道的。精英人才这块国家大把撒钱从国外雇人。只要是在国外名校上过班教过课的,国家就要。来一个给一分钱(百万级,具体记不清了,在大使馆网上看到的)。带着项目来的还有额外经费。应用人才采取的措施是取消了大量没什么含金量的资格证。提高了一部分专业创业门槛(要求公司人员里,某几个资格证要达到一定数量)。

能看到到国家是做了一些动作的。最后说句心里话。美国的科研人家积累了多少年,中国的科研才搞了多少年?就算美国开历史倒车,现在小学不教数学了。该牛逼的人家照样牛逼。中国的大国梦任重而道远啊。

共勉

知乎用户 魔法小精灵​ 发表

因为人家教的是数学。

咱们学的那叫做题技巧,只是咱们现在恰好通过数学作为载体来传授做题技巧,要是倒退个 100 年或者 1000 年,咱们是通过语文来学习做题技巧的,那时候,人们把它称之为 “八股文”。

知乎用户 艾米小刀 发表

我们的教育体系,不大会培养人,更多是在筛选人。

原因很简单,也很无奈。我们人多资源少,只能先把人筛选出来,再用资源去培养这小部分人。我们读书的各个阶段,小学、初中、高中、本科、硕士甚至博士,教育体系的筛选功能明显大于培养功能。学习写作不是为了更好表达自己的观点、学习英语不是为了对外交流、学习数学不是为了建立逻辑思维能力,核心目的都是为了考高分,至于思维能力、交流能力、阅读能力、表达能力… 都是顺带的

M 国人少资源多,最佳策略就是先培养再筛选,在孩子成长过程中,教育体系的培养作用大于筛选作用。

这会至少会产生两个重大区别,导致长期来看,肯定是 M 国这套更能出成果:

1. 孩子可以自己聪明,但科技是需要一代代传承探索的,不是靠筛选出一帮聪明人就能解决,况且很多发展科技需要的特征,也不完全和聪明、勤奋对应,更重要的还有兴趣。

2. 先筛选后培养的模式,其实假定了孩子都是线性增长的,即初中厉害、高中厉害、那就可以推导出本科厉害、硕士厉害、博士厉害,然后把资源按照线性增长的能力来分配。

但是孩子不全是这样的。孩子的增长方式有且不仅有线性、U 型、倒 U 型、指数、无规则…

我们的筛选方式,会偏好倒 U 型(伤仲永型)孩子、平等对待线性型(平凡型)孩子、虐待指数型(厚积薄发型)孩子

先筛选后培养的方式,也会把很多本来可以指数增长的孩子,逼迫成为线性孩子:同样的时间,我可以刷十道题,也可以深刻理解一道题背后的逻辑、思路过程、思维方式,但在考试中,选择题海战术的孩子大概率会获胜(参考某中学)。

而先培养后筛选的方式,甚至可以把线性孩子,引导成指数孩子。

而大成果,来自指数增长的孩子,所以我们的教育体系下,培养的孩子容易出现后劲不足的问题。

最后,还想说一点,M 国这套好不好,好,但是很无奈,实施哪一套不是由哪一套好来决定的。我们只能在给定现有约束条件下,做出**最优解**。先培养再筛选的方式好,但是现阶段我们学不来,未来或许可以。

为什么美国中小学生学的数学比我们简单,美国人却还能做出超级牛的东西?

——————————————————————————

欢迎阅读我的其它回答

复旦金融大一新生,如何实现年薪百万的目标?普通人真的有必要理财吗?

知乎用户 Vrijheid 发表

认为英美的数学及其他理科教育难度低于国内是出于无知的自大。

以英美主流的高中课程 alevel 和 ap 为例,英国的高中生一般要在高中时期考取 3 到 4 个 alevel 成绩以申请大学,美国的高中生可以在高中时期考取数额不等的 ap 成绩以换取大学学分(高中时期的考试成绩是被大学认可的,考出成绩的内容在大学就可以不用再学了)。

alevel 中我对数学部分比较熟悉,以 caie 的 alevel 考试要求为例,高中数学分为数学进阶数学两个 alevel 成绩。在数学(考试编号为 9709)中,分为纯数、物理数学和统计三个部分,要学习的内容除了国内高中所要求的知识以外,还包括

纯数中:1. 因式定理和余式定理(高等代数的内容),2. 三角函数与反三角函数(国内侧重于三角等式的计算,英国侧重函数图像,且国内高中几乎不涉及反三角函数),3. 隐函数求导和参数方程求导(求导的概念和多项式函数的求导是初中学的),4. 换元积分法和分部积分法,定积分求回转体体积,5. 平面向量(国内理科生会学,但英国最早是初中生就开始学平面向量了);

物理数学与国内物理中所要求的数学基本一样;

统计中:1. 直方图、累计频率分布图、线箱图,2. 正态分布的概念和标准化,3. 泊松分布,4. 连续型随机变量(概念和概率密度函数),5. 假设检验。

在进阶数学(考试编号为 9231)中,仍然分为纯数、物理数学,和统计三个部分。几乎所有内容都超过国内高中的要求。

纯数中:1. 在极坐标系内求定积分,2. 空间向量的叉乘,3. 空间平面、直线的位置关系和度量(包括两平面之间的夹角、直线与平面之间的夹角、两平面交线、异面直线间距离等;注意这不是立体几何的内容,是空间解析几何的内容),4. 双曲函数的图像和性质,5. 矩阵(包括特征值和特征向量,矩阵对角化),6. 反三角函数和反双曲函数的求导,7. 二阶导数,麦克劳林级数,8. 曲线积分和曲面积分,9. 微分方程(包含二阶微分方程)。

物理数学中,用微积分的工具,再定义和计算更复杂的物理问题,如变加速直线运动(变力作用下的直线运动),刚体,圆周运动,完全弹性和非弹性碰撞。这些内容虽然在国内高中也有,但是并没有应用高等数学的工具。

统计中:1. 连续型随机变量(用微积分处理概率密度函数和累积分布函数),2. t 分布与 t 检验,3. 卡方检验,4. 概率母函数等。

9231 的难度很大,几乎都是国内本科非数学专业都不会学的东西。能学到这并考出好成绩的学生是奔着牛剑的数学、计算机专业去的。

ap 中我对数学和物理部分比较熟。ap 数学只有微积分这一门,这门课基本上和同济大学的高等数学教材内容是重合的,难度低于同济大学的高等数学教材。包括极限的概念和性质,连续和间断,反函数、隐函数、参数方程的求导,微/积分中值定理,回转体体积,已知截面积求体积,斜率场,微分方程的应用(增长模型),级数的审敛法,幂级数的收敛半径和收敛域,泰勒级数和麦克劳林级数,拉格朗日余项和误差界。。。把目录都抄一遍也没什么意思,反正差不多就是高等数学那一整本书。

ap 物理部分分为四个考试,我对 ap 物理 C 力学部分比较熟,其内容就是用微积分的工具定义和计算我们高中除了电磁学以外的内容。比如这样的题:

9709 的内容大多是英国高中生在高一高二学习和考试的,9231 的内容是学有余力的学生在高三继续学习的。AP 微积分大多是美国高二的学生学习和考试的,AP 物理 C 一般要等微积分学到一定程度才能开始学。

国人对英美数学不好的印象源于 alevel 和 ap 均提供了大量选课这一事实。AP 共有 30 多门考试,能在高中阶段考出 8 个良好以上成绩就能得到 AP 国家学者这一考试荣誉了,大多数学生都会参加 3-5 个 AP 考试。Alevel 更是提供了超过 80 门选课,而牛剑的要求也不过是 3A*+1A 的成绩就可以了。诚然,几乎没有学校能把这么多选课都开出来,但是仍为不想学数学和物理的学生提供了大量的其他选择;而想学数学和物理的学生又能得到足够深入的学习。

与之相对应的,国内要求高而国外要求低的有对计算能力的要求,平面几何和立体几何数列。计算能力在我看来是有价值的,计算能力的训练过程中可以积累对数字和形状的感觉,但许多明显能以及明显会被计算器(机)取代的计算过程显得很无聊;国外的数学中也会有对计算能力要求较高的题目,但国际学校的学生普遍计算基本功没有国内经过中学训练的人扎实。平面几何和立体几何几乎是浪费时间,在座各位上过大学的有谁能说说大学阶段还在什么方面延伸了平面、立体几何的知识?国内中学生在几何问题上花费了太多的时间;几何问题可以代数解决,但国内中学几乎不学线性代数和解析几何。数列的话我说不好,英美对数列的要求很快就过渡到无穷数列和无穷级数上去了,而国内高中所要求的各种手法求数列的通项公式对大学进一步学习似乎也没太大帮助,欢迎各位分享自己的观点。

综上:国人对英美数学不好的印象来源于绝大多数西方国家的人不需要且可以不再深入学数学。而愿意在数学上学下去的人,英美数学教育的广度几乎是全面超过国内数学教育的。

看看这些我们大一大二大三才学的数学内容和本科不学的数学内容,别人是高二高三就在学了,我想很难认为英美的数学更简单。

知乎用户 Justin Lee​ 发表

美国能做出超级牛的东西,不是因为他们小学的数学简单,而是因为他们用全世界顶级的大学吸引全世界顶级聪明的人才去留学,而且用全世界最好的待遇和最好的实验室等工作环境,吸引全世界人才把自己的创造力贡献给美国。

2020 年联合国发布世界移民报告:2019 年全球 77 亿人口中有 2.72 亿人为跨国移民,其中近 19% 的移民流入了美国。

据 2018 年皮尤研究中心统计,美国外来移民总数达到 4370 万人,占美国总人口的 13.5%,每年大约有 100 万移民进入美国,其中技术移民的比重逐年上升,全世界 40% 的技术移民流向美国。

2004 年全世界近 40% 的国际学生来美学习,外国学生在美国高校毕业后,多数留在美国继续深造或工作,其中有一半人定居美国,这些人很多成为美国科技界的中坚骨干力量。据统计,1992-1995 年有 68% 的非美国出生的博士留在美国工作与深造,2000-2003 年这一比例上升到 74%,其中学成留美比例最高的国家为中国和印度。

2006 年全美共有 15947 个博士学位授予非美国出生的公民,这一数字占博士授予总数的 37.2%,而在科学及工程学专业这一比例达到 45.2%,其中工程学专业达到 67.7%,特别是电子工程专业更高达 77.2%,在全部科学及工程学博士学位授予者中,中国学者占 26.6%,印度占 10.4%,韩国占 7.4%。

在人才的最顶端,2000-20919 美国诺贝尔奖获得者,物理学奖获得者中有 42% 是被移民取得;化学奖这一比例是 36%;医学奖这一比例是 35%,各项合计 38% 是移民取得的。而且,截至 2018 年末,具有移民身份的美国诺贝尔科学奖得主 85% 留在了美国。

注意,后面这三段中的比例,只是非美国出生的学生的比例,如果再仔细去算算父母一代移民美国的人,他们提供给孩子的教育也为美国贡献了大量顶级人才——数据显示,2016 年在美国出生的 300 万个新生儿中,就有 75 万名宝宝来自移民家庭,也就是 1/4 的美国孩子,出生在移民家庭。

知乎用户 赛赛 发表

美国的教育资源还是很丰富的。对于学习能力最强的那波中学生来说,能够学到的东西是远比中国多的。

中国中学生的上限往往被局限在高考范围内了,并且需要在这个范围内不断的刷熟练度。对于最顶尖的那波中学生,把时间浪费在刷这个熟练度上,其实是非常亏的。少数同学可以去搞学科竞赛,但是学科竞赛的保送名额是非常有限的,一般来说估计着拿不到奖就不会搞选择竞赛了。还有不少情况是蒙错了自己的才能,选择的学科竞赛并不是自己最擅长的科目。说白了还是中学时候刷题太忙了,没有选择的时间,只能赶快看着哪个竞赛顺眼试一下,试一下不行继续回去刷题。

美国高中是选课制的,对于每个课程不要求刷熟练度刷到那么高。如果学习能力比较强,可以选层次更高的课程,而不是把时间花在某一层次课程的习题熟练度上。当然这种操作对于大学录取提出了更大的挑战,同时对于落后地区的学生不太有利。毕竟要想选课也得高中开这课程才行,显然资源丰富的高中可以有更多的课给大家选,资源贫乏的高中估计也没多少课可选,就算学生是天才也无法学到哪些更高的课程了。

我国由于区域发展实在太不平衡,不可能说让发达地区的高中开这些选修课而落后地区采用传统的高考方式。但其实民间已经有这种趋势,发达地区人家自己和美国学校接轨,选修美国课程直接申请美国大学。当然这只有发达地区的少数富裕家庭才玩得起。我估摸着等到我国更发达一些,这可能是未来趋势,也许过个二三十年我国的高中也是选课制了,然后大家也要为学哪门课焦头烂额了。

知乎用户 lqwang 发表

这个问题回答的人已经很多了,以一个华五毕业的前三流毕业生,目前仍旧在工程科研领域做基础工程研发的视角写点东西。

大学时高数的课本是自己学校出的,线性代数的课本也是学校主讲的老师自己编写的。大学高数成绩 80 分上下,线性代数成绩 70 分上下。属于是被卡 G 点的那类人之一。按照在学校的学业成绩,中等偏上,但是可以在保研的名单列表中那种。

问题来了,大学的课程对我这类人而言,很大程度上属于建立了个基本框架。微分方程学习了但是没有基础的建立方程的能力。我直到研究生毕业,直接进入工程领域内,等到必须要应用的时候才对物理场的概念有直观的概念认知,线性代数的空间概念也是看 3Brown1Blue 的视频才将矩阵、空间变化、这些概念串联起来。到自己能手推建立弦振动微分方程也已经硕士毕业两年后了。泛函变分的概念又耗费了一年多。

长期对物理概念缺乏直观理解,我一直归结于自己比较菜而且笨,数学功底差。后来因为工程试验需要,深感自己动力学概念不足,对试验的基础概念不清晰。找到 MIT 的 Walter Lewin 课程之后,才发现我们的大学课程真的懒透顶了。

【中英双字】8.03 - MIT Physics III: Vibrations and Waves Lectures by Walter Lewin_哔哩哔哩_bilibili

抽象理念的概念起源于现实物理现象而高于物理认知。我们的数理教育体系,对于基础的物理概念从来也没重视过,人老头为了解释多自由度振动系统可以弄个双质量摆模型; 一门课程将波涉及到的概念和相关试验悉数列出。这种教育课程下的学生,哪怕基础概念忘了,但是试验现象在大脑中的印记可是长远存在。

一定程度上来讲,从我看过的老外撰写的教材来看,人家就是照顾绝大多数人的思路。从基础概念写起来,有推导写推导,有物理背景很少直接上公式。80% 的人有可能真的能学清楚一些基础概念,深度方面也提供了往下走的可能。国内的绝大多数课本,一般就适用于前 10% 的高水平人才,这类人无论在哪种教育体系下都是耀眼的存;但是中间可能有接近 50% 的人,能拯救一波的在国内的理工科教育体系下就被废掉了。按照我们老板的说法,成了业内吹牛皮水平还可以,但是底色一般。

知乎用户 知乎用户 xh32ct 发表

亲身经历,和许多美国博士共事过,也给很多美国本科生做过助教。

先说学习简单这个问题。美国人,很多人是真蠢,我就不说红脖子,毕竟我接触的人有限,就说本科生,很多人可以蠢到没有基本常识。而且美国对人对这样落后的人是很宽容的,他们也不觉得自己缺乏知识或者不聪明有什么问题。更有甚者,美国人大都博爱,很多有学习困难的人在他们看起来应该得到更多资源去帮助。我并不是说有障碍或者困难的人不应该得到帮助,可问题是,我教过的很多学生,真正就是懒。我知道很多人可能真有学习障碍,但是你旷课不交作业还要我给你提供单独辅导考试还要双倍时间考完让还我给你通融,因为你有障碍,我真正觉得不应该。但是美国人,对他们太宽容了,说美国大学难毕业,但这种声称自己 “disable” 的,没有不让过的,导致很多人弄个 disable 成了策略,毕竟 mental health 的问题,不难操作。

再说精英这部分,他们聪明起来是真聪明,和很多东亚做题家不同,那种聪明不是后天习得的,是天赋异禀,而且在舒适无压力的环境下得到了充分发展。他们很多人都有丰富的爱好,擅长很多东西,很多和专业相关,很多无关,他们也不嫌浪费时间。他们对人对事都充满希望和热情,有时候觉得他们眼里有星星。他们在学习上一点就通,一说就会,甚至不说他也能自学,而且他们很多人真的只是热爱知识和学习,所以怎个过程轻松愉悦。毕竟他们生活压力太小,这种水平随便找个工作也是一毕业就中产上游。一栋房子花园泳池孩子车子,屋里有猫和狗,轻轻松松。他们学习,是凭理想和爱好,我从没在国内见过这样的学生。大概我在国内上的并不是清华北大这种顶尖学府,所以也没见过国内的大神,但是我在美国教的也不是他们的顶尖学生,就是普通学生中的优秀者都能如此之神,确实让我感慨。

我个人认为,因为他们的社会压力小,普通人也能过上挺好的生活,也因为资源丰富能对弱者施舍,充满同情,所以他们没有天赋的人是没必要逼迫自己学习的,而有天赋的人,在这样舒适的环境下可以找到最适合自己发展的道路,才华得到了充分培养。我一个尖子学生两年间,参加一个学生组织的慈善项目,跑非洲给他们建一个小桥,学生在工程师的指导下自己设计自己动手建造。他也往墨西哥跑,在那参加一个水利设施建设项目。他找学校师傅学了电焊,在实验室帮我们造一些对焊接质量要求不高的结构。他擅长养蜜蜂,自己设计建造了一堆养蜜蜂的东西。他喜欢天文,经常拿着他的望远镜去野营,讲到天文能如数家珍。他家有牧场,他放假就要照顾那些动物,他对动物的事情能侃侃而谈。他也擅长编程,我好多计算需要自己写的程序外包给他做。更重要的事,这样的人比比皆是。

这样的体制,缺点是明显的,优点也是。反之,我们亦然。没有必要妄自菲薄,也一定不能骄傲自满。

知乎用户 温酒 发表

美国是美国精英的美国,

中国是全国人民的中国。

美国有成绩是因为美国精英有能力有条件,

中国相对没成绩是因为有能力没条件。

知乎用户 babyquant​ 发表

首先,就数学而言,美国高中教育不浅。比如麻省理工、斯坦福都有数学相关的课程、冬令营、夏令营等,有大学教授指导做研究,都是正式的数学研究。其实很多猜想的解答也仅仅用到普通的线性代数,应用类的就更不用说了。这些学生可以在告知就接触到前沿的数学研究,这些经历比 IMO 金牌的含金量更大,对申请名校很快更有帮助,基本上 90% 被哈佛、普林、麻省、斯坦福本科录取。

其次,中国高中数学教育不是深而是刁。由于只能考高中内容,只能在有限的知识点上追求刁钻,很多都是为了选拔而非研究,内卷严重,创新不足。

最后,长远发展需要强烈的兴趣支撑。但高中这种在极窄的知识体系范围内反复训练,会极大地磨灭学生的数学热情,而且竞赛跟各种利益捆绑容易滋生功利心,这些都对长远发展不利。比如俄罗斯佩雷尔曼可以心无杂念在家啃老证明庞加莱猜想,在中国这么做则会面临强大的舆论压力。

当然,美国数学也有很多问题,其实这几年由于反智主义盛行,美国政府对基础科学的资助日渐减少,更多依靠西蒙斯、DE Shaw 等私人资助,稳定性很难有保证,他们本质上是靠金融交易赚钱资助基础科研,万一交易亏钱呢?政府税收其实更有保证。

知乎用户 俗不可耐 发表

因为

1. 工业化早

2. 有钱

我知道很多答案喜欢说美国的精英好厉害,培养他们都是大牛……

是,但是这些精英需要消耗多少资源呢?一个私立高中一个学期的学费八万美元。

那么问题又来了,这些被重金培养出来的大牛们,究竟对科研的贡献如何呢?

就看自然指数吧,在所谓中国大量人才流逝到欧美人均 GDP 不足美国 1/6,科研底子比美国要薄弱得多,而且学术山头大都是美国人的情况下中国的自然指数已经达到了美国的 2/3。

那么问题来了,美国的教育效率真的那么高吗?

举个例子,你用了二十年拥有了马云儿子 1/3 的财富,请问是你的教育成功还是马云儿子的教育比你更成功?

很多人以为科研就像百米赛跑,一声枪响谁快谁厉害。其实科研是接力跑,一棒落后,后面要一棒一棒地追。

尤其是现代科学,别的不说单纯就是把科研用的仪器用好用精都不是一件容易的事。更不要说自己搭系统了。

举个例子,我当年搞科研的时候就是想复制美国一个课题组的实验,结果横竖就是得不到同样的结果。后来师兄去加州大学实地研究,发现我们两个课题组不是差一点。就说我们用的镜头吧,从德国买要一个月才能到货。而他们楼下就是德国蔡司的分店,直接去挑个镜头,试着哪个好就拿来用,挑好了给钱。这只是其中一例。

那么问题来了,这种差距跟教育有多大关系?说白了,起步早,合作旧,经费多建立起来的互信,这是一天两天能建立起来的吗?

所以我们做个假设,如果美国教育体制,科研能力真的比中国强,在硬件上又有那么大优势,那么对不起,中国根本就不会有任何追赶的可能性,因为人家原本就比你领先,而且还跑得更快!

但是如今中国正以肉眼可见的速度在科技上追赶美国,这是为什么呢?这不正是说明中国的教育路子是对头的吗?

而且,假如我们排除掉美国吸引国际顶尖科学家的这个因素,单纯就看美国本土培养的科学家这些年来的科研共享,结果会更加难看。

许多人都有一种美好的幻想,觉得有一种教育方式,它轻松愉快,孩子们不用被迫去反复做基础练习,然后早早就找到了自己的方向,专注刻苦,舒舒服服地改变世界。

真实的情况是,不管你天分多高,五岁不练钢琴,这辈子别想成钢琴大师;六岁不压腿,这辈子别想成为舞蹈大师;七岁不练画,这辈子别想当画家…… 同理不从小开始在数学上反复练习,过了某个年纪就完全丧失了搞科研的能力。高强度的基础训练才能最大限度的培养具备科研能力的人才。

中国没有金融霸权,不能海外殖民,也没法从世界各地高薪聘用人才,唯一的方法就是拼人,就是培养更多的科研人才,工程师,一点一点的产业升级。

时至今日,日本,英国,甚至美国都在反思降低中小学生学业难度的负面影响。

以下是 2018 年,PISA 评估结果。有创纪录的 79 个国家参加,测评的学生样本人数为 60 万。

经济合作与发展组织由 37 个国家组成,当中大多数是像美国、日本和欧盟成员国等发达国家。

值得一提的是,中国虽然成绩最好,但是参加测试的只有北京,上海,浙江和江苏四个地区。不过哪怕就这四个地区也有 1.8 亿人,人口超过日本,比世界第八大人口大国还要多两千万人。

中国也正是凭借着教育,一点一点地在进行追赶。

我当然不是说中国教育体制就完美,我只是想说,美国式的公共教育减负,私有教育残酷绝对不是利国利民的道路,中国如果这么玩儿,最多两代也会每况愈下。

创造力必须要结合扎实的基础才能变成伟大的成就。没有扎实的基础,所谓的创造力不过是混沌无序的盲动。鼓吹创造力,天性,天赋而对扎实基础避而不谈,这是在误导大众。

其实没有天赋尚有扎实基础至少还能安身立命,找份工作糊口。如果连扎实基础都没有,只有天马行空的胡思乱想,这就是又熊又不老实。自己觉得自己很厉害,但其实什么都不懂。

看看美国现在那些反智的言论,今天已经影响到了国家的宏观政策,这就是前车之鉴。

最后,还是用一个简单的比喻来回答问题吧。

如果有一天,你发现自己拼劲全力却仅仅这北京买了套房,那么你是会接着让自己的孩子继续奋斗,还是让他去学王思聪天天玩儿网红把你那套房产也败光?

兴何以兴,亡何以亡,可不能本末倒置!

知乎用户 鱼昆​​ 发表

杨振宁现在回归中国国籍了,然而使他获得诺贝尔奖的可是他以美国公民的身份在美国的大学做出的成果。(经评论区指正,杨老是 1964 年才加入的美籍,1957 年获得诺奖时还是民国国籍,大家理解我的意思就好…)

“美国人” 的基础教育不见得和美国有什么关系,美国科研实力的强悍很大程度上是因为它挖走了各国的顶尖人才。这是原因之一。

原因之二是,那些真正的大佬之间的对比和普通学生之间的对比完全是两个概念。

IMO,IPhO 金牌和 top2 普通学生的差距大概与 top2 普通学生和一般一本学生的差距差不多大。这些人的能力强到如此地步,以至于为了研究他们的成长你需要重新建立一套评判体系。而题目中所说的 “基础教育的内容” 对这部分人的影响非常有限——无论中国还是美国的基础教育,对他们来讲都太过简单。基础教育阶段他们的成长只能严重依赖自学和师长的引导,而这又是一套完全不同的体系了。

或许还可以指出第三个原因: 在国内生活水平与美国有可比性之后出生的那些人还没到做出重要贡献的时候。

改革开放刚开始的时候,大家还在为吃饭发愁,这个时期经济发展的滞后决定了我国的教育产业——无论采用应试教育与否——是没有办法与美国相比的。在经济基础相对完善之后,教育体系的可比性才会显现出来,而那已经是上世纪 90 年代之后,甚至是本世纪的事情了。

然而,在经济发展起来后出生,接受基础教育的人,最年长的也不过三十几岁,这代人一生中的绝大多数贡献都还没有被做出。所以说从 “结果” 出发的对我国与美国教育体系的对比研究,还为时过早。

知乎用户 Terminus​ 发表

学的东西比较简单的时候,比如高中时期,中国学生被逼着往死里挖坑,比如一个圆锥曲线,非得用各种神奇的解法乱干,让你学三年都做不明白。你高中后还用过圆锥曲线那些傻逼解法吗?

甚至很多人都因此产生了数学没有实用价值的感觉,对数学产生了厌恶和反感。

等到了大学,各种解物理模型必须的三维积分,向量分析,多重积分,偏微分方程之类的,这可都是到了各个专业领域解决实际问题的最实用的数学,今天学完明天实验室做 IMU 算法就要用的,反倒不卡你了。高数课糊弄糊弄画画重点就放你过了。

本末倒置说的就是中国教育

知乎用户 小镇青年 发表

小孩上学之前,我也以为中国的教育重基础,基础扎实也不是坏事。

产生这种印象,主要是因为自己当年上学的时候还没有那么多应试教育

小孩上学以后,我发现现在小学的数学教育就是在成体系地摧毁中国人的数学能力。

真不夸张。

这还是在上海,某个口碑还不错的学校。

在知乎看到的一个人说得很好,现在的学生就是那种,能用公式把魔方的六个面都转出同一个颜色,但却不知道如何只转出一个面同一个颜色的那种。

你们以为中国的数学教育很差吗?我告诉你这还不是最糟糕的时候,会一代比一代差。

应试教育很糟糕,非常糟糕。

怎么说呢。

好的理科学习,主要是数学物理,要启发人思考,结果其实没有这么重要,思考的过程,研究的方法远远比一个 “Yes” 或者 “No”的结果有意思。

好的数学教育,应该尽可能启发小朋友去思考,并享受这个过程。

所以,小学阶段的东西,不能太难,必须是小朋友能想出来的东西。

现在的小学数学(不是课本上要求的,是老师出的)已经出现了太多超纲的东西,这些东西已经超出了小朋友自己 “能想出来” 的范畴。

但是,老师和家长又要一个结果(分数)。

怎么办,只能去套题型,把所有的题,看成一个 “套路”,看到题目,学生先回忆这是个什么 “套路”,老师有没有讲过什么解题步骤,能不能套公式。

真正天才我不知道啊。

一般聪明的小孩,钻到这个套子里,数学,基本上来说,就完了。也许他能凭借聪明或者努力,拿个高分,考个大学,也就这样了。

真正的数学是什么,他早就不知道了。

他知道的是,这是个什么题型,老师或者书上讲过什么东西,能不能套,套出来万事大吉,套不出来,就懵逼了。

我以前啊,应试教育的天罗地网还没有这么严密,我算是个漏网之鱼,能享受到学习的快乐,现在的小孩,从小就在套套套,套你麻痹。

我儿子跟我说。数学很无聊,从小就学数学的孩子很可怜。

我只能用自己的方法,把老师给他灌输进去的狗屎掏出来,再灌输点有意思的东西,感觉很艰难,努力吧。

没生小孩,如果你不能保证小孩不受到应试教育的荼毒,不生最好,利人利己。


回到正题,小学课本,简单点好,简单点好。

普通孩子,不会受到羞辱。

聪明孩子,不会被无聊的机械训练束缚思维。

有些人胡扯什么教育公平,你应试教育出来,就不搬砖了吗?

反正迟早都是搬砖的命,小学开心点不好吗?

知乎用户 空一格 发表

物质基础决定上层建筑。

美国数学牛,数学家也没几个本土出身的美国人。

美国以其地理上得天独厚的条件,一战前后,大量欧洲的数学家和人才跑到了美国。

二战后,美国作为未受任何侵扰的战胜国,又收割了一批欧洲的数学人才。

冷战后,苏联解体,原来的数学人才又被收入美国。再加上冷战后美国一家独强,全世界的人才都被美国强大的教育资本吸纳过去。

比如最近 AI 火热,多伦多大学号称 AI 曾领先(因其在 AI 未火前二十年来持续投入没间断),然而多大以前搞 AI 的几个人现在都到美国了。

由于惯性的作用,很长一段时间(可能半个世纪)美国的数学科研都将世界领先。

但这不代表美国的数学教育好。

什么鸟兴趣教学?兴趣能教好学才怪。

是因为美国强大,所以觉得它的教育好。不是因为美国教育好,所以它才强大。逻辑要搞清楚。

知乎用户 众生同心 发表

我曾经坚定的为我的国辩解,填鸭式教育是为了解决我们国家的文盲。

但是,当我综合了一切信息以后,我突然发现不香了。

填鸭式教育应该到了初中就能够解决当时劳动力的需求。

为什么到了高中?还要填鸭式教育?

高中,累死累活,算求导。其实一个洛必达法则就解决了。极限,微积分,深刻的解释了高中数学的某些费力不讨好的知识点。双曲线函数基本大部分人都用不到,能用到哪里呢?

当我回过头来,仔细回想,高中老师教的英语。我突然发现。英语不是难?而是有的人需要它难。因为高中英语,如果学到了点子上。基本一个月就能搞定语法,包括口语。听力可能需要练习。最多不过三个月。

也就是说,中式教育的高中完全是在脱裤子放屁,多次一举。

似乎高中就是特别设置的一环过滤器,服从性测试的第一环。

让服从的人,跟着某些人走。让不服从的人,活在最底层,这个国家需要安定。

这些服从性的人在以后,很有可能,即使身处高位,也不会搞出什么大乱子。很有可能会再次服从。

阴谋论有待商榷。

那么问题就来了?国家为什么要设置高中这一个脱裤子放屁的环节呢?

没有答案,至少我不知道。

就拿我自身来说,我如果不去主动学习编程。我可能永远也无法掌握对事物本质提取的能力,也无法去抽象整个世界逻辑。

我更有可能跟我的爸妈一样,上个工地,挣点小钱。然后一辈子活在自我否定和自我质疑的怪圈之中。

我问过许多人,他们都对自己有些极其严重的自我否定和自我怀疑。他们自己都认为自己不可能成功的做好一件事,他们一遍遍又又一遍的重复着,那个未知却又确定的答案。世界是他们的,我只是小角色。我只是个普通人。

他们在高中被老师一遍又一遍洗脑,人要有自知之明。被失败的成绩摧毁的内心似乎让他们认了命。他们渐渐的认清了现实。

世界很美好,生活也很美好。但不属于我。包括我自己,我会深深的陷入自我怀疑和否定中。

我承认的确会有那种天赋异禀的人存在,但这样的人很少,至少 99% 的人差异不大。然而,中国的教育人为的用三年高中,对没有进入筛选系统的人大肆进行身心都摧残。

最原本的状态难道不是应该是,即使我在高考输了你,但是别灰心,我会在下一场重新赢回来。

然而事实上,经过摧残以后,要么走了极端。成为读书无用论的坚定支持者,基本放弃了学习。要么一辈子活在自我否定,自我怀疑中,我们的确是普通人,但他并没有所谓的天赋异禀,他也是普通人。人都是一样的。基本上不存在个性太大的差异。

在 B 站,在知乎,在社会,在整个中国,学历歧视真实存在。我承认这个歧视,但是连我,都很奇怪,为什么我会比,985,211,清华北大低一头。从昨天以前,还一直用的是清北大佬。

大佬是我默认给清北打的标签。也就是直接在没有竞争的情况下,我已经认了输。也就是说,还没开始,我就已经输了。当然,现实也是,我似乎真的输了。

但当你讨论什么是输赢的标准时?你会发现这个标准是不准的?

初中毕业进厂的人,输了吧。

考了中专的人输了吧。

高中毕业进厂的人输了吧。

考上大专的人输了吧。

二本输了。

普本输了。

唯有,985,211,清华,北大赢了,这就是中国的标准。中国强制的把所有人的标准变为这个标准。但,最本质的标准却是。

那么最本质的标准呢?是什么呢?是在不破坏他人利益的情况下,随心所欲,每个人都有每个都追求。每个人都可以选择自己的生活方式。

这是一个竞争机制嘛,的确需要筛选。但筛选的过程和方式不对。我这里不谈筛选结果。

筛选过程和筛选方式却是服从性测试和废物测试,使得我们未来几十年都将会在人生中被刻下阴影。

为什么会说服从性测试?在智商上不存在特异性?只有在性格上存在特异性?也就是说。高中教育的测试标准,测试方式是符合某些人性格上的特异性。但并不是所有人都有这种特异性。那么,对于没有特异性的人来说,他们只能强制自己跟随标准去改变自己。而哪些没有去强制改变自己的人,就是差生了。

多元智商更符合实际情况,每个人都有其闪光点。

但是,中国的教育只认填鸭式的成功,文化课的成功。

我始终弄不懂的是?为什么会存在歧视艺术生这样一种现象和情况。

为什么大多数人的学习不好?除了,没有好的学习方法以外?更多的就在于,学习教的是不符合人的特异性的。也就是,中国的教育在基数上是反人性的。

服从的人筛选出来,大多数是服从的,不服从的人自然就是社会的底层了。

第二个就是废物测试,如果说服从性测试还能接受。那么废物测试简直就是灭绝人性,给一个十几二十岁的人进行身心的摧残。

就好像在阉割我们一样。

是的。我们的生活方式像被阉割过一样。

包括那些既得利益者,要么是经过了服从性测试,要么是被失败成绩摧毁了内心的人。

可怕的是后者,就拿我的一个朋友来说,她高中毕业。我跟她说,你自考,考上大专。考上大专以后,工作会体面一点。因为她要么做销售,要么就进厂。一来,她做不了销售?二来,进厂太磨人了。我跟她说,你先试试再说。但她压根没有这方面的信心。

不是大专和大专以下的人不愿意学习。而是被高中摧残过内心以后,坚定认定自己就是个废物。这是多可怕的一件事。中国的教育已经把人性摧残至此。使得,他们一进入社会,就比高学历矮了一头。

从各种相亲平台论述者中,我能感受到那若有若无的透露出自卑的心理。中国的整个社会,整个环境都是畸形的。

一个二十几岁的年轻人,本应该富有朝气和冲劲。

但是,你看看,高考失败者们,会怎么看待自己呢?

我是个高考失败者,我比不上那些高考成功者。我是个废物,从进入社会开始,就会踏踏实实比高考成功者低了一头。深深的自我怀疑和自我否定。但大部分看起来很正常,但是一旦仔细观察,会发现大部分其实是自卑的。

这是多么恐怖和可怕的一件事。事物有其自身的发展规律,不付出就索取回报是可耻的。但是,这是通过摧毁一个人的信心,一个人都立足之点,把所有人都变成了一个怪物。去摧毁了人性。

人在任何时候,都要在主观上承认自己的价值。客观上也许真的有差异。但是中式教育直接扭曲了一个少年的人性,使得,少年人在主客观上直接先认输。

你可以自行思考一下,没有被国家高等教育认可的那部分人基本不会去主动学习,也从来不会再次去重拾学习,他们也看不到问题的本质。

我们尊重高学历者,因为那是他们努力的结果,他们应该被我们尊重。但是,在他们面前,我们也无需自卑,他们成功在了社会需要他们成功的地方,而我们没有。

知乎用户 斩浪之刃​ 发表

用一个切身体会来讲这个问题吧,这可以解释某些关于美帝教育的问题。

我的高中是在美帝读的。作为中国人,我的数学和学校里那些成绩不错的美国童鞋一样,学到了 AP Calculus BC。由于同课的大多数童鞋都是 senior,都已经拿到了大学录取通知书,马上就要毕业了,所以许多人已经提不起精神去准备 AP 考试了,觉得实在大不了去大学再学一遍就是了,说不准还能用来刷 GPA 呢。

为了激励同学们,老师拿出了一套 1969 年的 AP Calculus AB(BC 的预备课)的题目给我们做。刚开始同学们觉得老师是在搞笑,然而题目拿到手以后都懵了:谁也没想到 1969 年的 Calculus AB 居然会比最近几年的 BC 还难,而且是难太多以至于连选择题也全得靠蒙。下课后老师语重心长地对着一脸懵逼的我们说:“千万要好好学习,要知道相比于你们的前辈,你们差的还很远。”

后来我查了一下才知道,本来美帝的教育一直都是很有难度与挑战的。然而自打智商可能有点捉急、小时候学习可能有点吃力的小布什上台后,为了给孩子们减负,提出了 “No children left behind” 计划,这其中最重要的一项就是强制要求各个学校必须保证学科通过率与毕业率。

但这不代表学生们就一定能把题做出来,于是各个学校就开始不断降低课业难度以保证这一点,这才为今天的美帝教育带来了无数槽点。

很多人吐槽过美帝教育,譬如说认为完全是贵族教育,以及发现越来越多的年轻教授与 RA 都是来自其他国家;而为美帝教育辩护的人则说美国仍然是世界科技强国。我个人认为两种解释都对:那些真刀真枪杀出来的老一辈教授与科学家仍然是美国的科学基石并继续带领美国走在世界前沿,然而现在这些在贵族教育环境下出来的孩子已经很难看到像他们前辈那样的能力了。

现在的中国教育,处处可见当年美国教育的影子。然而未来的中国教育会不会看到今天美国教育的影子?不好说了。

————

谢赞~

有人质疑说耶大毕业的小布什不会智商捉急。这是因为在中国就算你的一大家子人都是北大毕业的也和你进北大没什么直接关系,然而在美国,如果你家有哪怕一个耶大毕业生,也都可以显著提高你申请耶大的成功率,更不算如果你父亲爷爷都是耶大的,那么你进耶大也基本上就是顺理成章的事(就算不是,考虑下布什家族的实力,也差不多了)。

知乎用户 高斯白乎 发表

这个问题非常重要。

我自己不知道答案,也不确定已有的回答是否就找到了答案。

但非常希望有人做个调查,不光数学,也包括其它领域,看最顶尖的那些人的教育经历:什么样的家庭背景与家庭教育,在哪里受的什么样的初等教育,在哪里受的什么样的高等教育,在哪里做出的重要成果,如此等等吧。

先不要下结论,先找到数据。然后再思考为什么这样。相信有了这些数据,更一目了然。

咱么这一代已经过来了,为的是下一代。一方面希望现在的应试教育有所改观,一方面希望不要走到另一个极端:反智。

我回答这个问题,虽然只有几句话,也还是战战兢兢,生怕误导别人。因为这个问题实在太重要了。

知乎用户 peter.z 发表

我一直觉得,这个问题是个伪问题。

一个国家的科学技术水平,甚至包括社会科学、人文等学科的成就,跟中小学甚至大学本科教育到底有毛线关系呢?

美国在科学技术领域的成就,难道不是因为科研人员的待遇更优越、社会地位更合理,国家对科研的投入更大带来的吗?跟中小学教育到底有毛线关系?

知乎用户 梅华龙​ 发表

我在国内上到本科毕业,在哈佛读了硕士,目前博士末期。

此答案有事实层面的猜测与实际不符或无法证明。

第一,应试教育确实可以让学生产生分数上的满足感,但这和好奇心求知欲不矛盾。我自己和不少我熟悉的同学都对当时课内和课外的话题产生了很强的好奇心

实事求是地说,我感觉这种好奇心来自于学龄前时期,在上学时已经基本定型。但其他同学未必。比如,我们学校每个年级当时都有人在初中或高中某个时期开始自主学第二外语或语言学常识,而我们全都是每个月要考试的应试教育学生,并且大部分都是从零下十度到零上三十五度都要自己骑自行车上下学的平民老百姓子弟。

我在中学期间自己买了法语、德语(我们中学有慢速兴趣班级别的二外课)、西班牙语的教材,荷兰语小册子,甚至还有一本塞尔维亚语教材但这本基本没学过。买了汉语方言学和普通语言学的书籍,让我爸去单位打印的小语种资料(当时可没现在照电子材料那么方便)涵盖另四门以上的语言。绝没有老师留作业让我去做,更无考试压力。

应试教育这块我也基本应付着,高中考试考过几次年级第一。

我没觉得二者之间有任何矛盾。我认为应试教育是基础。应试教育的语文和英语你搞不清楚,你可能真有兴趣和资格在中小学学二外吗?中学数理化一塌糊涂的话,你有可能对世界充满兴趣吗?历史会考几道大题背不下来的人能自发去研究古埃及或早期中国?这不是扯淡是扯哪儿?

到现在我都觉得应试教育给我的知识是我一辈子最重要的基础,并且也是一个人好奇心和上进心的来源。

第二,美国真的没有应试教育吗?他们只不过用一个选拔方法,使得成绩最好、最有潜力、最努力、最有好奇心的学生都不足以依靠自己在学业上的成绩或天赋考上这些精英校,如哈佛。你要有一系列的因素被考察,而每一项归根结底都变成家庭背景、经济实力。这就是所谓精英化。

但这批人到大学之后并不是个个成才。恰恰相反,很多人基本的学习能力以北大清华的标准看是不合格的。

不过这些人比中国学生还在乎成绩。因为法学院医学院乃至精英圈子找工作都看 gpa。我助教过的几十个本科生,有好几个对成绩的痴迷如同病态。除了体育生之外所有人不给他 full A 就恨不得跟你打架。抄袭作弊两人次,其中一个相当于洗稿,就这哈佛老师都不管。消费者是上帝啊谁敢管,人家爹妈爷爷奶奶说不定都比你一个哈佛教授是人物。

还有为了高分,平时不学习,考前找我谈话四小时的。

也有拿了 a - 给我写信一年我不理的。

还有哭天抹泪说自己阿姨死了能不能照顾照顾给高分的。

还有哈佛中国同学给精英子弟当中学伴读,她是河北省考生,都觉得那个美国孩子一天的生活太枯燥死板了…… 精确到半小时。你们家的素质教育……

他们的求知欲我是没看出来。求分欲比我这个过去总算计这次能考第几啊别让老师笑话啊的人厉害五百多倍吧……

至于美国科研为什么仍然厉害?

科研也不是这群小时候精英教育中的大多数人弄出来的啊。有一些是肯定没错,但大多数人家教育是为了保持在同一个阶层,资源留住,谁要去做科研啊?

一线科研美国钱多,资源多,时间做得长。过去从德国、苏联也继承了一些。靠金融和军事霸权维持较高的生活水平吸引全世界科研人员。

中国科研人员工资一涨就回去一批,人家美国这方面的投入这都多少年了?

好奇心求知欲我觉得还是看大众层面。一个国家不会用一切资源,归根结底是短视的。为什么妇女能顶半边天?按照旧社会,妇女在家呆着呗,上什么学,那全社会的创造力和劳动力就少了一半。同样,靠自私自利地精英阶级和雇佣兵维系霸权是有风险的。

人家美国可以一路走来很开心,但我们中国没资格更不应该去学啊。

鼓吹应试教育无用者,你们的言论对我们国家真的没有好处,对知乎上我们这些普通人没有好处。你们还是去国外鼓吹吧。

平民教育难道不是我们中国国家最大的功绩吗?本身科研就是研究生以上的人来做的,学历就已经分级了。怎么着还想从阶级上分层?中国古代有这方面的经验,效果如何?

知乎用户 invalid s​ 发表

编个故事……

我是游戏迷。当年玩魔兽世界,打英雄本,带我公会几个新人玩英雄本。

当时英雄本难度挺高的;但相比于 RAID 本,那就不算什么了。

公会嘛,目标自然是 raid 推 boss 去。所以能全部带成 raid 骨干最好。

然而不同玩家水平、目标不同。有的就是喜欢看风景、爬山,有的想抓拉风的宠物,有的能一天 24 小时蹲 AH(拍卖行)低买高卖倒腾物资…… 愿意挑战超高难度的 RAID 的硬核玩家是很少的。

那么,如何尽可能的把他们更多人带进 RAID 呢?

当然,想要进 RAID,他们需要掌握:

1、配装知识

2、boss 技能作用原理及各职业对策

3、输出循环

4、装备属性收益计算方法

5、装备收益递减公式

6、一次掷筛二次掷筛的区别与联系,以及由此给坦克装备选择带来的变数

7、技能间的互动机制(本职业)

8、不同职业技能间的交叉互动

9、技能数值设计及收益递减排序

10、怪物仇恨机制和嘲讽作用原理

11、回蓝 / 能量恢复和输出 / 治疗的关系

12、不同 boss 不同阶段不同位置应对策略

那么,怎么讲呢?

策略 1:我写一本书,把以上内容全列出来,爱看不看!

策略 2:来,乖,你现在上幼儿园,别看你的微积分了!我们今天算 10 以内加减法,来,先把这份卷纸做 20 遍!

策略 3:公式太难背?没关系。来,跟我念顺口溜…… 什么?公式有什么作用?你先背!10 年后参加工作了就知道了……

策略 4:不行!不能贪玩!电脑都收掉,不把公式背完不准摸电脑!

策略 5:来,这里有五百万道题。统统给我刷完!就不信你们学不会。

你猜这样能不能教出来 “硬核玩家”。


你猜我是怎么教的?

来。跟着我,开打。

打完,xxx,知道你为什么会死吗?不明白?怪物有仇恨概念,仇恨大概和伤害量成正比。所以坦克起手引怪是没有多少仇恨的,你直接一个大火球呼过去,你不死谁死!

记住了,看坦克给怪物上破甲,破甲是一种伤害和仇恨换算比例极高的技能。上了三层破甲,随便你怎么打。

对,嫌麻烦的话,记住 “三破开打” 就好了。别抢输出。

还有,治疗也一样,要么开战前你给坦克加满,要么等坦克上完三次破甲你再给他治疗。坦克还没接触小怪呢,你先丢个 hot,怪物不找你找谁?

你们记住这点就够了。差不多到 raid 都够用。当然,想玩极限输出、赚 DPS 奖金或者治疗补贴的话,我可以给你讲的更深一些。这里面好玩的就多了,我先给你说说远程 OT 和近战 OT 的区别吧……

讲完了?没有。这里面东西多着呢。比如 raid 灭团后,为什么总有人能赖着不死?

这个先不讲,耽误时间。你看 xx 都急了。我们继续刷副本,有兴趣可以到 nga 看帖,也可以刷完本私信问我。现在我们先玩。

看出问题在哪了吗?

这几个玩家,他们的目标不同、兴趣点也各不相同。

其中,A 不喜欢理论,但挺享受征服副本的过程。所以我不事先给他讲原理,等他犯错了才点出来。

而 B 呢,好奇心特重,理解力超强,好奇宝宝一样,总想打破砂锅问到底。而且你直接给他讲公式他都听得懂,知道往哪用。但和他讲多了,A 就有意见了:“要睡着了…… 还打不打啊?”

另外两个,C 和 D 呢,和这两个又不一样。他俩没兴趣玩,就是个拖油瓶,过来混个装备打个酱油。你给他们讲,他们听不懂也不想听。挂了?挂了就挂了。反正你强你能单挑,我们就是来混装备来了……

混装备也行。我给讲 ninja 规则,讲为什么不是自己能用的装备都可以需求——混装备本身并不惹人反感;连混装备的规则都不知道,将来连我们公会都会被人指着脊梁骨骂的!

什么?公会要这些混子干嘛?

什么叫混子。这叫生活玩家。看看公会仓库满满当当的顶级药水、金币……

没这些人,这东西从哪来?

好了,最后,公会会长问了:都带的怎么样了?

怎么回答呢?

C 和 D 小学毕业了。有了基本的装备,也知道基本的混装备礼节,不会丢我们的人了。之后随他们吧。当然,愿意继续跟着我刷本也行,他们还有很多东西差的远。

B 现在就可以吸收进团,强 X 给他点装备,当主力培养吧。没错,可以出师了。就差到 RAID 灭几次团了。

A 还不太行。现在的副本压力还不够大,他的很多问题自己还意识不到,现在进 RAID 就是个 “灭团发动机”,我带他再专门打几天那个最阴间的英雄本,让他死出新花样、新高度。等死出来了,在 RAID 里再带带就会了。

你看,C 和 D 那样的,你硬拔苗助长也拔不起来。又费精力又费神,还平白惹人家恨你——这货,嘴怎么这么碎!

而且,你硬拔 C 和 D 时,A 和 B 就流失了。这两位可是难得一见的硬核玩家啊!

结果你硬按着他俩,陪着 C 和 D 一遍遍的死记硬背 “选怪物做标记时必须用左键、左键、左键!别用右键点,一点你的箭就射出去了,会灭团!灭团!灭团!”

这样背一天两天,他们能耐住性子;可这样背半年……

如果你能硬摁住 A 和 B,那么你会把他们教成 C 和 D 一样、连英雄副本挑大梁都做不到的低端玩家。

更可能的是,人家捏着鼻子坚持三天,之后就该果断退会、到别的公会碰运气去了。


那么,如果你来评价我的教学,会如何评价呢?

1、考核太简单了!不 ninja 就能过?这种垃圾老师带团,怎么可能带出来人才…… 什么?他给公会带出来的高端玩家最多?公会最强的新人一大半都他带出来的?不可能,这世界错了……

2、不是…… 这上课都上的什么啊?给第一次玩魔兽刚满级的人讲二次掷筛?不是,这东西一流大公会有几个人知道?给新人讲,不是浪费时间吗?

不过,恐怕也有人已经看出我的 “心法” 核心了。

什么心法?

1、考核最低标准依照 C 和 D 设置,以 “不丢人” 为目标

2、教学以 A 和 B 为中心,以 RAID 为目标

3、不因为 C 和 D 去限制 A 和 B,更不为了 C 和 D 达标而让 A 和 B 陪着受罪——恰恰相反,为了培养 A 和 B,宁可让 C 和 D 听点天书。反正他们自己会忽略的。

4、着重培养自学能力,“课堂” 上不讲解过深,点出问题核心,给出解决线索,让他们自己去查阅权威资料——课堂是答疑解惑、培养兴趣的所在;生动活泼、激起兴趣最重要,绝不能搞成死气沉沉的 “劝退营”。

这就是我的取舍。

所以,你看我考核标准低、课堂 “乱讲一起”、“超纲内容” 满天飞…… 但 ABCD 他们都喜欢。

反之,你大手一挥,“以五人英雄本挑大梁为基本标准”“按过线率计算 KPI”——那惨了。

首先,A 和 B 我就被迫要忽略了。反正他们怎么都能过线,没必要培养他们。

而 C 和 D 就要成为重点培养对象了。把他俩拉起来,我过线率 100%;拉不起来,50%。我当然得拉啊。

为了拉 C 和 D,我不得不给他们灌输,耳提面命;以达标线为纲,考察他们 “哪里没学会”——然后针对性训练,题海淹过去!

不光 C 和 D 要受罪,A 和 B 也不得不跟着受罪、跟着背诵 “XX 副本的二号 boss 有 X 个技能,应对方案分别是……”

你看,本来,A 和 B 自己弄明白机制,他们自己就知道该怎么打,完全没必要教的。现在为了强行把 C 和 D 带起来,A 和 B 不得不跟着死记硬背、在自己完全用不到的题海里硬泡!

甚至于,凭 A 和 B 的机灵劲儿,他们完全可以在打这些难度并不特别高的英雄本 boss 时随机应变、根据队伍配置调整战术的;但现在,他们不得不像 C 和 D 一样,把我为菜鸡玩家设计的垃圾战术硬生生背下来!

因为,如果不让他们背,C 和 D 会有意见的:为什么我们就必须 “书海无涯苦作舟”,而 A 和 B 可以想怎么玩怎么玩?不是,你给他们教的那个窍门好简单啊我干嘛要学复杂的…… 哦,那个窍门需要特定职业啊?我再练一个行不行?

所以,没办法。我们必须毁掉 A 和 B,换来 C 和 D 的达标。

最终,我们成功的毁掉了 A 和 B,换来的却是…… 把 C 和 D 带过了他们并不在乎的 “达标线”。

甚至于,因为 C 和 D 脑筋死、没有战斗天赋,他们仅仅是能够在特定队伍配置下、生搬硬套的通过考核而已。

真放出去了,别说英雄本挑大梁,普通本他们都不行。

不光不行,还满脑子条条框框——本来我随便带带,足够他们普通本挑大梁随便打了;现在这么一背,完蛋,普通本他们都能带灭团。

带灭了还振振有词:外面那些玩家都特么傻!boss 机制不懂战术安排不想听,打一次灭一次还嫌我讨厌!

而 A 和 B 呢?

他们满是沮丧。因为他们实在背不下那些枯燥的战术——不是,老大,这英雄本我起码有 7 种方案打过去,干嘛一定要背那唯一的一种?而且我自己组织语言说出来还不行,还一定要学你的措辞!

不是,你为什么不关心我会做什么、只关心我还没背下那几句废话!

你看,思路一换,大家都轻松了。

对 C 和 D,你再带新人我跟着混团就好了,英雄本紫装也不错了,再混两拨人就能毕业。挺好。

对 A 和 B 呢,再没人提着耳朵逼他们背、考察他们 “哪里还没背会” 的“缺陷”了——不不不,现在他们每天都有成就感:

——老大,我总算把二次掷筛原理搞懂了!

——老大,我明白仇恨机制了!

——老大,这个递减机制我还有点不明白…… 哦哦哦,太棒了,我终于懂了!

——哎呀这个游戏也就这样嘛。基本都玩明白了…… 什么?商业?嗯嗯,有时间看看……

很多所谓的 “短板” 并不是 “短板”;何况,“短板” 未必重要:毕竟,木桶的容量,取决于最长的那块木板。你何必一定要逼着杨振宁学会演电影呢。

知乎用户 匿名用户 发表

转两篇文章

作者大神的知乎似乎是 Zeldovich Yakov - 知乎

人人网,中国领先的实名制 SNS 社交网络。加入人人网,找到老同学,结识新朋友。

前面本人转发了麻省理工学院的交换代数期中考试,有些人不明白所谓的黑点在那里。其实说老实话,考卷水,这未必说明学生学得差。但是之所以把麻省理 工学院的考卷发出来,这是因为,明明考卷题目十分水,到了让人看不下去的地方,这点,但凡懂行的一看便知,可以这么说,国内那怕是普通 985 大学,考卷怕 都不会这么水。可是还因为学生分数差,说要调整总分计分方法。实际上,MIT 以前这门课的考试难度更低,这已经得到某曾经在 MIT 就读的中国籍国际奥数金 牌的证实。
需要调整计分方法,也从一个侧面说明了这算是 MIT 比较难的考试。
而且,交换代数是研究生课,选这门课的学生,绝大多数肯定是存在继续学习基础数学的意愿的学生,即便有一些外系的学生,也多半是来自理论物理等需要用较多数学知识的方向的学生。所以也排除了,所谓有大量所谓辅修数学混杂其中的可能。
因此,我认为,这件事虽然是个单独的事件,但足以说明,MIT 那怕是程度较好的学生,其程度也绝对没有很多人想象的那么好。所以网上很容易找到不少美国大学的考卷,的确几乎找不到什么有难度的考卷,但是都很难被拿来说事,这张卷子则完全不同。
当 然,有些人可能可能又会重弹今年来泛滥的所谓 “素质教育”、“高分低能”、“创新能力” 的老调,甚至会拿美国大学培养了多少位诺贝尔奖得主、菲尔兹奖得 主、出来说事。但是这些人都忘了一点,那就是,那些被用来证明美国大学培养质量的人物,大多数都是几十年读的大学,当年的美国大学不可能是和现在一样的 水。本人见过 1963 年美国数学会委托 Peter Lax 等人制定的数学专业课程要求,就算以现在的标准来看,要求都不低。特别是最高标准的课程要求,还包括复流形等内容。此外,J-P.Serre 的李群 李代数(LMN1500)一书,书后有他 1964 年秋季在哈佛上课时的期末考试题,那题目看起来也很正常。
实际上,并不是所有的先进国家,教学都和美国一样水。比如同样是被业内公认是世界顶尖的波恩大学数学系的交换代数课的期末考试试卷就挺正常:
http://www.math.uni-bonn.de/people/aosoldat/Klausur.pdf
而且要注意的是,这门课在 MIT 是研究生课,但是在波恩大学只是本科生课,没有被列入研究生课,他们的培养手册建议也是大二第二学期选课。
众 所周知,德国的大学本科入学几乎没有什么门槛,多数学生都是选离家近的大学入学。即便是波恩大学数学系这样公认在世界上处于顶尖水平的地方,恐怕有些学生 进来的时候,也只是国内二本的水平。总之,本科生源是不能和中、美两国的顶尖大学比的。从他们线性代数课的选课容量来看。波恩大学数学系招收的新生不超过 250 人。(https://basis.uni-bonn.de/qisserver/rds?state=verpublish&status=init&vmfile=no&publishid=100165&moduleCall=webInfo&publishConfFile=webInfo&publishSubDir=veranstaltung
但是交换代数课的选课人数竟然多达 86 人(http://www.math.uni-bonn.de/people/aosoldat/commalg_V3A1_SS15.htmpl?language=en)。而作为后继课的代数几何 1 的选课人数也是 83 人(http://www.math.uni-bonn.de/people/aosoldat/alggeom_V4A1_WS15.htmpl), 我稍微看了一下两个名单,名字重合的人数不少,这些学生可以肯定是认为自己交换代数学得还可以的,否则也不敢去选代数几何 1,因为波恩大学的代数几何 1 这 门课是直接讲概形理论的,没学好交换代数的学生,不可能听懂。一个招生人数 250 余人的数学系,有这么多人选交换代数和代数几何,这个培养效果还是很惊人 的。当然,这 250 人中被淘汰的学生也不少,他们大二第一学期的代数学引论课的选课容量就只有 133 余人(https://basis.uni-bonn.de/qisserver/rds?state=verpublish&status=init&vmfile=no&publishid=100163&moduleCall=webInfo&publishConfFile=webInfo&publishSubDir=veranstaltung)。也就是说差不多一半的人都在第一年结束以后被淘汰掉了。
但不管怎么说,我相信,这些都可以说明,以波恩大学代表的德国一流数学系培养的学生的整体水平是大大高于美国的所谓名校的,我相信这也是德国近几年涌现了不少数学新星的原因,也给目下唯美国是从,盲目崇拜美国模式的国人好好上了一课。
目 下,不管大陆还是台湾香港,对于很多国人,外国就约等于美国,最多再加个英国,对美国有大量不切实际的幻想,大中小学教学全部以美国为样板,大搞所谓 “素 质教育”、“通识教育”,甚至盲目否定我国的教育传统,不加分辨的引进美国的错误经验,结果是教学质量每况愈下,学生程度一年不如一年。学生留学也只盯住 美国,宁可去美国二三流大学,也不肯去欧洲日本以色列等国的一流大学,已经到了十分可笑的地步。这种情况是必须得到纠正的。

人人网,中国领先的实名制 SNS 社交网络。加入人人网,找到老同学,结识新朋友。

在当今世界,美国是毫无疑问的头号强国。在科学领域,迄今为止,美国获得的诺贝尔科学奖和菲尔兹数学奖人数都高居榜首。很多人理所当然的认为正是因为美国 的教育和科研制度也是世界上最合理的,所以美国的诺贝尔科学奖和菲尔兹数学奖人数才能高居榜首。以至于当前无论是大陆还是台湾,中国的科技界和教育界言必 称美国已经成为了常态,对美国模式的迷信几乎无所不在。美国的教材被大批采用,美国的各种做法成为大家模仿的对象,有志于科学研究的学生纷纷前往美国的大 学,希望在那里能够实现自己的科学家梦想。 但是如果我们仔细审视一下诺贝尔科学奖和菲尔兹数学奖的获奖名单就会发现,情况似乎远不是那么简单。我们以大家通常认为难度最大、含金量最高的诺贝尔物理奖和菲尔兹数学奖为例,看看其中是否存在着什么值得注意的情况。
如果按照总数来说,各主要科学强国获得诺贝尔物理奖的总数如下:
美国 85
德国 25
英国 22
法国 13
苏联 / 俄罗斯 11
日本 10
的确,初看起来,美国优势明显。但是如果我们再看各个时间段的获奖人数,会有不一样的发现。一战以前各主要科学强国获得诺贝尔物理奖的人数是法国 4 人、德 国 4 人、英国 2 人、美国 1 人。显然在一战以前,法国和德国的物理研究水平领先世界,英国紧随其后,美国就要弱一些了。
但是到了一战开始以后,英国多次获奖,截止二战爆发前,迅速跃升至第一。战前的物理大国法国则萎靡不振,而德国保持了势头,美国则进步神速。从一战开始以后,到二战爆发之前,获奖人数为:英国 8 人、德国 7 人、美国 5 人、法国 2 人。
等到二战爆发以后,法国的物理长期低迷,从 1939 到 1965,长达 26 年没有人获得诺贝尔物理奖,美国则迅速成为获奖人数最多的国家,而日本和苏联也有多人获得诺贝尔物理奖。
二战结束以后,获奖人数为:美国 79 人、德国 14 人、英国 12 人、苏联 / 俄罗斯 11 人,日本 10 人,法国 7 人。可见二战以后,美国摇摇领先,而欧洲却开始 落后了。那么这中间发生了什么事情导致了这种局面呢?是否是因为美国的制度先进才形成了这样的局面呢?我认为这是一个需要研究的问题,而不能简单的说,美 国科学发达就是因为美国制度优越。因为科学家并不是生活在真空里的。科学的发展和经济和政治局势有很大的关系。一个国家的科研教育制度不论再合理,如果国 家混乱不堪,局势动荡,学术研究时常遭到政治原因的干涉,也很难出现好的科研成果。而在各个主要科学强国中,只有美国因为乱离动乱中心,在百余年来维持了 长期的稳定局面,即使在两次世界大战中,美国也没有大伤元气,反而因为战争物资生产对经济的刺激,以及由于战争导致大批其他国家的科技人才为了躲避战乱涌 入美国,大赚了一笔。而其他国家又是什么情况呢?
先看第二次世界大战之前公认科学研究水平最高,也是现代研究型大学诞生地的德国。德国在一战和两战中两次战败,加上纳粹党上台以后迫害犹太人,导致大批成名科学家及青年才俊纷纷外流,这一段历史对很多人来说都是十分熟悉的。
法国在第一次世界大战中元气大伤,尤其是大批巴黎高师和综合理工学校的学生入伍参战,死在了战场上。大批未来的科学家死在了战场上,这对法国的科研人才储 备带来了毁灭性的打击。一战结束以后,法国政局又长期不稳,1919 年有海军水兵暴动,1920 年铁路大罢工,1930 年又陷入经济危机,到了 1934 年 甚至闹出法西斯右翼组织发动叛乱,企图占领国会实施政变的重大政治危机,差点就给法兰西第三共和国判处了死刑。二战中,法国被德国占领,科研和教学工作当 然无法正常进行。菲尔兹奖得主 Alexander Grothendieck 和诺贝尔奖物理得主 Georges Charpak 甚至在二战期间被关入纳粹集中营,算得上是死里逃生。在这样动荡不安的环境中,科研和教学又如何正常进行呢?
而英国,虽然在一战中损失不大,但是二战同样也是大伤元气。英国二战以后曾经一度实施配给制,一直到 1954 年,英国实施的食物配给制才最终取消。试问, 连饭都吃不饱,还怎么做研究呢?一直到 1970 年,英国的人均 GDP 也仅为 2246 美元,而美国则是 5126 美元,经济总量上的差距则更大。对于科研和教 育的投入自然远远不如美国。
而俄罗斯,在帝俄时代本来就是欧洲相对最落后的国家,第一次世界大战以后,又经历了革命和内战,直到 1922 年苏联内战结束,才有比较和平的局面。但是苏 联作为一个极权主义国家,政治对科研和教育又经常产生破坏作用。尤其是 1934 年开始的大清洗,包括 Lev Shubnikov、Lev Landau 在内大批优秀物理学家都被捕入狱,Lev Landau 比较幸运,被捕一年以后就被释放,而另一名在低温物理方面做出卓越贡献的年轻物理学家 Lev Shubnikov 则在被关押了 8 年后,死在了监狱里。Lev Shubnikov 被捕时年仅 36 岁,介于他之前已经取得大量科研成果,尤其是对第二类超导体和反铁磁态的研究,我们完全有理由认为,如果他能继续从事科 研,很有可能会获得诺贝尔物理奖,甚至有可能创立一个和赫赫有名的 Landau 学派一样人才辈出的物理学派。在艰苦的第二次世界大战中,苏联最发达的欧洲 部分大部分都被德国占领,战争带来的破坏需要很长时间来医治。出生于二战结束后相对和平时代,受到系统严密的科研训练的俄罗斯物理学家,在他们最年富力强 的时候,又遇到了苏联解体带来的灾难性的经济崩溃,科研条件一落千丈,大批俄罗斯物理学家被迫放弃科研或者背井离乡,留下来的人,则生计窘迫,设备缺乏。 在这样的条件下,俄罗斯物理学家要取得同样的成就,需要比他们的美国同行克服更多的困难。
至于日本,本是一个落后的亚洲国家,直到明治维新以后才开始从西方引入近代科学。日本第一个有世界影响的物理学家仁科芳雄,1918 年才从大学毕业。中间 又经历了第二次世界大战的破坏,日本物理学家在这样低的起点和面临着如此多的困难的前提下还能取得如此多的成就,不能不让人肃然起敬。
诺贝尔物理学奖得主获奖的工作通常是若干年前做出的,考虑到在二十世纪前半叶,除了美国以外的主要科学强国的科学发展都曾经面临过各种各样的非科学因素的 干扰。我们完全有理由认为,简单考察诺贝尔物理奖得主总数完全不足以说明各国物理方面教育和科研制度合理性。毕竟要其他国家的物理学家在时局动荡或饥寒交 迫中还要做出和美国物理学家一样的工作来,实在是太强人所难。但是如果我们仅仅考虑出生在 1950 年以后各国经济恢复和相对和平时期的诺贝尔物理学奖得主 人数就会发现,美国的优势就开始缩小了。
到目前为止,出生在 1950 年以后的诺贝尔物理奖得主有:
Robert B. Laughlin,美国物理学家,现年 64 岁,加州大学伯克利分校和麻省理工学院毕业。 Eric Allin Cornell,美国物理学家,现年 53 岁,麻省理工学院和斯坦福大学毕业。 Carl Wieman,美国物理学家,现年 64 岁,麻省理工学院和斯坦福大学毕业。 Saul Perlmutter,美国物理学家,现年 55 岁,加州大学伯克利分校和哈佛大学毕业。 Brian Schmidt,美国物理学家,现年 48 岁,亚利桑那大学和哈佛大学毕业。 Adam Riess,美国物理学家,现年 45 岁,麻省理工学院和哈佛大学毕业。 Frank Wilczek,美国物理学家,现年 63 岁, 芝加哥大学和普林斯顿大学毕业。 Wolfgang Ketterle,德国物理学家,现年 57 岁,慕尼黑工业大学毕业。 Andre Geim,俄罗斯物理学家,现年 56 岁,莫斯科物理技术学院毕业。 Konstantin Novoselov,俄罗斯,现年 40 岁,莫斯科物理技术学院毕业。 天野浩,日本物理学家,现年 54 岁,名古屋大学毕业。 中村修二,日本物理学家,现年 60 岁,德岛大学毕业。
其中美国物理学家有 7 人、俄罗斯和日本各 2 人、德国 1 人。美国的优势明显缩小了。如果考虑到各国人口的差异,日本的表现其实完全不逊于美国,尤其是美国有 3 人年龄超过 60 岁。如果考虑到 60 岁及以下的物理学家,日本恐怕更胜一筹,而美国获奖人数则变为 4 人,不再具有绝对多数。在 60 以下的诺贝尔物理学家 中,成立于 1955 年的莫斯科物理技术学院物理系和大名鼎鼎的麻省理工学院物理系均有两位校友获奖。在国人心目中名不见经传的莫斯科物理技术学院物理系, 其培养的毕业生表现丝毫不逊色于能够利用资金优势吸引全世界最优秀的物理学家和学生的 MIT 物理系。其实除了这两位实验物理学家以外,年轻的莫斯科物理技 术学院物理系还培养了 Mikhail Shifman、Alexander Polyakov、Rashid Sunyaev、Nikita Nekrasov 等当代最优秀的理论物理学家,他们将来如果获奖,我想也并不会让人感到意外。但是有人也许会说,我们的样本太小,不足以说明问题。但是如 果再往前算,更早出生美国的科学家能够在和平安静的环境中成长和做研究,而他们的德国、英国、法国、俄罗斯、日本同行却不得不在战火纷飞中或是在战后百废 待兴的艰苦环境中成长,这样的比较,对于其他科学强国来说,实在太不公平。实际上直到 1970 年,美国人均 GDP 为 5126 美元、德国人均 GDP 为 2636 美元、英国人均 GDP 为 2246 美元、法国人均 GDP 为 2815 美元、苏联人均 GDP 为 1788 美元、日本人均 GDP 为 2015 美元。实际上其他 国家的科研和教育投入根本没有办法和财大气粗的美国相比。美国即使有优势,也说不清是因为投入多、设备先进、政治局势安定还是因为制度更优越。
相对于物理、化学等实验科学,数学对于物质上的要求更低,且更依赖于一个国家教学体系培养出的学生的水平和科研共同体的体制优势。在物理、化学等实验科学 领域,美国科学家可以靠着本国经济优势,获得更多的经费,购买更好的仪器从而让欧洲和日本科学家直接输在起跑线上,但是在数学领域,美国经济优势带来的加 成则大为减少。而且数学菲尔兹奖有个特点,就是限制年龄,只有 40 岁以下的数学家才能得奖,因此可以更多的反应出比较近的时代各国国家科学教育和科研系统 的效率。
如果我们打开菲尔兹数学奖得主名单,美国一共有 13 名数学家获得菲尔兹奖、法国有 11 名数学家获得菲尔兹奖、苏联有 9 名数学家获得菲尔兹奖、英国有 6 名数 学家获得菲尔兹奖、日本有 3 名数学家获得菲尔兹奖、德国有 1 名数学家获得菲尔兹奖。我们注意到,美国虽然仍然有一些优势,但是优势已经大大被削弱,而法国 人口仅为美国五分之一,菲尔兹奖得主却和美国相差无几,如果论培养和科研效率,对美国的优势可以说十分明显。
苏联虽然少于美国,但是在苏联时期得奖的数学家仅为 3 人、而苏联解体后反而多达 6 人,这和通常大家认知中,苏联时期的数学的教学和科研水平更高的感觉完全 相反。尤其我们注意到,很多水平被大家公认高于绝大多数的菲尔兹奖得主,而且很早就成名的苏联数学家并没有获得菲尔兹奖,如被公认为活跃在二十世纪下半叶 最伟大的数学家的首届沃尔夫数学奖得主 Israel Gelfand、本科期间即在国际数学家大会上做 45 分钟报告的著名数学家 Vladimir Arnold,而没有得奖的原因,常常是非学术性的。如 Vladimir Arnold 是因为参与写联名信支持被苏联政府迫害的数学家,被秉承苏联政府旨意的 Pontryagin 想办法否决掉了。事实上,当时苏联政府经常禁止一 些数学家出国。如苏联第一位荣获菲尔兹奖的数学家 Sergei Novikov 和第二位荣获菲尔兹奖的数学家 Grigori Margulis 均未被允许出国领奖。首届沃尔夫数学奖得主 Israel Gelfand 就是获奖多年以后才得以出国领奖。甚至像他的老师 Andrey Kolmogorov 这样的苏联国宝级数学家,都未被允许出国领奖。要知道 Andrey Kolmogorov 逝世时,苏联政府给予了和苏联最高领导人一样的国葬级待遇,治丧委员会组成人员包括苏联最高领导人 Mikhail Gorbachev、总理 Nikolai Ryzhkov、苏共中央书记处书记 Yegor Ligachyov 在内的所有苏共政治局成员。连 Andrey Kolmogorov 都会无法领奖,其他苏联数学家是否能够去领奖,自然对评奖委员会来说是一个问题。所以很有可能因为他们经常不被允许出国领奖,评奖委 员会也经常懒得给苏联数学家发奖,因为这样大概会省很多事。
同样人口仅为美国 5 分之一的英国,菲尔兹奖得主却达到了美国的一半,其效率也远高于美国。日本人口为美国 40% 左右,虽然论人均菲尔兹奖得主少于美国,但 是同样的,我们必须指出,存在水平被公认为远高于菲尔兹奖得主平均水平却没有得奖的大数学家,比如柏原正树。我们更要注意的是,二战前,被公认为数学水平 最高的德国,有大批优秀数学家前往美国,包括 Hermann Weyl、Emmy Noether、John von Neumann、André Weil、Richard Courant 这些重量级数学家,可以说当时世界上最优秀的数学家绝大多数都到了美国。反观另外两个数学大国法国和苏联,法国全国被占领,除了年过七旬、 早已退出科研一线的 Élie Cartan,在法国当时唯一能和这几位相提并论的,且在科研一线活跃的大数学家 Jean Leray 正在监狱里面面壁。而苏联只有一个 Andrey Kolmogorov,他的学生 Israel Gelfand,当时还只是一个二十多岁的愣头青。美国在人力和物力拥有压倒性优势的前提下,表现却并不那么出色。
而进入 21 世纪以来,所有得奖数学家中,不考虑具有巴西和法国双重国籍的 Artur Avila,法国数学家有 4 人、苏联数学家有 4 人,美国数学家仅 1 人,就算算上在本国读完大学后去美国读博士和工作的伊朗数学家 Maryam Mirzakhani 和澳大利亚数学家陶哲轩,美国体系下出来的菲尔兹奖得主也不过 3 人。有人或许会说,俄罗斯数学家 Stanislav Smirnov 和 Vladimir Voevodsky 不是在美国读的博士么?但是请注意,Stanislav Smirnov 的导师 Nikolai Makarov 和 Smirnov 一样,是来自圣彼得堡大学的俄罗斯数学家,而 Vladimir Voevodsky 的导师 David Kazhdan 和他一样,是来自莫斯科大学的俄罗斯数学家。一个俄罗斯人,在莫斯科大学或者圣彼得堡大学完成了全部的课程学习(实际上这些课程比美国大学 开设的研究生课程更加系统、深入和严格。),去一所美国大学跟一名前莫斯科大学或者圣彼得堡大学教授写了一篇博士论文,等于只是他们把办公室搬到了美国而 已。尤其是美国大学数学系通常规模较小,所以很少会有两个教授的研究方向基本一致的现象,他们要讨论问题,也只能找他们的俄罗斯导师及其学生讨论。美国大 学如果想要跑去和莫斯科大学和圣彼得堡大学抢功,实在缺乏底气。2002 年及以后得奖的数学家,最早也就是出生于 1962 年,而进入大学则差不多是 1980 年以后,这个时候距离二战结束已经 35 年,从欧洲前往美国工作的大批优秀科学家基本上已经退休,而美国的 GDP 则已经从 1970 年占全球的 3 分之 一下滑到不到四分之一,经费上的优势大大缩小,若论土生土长的美国科学家,美国在数学菲尔兹奖方面和法国和俄罗斯比起来,真是输得裤衩都不剩。
如果说只看菲尔兹奖,样本还太小,那么在数学界,被大家公认可以衡量各国数学培养和科研水平的指标中,还有一样可以作为参考的就是国际数学家大会的报告 人。在 2014 年举行的国际数学家大会上,做 1 小时大会报告的美国数学家仅有 2 人,而法国数学家却有 4 人,苏联数学家也有 2 人。苏联解体以后,人才外流和 经济崩溃的影响已经显现,但是苏联数学家的表现仍然不逊色于美国数学家。其中做 1 小时报告的俄罗斯数学家 Alexei Borodin 是 1997 年从莫斯科大学毕业,然后去美国跟随前莫斯科大学教授 Alexandre Kirillov 做博士论文。在他在莫斯科大学读书的时候,本人也恰好是莫斯科大学数学力学系的副博士研究生,所以我很清楚当时的莫斯科大学数学力学系是 怎样的情形。当时是俄罗斯经济十分困难的时代,后来获得 Nobel 物理奖的 Ginzburg 院士,工资还不如我在法国拿到的奖学金多(本人同时在法国的大 学注册,拿法国的奖学金。)。莫斯科大学数学力学系看起来宏伟壮丽的主楼,里面看起来十分破败,我第一次去宿舍的时候,宿舍脏乱差不说,厕所里的马桶盖都 整个没有了。我们教研室的研究生大多经济窘迫,研究生奖学金少的连饭都吃不饱,有些人不得不打零工赚钱糊口。在这种情况下,教授能够坚持教学和科研、学生 能够静下心来学习数学,实在是需要非常大的决心和毅力。而经济上没有问题,可以让大家安心做学问的法国,即便不考虑人均问题,看总量,法国数学家的表现也 已经明显优于美国同行。
虽然我们不能分析更多的学科,但是综上所述,要把美国目前在科学上的优势归功于他的科研和教育体制比其他科学强国更加优越,实在是缺乏说服力。在数学领 域,法国和俄罗斯的科研和教学系统,明显交出了比美国更好的答卷。美国的科技优势,究竟仅仅是因为乱离动乱中心,没有经历过大规模的革命和战争,政治形势 稳定,在两次世界大战中都能独善其身,同时可以发战争财,还是因为其制度合理,可以选拔和培养出更多的科技人才和鼓励科学家的科研积极性,让他们可以更高 效的工作,达到物尽其用,人尽其才,我们必须要打上大大的问号。
如果我们仔细深入美国教学和科研体制,则会发现这里实际上存在着大量的问题。
首先就是美国大学存在不顾师资力量,大量扩招研究生的问题。因为数学系的研究生数量较易查询,所以我们先看两所美国顶尖大学数学系的人员构成(这些数据可以参见各系主页的通信录):
普林斯顿大学数学系的 tenure track(终身教职候补资格)和 tenured(终身教职)的教师一共是 43 人,而该系就读的博士生总数为 63 人,平均每个年级为 15.75 人(该系博 士生是四年制)。tenure track(终身教职候补资格)和 tenured(终身教职)的教师人数和每个年级博士生人数之比是 2.73:1。
斯坦福大学数学系的 tenure track(终身教职候补资格)和 tenured(终身教职)的教师一共是 30 人,而该系就读的博士生总数为 80 人,平均每个年级为 16 人。tenure track(终身教职候补资格)和 tenured(终身教职)的教师人数和每个年级博士生人数之比是 1.875:1。
麻省理工学院数学系的 tenure track(终身教职候补资格)和 tenured(终身教职)的教师一共是 71 人,而该系就读的博士生总数为 120 人,平均每个年级为 24 人(该系博士生 是五年制)。tenure track(终身教职候补资格)和 tenured(终身教职)的教师人数和每个年级博士生人数之比是 2.96:1。
我们再来看看作为欧洲顶尖大学的巴黎六大和巴黎七大是什么情况,巴黎六大和七大的数学和计算机科学专业组成的巴黎中心数学研究联合体一共拥有终身教职教师 和研究员 437 人,博士生 342 人,平均每个年级 114 人。终身教职教师和研究员人数与每个年级的博士生人数之比是 3.83:1。(参见:http://www.federation.math.jussieu.fr/
我们在这里是用普林斯顿大学、斯坦福大学和麻省理工学院的数学系去和包括数学与计算机科学在内的巴黎中心数学研究联合体做对比。实际上数学系的博士生扩招情况远不如计算机系严重。如果去看美国大学的计算机系的情况,那我们会发现情况更加夸张。
我们没有查到麻省理工学院计算机专业的相关数据,而斯坦福大学计算机系有 tenure track(终身教职候补资格)和 tenured(终身教职)的教师 55 人,而研究生数量多达 218 人,平均每个年级 43.2 人。tenure track(终身教职候补资格)和 tenured(终身教职)的教师人数和每个年级的博士生人数之比是 1.27:1。
普林斯顿大学计算机系仅有 tenure track(终身教职候补资格)和 tenured(终身教职)的教师 35 人,而研究生数量多达 138 人,平均每个年级 27.6 人。tenure track(终身教职候补资格)和 tenured(终身教职)的教师人数和每个年级的博士生人数之比是 1.27:1。
而且普林斯顿大学数学系还不包括运筹学、统计和金融工程(巴黎中心数学研究联合体的数学专业则包括这些方向。),如果去看普林斯顿大学运筹学与金融工程 系,情况同样夸张,该系仅有 17 名 tenure track(终身教职候补资格)和 tenured(终身教职)的教师,而研究生数量却多达 47 人,平均每个年级 9.4 人。tenure track(终身教职候补资格)和 tenured(终身教职)的教师人数和每个年级的博士生人数之比是 1.81:1。
非常明显,即便在统计数据中包含了博士生相对规模会更大的计算机科学专业,巴黎六大和七大与普林斯顿大学、斯坦福大学和麻省理工学院相比,其博士生相对规 模仍然更小。如普林斯顿数学系的正式教职教师仅为巴黎六大和七大的十分之一,每年毕业的博士却超过巴黎六大和七大的七分之一。可见,巴黎六大和七大的博士 招生人数比起美国大学受到了更好的控制。而美国的三所大学滥招研究生则非常严重。
现在很多国家,博士多而教职少已经成为一个普遍现象。像计算机科学这样的工程学科的博士,毕业以后可以进入企业就职。教职少对他们还不算是个大问题。但是 像纯数学这样的基础科学专业的博士,找不到教职则只能改行。如果一个人 18 岁读大学,那他博士毕业的正常年龄应该是二十六七岁。一个快 30 岁的人,读了那 么多年书,读的还是数学这样难读的专业,毕业时却面临着毕业即失业的窘境,不得不改行,这是何等残酷的一件事?像数学专业还比较容易改行去搞金融、保险之 类的行业,但是像生物专业,改行恐怕只能去买保险。生物专业是美国大学博士培养规模无序扩大的一个典型。生物博士就业之难早已人尽皆知。许多人博士毕业以 后,博士后做了一期又一期,四十多岁还在做博士后,根本看不到拿到教职的希望。生物专业的“千老”(千年老博士后)现象,也让大批对生物研究还算有兴趣的 学生,出于生计考虑,不得不远离生物,实际上严重影响了生物科学的发展。而美国教职少博士多的状态,使得美国生物学的 “千老” 问题更加的严重。
美国为什么会出现这种状况呢?在这一点上,我基本赞成 Noam Chomsky 在 “The Death of American Universities” 一文中的看法,即和美国大学的过度商业化有关。美国大学的管理者,为了使学校可以快速扩张,并且在校际竞争中取胜,想尽办法开 源节流。为了开源,四面出击想办法获取经费和捐款,为了节流,则想尽办法节约成本。出于节约成本考虑,美国的大学越来越多的使用没有长期教职或者只有短期 合同的研究生、博士后等从事教学和科研工作。因为这些人的工资比有终身教职或者终身教职候选资格的教师要低得多。然后可以把省出来的钱用在一些明星教授身 上,给这些著名教授更高的薪酬或者是修建校舍和增添设备上。事实上,在一些开始盲目学习美国制度的国家,也出现了类似美国大学的状况。如在德国,拥有终身 教职的学者从 2005 年到 2012 年仅仅增加了 0.04%,而与此同时的是短期合同的大学全职雇员从 50% 增加到了 58%(参见德国 Regensburg 大学 Björn Brembs 教授的文章:http://bjoern.brembs.net/2015/01/booming-university-administrations/)。 在没有学习美国制度的法国和俄罗斯的大学里,给学生上习题课、答疑和批改作业的工作大部分是由具有终身教职的讲师、副教授等来承担的。而在美国的大学里, 承担这些工作的清一色是在读博士,在个别学校,甚至课也让在读博士来上。在读博士们一边要完成自己的科研工作,一边又要承担繁重的教学任务。相比传统的欧 洲大学的博士,美国的博士要承担更多的教学任务。教学任务过多,势必不能 集中精力于科研,严重影响博士生的培养质量。而博士生自己也是学生,教学经验和学术水平都不高,其教学效果当然也比不上有教职的老油条们,这也必然会影响 教学质量。大学出于自己节约成本的考虑,不顾社会需要,盲目扩大博士招生数量,大批博士毕业以后找不到对口的工作纷纷转行,这种现象是对社会资源的严重浪 费,破坏了正常的学术环境,非常的不合理。
同时,我们也看到美国大学过度商业化的模式,导致了大学日益行政化。为了对教学与科研实施更多的控制,以及更多的和商业机构打交道,大学行政人员增多,从 而使行政部门的力量日益加强。这会对大学的学术自由产生破坏,导致外行领导内行的现象。Noam Chomsky 在 “The Death of American Universities” 一文中就提出了这个问题。而这也被 Björn Brembs 教授给出的对开始引入美国制度的德国大学的数据所证实。在有终身教职的学者几乎没有增加的同时,德国大学的在去掉图书管理员和其他技术支持人 员后,大学的非科研支撑行政人员在 2005 年到 2012 年之间增长了 17%,对比两组的人数,管理人员和科学家的比例从 2005 年的 0.57 上升到 2012 年的 0.64。
事实上,过度商业化的模式不但使短期合同教学与科研人员受到影响。连有终身教职的教授的行为也受影响。对比传统欧洲大学和美国大学的教学,我们会发现,在 传统的欧洲大学,基本上不存在教材,教师一律使用的是自己编写的讲义,同时按自己的理解和想法去教学。对于理工科专业,习题课上发给学生的习题和作业也是 助教们自己准备的,而不是按照某本教材或者现成的习题集。而美国的大学,大部分课程却都存在教材。很显然,相对于欧洲的大学,美国的大学,即使是有终身教 职的教授,也不太愿意多花时间去动手编写讲义和准备习题。我认为这和商业化的模式下,教授更愿意把时间花在科研上,以便于获取更多的经费有关。
美国的大学单个系科的教师规模非常的小,被美国人认为是巨型大学的加州大学伯克利分校的数学系,终身教职候补资格和终身教职的教师也不过 65 人。实际上完 全是一种小国林立、诸侯割据的局面。而这样的状态,其实根本不利于科学的发展和科技人才的培养。因为搞一堆小数学系、小物理系,而不是集中成一个大型的数 学系或者物理系,这种做法会导致基础课重复开设,而高级的前沿课程却因为没人教或者选课人数过少而根本开不起来。基础课通常内容比较成熟,要找到参考教材 覆盖课程内容不难,所以要自学还相对简单。而高级的前沿课程则基本上只能参考一系列论文和专著,要自学就麻烦的多。这种状态必然会阻碍科研人才的培养。
另外一方面,单个系科的教师规模小,也导致根本无法相成集中的科学学派等科学共同体。你很难找到和你做的方向差不多的教授进行交流。不同方向的教授隔行如 隔山,也很难彼此交流科研问题。每个教授基本上都出于各自为战的状态,缺乏及时而有效的科研交流。这样的局面会严重地影响科学的发展。在数学领域,法国和 俄罗斯数学的优异表现,我认为这和巴黎与莫斯科是全世界数学家最集中的两个城市有十分密切的关系。因为大批数学家集中在一个城市,所以你很容易可以找到其 他懂行的专家一起讨论学术问题。而且两国的数学系都规模相对于美国更大,自然也有足够的力量开设更多的前沿课程,帮助学生尽快的到达学科前沿。如 80 年代 的莫斯科大学数学专业,每年开设的高级课程多达上百门,常年举办的讨论班则多达 200 多个,这样的环境显然比美国的小数学系林立的局面更加有利于年轻学者 的成长。
同时,因为用大批小规模系科代替少数大型系科,也使科研设备、图书、期刊等存在重复购买现象,造成了资金使用上的巨大浪费。同时有些科研设备、图书、期刊等又有可能因为使用较少,而没有购买,从而影响科研的进行。这样的学科布局实际上并不利于科学的发展。
近年来,美国大学的 tenure 制度引起了很多人的兴趣,很多中国的大学和研究所都试图引入 tenure 制度。但是这一制度真的就合理么?据某些美式大学 的鼓吹者的说法,美国大学的 tenure 制度看起来很美,但是在美式大学制度下,一方面大学狂招博士生,导致毕业的博士远远多于新增的教职,这使得即便是 最优秀的博士生,不做上几期博士后,都别想找到教职。另一方面,就算年轻人千辛万苦找到教职,作为 tenure track(终身教职候补资格)教职的 Assistant Professor 也不是终身教职,而是需要在 5-6 年后晋升,取得终身教授,否则安装所谓 “非升即走” 的原则就必须走人。假如一个人 27 岁博士毕业,做 5 年博士后,一帆风顺做了 5 年助理教授,拿到终身教职也已经 37 岁了,这还是非常理想的状况。据说这样的制度有利于调动年轻人的科研积极性,但是科学研究 是需要有耐心的。事实上愿意花时间从事科研 的人,就算没有外界压力,也很少有人会不努力做研究,而是在那里吃闲饭。如果说应用科学领域,这种做法或许还有一定的合理性。那么在基础科学领域,则根本 看不出 这样做的合理性。在完全没有采用非升即走的 tenure 制度,讲师就是终身教职的法国,科学家们难道就不干活混饭吃了么?我们特别需要指出的是,法国的教 职制度完全是一个社会主义大锅饭的典型,教师和研究人员的工资只看职称和工龄。照某些人的看法,法国的科学家们应该天天吃闲饭,发不出几篇论文,一定会被 美国同行在科研水平上秒杀。但是在对资金投入要求最低的数学领域,实际情形恰恰相反,考虑到两国的整体科研规模,科研水平上法国数学家们反而略胜一筹。同 样是采用这样的社会主义大锅饭的教职制度的苏联,在科研经费能够得到保障,科研能够基本上正常进行的时代,在科学上同样取得了长足的进步,出现了一大批具 有深远影响的科研成果。而在日本,在 2004 年推行国立大学和研究机构法人化以前,讲师和助教也是等同于终身教职的无聘期人员,日本近年来屡获诺贝尔奖, 这些科研成果实际上都是在国立大学和研究机构法人化以前做出来的。相反,在国立大学和研究机构法人化以后,采用了类似于 tenure 制度的教职制度,反而 闹出了小保方晴子事件这样的丑闻,让日本科学蒙羞。从以上对比可以看出,其实所谓 tenure 制度可以在整体上刺激科研人员的工作积极性,提高科研效果这 点,实际上是缺乏实证的。
另一方面,我们可以发现 tenure 制度其实存在着非常多的弊端。因为在非升即走的制度下,对于一个 30 多岁有家有口的科研人员,失去教职可能就意味着全 家没有饭吃。 压力一大,就很容易去想一些歪招。首先,对于未成名的年轻科学家的科研成果,一般不容易引起注意,为了让自己的科研成果容易引起注意。很多人就忙于到处开 会,推销自己的科研成果,而不是在办公室和实验室埋头苦干。很多人不是努力解决科学中的基本问题,而是忙于跟风凑热门和在科学期刊上灌水。和他们的欧洲和 日本同行相比,他们自我宣传的本事可比他们搞研究的本事更好。同时,在这种心态下,学术造假和学术腐败事件也在所难免。
事实上通常而言,重大科研成果通常不是由年长的成名科学家做出,而多是由年轻的科学家做出。而 tenure 制度事实上有利于年长的成名科学家,而不是有利 于年轻的科学家。这样给年轻科学家制造过高压力的做法,其实根本不利于年轻科学家的成长。因为他们必须要在 5-6 年内拿出足够的科研成果。这种明确的时间 限 制,使得他们不敢去攻克比较困难的科学难题。像菲尔兹奖得主吴宝珠,在获奖时只发表了 15 篇论文,他如果在美国,估计早就被迫离开数学研究或者也不得不跟 大家一起搞一些短平快的题目,跟风灌水了。同时,这样的制度也不鼓励他们去开拓新方向,因为这需要足够的准备工作。比如沃尔夫数学奖和菲尔兹奖得主 Sergei Novikov,他年轻时候曾经花了两年时间去学习物理,准备开展数学物理的研究,而没有发表什么论文。如果他在美国做 Assistant Professor,我想他也绝对不敢这么干。实际上,有些重要科研成果的重要性,甚至是具有里程碑意义的工作,有可能一开始不被理解或者存在争议,尤其 是这个成果是一个无名小卒做出,而且又很与众不同的情况下。一个大家熟知的例子就是数学大师 Abel,他关于群论和椭圆函数的工作在他身前根本没有得到重 视。他关于椭圆函数的工作被大家重视的时候,他都已经去世十几年了。日本数学家望月新一关于 abc 猜想的证明如果被确认是正确的,也是一个例子。所以为了 避免这种情况,一个理性人的选择自然是避免争议,也不去发表大家很难理解的工作,而是跟风随大流。如果每个科学家在最有创造力的时候都这样做,自然会导致 科学研究的平庸化,根本就不利于出现科学上的重大突破。没有一个一个科学上的重大突破,人类社会又如何前进?
tenure 制度是一种很残酷的制度,这种搞法,只会让年轻人害怕科研,从而导致优秀人才不敢从事科研。除非这个国家和美国一样,有大批其他国家的年轻人跑过来当炮灰,否则,这个国家的科研队伍的长远发展绝对堪忧,因为优秀的年轻人只会大批的逃离科研。
美式大学的鼓吹者经常说美国大学的教学质量好、学生水平高。那么是不是这么回事呢?不能不否认,美国大学的管理者还是很重视教学的,因为和依赖于政府拨款 的欧洲大学不同,美国的大学运转很大程度上靠的是来自社会捐款的校产基金。管理者自然也希望让更多的校友取得成功,从而获取更多的捐款。于是在这种经济利 益的驱动下,美国学校为了提高毕业生水平想出了各种办法。除了四处出击拿出高额奖学金抢夺优质生源以外,美国大学的管理者自然想出了各种办法来提高教学质 量。要提高教学质量,首先要对教学质量进行评估。那么美国大学是怎样对教学质量进行评估的呢?办法就是对学生进行匿名调查,让学生去评价老师。此外美国大 学的课程大部分是选修,不像欧洲的大学,基础课绝大多数都是必修课。管理者认为,如果学生给的评价差的老师就不是好老师,没人去选他的课的老师也不是好老 师。于是教师为了讨好学生,根本不敢像传统的欧洲大学一样对学生严格要求,生怕被学生打低分或者不去选他的课。
美国人认为是很难很严的课,实际上既不难,也不严。比如很多在美国学数学的学生都听说过哈佛大学数学系的 Math55 这门课。哈佛大学数学系的主页上说(参见:http://www.math.harvard.edu/pamphlets/freshmenguide.html),这门课是美国最难的本科数学课,需要每周花 24-60 小时去做作业,而且要学过一年大学水平数学的尖子生才适合。而哈佛大学一份校报(http://www.thecrimson.com/article/2006/12/6/burden-of-proof-at-1002-am/) 则说这门课的作业加上每周 3 小时的上课和 1 小时的习题课,基本上相当于在干一份全职工作。而且这门课让哈佛大学的学生畏惧不已。有人回忆说,第一天上课还 有 51 人,两周以后就只剩 23 人。听起来让人很害怕。但是如果我们真的看看这门课实际的教学内容和作业,又是什么情况呢?这里有 Noam Elkies 讲授这门课的课程主页:
http://www.math.harvard.edu/~elkies/M55a.10/
http://www.math.harvard.edu/~elkies/M55b.10/
如果这样习题还需要一周 24-60 小时,我只能说哈佛大学数学系的学生实在太差了。这样的课程如果能算是美国最难的本科生数学课程,美国的数学教育水平实 在就是个悲剧。莫斯科大学卓里奇的数学分析上面,要找这样难度的习题,可以说一抓一大把。如果这都需要像干全职工作一样,让他们去莫斯科大学估计全年不需 要睡觉了。如果按这个速度做题,让他们去考巴黎高师的入学考试,恐怕得把考试时间从 6 小时或 4 小时延长到至少两星期。
笔者没有找到他们的考题,所以无从判断他们考试是否严格,但是同为美国顶尖数学系的普林斯顿大学数学系,本科生的复分析期末考试,考题全部来自他们的教材 (Stein 的复分析)上面的习题。更为好笑的是,居然还是开卷考试。这和给学生送分,我看也没有什么两样。美国最顶尖的大学如此,普通的美国大学则更加 严重。所以美国大学的学生成绩都出现了所谓的 Grade inflation,也就是分数膨胀现象。在 Henry Rosovsky 和 Matthew Hartley 撰写的美国艺术与科学院(American Academy of Arts and Sciences)的报告 “Evaluation and the Academy: Are We Doing the Right Thing? Grade Inflation and Letters of Recommendation”(参见:https://www.amacad.org/multimedia/pdfs/publications/researchpapersmonographs/Evaluation_and_the_Academy.pdf) 中,他们指出从 1960-1974,他们调查的 180 所学校,平均 GPA 增加了 0.432。而通过对 4900 名大学生的调查,显示获得 A - 及以上成绩的比 例从 1969 年的 7% 增加到 1993 年的 26%。在普林斯顿大学,1973 年,所有成绩中,有 30.7% 是 A,而到了 1992 年,则增加到了 42.5%, 同时,只有 11.6% 是低于 B。而根据 Ross Douthat 的 Privilege: Harvard and the Education of the Ruling Class 一书,到了 2000 年,哈佛一半的学生成绩都是 A。以下这张图来自经济学人(http://www.economist.com/blogs/freeexchange/2014/08/economics-university-grading),反映了美国部分名校分数膨胀的趋势。
那么是不是因为美国学生的水平提高了,所以成绩才增加了呢?Henry Rosovsky 和 Matthew Hartley 的报告在研究了全美学生的 SAT 成绩等统计数据以后,认为这种看法是毫无根据的。实际上,越来越多的大学生因为学力低下,被大学要求参加补 习,参加补习的学生从 1990 年到 1995 年增加了 39%。另外,根据美国高等教育研究所的一项对大学教授的调查,只有 25% 的教授觉得他们教的学生在学 术上有足够的准备。
事实上,除了少数明星学生以外,即使是美国的顶尖大学的本科,大部分学生的水平究竟如何,都是需要打一个问号的。据本人了解,麻省理工学院物理系和加州理 工学院物理系去法国综合理工学校(École Polytechnique)物理系高能物理专业进行交换学习的本科生,在综合理工学校的考试成绩基本上都是垫底的。而这些学生在麻省理工学院和加州理工 学院物理系的全体学生里面起码应该是中等水平以上的的学生,否则也不敢去学高能物理专业。况且成绩太差的学生,综合理工学校也绝对不会接受他们来进行交换 学习。
同时 tenure 制度加剧了分数膨胀现象。Jean Filetti 的 “Grades and Ranking: When Tenure Affects Assessment” 一文(参见:http://pareonline.net/getvn.asp?v=15&n=14) 给出的统计则显示,相对于有终身教职的教师,只有 tenure track(终身教职候补资格)的教师明显更喜欢给高分。这种现象并不让人意外。首先,只有 tenure track(终身教职候补资格)的教师为了得到较好的学生评价,避免对自己取得终身教职产生不良影响,并不愿意得罪学生。其次,只有 tenure track(终身教职候补资格)的教师,能不能在这个大学长期待下去,是需要打个问号的,所以他们心里面常常把自己当成这所大学的局外人,行为自然会短期 化,不会有长远的考虑,自然对于学术标准的坚持程度远不及那些有终身教职的教师,因为他们常常觉得自己是一个局外人。
事实上,美国大学的这些做法,导致了严重的劣币驱逐良币的现象。坚守学术标准,认真教学,严格要求的教师越来越少,而给分宽松,教学简单的教师则越来越多。这种状态必然导致大学的教学质量严重下滑,根本上危害科学的进步和社会的发展。
同时分数膨胀也使得优秀学生难以脱颖而出,研究生院和雇主根本搞不清那些学生更优秀,对分数失去信任。于是只好靠推荐信、出身学校、排名之类的东西来选择 学生,结果导致拥有良好的社会网络的学生跟容易被看中。学生学得好,不如关系搞得好。这种做法严重损害了社会公平。比如对于研究生院的录取,既然学校的成 绩无法区分出谁是更好的学生。那只能靠推荐信,或者看学生毕业的学校,或者看学生发表的论文。可是推荐信这种东西,完全取决于写信人和被写信人的关系,能 拿到好的推荐信的,未必是程度最好的学生,而可能是最会拉关系、走门路的学生。这种做法,本质上和汉代的 “举孝廉” 差不多,其弊端是显而易见的。而看学生 毕业的学校来决定是否录取,也是十分不合理的行为。谁都知道好学校也有可能有很差的学生,很差的学生,也不见得没有出类拔萃的学生。尤其在美国的制度下, 看学校出身更加会严重影响社会公平。因为美国各级学校都不存在入学考试,而是采用审核学生学习档案来决定是否接受申请。在这种录取办法下,来自比较差的学 校的学生自然会吃很大的亏,根本不能实现唯才是举。
尤其要注意的是,美国的基础教育存在很大的不平等问题。在美国,由于不论是联邦还是州一级都不存在统一的教学大纲和教材,所以教学基本要求无从谈起。中小 学阶段,不同的学校的差距相当之大。美国中小学分成公立学校和私立学校,大约 90% 的中小学是公立学校,10% 是私立学校。对于公立学校来说,其经费的主 要来源并不是中央政府的财政拨款,而是来自于州政府拨款和学区税收,特别是财产税。如 2010-2011 年度,美国中小学教育经费中 44.1% 来自州政府 的拨款,而 43.4% 来自学区的税收,特别是有 35% 来自于财产税。(参见美国国家教育统计中心的教育统计摘要:http://nces.ed.gov/programs/digest/d13/tables/dt13_235.10.asp) 全美有 13506 个学区,平均每个学区只有两万余人。同是公立学校,比较富裕的学区,其财产税收入就更多,而比较贫困的学区,显然其财产税收入就少。因此 富裕学区公立学校和贫困学区公立学校的经费收入存在不小的差异,这首先体现在教师工资上。如同在马里兰州,Calvert 学区 2010 年 10 月的教师平均 年薪为 73986 美元。与此同时,Dorchester 学区的教师平均年薪则仅为 51789 美元。(参见:http://www.marylandpublicschools.org/NR/rdonlyres/F5B571DF-A470-47BE-9516-05DC870DEEBA/31303/analys13.pdf)。实际上上州与州之间的教师平均年薪同样存在相当大的差距。2012-2013 年,纽约州的教师平均年薪为 75279 美元,而南达科他州的教师平均年薪则仅为 39580 美元。几乎相差一倍。(参见美国国家教育统计中心的教育统计摘要:http://nces.ed.gov/programs/digest/d13/tables/dt13_211.60.asp
非常明显,在不同学区间经费差异巨大且不存在统一教学基本要求的前提下。对于公立学校,富裕学区公立学校请得起更好的教师,有更好的设备,所以其教学质量 也就更高,周围富人就会更多,房价自然也会更高,穷人很难在这里住的起。而穷困学区公立学校的教学质量则难以保障。而对于私立学校,自然只有有钱人的子弟 才交得起学费。这样一来,势必会导致普通家庭的子弟输在了起跑线上,而且很难靠自己的努力去扭转局面。能否进入名校,则越来越不取决于个人的才能,而是依 赖于家庭出身,也就是俗话说的 “拼爹”。这样做则必然会导致社会不同阶层之间上升通道的阻断。
而看论文来决定是否录取,这种做法同样存在问题。且不说较差的大学的本科生很难有这种条件,就算是名校的本科生,能够得到 足够的指导和具备一定的科研环境。但是本科生的程度真的足以发表论文么?对于实验科学,本人不敢多说。但是对于数学和理论物理专业的学科,按照美国学生的 程度,有几个学生有能力做出足以发表的论文?德国 Bonn 大学数学系教授 Peter Scholze 这样读了五年大学就拿到博士学位,博士毕业直接被聘为德国数学第一的 Bonn 大学最高级别的 W3 教授,而且博士毕业两年后就应邀在国际数学 家大会上做邀请报告的人,他发表第一篇论文也是进大学四年以后。美国最顶尖的大学的绝大多数的本科生,其水平与 Peter Scholze 相比,显然远远不如。于是大部分论文实际上只能是教授把东西做的差不多,然后让本科生来结个尾,最后就变成了教授给学生送结果。美国国家科 学基金(NSF)大概是为了鼓励本科生发表论文,于是资助了各种 REU 项目 (Research Experiences for Undergraduates),但是至少在数学领域,这种搞法已经变成了大面积资助教授给学生送论文的项目。有一些数学论文的署名作者算上指导教授多达 七人,内行人一看就知道是怎么回事。
需要说明的是,本人并不反对本科生尽早参与科研,相反,本人十分支持这种做法,但是把本科生科研搞成了教授给学生送论文,这样的本科生科研,意义何在?苏 联的顶尖大学,传统上也是鼓励学生尽早参加科研。通常按照教学计划的要求,每个学生在大二结束前都要选择导师,然后从大三开始在导师指导下开始参与科研活 动,比如参加讨论班、参加实验室工作等,同时从大三到大五都有学年论文和毕业论文的要求。事实上,也的确有出现过本科生做出了让学界震动的重要科研成果的 例子。 在著名的 Hilbert 的 23 个数学问题中,就有两个是被苏联的本科生解决的。分别 Hilbert 第 13 问题(由当时是莫斯科大学本科生的 Vladimir Arnold 解决。)和 Hilbert 第 10 问题(由当时是圣彼得堡大学本科生的 Yuri Matiyasevich 解决,后来成为俄罗斯科学院院士。)。Vladimir Arnold 和 Yuri Matiyasevich 也都在本科期间,在国际数学家大会上做了 45 分钟邀请报告。但是这些科研活动都是在教学计划 要求下有计划的进行的,而不是试图搞大跃进,以发表为目的,最后演变成教授给学生送论文。这种教授向学生送论文的所谓本科生科研,根本不能训练学生的科研 能力,也无法用来分辨学生是否具有科学研究的潜力,靠这个来录取学生,其实比靠抽签来决定录取也不见得可靠到那里去。
分数膨胀现象也让美国大学生对自己产生误判,明明没有学好,却自己以为学好了。他们也就不再花时间吃透基本理论,加强基本功。学习积极性高的学生,则开始 拼命向前学。很多人高级的东西学了一堆,但是却搞不能灵活运用基础课程的知识。有人或许会说,美国的研究生教育很好,他们到了研究生阶段,也会把基础补上 来。那么美国的研究生课程如何呢?实际上,美国的研究生课程除了博士生资格考试的科目以外,其他研究生课基本上都不存在考试。而美国大学的博士生资格考试 的范围,充其量相当于苏联大学五年制本科基础必修课或者法国大学 M1 课程的范围。而相当于苏联大学五年制本科专门化课程或者法国大学 M2 课程的范围的研究 生高级课程,如数学专业的代数几何、物理专业的弦理论都是没有考试的,其教学的严格程度远不及同级别的苏联大学的五年制本科专门化课程或是同级别法国大学 的 M2 课程。在这样的教学制度下,除了资格考试相关的几门课,美国学生的程度如何,必须要打上一个大大的问号。事实上,美国不少教授并不希望研究生多花时 间在基本理论上,恨不得学生不上课,因为这会影响研究进度,所以不少美国学生在基本理论上的掌握上是存在隐患的。
也许有人会认为,给学生一个较高的分数会给学生更多的信心,不过分的打击他们。然而这样做对于学生真的好么?美国大学分数膨胀现象导致学生的程度低下, 不合格的学生根本不会被淘汰。在欧洲,法国的大学,大一的学生平均只有 42% 可以升入大二。德国 Bonn 大学数学系,今年大三的这批学生,入学时有 500 人,两年就淘汰了 400 余人。如果美国大学的教授想像他的欧洲同行一样做,搞不好会卷铺盖走人。于是大批不合格的本科生就带着很高的 GPA 走出了校门,但 是却什么都不会。对于理工科专业,从业的标准不是可以随意降低的。不管他们是就业还是继续读研,这样的本科生,很快就会吃到苦头。尤其是准备攻读研究生, 从事科研工作的学生。他们很快会发现,他们根本没有为读研究生做好准备,只能转行。有些人甚至到了博士毕业才发现自己根本不是这块料。试问,究竟是严格要 求,在低年级就把不合格学生淘汰掉,让他们另谋高就比较人道,还是等到他们快 30 了,才告诉他们,其实他们根本就是不合格学生,必须要改行比较人道?我想 答案是显然的。
或许有美国大学的崇拜者会质疑说美国的大学生程度既然如此之差,为什么还有不少美国科学家能够在科学领域做出不错的成绩?首先,现在我们所熟知的美国科学 家, 他们求学的时代是几十年前,而不是现在,他们的成功并不能用来证明现在美国大学的种种做法是合理的。在上个世纪,美国科学突飞猛进的时代,美国大学对学生 的要求绝不会像今天这么低。50 年代的哈佛大学可不像今天的哈佛大学,大部分人都是 A,当时的平均成绩也就是 C。其次,少数美国高材生,未必是美国学校教 育的结果。比如很多人会想到的前几年普林斯顿大学数学系那个在数学界权威杂志 “Annals of Mathematics” 上面发表论文的本科生 John Pardon。他的父亲是 Duke 大学数学系的正教授。我看与其说他是普林斯顿大学数学系培养出来的,不如说他是他爸培养出来的。可是对于出生于普通家庭 的学生来说,你爸是美国著名大学的教授么?
对于美国的教育与科研制度,我们要清醒地看到其中存在着大量问题,绝不像美国的盲目崇拜者所宣称的那么美好,切不可以一味仿效其做法,更不能简单照抄。否 则这种盲目崇拜美国的科研和教育制度,照搬照抄美国的做法,必然会给一个国家的教育和科研事业造成破坏。事实上,这一个绝非危言耸听,而是在不少国家的教 育和科研领域已经发生的事实。除了前面说的一些国家盲目引入 tenure 制度导致科研人员急功近利,影响科研和教学质量以及学习美国大学在教学和科研管理 上的一系列错误做法,如让学生给教师授课打分等问题以外,传统上中小学教育严格规范的一些亚洲和欧洲国家,学习美国搞的所谓的“轻松教育”、“素质教 育”,也已经严重的影响了国家竞争力。如李远哲在台湾推行的所谓教改,就造成了台湾学生素质的严重下滑,在台湾岛内引起了许多人的忧虑。此外,日本仿效美 国搞所谓 “轻松教育” 也使日本新一代学生的程度出现了下滑。同时家长害怕学生程度太差,有财力的家长,便纷纷给学生报补习班,反而大大加重了学生负担,同 时也造成了不合理 的两极分化。因为这样一来,学生家里如果没钱去报补习班,自然会比有钱报补习班的学生吃亏。最后产生两极分化的原因并不是学生的天赋和努力程度,而是家里 有没有钱,实质上是大家一起来拼爹。甚至在中国,所谓的减负和素质教育也让中国的科学教育混乱不堪,现在的中学数学甚至不教授复数的系统知识,中学物理不 讲热学。诸如此类不能理性的看待各国制度优劣,放弃自己的长处,盲目崇拜美国,唯美国马首是瞻,美国怎么做,自己就怎么做的做法如果继续蔓延,只会走上机 械模仿美国的死路,从而摧毁国家的科学和教育基础,拖累科学和文化的发展,必须得到有效的警惕。

知乎用户 Yupeng​ 发表

大学之前叫算数,大学之后叫数学…… 我们算数比他们强太多,数学比他们还是有些距离。

知乎用户 阿成​ 发表

靠移民啊。

冷战期间美国有部电影叫 the right stuff,描述的是美苏科技竞赛。

参议员 Lynden Johnson(后来肯尼迪的副总统)问一个科学家,

“我们靠的也是德国人,苏联靠的也是德国人,我们是不是危险了。”

结果那个科学家回答:您放心,

Our Germans are better than their Germans.

当时我就笑的不行了。

视频片段:

https://www.youtube.com/watch?v=rYco0UsWhLc

我的看法:

1. 美国现在的初等教育根本无力产生足够的数学家。

2. 美国遇到过两次千载难逢的机遇:二战和苏联解体。

欧洲和苏联的伟大的科学家大量来到美国。

这样的机遇如果不出现,美国根本不可能碾压其他发达国家,取得现在的学术地位。

3. 美国开明的文化和优渥的物质基础保证了移民科学家能最大程度的发光发热。

并且两次机遇留下的红利持续激励着世界各国优秀的研究者来到美国。

4. 美国也有过高强度的数学初等教育。

一个教授告诉我,美国个别地区甚至试验过从小学开始公理化数学教育。

但随着高素质移民增多,逐渐造成了路径依赖,本土中小学越来越放纵。

5. 不要鄙视中国的数学初等教育,不信 10 年后我们再看。

我个人认为初等教育就应该强调数学基本功。

不然你以为英国为啥要来华取经?

6. 如果美国人不认为自己的基础教育有问题,

现在就不会强调教育改革,也不会大力鼓励 STEM 移民了。

7. AP 什么的,你确定比竞赛培养更有用?

统计 1936-2014 年间,所有美国籍或在美国工作的的菲尔兹奖得主。

看看他们初等教育是在哪。(手动统计,难免错漏)

Fields MedalLars Ahlors, 芬兰
Jesse Douglas, 本土
Atle Selberg, 挪威
John Milnor,本土
Paul Cohen,本土
John Thompson, 本土
Sergei Novikov,苏联
Enrico Bombieri,意大利
David Mumford, 本土
Pierre Deligne, 比利时
Charles Fefferman, 本土
Daniel Quillen,本土
Grigory Margulis, 苏联
Alain Connes,法国
William Thurston,本土
丘成桐,中国香港
Simon Donalson, 英国
Gerd Falting, 德国
Micheal Freedman,本土
Vladimir Drinfield,苏联
Vaughan Jones, 新西兰
Edward Witten,本土
Efim Zelmanov, 苏联
Richard Borcherds, 英国
Maxim Kontsevich,俄罗斯
Curtis McMullen,本土
Vladimir Voevodsky,俄罗斯
Andrei Okounkov, 俄罗斯
陶哲轩,澳大利亚
Elon Lindenstrauss, 以色列
吴宝珠, 越南
Manjul Bhargava, 印度
Maryam Mirzakhani, 伊朗

22/33 不在美国接受初等教育。

而且趋势越来越不乐观,最近 20 年(94- )只有 1/11 的得奖者出自美国的初等教育。

另外推荐答案

为什么美国学生学的数学比我们简单却还能做出很牛逼的东西? - 丁浩的回答

据 mitbbs 网友研究, A 校:清华,B 校:麻省理工,C 校:哈佛,D 校:宾大

知乎用户 老杨叔聊志愿填报​ 发表

最根本的原因不是教育而是国情。

虽然都是美国人,但不同阶段的美国人含义差别很大。

中小学阶段的 “美国人” 大都是美国人。

能做出超级牛的东西的 “美国人” 有美国人,也有印度人、中国人、日本人、韩国人……

中国是聚全国英才而均之。

美国是聚天下英才而用之。

知乎用户 Bill Cheng​​ 发表

原因有几个:

1. 国外计算机领域的发展时间要比国内长远很多,有很多技术累积,这些累积在大学课堂、开源资料上面都有很深的体现

2. 一个系统的开发,并不需要那么多数学牛逼的人,往往需要的只是一个数学非常牛逼的人,比如说凭一人之力开发出各种牛逼引擎的卡马克大神

3. 中国数学牛逼的人很多都去国外了。我在国内和国外都见过很多数学很牛逼的『程序员』,和这些程序员聊到最后,总能聊到『出国、移民』的话题,而其中大部分人决定出国之后就不回来了……

4. 中国顶尖的程序员都在干一些无法理喻的活。据说某个 T 开头的公司的大楼有一层,在那层楼里面上班的是全中国最牛逼的程序员,而这些程序员的工作,很多时候只是拿到一个软件,然后通过软件发出的系统指令集写出可以完美复制的代码…… 江湖上还传言另外一家以数字开头的公司,里面有中国最牛逼的系统工程师,而这帮人每天都在研究怎么让自己的系统钩子不被别人发现……

5. 很多时候你会发现,一个牛逼程序的诞生,并不需要作者有多么牛逼的数学能力,特别是个人软件领域。更多的时候,创意战胜一切,当然在某些地方,营销战胜一切…… 很遗憾的是,前者我们基本上没有,后者是大公司的特权……

想了到其他的再补吧……

=========== 体内酒精含量:0.4%==============

关于江湖传言这种东西,信者则信,不信笑笑便好了

就算你问我,我也只会告诉你不知道

知乎用户 茉茉 发表

因為(一些說法)說的是「算術」,不是數學。

知乎用户 一直住顶楼​ 发表

这个题目下基本上的回答都是在说中美高中教育,我没在美国读过书,但是如果撇开教育实践,从统计上看这也是一个逻辑问题吧?

中国执行的是 9 年义务教育,美国执行的是 12 年义务教育。

中国升高中的学生虽然看起来毛入学率很高,但是实际上被各种中专和技校(还包括直接进厂的农民工)分流的数量之后可能也就是 50%-60%。按照今天中国的教育体制和社会取向,我们可以合理的认为,升高中的学生是初中毕业生里面最好的那 50% 或者 60%(如果数学好大概率高中会读下去的,不会去读技校,对吧?)。

我个人一直认为智商这个东西在不同人种中的分布曲线是差不多的,所以说如果我们假设中国和美国的初中生水平相同,那么高中部分其实是拿中国的前 50% 平均水平去比美国的 100% 的平均水平,这个如果在统计结果上如果还会看不出智商差异,那就是自恨了。

下面这个可以帮助阅读理解:怎么看待中国城镇居民净资产均值高于美国家庭中位数。

如何看待央行报告城镇居民家庭净资产均值为 289.0 万元,中位数 141 万元?

如果想排除采样的干扰,比较双方高中 TOP 10% 才是合理的。

知乎用户 请勿关注​ 发表

我觉得是大环境的原因,我下面写了四句话,无论那个国家能做到,那个国家的科学技术就能真正强大。

因材施教,学以致用,尊重知识,充分竞争

一,因材施教除了孔子搞过小规模实验外其实从来就没有大规模的,官方层面搞过,一直到现在也是,我们要么是统一,要么是平均,冒尖都是极其困难的个人行为,从来都不是受鼓励的大众行为,吃大锅饭平均主义一直是我们主流的群体想法。

现在教育思路更是把这种平均主义发挥到了极致,因为优秀的人永远是少数,如果让优秀的冒头大多数人都嫉妒,就有有人投诉,所以在制度建设上把矛盾下移,让优秀的人在人群中艰难的突破,而不是用制度把优秀的尽早的挑出来培养,因为这样做会动摇基础,毕竟我们的基本盘跟别人不一样。

二,学以致用这点就不细说了,爱专业的不一定能学专业,学专业的不一定干专业,干专业的不一定能一直干专业,一直干专业的靠的不是待遇而是靠信念活着。

三,大家扪心自问,我们真的尊重知识吗?我们真的尊敬知识分子吗?我们享受知识带来便利的同时我们感谢过创造他们的人吗?不要回答我,对着自己的心说,有几个人真正感谢他们?

四,充分竞争的结果是什么?赢家通吃,胜者利益最大化,这才是知识就是生产力背后最大的动力,但是基于社会问题我们做不到,那个国家都做不到,就是松紧程度不一样而已。

我想说分配赢家财富可以,甚至是共识,但用我消灭你与你无关的态度操作就不是分配财富了,那是毁灭财富。

不知道多久被删除,也不知道会有多少人骂我,表达才是重要的。

知乎用户 知乎用户 pODQZx 发表

讲两个事情,不是在美国,都是在法国,不过可以类推一下的。

我们经常会嘲笑法国人数学差。为什么?不会算算术。

亲身经历,我去一家很大的跳蚤市场,淘了 35 欧元的货,递给收银员一张 50. 可怜的小伙那个愁啊,算了老半天,找给我 25 欧元。。。

我递还给他 10 块纸币的时候,他反应不过来啊。真的是一脸茫然地看着我。我只好说:“你找多了。” 然后深藏功与名。

但是我们经常说法国人数学好。为什么?也有一个不会算算术的梗。

我估计这个笑话在留法中国学生里传播甚广:有人问一个小女孩,3+4 等于几啊?小女孩说:“不知道,但我知道 3+4 等于 4+3.”

这人只好接着问:“为什么呀?”

“因为整数与整数加法构成了阿贝尔群。”

就是这样。

知乎用户 贾明子​​ 发表

更新:(鉴于有人说我有炫耀之嫌,干脆删除具体考纲)

我女儿今年高一,刚刚参加完美国大学先修课 AP 微积分的考试。课程基本上可以覆盖工科大一高数一元微积分内容。

这是美国课程体系中,每个中学生只要愿意学,都可以学,并且有标准化考试衡量水平的课程。这些课程可以带来申请大学的优势,甚至可以换取将来大学的学分。你觉得这些课程范围比我们学生的简单吗?

关键的问题是,你只要愿意学,你就有自由、有资源去学,学校虽不要求,但是会鼓励你去学。并且学习有升学的好处。这里可没有什么 “黑名单” 的限制。

评论区中有好多 “牛人” 大谈 AP 考试如何简单,零基础一个月搞定 5 分,甚至 10 天搞定的,并且由此说 AP 课程很简单,比高中数学简单。我姑且认为这些人说的都是实话。但是这恰恰反映出为题主问题的根源,这是经过了多少考试才造就了这样的应试思维啊。考试的目的是学习课程,而不是学习课程的目的是为了考试。如果你真的认真学习了,你不可能说出什么 “零基础 10 天搞定” 这种胡话。你十天搞定的,不过是针对考试的训练,而不是在学习。用一句大家常说的话,就是:它们的课程下限低,但是上限高。我们有些人觉得轻松考 5 分就是简单,其实是在主动地用下限来衡量自己。而真正那些能够 “做出超级牛的东西” 的人们却不是这样。他们的课程摆在那里,想学的,以上限为目标的,自然不会认为这些课程零基础 10 天搞定(并且高一入学的时候,基本的初等函数都没学全。你 10 天能把三角函数搞定就不错了)。如果你以达到下限沾沾自喜,你大概率做不出 “超级牛的东西”。

我可能比较愚鲁,但是我自认智商还能达到中等水平,我在大一的时候,光是 epsilon-delta 语言就消化了不止 10 天 - 但是我在第一天就可以用它解题毫无压力。我可以说它很简单,看一眼就可以套它来解题考个不错的分数,也可以说它很难,因为它背后带来的数学思维的转变是全局的。

下限低带来的后果是拉低平均水平,产生很多学渣。但是与下限过高相比它有它的好处:使得那些某个方向有特长或有志向的人,能够有更多的精力专注于他所属意的方向,以整体水平的降低换来某些方面的突出发展。而恰恰是这部分 “突出发展” 才是能做出超级牛的东西的动力。

评论中有人说很多 AP 课老师根本不讲 episilon-delta 语言,甚至不讲极限的定义。可是你看看参考书中,明明都是标准的内容,为何不讲?因为整个极限的内容考试只占 4%~7%,那么极限的定义在考试中更加无足轻重。但是这种思维忘掉了,极限的严格定义,才是整个微积分中最精髓最基本的东西。这些老师的目的说白了不是教授知识,而是教授如何考试。如果说为考 5 分而努力的学习,是学习的下限,那么教授如何考试的教育,就是教育的下限。

我在另一篇回答中说过,数学上,高一水平和大一水平并没有什么了不起的门槛。有很多在前微积分阶段追求所谓 “深度” 的做法,其实和在学习方程之前去做什么鸡兔同笼、在高年级数学之前去学习速算技巧这类做法相差无几。真正重要的,是微积分所带来的的数学思维转变,而不是总结各种解题技巧和套路。

我理解我们教育为了整体公平,为了选拔门槛,把初等教育的下限提得很高,这是实际国情。这种模式有没有不合理的地方?肯定是有的,但是不是要改,如何改,这是一个社会问题而不是一个简单的数学教育问题。我不批评也不过多评论。我只是说,下限高这个客观现实给我们我们一种印象:就是我们的数学教育水平远远高于美国。我们的确可以这么说,因为大众教育是教育的重要目标。但是 “做出很牛的东西” 往往来自精英教育。在这方面,我们真的没有什么自信我们的教育水平高过人家。

知乎用户 小舜利 发表

**中国的教育模式,延续了普鲁士和苏联的教育传统,是不折不扣的「平民教育」。**教育的根本目的,是为了提高国民教育水平,以最快的速度生产出一批批零件,从而投入到社会大机器生产中。

在这样的指导理念下,学生通过填鸭式的教育迅速掌握各种知识,不少人甚至因此实现了阶层越升。

但是同样的,也正是因为应试教育制度僵化的特点,长期忽略了对人格和兴趣的培养,导致很多天才的学术潜力并没有充分发挥出来——

即便孩子们的成绩再出色,他们最多也只能从考试中获得成就感,而难以体会到求知的满足感。这是应试教育的弊端。

有人会问,为什么中国的基础教育这么好,但是科研干不过美国呢?

很简单,因为基础教育扎实,和科研取得重大成就,这两者之间的关系不一定是正相关的。

**科研成就的高低,是一个国家智力精英数量和质量的集中体现。**众所周知,尽管做科研的相对难度并不高,但是要想出成绩,甚至说取得影响全世界的重大突破,这么高难度的任务,只有那些万里挑一的智力精英们才能够胜任。

说白了,那些在科学史留下浓墨重彩一笔的天才们,个个都是星宿下凡,基因突变的怪物:他们之所以能够取得伟大的成就,并不是因为他们后天多么努力,更不是因为他们小时候受到了多么良好的教育,这些因素很重要,但都不是主要原因——尖端领域内,1% 的灵感比 99% 的汗水重要得多的多。

归根结底,天才之所以是天才,并不是因为他们有扎实的基础教育,而是因为教育给予了他们学习的机会,从而帮助他们发挥了自己的天赋,没有这样的启蒙,天才们是不可能出头的。

这就好比中国 14 亿人,绝对有很多足球天赋不输梅西的人存在,但是这些人很可能从来都没有踢球的机会——中国梅西可能在工地搬砖,也可能在 KTV 当服务员,因此即便他们再怎么天赋异禀,那也是英雄无用武之地。

这恰恰是基础教育不可或缺的价值所在:基础教育的目的,不是为了培养天才,而是通过普及知识,让更多人获得受教育的机会,由近及远,循序渐进,如此才有可能发现更多的千里马。所谓良好的教育环境,也必然是朝着「发现并培养」天才,而不是「压制并埋没」天才这样的方向前进。

诚然,在发挥人才潜力的方面,素质教育要远胜于应试教育。然而虽然很多人都称赞美国的素质教育好,殊不知美国的「素质教育」,其实是一种以金钱为门槛,资源分配不公,带有较强阶级属性的教育模式。

比如,美国的穷人读不起私立学校,所以他们的孩子很难考上好大学,即便有个别的漏网之鱼,也很可能因为昂贵的学费,不得不背负上学生贷款的沉重包袱。

**富人则没有这种烦恼:**他们从小请家教,学习各种高大上的马术、击剑、高尔夫、大提琴,在推荐信制度的亲切关照下,他们的孩子考一个好大学,那是顺理成章,水到渠成的。

说白了,美国的教育是层次分明的——那些有钱的或者读书特别出色的人,享受最好的资源,比如:实验室随便用,奖学金随便拿;但是面对家里没钱或者读书一般的人,对不起,我们只负责扫盲,其他真的帮不了你。

事实上,美国的教育系统导致精英学生和普通学生在高中毕业后分流巨大,所以大部分人的出路一般都是社区大学或者本州的公立大学,只有极少部分,大约 5% 左右的学生,可以考进 Top30 名校。

另外,大多数美国高中都提供 AP 课程(衔接课程)——这主要取决于你高中选修的速度。如果速度快,那么理论上在十年级(中国高一),你就可以上大学的数理化课。

这对于上课吃不饱的智力精英们来说,无疑是极大的利好。值得指出,由于选课制度的存在,学生们都选修不一样的课程,因此每个人的能力和差距都非常大。

有一个独特的规律是:一个大学的水平越高,那么高智力精英的学生数也就越多

比如,当年我在新泽西州立读书的时候,门门课都是 A+,考试随便做,一般都可以甩开全班平均分 30 分以上;结果一到密歇根,学业强度和压力陡增,班里随便一个美国人,都是当年高中排名前 3% 的尖子生,竞争的激烈程度就不可同日而语了。

可想而知,Umich 尚且如此,那么 MIT 哈佛这样的顶尖院校呢?

想必这些学校的高智力精英,只会更多。


干货内容:

美国人心目中的美国大学排名是怎样的?为什么美国名校的中国本科生普遍不愿意加入中国学生会(CSSA)?在美国千万别做什么?

知乎用户 芙兰朵露​ 发表

其实原因很简单,那就是科研和做题真的是两回事,尽管很多人,甚至我们的教育体系都把它们混为一谈。

就拿我自己举例子。我应该不算个笨人,初中的时候全班排 10 名左右。中考算我的人生巅峰,不过进了高中就开始拉垮。我进高中的时候,成绩是我们班所有本市人第一,所以学号也排在很前面(4 号),不过成绩迅速滑落至班里 20 名。如果我按这个水平去参加高考,根据我们学校的录取率,运气好上个山大,运气不好也许连个 211 都去不了。

如果单论做题能力,那么我绝对算不上个人才。但当我开始做科研,我发现自己身上的某些 “缺点” 反倒变成了优点:

  1. 我是兴趣驱动型的人。对自习感兴趣的事,我走路,吃饭,洗澡都在思考如何做好;相反,对我不感兴趣的事,我无法强迫自己去做好它。这导致了在考试中,对那些我不喜欢的学科,比如政治,我几乎拿它毫无办法。但在科研中,只要老板愿意给我足够的自由,我就能追随自己的兴趣去做研究。
  2. 我绝对算不上一个很聪明的人。事实上,我小学初中参加奥数竞赛也基本上是刹羽而归。但是,我的发散思维很强,并且能够在繁杂的外部条件中抓住问题的本质,并举一反三的提出解决方案。这种能力在封闭式的考试做题中,并没有太大的用处,但对开放式的,有多种解决方法的科研却有巨大帮助。

这两点让我在科研中如鱼得水。虽然现阶段,我很难声称自己是出色的科研工作者,但至少我的工作得到了导师,周围教授和同事的一致肯定,PhD 也按时顺利毕业(跟我原本的计划相比算是提前毕业)。

当然,我写这个答案的目的绝不是为了贬低我国的基础教育水平。事实上,我在国内所接受的基础教育和培养的逻辑思维能力,对我科研上的职业发展也增益良多,但科研能力和做题能力的确是不能直接画等号的。

用一个比较通俗的比喻,做题就像读书,科研就像写书。虽然读书是写书必要的积累,但并不能说读书多的人就一定能写好书。

知乎用户 萝魏紫​ 发表

做出东西,那就是工程了,科研另算。工程方面要超级牛,你得超级有钱……

知乎用户 Alan Zhang 发表

数学家并不是那种能够轻易摆布数字的人,他们常常做不到这点。数学家的主要技能是擅长在一个高层次上运用符号逻辑,尤其他们有着优秀的直觉判断力。
《信息简史》

我们的教育崇拜的是前一种人。

知乎用户 赵惜墨 发表

你们不是学数学唯一的目的就是买菜么

知乎用户 孙鹏. eduzhixin 发表

问题分成好几个部分来看。 先看是不是的问题。

  1. 美国大部分公立的中小学,数学的难度和水平和中国一二线城市的重点中学的数学难度水平相比, 弱。 如果和中国的乡村中小学相比,不好说哪个强。
  2. 美国的顶尖私立中小学,数学 “天才班” 的数学难度和水平和中国一二线城市重点中学的数学难度水平相比,强。 如果和中国顶尖中学,比如一系列附中的重点班相比,不好说哪个强。

美国整体上数学、物理等科目,尖端学生占比的确不高。 顶尖的比赛参赛人数,以及比赛试题难度上都可以对比出来。 物理竞赛为例,美国会花时间学习和准备物理竞赛的人群一届大概 1w 人左右。 中国复赛难度层级 4~5w 人左右。 按照人口比例来算,是相当的。 他们的顶尖人才的水平和中国相比是差不多的。 说白了就是美国是一个在数学、物理学科方面学习水平 “贫富分化” 很严重的国家。 中国相对来讲没有那么严重。 最顶尖的部分,美国本土人口贡献也就一般。

看到的大学、研究生阶段,很多极其优秀的一些数学、物理人才,有相当大的比例是从其他国家吸引过去的。 这个吸引力有多方面的因素。 更重要的是,美国可以吸引这些人留在美国。越厉害的被吸引留下的概率越大。

很多领域的发展还需要看积累。 比如生物医疗,比如芯片工业。 这些都不是一朝一夕就能赶上的。

嗯对了。 还有个很重要的就是,顶尖的研究成果,不一定需要最顶尖的中学作为铺垫。 需要的更多的是多样性,以及大学、研究生阶段的研究机会、资源、交流、自由。 中学阶段只要不限制太多就行了。 比如,不要逼着不擅长文科记忆的数学物理的天才,天天背诵大量古文啥的。

知乎用户 Dr.Strangehate 发表

在美国读物理博士,告诉大家,本科阶段和博士阶段的数学和物理都不简单。

首先要分清楚,美国大学分为公立和私立学校。公立学校因为学生多,要保证通过数量,所以课程和考试相对简单一些。但私立学校,一般比较好,学生相对少,精英教育,课程和考试都会比较难些。

美国人没有中国人的乘法口诀,但照样很多学生可以很快背出几乘以几得多少。

我们学校物理系有一门研究生的数学物理方法,老师非常优秀,讲的也难。作业多,比较难。期中考试,很多人拿零分,或者十分。一个原因是只写答案,没有过程分。所以需要学生记忆大量公式,对于记忆,很多中国学生有优势(除了我之外)。

不过即使这样,照样有很多本科学生修这门研究生的课。

国内本科物理一般会学热学、热力学和统计物理两门课,我所在的这所很好的私立学校的本科物理课就只有一门热力学,但就这门课讲的很快,一学期把厚厚的一本书(比国内那两本书还厚)讲完。。。进度很快。教授也认真,早上八点的课,他会七点给愿意来的学生做半个多小时的全班性辅导。

国内教育是小学到高中教育,乃至好的大学的本科教育都不错(从考试成绩和知识记忆角度来讲),但其他大学的本科教育以及大部分学校的研究生阶段就不行了,毕竟小学到高中都是老师学校和家长压迫学生出来的成绩,大学后没人压迫就没有了动力。

而美国教育强调个性、全面、创新发展,年轻时发展创造力和兴趣,到了本科大部分学生都选的是自己真正喜欢的学科,就算不喜欢换专业非常容易,到了研究生阶段都是自己非常执着的了,所以干劲十足。我所在的学校,大一是没有专业的,随便自己选,大二才开始定专业,可以加一门副科。

知乎用户 Jackie Lee 发表

我觉得原因有这么几点:

  1. 不是所有学科都需要复杂的数学。
  2. 四五十年前,美苏争霸时期,美国人学的基础数学远比现在要难,现在这一批美国本土科学家很多是那个时候接受的教育。赞成这个回答:为什么美国中小学生学的数学比我们简单,美国人却还能做出超级牛的东西?
  3. 吸引其他国家的移民(也跟冷战有很大关系)。我导师是波兰人,这学期上的随机过程,教授是希腊人;以前上过的课的老师也有希腊人、克罗地亚人,以及 xx 洛夫(应该是东欧人)。以力学举例,著名的力学家、工程师,铁摩辛柯,也是苏联人,后来跑到了美国;爱因斯坦、冯 • 诺依曼是德国人。

根据我带一年级博士生课的 TA 经验,美国学生的平均数学水平确实不如国内(我们还是计算数学系)… 要吹数学教育,我觉得还是法国、俄罗斯比较值得学习,再就是加上德国日本,美国绝对是反面典型…

知乎用户 哈皮 发表

It’s the 1% of human being that pushed the civilization forward. To be that 1%, you need intelligence, perseverance, curiosity and enthusiasm. And the education system of China is doing a good job to destroy the last two. (or the last three.)

知乎用户 匿名用户 发表

更正一下,那些超牛的东西是在美国做出来的,不是美国人做出来的

知乎用户 郑雷 发表

不请自来。本人学画画的,学了 10 年了。如果我一年内不画画。一年后提起笔照样可以栩栩如生。

你让一个学了一年天天练习的人和我比,就算他再勤奋也白搭。

规律是所有发达国家高等教育普遍水平高,而不是只有美国。靠的是基础累积。也许你在美国高校看到的小哥美女,基础不怎么。也许人家的父母可是某方面的大牛。昨天耳濡目染,以后不牛才怪。而我们的父母大部分是小职员活着农民。昨天想的事、做的活和人家就不一样。

别迷信所谓素质教育、任何发明创造都是建立在前任基础上的。你不可能让毕加索 30 岁有 70 时那么挥洒自如。中国的科技整体水平不在世界前茅,你不可能有普遍持续的创新。我们的创新还在复制别人走过的路。别杞人忧天了。

你把马斯克放到他的出生地:南非,他做多作出穿天猴,放到中国最多能做成比亚迪。他的 spacex 和特斯拉是建立在美国高科技基础上的。

知乎用户 杨傻大 发表

转的

感受一下硅谷初中里的讲座和小朋友做的作业

知乎用户 zhiser​ 发表

因为美国的精英教育,比国内的教育不知要严格多少。

精英教育花钱多,人家学校也要为学生负责,将来是为了培养社会精英做准备的。比如比尔盖茨,从小就是在贵族学校上学,他才有机会在小时候就能接触计算机,那还是六七十年代的事。比尔盖茨在父母的引导下,学了编程。再后来,进入哈佛学习。

在哈佛求学期间,比尔盖茨迷上了编程,并辍学于 1975 年创办微软。能做到这些,都是在前期的铺垫下才有的机会。

我们呢,2000 年左右才开始见到微机,还是 386、黑白屏、插软盘、没硬盘、跑 DOS 系统、只能打字的那种微机。

我也没人教,自己自学编程,后来也没有成为程序员,反而做了设计师。

我们学校也没开展计算机课程,完全是老师为了赚钱,搞了一批银行淘汰的电脑,我们才有机会学计算机。

人家精英教育,将比尔盖茨培养成了精英,我只能成为搬砖的。

我数学好不好?我小时候参加奥数得过省一等奖,数学老师一个初中毕业生,我连奥数的题库都没见过,参加奥数比赛才第一次进我们县城,在以前连我们镇都没去过。

你看,这就是命运。

所以,题主,美国的教育是两级分化的:一部分富人,将孩子当精英培养。另一面,忽悠穷人,让他们享受美国的快乐教育,未来成为快乐的工具人,以维护美国社会的稳定性。这没什么不好的。

你看,美国那么多科学成就,不都是重视教育的白人和亚裔搞出来的?黑人和老墨接近 30% 的人口比例,有什么科学成就?屈指可数吧!

他们都去娱乐到死,比如打篮球、唱歌、跑步、橄榄球之类的。黑人搞顶级科学?想想都觉得搞笑吧?不存在的。

其实,国内的教育也存在严重的不平衡。

我所在的武汉,有些私立学校重高率 90% 以上,而这些重高一本率又高达 90% 以上。而大多数普通初中,重高能达到 40% 就不错了;普高的一本率就很低了,20~30% 是常态。

但这些私立中学是有门槛的,然后每年还有花好几万学费,全程封闭式教学,管理特别严格,校风也不是普通初中能比的。

你看,将来要进入一本,从小就要付出很大的代价的。天天快乐教育?不存在的。

我女儿是现在武汉某初中的重点班,即使现在疫情很严重,每天的课仍然一节不落,周考、月考、学期考试正常进行,每天做不完的作业。这是为了她们将来都能进重点高中应该有的代价。父母也不用和她们说这些,她们自己都明白,所以也特别努力。

我小时候是在农村的小学初中毕业的,鄙人不才,只考上了本县唯一一所的重点高中。我们小学到初中会有一半人辍学,参加中考的人数只有上初一人数的二分之一,重高率只有个位数。本人全校第 6 名成绩(中考 150 人左右),勉强进了县一中,够不够残酷?

师资差得一塌糊涂,老师很多就是一个中专毕业生,比如我们的英语老师,就是一个师范生,讲一口家乡话的英语。我现在英语也很不标准。语文老师高中毕业,用我们那里的土话授课,也也是醉了。我早自习经常蒙着耳朵用普通话背课文,有时候背到忘形了,发现全班都在看我,然后大家都笑了。他们都在笑我用普通话背课文,觉得我是奇葩。

不过,我读书的时候,有时候确实是一个逗比,经常扰乱课堂,和老师抬杠,有时候老师说这样解,我说那样解,和老师争得面红耳赤,有时候让老师下不来台。但老师有时候也服我,经常别人回答不了的数学题,大多数会回到我这里。

然后,我初中数学以满分毕业,物理全县第一。但英语和语文差强人意,语文甚至没及格,丢尽了我们语文老师的老脸(老师原话)。

对了,我读初中时也不是什么乖乖宝,早上迟到是家常便饭,因为离学校 6 里路,得走路一个小时,我又起不了那么早,所以经常迟到。

有一次到学校,都上第一节课了,测试物理,都过了 15 分钟了。老师让我考,我一口气写完,还用了 5 分钟检查,老师全程就站在我身后看我写,那次还得了第一名。物理老师也特别宠我,虽然我特别调皮捣蛋。我觉得我是有理科天份的。无奈,语文和英语基础太差,最后只能搬砖。

想想我以前的教育经历,再看看我女儿现在的教育条件,简直是天上地下的区别。

美国哈佛大学,在诸多世界排名中,都是世界 TOP3 级别的综合大学。人家的学生不是天天快乐学习,而是天天泡在读书馆,一天十几个小时,要读浩如烟海的专业书,才能写出合格的论文,才能从哈佛毕业。所以,大家都没时间玩。哪像我们国内的一些大学,管理特别松散,学生就是混毕业的,毕业后脑子里比高中时还要空。学校也害怕学生毕业率太低,影响教授的绩效,所以,教授经常睁只眼闭只眼,放一些混日子的学生毕业,甚至有不少抄袭行为也不过问。

我曾招了一些国内知名大学机械或电子专业的学生,一本也有好几个。有些一本的毕业生,缺乏独立思考的能力和自学能力,需要领导推着走。

写标准文件也是逻辑混乱、词不达意。有时候让我很失望,甚至怀疑这就是一本的水平吗?毕业论文是怎么通过的?

我如果是他的教授,我就让他毕不了业,真的。

很多人不是不聪明,就是懒惰,需要逼一下,才能成才。

你对他放松,他比你还要松。

有些人走出社会,经历社会的毒打之后,就会知道社会的残酷,才明白能被学校逼着做作业是多么幸福的事情。

精彩推荐:

zhiser:为什么华为不上市呢?华为有哪些好用的产品?

zhiser:立式空调柜机哪个牌子好?立式空调柜机推荐

zhiser:拍照手机什么牌子好?高评价拍照手机评测

zhiser:笔记本电脑什么牌子好?2020 年高评价笔记本电脑评测和推荐

zhiser:智能手机哪个牌子好?2020 年高口碑、高销量智能手机推荐

zhiser:扫地机器人哪个牌子好?2020 年优质扫地机器人评测推荐

知乎用户 金陵小老头​ 发表

这只是一个天赋和舞台的关系

上帝给了我们每个人独有的天赋,遗憾的是我们也不知道该怎么用…… 甚至不知道自己的天赋是什么……

这就好像你我各有一发子弹,却因为无知而不知道这子弹是什么枪用的。

而美国的特长就是什么型号的枪都有,这些枪还喜欢主动去找好子弹,比如 nba 的选秀…… 也有最好的枪手,比如教练,经纪人,经理。他们让子弹更好的完成自己的使命,

现在我们知道了,关键点在于子弹和枪。

先说子弹,美国的长处是多元化,什么类型的子弹都有,在那里有几乎地球上的所有种族,没有明显的短板,还有庞大的数量,这是美国有那么多成就的基础。

中国的长处是子弹多,多到浪费一些天赋也无所谓的程度。十几亿发子弹,是任何一个国家都无法比拟的。

至于枪,我们建国其实还很短,可以用来给大家施展天赋的平台比美国少很正常,但是最近 30 年平台越来越多,单比枪的数量,我们和美国的差距已经没有以前那么大了。

真正的差距在于枪手,枪手要替枪去找合适的子弹,很遗憾我们这事做的不咋样,枪手还要把子弹准确的打入目标,这事我们起码在进步。

为什么会这样哪?是因为我们这些子弹大多不在自己该在的位置,不在该在的位置就算了,还不敢离开现有的位置,还为了保护占有的位置而无所不用其极……

知乎用户 Chen Qian 发表

在美国留学过的就知道,中国的 985 好学生在美国公立大学大概能超过 90% 的学生,但有那么 5% 左右的,跟你一样甚至比你更厉害。美国的科学就是靠那些人推进的。

那理科不如你的 80% 的人的工作跟数学基本上没什么大关系,就算有也可以靠计算器和 excel,复杂一点的用 matlab。需要学那么多三角函数,鸡兔同笼,牛吃草,添加辅助线干什么用呢?

知乎用户 尼克六六 发表

这个题目的高赞答案已经说得很充分了,想了解的请移步高票答案,我不再赘述。

我写此答案的唯一目的是反对那位哥大教授的答案。

雅宁心教育:为什么美国中小学生学的数学比我们简单,美国人却还能做出超级牛的东西?

说实话您夸美国其它方面都行。但是中小学基础教育?还是数学方面?

对不起,简直没法忍。

逐段说吧。

我在美国教了 10 几年的书,什么科目都教过了,从幼儿园到高中都教过,数学也教过。的确,在小学阶段我看到的也是这个样子,为什么小学数学程度可以差亚洲 2 年甚至 3 年!心算速度很慢,加减乘除也慢,看我自己的孩子,我也是看的很心惊胆跳,3 个孩子在美国土生土长。
但是没想到
到了初中阶段出现了情势大逆转
,当我的孩子还有我的学生进入了 7、8 年级的阶段,我看他们的数学,把我愣住了,我自认数学是满好的,也是在亚洲成长的学生。但当时孩子拿题目来问我,我竟然跟孩子说,「大家退后 3 步!!!!让我好好想这一题」。10 分钟后,我居然还想不出来,这是 8 年级的数学,我想怎么可能呢? 我数学那么好怎么可能做不出来?

首先您说从幼儿园到高中都教过,数学也教过,而且自认为数学满好,这个我可以相信。但是我无法相信的是,您当过数学老师,但是您孩子初二(8 年级)的题目却做不出来。怎么可能呢?

如果是个普通人,或许忘记了还能解释得通,但您不是教过数学的老师吗?

如果真是这样,那只能说明您这个老师的资格… 值得怀疑。自己不会怎么教学生?至少我拿着我的初中数学卷子问全校任何一个数学老师,都能得到答案。

其次,这题目到底有多难我想大家心里有数吧。不说特例,就平均难度而言真心简单得令人发指好吗。国内的学生只要认真复习了,去考个 SAT 数学部分没拿满分都挺丢人的。

论证观点就好好说,别动不动就搞个大新闻。

老师都不会做学生的题目,美国数学教育真是亚克西

第二天我回到教室去看到学生上课,我忽然懂了,美国的数学到了初中、高中,完全是小组式的讨论,老师们将题目交给学生,由学生组成小组自己去想办法解决,老师是不给公式的,就算有公式,也是让学生自己把它归纳出来。学校的功课也并没有回家做 50 题,也不考你速度,要的是训练你思考解决问题的能力。
犹记得在国内的数学课,老师一定会给大家很多很多的公式,大家把公式背的滚瓜烂熟,到时候套公式就好了,这一招呢,在高中之前非常有效,但其实就是走捷径的方式。老实说在高中之前的数学就是在比速度,就是代数的运算,你要是对数字有敏感度,不用算,答案都看的到。
所以说考这样的数学的话,中国学生是非常吃香的。但是美国人不走这一套,因为他们知道公式背久了也会忘记。反观我在美国的学生,也许在小学中学时代,在我们看来很像是龟兔赛跑的乌龟,又慢又没有效率,但是很多孩子到了高中的时候突飞猛进,因为他们不需要背公式,他们脑子反而非常灵活,他们脑子反而没有被各种公式及规则塞满,而是在找各式各样可能性。

数学家花费多少年功夫推导出来的公式,初二学生「小组讨论」一下就归纳出来了?

我只能双击 666,老铁您这学生都是千年难遇的天才啊。

我知道您想强调美国的教学方式是教学生逻辑思考的能力,但讲道理也都是老师一边讲解推理的过程,一边启发学生思考,您这上来就学生自己讨论一下就搞定了,那要老师干什么?

再回到国内。就我自己的经历,我初中的数学课,所有的公式也是老师在黑板上一步步推出来的。没有老师丢给你一个公式就让你做题,因为根本做不出来。

您这句「到时候套公式就好了」,搞笑的吧?

我写作业套公式都搞不定,考试还想直接套公式?在国内,如果你想考试得高分,必须要理解这个公式背后的一切。国内的考试题目都是要转 18 个弯以后才能推到最后的套公式环节好吗!

反倒是在国外,至少就我在加拿大的体验来看,那题目才是真正的直接套公式就可以了, 顶多转两个弯,再复杂一点全班平均分就要挂了。

这样的孩子数学在小学时如牛步般缓慢,但之后非常惊人。我今年回到亚洲演讲,到一所全国最好的女子中学讲留学的事情,我跟他们说到数学要考微积分,但学生惊讶的说:「还没教啊!」,我说怎么会还没教呢? 美国 11 年级开始就上微积分了啊! 我突然理解,这就是所谓的,慢即是快;快即是慢。亚洲的数学教育在初期突飞猛进,但这个表象是源自于大量的练习,这个练习是很低阶式的,训练速度与敏感度,但是对于对数学最重要的逻辑、思维还有思考能力,在我看来是不存在的,因为这个思考能力是完全需要长时间培养,也须要忍受前期看起来没有进展的感觉。这就是为什么,美国的孩子到了高中之后,数学进入高阶后,让我非常瞠目结舌。

真是恶心坏了。

高二学生还没学微积分,是因为您所谓的「快即是慢」?是因为亚洲学生不存在「最重要的逻辑、思维还有思考能力」?

这特么也行??????

国内高中学生不学微积分的唯一的原因是教育部没有把微积分放入高中教学大纲里好吗!!!

还扯上了逻辑、思维、思考,我真是醉了。

而且美国高中学的那点微积分,要多浅有多浅。

看您这意思是因为亚洲人练习「是很低阶的」,所以到了高二还没有能力学微积分?

逻辑感人。

我不知道这位哥大教授是什么意图,如果真的从事一线教学工作十几年,能写出这么幼稚的话我也是很佩服的。这一套夸赞美国亚克西的说辞可能十年前还好用,现在还想忽悠人?

不存在的。

最后说几个数据吧。

PISA (Program for International Student Assessment)每三年会对全世界各个国家和地区的 15 岁中学生做一个综合测评,包括数学,阅读,和科学。最近一次是 2015 年做的,我把数学的部分贴出来。(图片来自于 Business Insider)

在国外读书的同学,应该对「新加坡学霸」这个名词有了解吧。他们读书非常努力,丝毫不比国内的高考轻松,成绩自然也可以反映。

然后不出意外,亚洲的国家和地区名列前茅。

香港澳门和台湾教育体系不太一样,也可以放在这作对比。

中国那里有个星星,因为样本选择是北京、上海、江苏、和广东。

我仍然认为这个是可以代表中国的,毕竟湖南湖北河南四川等地的数学水平并不在这些经济发达的城市之下。

即使综合全国来看,就算低一些,也不会比这个低多少。

找不到美国?嗯,在平均线以下再找找。

全世界平均 490 分,中国同学们 531 分,美国同学们得了 470.

说好的「数学逻辑思维和思考」哪里去了?

归纳公式的惊人能力哪里去了?

以上是全国平均水平的对比,咱们再来看看两个国家最顶尖的数学高手。

拿中学生国际数学奥林匹克竞赛作为对比。

年份和排名如下。(图片来自于维基百科)

哎不用我解释了,看完图直接点赞吧。

可是为什么那个莫名其妙的答案现在居然 2000 多赞了!!!到底都怎么想的???

知乎用户 Draco 发表

中国高中做的很多事情,就是用初等数学去解决高等数学或者高等数学衍生出来的问题。

这是个很可怕的事情,以最简单的证明勾股定理这件事来举例:

在证明勾股定理之前,数学至少存在了上千年,在数学存在上千年的前提下,有人 “偶然” 发现并证明了勾股定理。

而高中生 “需要在 10 分钟之内就把这件事搞定。”

能够解决这样的问题的唯二方式:

1. 你的运气极好,恰好想到了。

2. 你之前做过一样的或者类似的题目,照猫画虎。

注意到了吗?这两者都没有 “思维逻辑的循环演进,不断深入。”

现代科学,根本不用求快,只要你的逻辑严谨正确,而且可以不断深入,集中一点,用一辈子去突破都没关系。

就跟程序员编程一样,这里去抄一点,那里去看一点,只要整个程序的逻辑正确,这些小模块的抄抄看看都是为了省时间。中国高中学生,把时间全拿去证明抄抄看看的东西了。

知乎用户 顽皮的虎鲸 发表

看了几个高赞,不太满意。

我列几点供大家批评指正。

1. 查一下做出 “超级牛的东西” 的美国人,有多大比例是在美国本土出生并完整接受美国教育的。

2. 在这些本土出生接受美国教育的大牛中,有多少是在 1980 年之后上学的。我的意思是,你需要查一查,有多少所谓大牛,是在美国的数学教育改成屎之后,完整接受了美国本土数学教育还成为理科方面的大牛的。

以上两点,可以回答是美国理科教育厉害,还是吸引人才厉害这个问题。自己动手查一查,查完你会有自己的思考。

3. 就美国的理科教育而言,我认为它做得最好的是它不仅不限制有天赋有兴趣的学生超前学习,还鼓励这个东西。我认为这个是我们应该学习的——但最近这几年的教改总让我觉得好像离这个越来越远了。

这个很多高赞有提到,但其中一个拿去宾大当例子说牛人就过分了,如果以 “超级牛” 作为标准,那么能去宾大的学生数学基础跟屎没有什么区别的,除了哈佛麻省加州理工斯坦福普林斯顿伯克利之外,其它学校的本科生将来几乎不可能在数学和物理上做出建树。因为这些学生中学的数学基础太差,本科阶段,由于大部分人数学基础太差,教的东西也很难有多深入。所以这些人跟 “超级牛” 是没半毛钱关系的。

4. 我认为客观上造成了少数美国本土学生超级牛还有一个原因,是他们学校的反智主义。我指的是天才被当 geek 当 nerd 不受欢迎甚至广受霸凌的这种反智主义。能够在这种环境下坚持自己的兴趣理想不改初心乐在其中的,超级牛的概率明显比一路顺风顺水被人表扬敬仰呵护仰视下成长的天才要大。——野花本就温室里的花生命力强得多。

知乎用户 长风​ 发表

高中的数学你还能记住多少?你还会解解析几何吗,那些做自认为奇妙的做辅助线的方法你还有印象吗?

高中毕业你可能比美国学生数学好,但 4 年之后,已经没有差别。这是应试教育的悲哀。

高中做了很多解析几何的题目,但现在已经完全没有了印象,大一时学微积分,没有做过几道题,但对微积分有很多的思考,这些思考现在还很清晰。

总之就是缺少思考,用自己的思维去思考。

知乎用户 走召​ 发表

曾在国外上过高中
也读过国外高中的数学
理解题主的意思
但根本的原因是
我国与西方教学理念是不同的
我国主导的是应试教育
所有考试题目,有它一定的套路,有它一定的公式
所以每个学生,其实就是等于机械化般的做题,然后记套路
最后在考场上能完美写出答案
但西方教育
主导的是个人独立思考和 Critical Thinking( 批判性思维)
不否认国外高中的数学大部分简单
注意,是大部分
还有少部分,其实是很难很难的
就拿我当时国外读高中做例子
从 Year10 开始开始可以学高考所需要的学科
数学这一大类,当时被分作四门学科来学习
从简单到困难
General Maths, Further Maths, Maths Methods, Special Maths
下面我都会缩写用头字母代替)
如果让我翻译,应该是
基础数学,应用数学,数学研究,以及高阶数学
GM 和 FM
GM 学的就是最简单的几何图形,一元二次,二元二次
FM 学的实际上就是应用题,将所有数学文字化,以及一些统计,概率的方法
然后自己去理解做题,
难一点开始的
先说 MM
MM 学的是三元二次,三元三次,十字函数图,到后面忘了,大概就是国内高 2 水平的数学
SM 开始,就是难了
高数,微积分等 ··
按照我当时在国内读完初中就过去的数学基础
可以闭着眼做前 GM 和 FM
MM 的话,懂一点,但中间开始就要开始看书学习做题了
SM,说实话,我自己是不敢碰,太难了
看过同学的 SM 书籍,第二章开始整个人就是懵圈的状态
同样我身边的同学,以及认识的留学生中
大部分的人
前三门,都能拿到不错的分数
唯独第四门 SM
10 个人里面,可能只有 2-3 个人会去读
这些人也是我们之中的学霸
而对比老外
会发现,前面三门学科,他们很多都比不过我们留学生
在班上,留学生在总成绩都能排到前 20%
只有第四门课,会发现老外反而有许多比留学生强了
而且强的不是一点半点
为什么?因为别人会有深入思想,有独立思考的方式去解决更深的问题
就好像,无论在什么领域,你都会发现
在某一个领域中,一些老外,真的 NB 到不行
因为他们会把某一样,他们感兴趣的东西
研究到底,学习到底
但抛开这个领域了,无论是其他领域,还是生活行为作息上
这人就是个二货
所以这就是国内教育与国外教育的差别所在
中间的 GAP,就是思维方式导致的差距
如果当国内的教育,在高中开始教导自主性思维和反叛性思维的化
我相信中间这段 GAP,很快就会填补上的

知乎用户 恩恩​ 发表

儿子 2 年级,有一天,数学有道题,要求是 “按照例题,在两种方法中挑一种你喜欢的方法完成下题”。

儿子跟我说,他两种方法都不喜欢,用了自己的方法,所以老师算他错。

我给予鼓励。

在辅导儿子数学的时候,我反复告诉他,数学思维讲究通用做法,而不是特例特解,你做完一道题后,要想想如果问题中的数字换成了别的,或者条件发生了轻微变化,是不是可以用同样的方法解决。

他是听进去了,然而学校在教学过程中,却痴迷于让学生记住那些特型和讨巧。。

知乎用户 王星泽 发表

我理解问题问的其实是为什么美国人做出比中国人牛的科研成果。几点潜在原因:

  1. 话语权。是不是超级牛呢?评价一件科研成果有多牛,裁判权大多在美国人手里。
  2. 移民。是不是 “美国人” 做的呢?相当一部分科研成果是由没有接受美国中小学教育的移民所做出来的。高端人才,日积月累,形成的良性循环,是目前我们望尘莫及的。
  3. 传统。中国的体制更鼓励近亲繁殖,如果不小心掉坑里、更难否定自己、从而更难创新。
  4. 市场和资本。美国人有钱敢冒险(包括私人的、军费等);在中国,毕竟我们还是发展中国家、产业升级还没有完成,就算有能力做出超级牛的东西,市场往往会教育你不要太超前。
  5. 知识结构。对于发明创造而言,批判性思维、不惧权威、沟通能力等等可能比学会严谨的三角函数公式更重要。

知乎用户 有鱼 发表

因为这是个伪命题。

美国中小学生学的数学广度极大,这就为后续发展奠定了很多概念性的东西,准确的说他们的数学拓展性和抽象性很强,对算术要求不高。

中国中小学生学的数学本质上不是数学,是算术,即使到了高中阶段,还是属于算术游戏,不是数学。数学指的是以抽象概念为基础进行的纯粹逻辑推导。算术指的是计算和求解。

因此,在两种体系的训练下,中国学生表现出较高的计算能力;欧美同学表现出较高的数学水平。

大学之后,开始现代数学训练,立马就可以看到这种效果了;有的同学自认为高中数学挺好,但是大学里的高数都考不及格,或者表现平庸,其实本质上不是这个人变懒了,而是因为他适应不了数学。

后续中国和欧美科研水平(数学物理水平)相距十万八千里,做出来的东西自然就相距十万八千里。

本人从国内 - 到国外这一套流程都跑完了,以上属于亲身感受。

知乎用户 壮士请收下我的赞 发表

你在拿中国高中生和美国高中生对比的时候,

已经默默地把中国一半同龄人开除国籍了。

美国是十二年义务教育,接近 100% 的人都上了高中,

而中国初中毕业生的高中录取率至今只有 50%,

以 2018 广东省为例,

全省初中毕业生 109.76 万人,

高中阶段(含技校)招收 105.5 万人,

普通高中招生 53.8 万人,

在初中毕业生中,高中参与率为 49.01%。

各省份的数据,各位有兴趣可以自己去查,大致情况是 40%-55%,

有几个大城市能达到 60%-70%。

换句话说,你是在用美国所有适龄学生,对比中国的 top50%,

针对所有人的教育,下限自然要放低一些,保证绝大多数人能理解,

而因为高校能吸引全世界最优秀的高中毕业生,上限自然也要高一些,

这都是很自然的事情。

知乎用户 梦羽灵泉 发表

不是美国人不做题,而是中国人需要靠做题来突破自身阶层

其实说白了,美国上下也四亿人,中国的三分之一,而且美国人家是大豪斯国家,实力还是要雄厚一些的

那么同样的框架下按比例来算,美国其实并没有和中国差异那么大,如果只是一个千八百万人口的小国,教育产业不完善,你有系统性偏差还可以理解,但美国基本上可以和中国平行比

所以什么东亚智商论其实有点儿,结果主义的意思

基础智商谁还比谁差多少?

你是因为你上了大学,然后觉得说中国人数学好,那是因为你遇到的人都是数学好的筛出来的

你记不记得你小学有多少不念的

所以其实大家都差不多,只不过美国的测试是基础,从低下砍,你不太笨,有基本能力就行

但中国不行,因为中国人太多的同时,需要集中优秀人才干大事儿,同时又没有足够充分的教育资源给所有人大豪斯,所以需要做题

爱因斯坦十五岁自学微积分,你还真觉得人家教育水平差了?

知乎用户 Mr Poopybutthole​ 发表

(* 欠打式提问一般都是知乎小编的 KPI 提问,而且非常暴露知乎的底色。这个问题目的也是让各位在回答中大放彩虹屁吹美国,所以我态度比较差,我也懒得迁就谁。我从小到大闻得美国彩虹屁多了去了。但是本回答还是就事论事,不吹不黑,我也不打算吹一番中国教育的彩虹屁,因为无屁可吹。彩虹屁诸位要放,请在答题区放,别搁我评论区放。有点能力去 Facebook twitter 放。不过事先说清楚,彩虹屁这玩意在美国可不是财富密码,还会被洋人尤其是左派嘲笑为 self hating Asian,反倒是红脖子有可能给你投来赞许的目光,但是内心里骂你是个脏东西)

第一,你为啥不先数数,身在美国的那些做出很牛东西的人,特别是负责数学部分的人,有几个人是在美国接受的小学数学教育的呢?

第二,如果你真接受过多年数学教育,你可以回头看看中国的小学,中学,大学课本。内容递进是线性的么?中学 6 年的数学内容深度广度是小学 6 年的几十倍上百倍不过分吧?大学内容又是中学的几十倍,没毛病吧 (前提是你认真学)?硕士博士搞的研究要是需要数学知识,那学的又是大学深度的不知道多少倍,不错吧?那你前面 10 米比别人多出了一个身位,凭啥就觉得你马拉松就能赢么?想多了吧?

第三,就算是同一个人,同样的能力和态度,在不同的平台做出的东西的牛逼程度也可能不一样,取决于所在的平台有没有巨人的肩膀可以站。好比做游戏,同样的工作量,在大厂有自家牛逼引擎,有厉害的编剧,有给力的美工,有大笔预算,你也许能做出巫师 3 的一个关卡,换个厂也许就只能写个斗地主倒卡布奇诺功能。现在的中国人比 20 年前的中国人数学更好么?不见得吧?但是现在中国人做的东西是不是比 20 年前牛逼?

码农喜欢说 “造轮子” 你知道这个词语背后的含义么?Reinvent the wheel 是一句英语俗语:轮子是人类文明上最具里程碑意义的发明之一,“重新发明轮子”实际是一种挖苦 – 发明轮子很了不起,但是把已经存在的东西再发明一遍就属于浪费精力。码农所说的 “造轮子” 很多情况下也是无奈的自嘲。已经很成熟的东西,出于种种限制不能直接拿来用,只能反复造轮子。你看哈工大,本来轮子用的好好,半路被人砸了,只能再花宝贵的精力去重新发明轮子。他们是数学能力不够,才不能造机车的么?

综上所述,有此疑问的人,应该都是数学不太行的,脑子里因果关系比较乱。

知乎用户 粒粒赵 发表

对一个事物的兴趣决定了你探索的深度!

我在美国教的数学很简单,但是内容很广泛,三年级就已经涉及 3D 图形了。 当然,美国学生不好教,还有点儿笨,有点儿懒,有点儿。。。所以要求老师不只提供一种方法,还要提供多种方式,游戏啊游戏啊还有游戏啊!这样他们就不会厌倦。

而且,他们学会了探索!!!就是这个神奇的玩意儿,我们国内的孩子几乎木有啊!你若不是要参加奥数或是科技比赛,整那玩意儿干啥?

美国人的探索就如同各种折腾,自己折腾的高兴,根本不理会周围人的看法,越 crazy 越个性,周围的人呢, Great Job! Excellent! 家长也喜欢参与。我学生的很多 project 都是家长帮着给主意。。。全民爱折腾!

从儿童教育方面,美国是给足了孩子发挥的空间。

知乎用户 詹姆斯. 通​​ 发表

我的观点是,基础教育是基础教育,高等教育是高等教育。

美国本土科研工作者的主要数学能力是在本科和研究生阶段培养的,而不是中小学。科研能力也不等于做题,你要说我们自己的中小学生数学好,我觉得咱们只是做题又快又好,尤其是三角函数题,那简直是秒杀美国人,但科研能力真不见得有多好。我是看着很多连二元一次方程都算不好的美国孩子,在本科学了几年后,可以和我聊两句线性代数了。我承认他们的本科生也还很嫩,但咱们中国的本科生也不是各个数学学的多好啊。而且他们的本土 PhD 们就能经常聊些我不懂的数学话题了,另外这些本土 phd 们明显在竞争教职以及以后申请项目经费等方面更有优势,只是人数有点少。

有些朋友还提到,美国 “有钱人” 的孩子可以享受更好的基础教育,其实他们享受的不是基础教育,而是提前的高等教育。比如说所谓的高中 AP 课程,这个其实是大学的课程提前开到高中里,让有余力的孩子可以早点学习到更难的东西。有很多好处,还可以冲抵大学的学分。但这也不是啥了不得的东西,也没人说因为一门 ap 当上数学家的,更多还是作为申请大学的敲门砖。

数学知识像海洋一样,大头其实在后面,高中那点东西既不够广,也不够深,也不严密。中学时代数学学的再难再多也没有太大用,除非真是天才,你自己愿意,否则做太多题反而抹杀了孩子们大学以后继续学习的兴趣。就拿题主所说的计算机图形学来说,会用到很多中学的代数、几何、三角函数这些东西,但这些并不难,如果你真是干这个东西的料,你中学不用花多少时间学,大学里看看就好了。然后在大学里你会继续学习微积分,线性代数,信号与系统分析,射影几何,微分几何等等。这是个非常宽广的领域,并不在基础教育的范畴,如果你不是搞这个专业的,其实也没必要去学。学了你也学不会,学会了你也记不住。

但上面说的还只是应用,如果你要在上面的某个领域做科研并寻求突破,你得花个几年读个 PhD 吧?其实前几年也就是跟着导师灌水,等你真有点想法了,又着急毕业了。等毕业了,你得找教职吧,教职得评职称吧,这折腾下来,等你可以安心坐在办公室里,端起一杯清茶,想捡起自己年轻时候的想法的时候,你老婆一个电话告诉你女儿要交学费了,你说这玩意不是真爱你能搞不?你高中学的再好也没用啊。

知乎用户 哥淡定 发表

这样说吧,能做出超级牛的东西的人是极少数中的极少数,可以说是凤毛麟角,有些简直可以说是不世出的天才。

能让这些极少数的牛人脱颖而出,发光发热,并做出超级牛的东西,则是弊端重重的美国(公立)基础教育体系的一个优势。

99% 的美国中小学生(这里主要讲公立)学的都是极其简单的数学,其深度、广度、难度和各项要求甚至在逐年降低。具体体现在广大美国成年人的数学水平、乃至逻辑思维水平这些年来越来越差得令人咂舌。从某种意义上说,美国的中小学基础教育是向 “差生” 看齐,“差生”没学会,老师就会尽量放慢全班节奏、降低难度,以达到不让 “差生” 难过自卑的目的。加之美国中小学一般的作业、练习也少,故而普遍数学水平低也是必然。

学校教的简单,学习压力也不大,对于那些极少数的天才学生来说就是即 “吃不饱”,也有充分的时间和精力去继续培养自己的兴趣和特长。美国是发达国家,科教资源丰富,尤其是条件良好的图书馆遍及每个乡村小镇,这都给这些孩子创造了非常好的客观环境。钢铁侠马斯克,加来道雄这些人都说自己小时候已经把当地图书馆里自己感兴趣的科学书籍都读完了,还在附近大学图书馆里借书。很多人也提到,美国的中学有 AP,学有余力的尖子生可以选择,而数理化的 AP 很多内容已经是大学一二年级的内容。油管发明后,更加海量更加高质量教育资源数不胜数。

所以在美国如果你有天资、兴趣、可以持之以恒不断上进,家庭条件不错可以提供必要的支撑,那么你有很多可以学习高深课程与知识的机会。

美国的这种向下与向上同时 “宽松自由” 的模式,一方面会制造出大批反智低能的普通民众,一方面也会让极少数的杰出精英脱颖而出。加之美国从全世界吸收顶级移民人才,这些美国本土与移民精英才是 “做出超牛东西” 的人。

其实这也是美国的现状,什么样的教育体制培养出什么样的国民。有特朗普以及静脉注射漂白消毒剂的红脖,也有比尔盖茨、马斯克(他是移民)这样的精英。

知乎用户 yilin wang 发表

这两天看了一句话,说美国的教育讲究的是输在起点,赢在终点,而中国的教育是赢在起点,输在终点。
美国是世界上对基础教育投入最多的国家,我们没有任何理由认为美国人是傻子,傻子是无法建立一个世界最强大的国家的(对不起我又跪舔美爹了),美国的基础教育是建立在大量对儿童心理和生理的研究之上的,而中国所谓成功的基础教育,是违背儿童发展规律的。
第一,对于儿童来说,运动比学习知识更重要。近年来国际学术界提出了儿童发展 “感觉处理” 的概念,也就是说,孩子只有能很好的控制自己的身体,才能更好的感受世界,获得并且良好的整理的感觉输入,这是智力发展的基础。孩子如何能够更好的控制自己的身体呢?说白了,就是运动,孩子能跑,能跳,能翻跟斗,会荡秋千,爬高,会拍皮球,踢足球,就是孩子了解,并且运用自己身体的重要方式,有一些特殊儿童,比如自闭症的孩子,还应该专门进行这方面的训练,这叫“感统培训”。而且特别要注意,这种培训是有窗口期的。
我在日本出差的时候,去过一个体育馆,看到很多家长把一些八九岁的孩子送来训练,训练什么呢?就是走一段平衡木,然后翻一个跟头。我当时非常不理解这是在练啥,专业练体操又好像太简单了。后来自己有了孩子,看了一些书,才知道翻跟头,走平衡木这些训练,能够很好的刺激孩子的前庭,对孩子集中注意力有好处。而且,这种对身体的训练对孩子的一辈子都有好处。我们的同龄孩子在学什么呢?在学奥数,非常痛苦的学几年,学完了这辈子 恐怕都用不上(初高中学习的初等代数完全是另外一个体系),孩子还小,为什么我们就那么着急出发呢?为什么不能让孩子把身体锻炼好,四肢协调,能够集中注意力之后,再去学习呢?
所以说,我们孩子所谓的数学成绩好,完全是以剥夺孩子运动和户外活动的时间换来的,让孩子掌握了很多无用的知识,却错过了发展身体和大脑的窗口期,非常不值得。如果孩子在幼年时期,花时间运动,把大脑和身体调整到最佳状态,再开始高强度的学习,难道不好吗?
第二,我们完全不顾孩子的发展特点。中国人总有一种说法,说小孩子学东西容易,这是大错特错的。小孩子学东西很难,哪怕是成年人眼中最简单的事情。有一个基本的概念,就是孩子发展是有阶段性的。这个阶段不可能做下一阶段的事情。但反过来,高阶段的孩子做初级别孩子的事情,就觉得很简单。比如,你让 6 岁的孩子做二十以内的加减法都很困难,无论你怎么练,孩子可能会有点进步,但始终会觉得困难,而且你无论怎么练,怎么教,孩子也不可能掌握两位数的加法或者简单的乘法(神童不在考虑范围内)。但是,你让三年级的孩子做这些,可能就很简单。
再举一个例子,我自己读书时就发现,其实班上小学的时候最差的学生,只要升到初中,也会做小学阶段的拼音,算数和阅读。但这些学生在读小学的时候,学的非常痛苦,老师天天逼,父母天天骂。我当时就觉得,其实这些打骂完全是不必要的,孩子长大了之后,这些东西其实都能掌握。
但我们的教师和家长就特别着急,一定要孩子必须立刻,马上,完美的掌握现阶段的知识。比如小学一二年级,甚至三四年级的学生始终有一个教学任务,就是不停的做口算。大篇大篇的做。但实际上有必要么?孩子现在做这些题觉得困难,甚至做错事正常的,这是他发展年龄决定了的,再怎么练,提高不会很多,其实你让孩子大一点再去做这些题,自然就好了。其实这种枯燥的,重复的训练对儿童能力的提升非常小,你做的再熟练,能熟练过计算器么?但机会成本非常高,因为他过多占用了孩子的时间,让孩子失去了了解信息,探索世界,发展兴趣爱好的可能。

第三,始终有淘汰思想。我们的教育始终有一种选拔机制,认为不合格的孩子就是要淘汰掉,而孩子为了避免被淘汰,必须要花大量的时间在考试上面。而考试为了选拔,就可以用非常简单的知识,出非常难的题。其实这些刁难人的难题做不出来,并没有什么,并不意味着这个人就是失去了追求知识的资格。世界上老师做不出来的难题多了去,也没见老师主动辞职的。但老师却主观(当然,也是被这个体制逼的)的认为,做不出来这些题,就没有资格升高中,升大学,这就逼着孩子做难题。我高中三年最大的感悟就是,我们不是在学知识,其实是在和出题老师搞斗争,光懂得知识是不够的,重要的是搞清楚出题人的套路。这样 给孩子造成一个非常不好的后果。第一,不会提出问题。整个学生年代就在不停的解决老师出的题,压根没有办法自己提出问题。第二,没有解决问题的能力。表面上看我们做题时在解决问题,其实不是的,我们是在出题老师的规定和引导下解决问题。第三,认为知识是无用的,只能用来做题,不会用知识解决生活中的问题。第四,对于有些发展较慢的孩子,很早就被淘汰出局了。

第四,文科教育极其失败。整个教育阶段,中国学生的阅读量是严重不足的,如果只读那么薄薄的基本语文书,基本上等同于文盲。更重要的是,历史和政治教育的缺失(我们现在那叫灌输,不叫教育),让孩子基本上失去了对世界的框架性认识。他们基本上不了解,我们社会为什么要这样运行,是什么导致了我们这种运行方式,以及这种运行方式是否合理。

这些弊端都导致,我们的孩子学到后面,就后劲不足。大部分大学生,从中学进入大学之后,整个人就散架了,完全失去了追求知识的动力。而事实上,二十岁开始,人才真正进入了学习的黄金时期,因为个人在生理上,心理上都发展成熟了,学习起来效率特别高,心理状态特别好。但可惜,大部分中国学生在这一阶段放弃了。所以说我们的教育是一种输在终点的教育。

知乎用户 匿名用户 发表

我在中国接受的从小学到大学的基础教育,在美国念的 PhD。对于中美两个教育系统也算是深度参与者。

先说说数学,没错,中国的小学,初中,高中的数学都比美国的数学难。但是,大学的数学,美国的绝对不比中国的简单,到了研究生博士生级别,美国研究的课题的难度甚至要超过国内。为什么会有这么大的区别呢?我觉得在于两国教育的定位不太一样:在中国,教育的关键似乎在于选拔,无论是高考,还是奥赛,都要排个名次,这一点是深受古代科举制度的影响,自古状元都是衣锦还乡,况且中国人数这么多,如果简单了,每个人都满分,清华北大该怎么招生?在美国,大学以前的基础教育真的就是基础教育,数学的话并不是每个职业必备的知识,比如一个学生如果未来的职业是律师,就没必要去学如何根据椭圆的参数计算椭圆面积,或是拉格朗日中值定理之类的了吧?但是在美国,到了大学,如果是数学系,那真的就是扎扎实实把整个数学的根基学好,尤其是到了研究生博士生,那么研究的问题很多甚至都是国际上 open 的问题,到了那个层次,其难度绝对不比中国的同等层次的研究机构差。

再说说 “为什么很多需要数学的东西(比如计算机图形学领域)都是人家做出来的而不是咱们?”。这个结论非常片面和武断。我真的是很反感没做任何调研就发表结论说 “都是人家做出来的而不是咱们”。既然题主点名举例计算机图形学领域,那我就说说计算机图形学。我恰好是学计算机的,虽然我的专业是计算机网络,但是我 07 年在微软亚洲研究院(MSRA)实习的时候,是深知 MSRA 如何在 SIGGRAPH 上辉煌的。ACM SIGGRAPH 是什么?学计算机的都知道,凡是 SIG 打头的都是超级 NB 的学术会议,SIGGRAPH 也不例外,是 computer graphics 最牛的一个学术会议,而在上面发表论文,对于图形学领域来说,就相当于 Science 和 Nature,每年在 SIGGRAPH 上发表论文的 MSRA 基本上都少不了,里面作者的名字放眼望去很多都是我们中国人(我随便翻了一下 SIGGRAPH 2015 就是这样 SIGGRAPH 2015 Papers) 如果硬要说办公室在北京,招的也是中国人的 MSRA 所贡献的知识是美国的公司微软的,因此也算 “人家的”,那我就没招了。

另外很重要的一点,在美国,很多的学术贡献也未必是美国本土的基础教育系统培养出来的人。美国是一个移民国家,这一点和中国很不一样,美国有来自世界各地的大脑来给他们做学术贡献。比如中国,我在清华那会,基本上理工科每个系学习成绩好的都优先考虑去美国的名校留学。值得一提的是印度,虽然被国人各种鄙视,但是阿三的智商和勤奋程度是不容小觑的。我在读博的时候,同学分为 3 类,一类是国人,一类是阿三,一类是其他的人(包括很少的美国 local)。硅谷的 IT 公司,一堆 CEO 都是阿三,很多 NB 的互联网公司,阿三的数量超过了三分之一。千千万万阿三的学术贡献也算是美国的基础教育系统的贡献吗?

所以问题不在中国人的智商不够高,也不在于中国的基础教育系统太差(当然也是有值得改进的地方,但是不是致命性的),而在于中国的大学(尤其是博士阶段)和研究机构的学术研究的氛围和机制,同时也和资源有关。所以与其去盯着中国的中小学看,还不如直接点,盯着现在中国的大学和科研机构,该如何改革,如何吸引最优秀的人才(全世界范围的),如何改变机制。

最后也希望大家能看到中国的进步,不要太妄自菲薄,觉得我们啥都比老外差。我认为中国人是世界上平均智商最高的人种之一,如果机制够好,勤奋程度足够的话,中国可以做出很多题主问题里说的 “牛逼的东西”。

知乎用户 还是尔等最秀​ 发表

因为中小学的数学内容对做出超级牛的东西帮助并不大。而到了大学里面学的东西,才是真正有助于出成绩的。

中国的中小学生,数学再好,有多少是出于真正的热爱?有多少只是为了考试得高分?高考一过,就是所谓的解放了自由了,到了大学里面还有多少是可以坚持钻研全心投入的?中国的教育很大程度上是本末倒置,中学的时候,非要把基础数学复杂化,用 n 种方法解平面几何,三角函数,数列,出题老师变着花样的折腾学生,搞得大部分学生完全没有对数学的兴趣只是被动的学习接受。等到了大学,真正需要对数学彻彻底底的进行钻研的时候,真正喜爱数学的已经寥寥无几,一到期末就划重点六十分万岁九十分有罪,还能指望什么?不用拼命的时候逼着学生玩命,透支学生的兴趣和热情,真到了需要玩命的时候却是一副潇洒自由轻松的环境。怎么搞?

从大环境上看。如今高考毕业生,一门心思奔着就业率好的专业去,只想着毕业后找份待遇好的工作,金融,互联网专业年年爆满,相比之下医学,教育,物理数学这些需要沉下心来研究的专业遭到冷遇。但是话说回来,整个社会的大形势就是这样,弥漫着一股金钱至上的风气,一个明星随随便便拍个烂片就是百万级别的收入,真正做学术钻研创新的学者,待遇让人心寒。这个问题不解决,白搭。

知乎用户 匿名用户 发表

我最近看见有讨论说国外数学教育强调逻辑,而我们强调计算,但我自己反思自己:自己都无法理解数学逻辑,怎么教育孩子?于是要了美国同学上 3 年级的孩子的数学教材来看,印象是:美国孩子的数学一点儿都不简单!(抱歉不能提供实际的素材佐证,因为他们教材有版权,我也是登陆他们的网校看的,也承诺过不能扩散。另外,也需要说明,他们的数学教材也是自己学校编的,只能代表他们学校,不能代表全美国。)

先说为什么有些人觉得美国孩子的数学简单。同学儿子上 3 年级,以我们的眼光粗粗来看,他们学的是:

1)10 以内的加减法

2)1000 以内数字的读写

这是什么级别?我女儿学校中班就已经教这个了 !

但我看了 3 个小时的教材,越看越汗。他们教的我们没教的:

1)同样从 1 数到 10 再 11,我们是单纯的数,数到 10,100,1000;但他们数到 11 后,开始讲进位、十进制,开始引入二进制、五进制等;

2,讲 10 以内的数字,区分数字的用途,同样是数字 3,可以是 3 个房间,第 3 间房,房间长 3 米,他们有什么不同?

3,在数字的用途一节后需要写 paper:我们说华盛顿有人口 3,454,456(数字是我随便编的),这个数字是精确的嘛?还是估算的?为什么?怎么证明?

4,有一些逻辑题目,类似于 “教授的隔壁是医生 医生喜欢蓝色” 这种问题,我们孩子是从小作为智力题目来做的,他们则介绍了只需要用到 1-10 这几个数字的表格,介绍如何将这些条件填入表格,最后如何出答案。

5,其实还有更多…. 但我对自己非常失望的是,我这个大学本科毕业学过高等数学的人也没看懂他试图解释什么数学思维,但肯定是有的放矢。

总体感觉这个的教材:

1)强调数学的基本概念;

2)强调逻辑思维

同样是逻辑问题,我们一直强调的是用脑子解决,会的就真是个聪明的孩子,不会就你怎么这么笨。而他们从小就拿出来,教孩子用工具解决。

国人一向号称自己是最聪明的民族,但创新实力却很弱,我相信我观察到的这一点是原因之一。

3)细致。

我们长大后,看老外的文档,事无巨细,是不是特别佩服?可为什么我们从来没有耐心看下去,写出来呢?我想,这就是原因,我们都没有耐心为孩子细致的解剖这个世界,孩子长大后怎么能细心?

知乎用户 nan hu 发表

因为初高中的数学,在其他科学技术领域中用到的数学和搞数学研究是三个完全不同的东西。

知乎用户 Quantum Engineer 发表

哎咱别尬吹美国教育了。美国理科教育就是不行,小学中学大学都不行,尖子也不行。美国人都不学理科了,全靠外国人搞。

我在国内 top2 读本科,美国 top1 博士,物理专业。见到了一些这里的本科生,这是毫无疑问的尖子了吧。水平真不怎么样,我老板交一个高年级本科生课,经常吐槽学生基础太差,线性空间、矩阵这种概念都不知道,搞得她不知道怎么讲课。做过 ta 的都知道美国本科生水平差不说,找你要分脸皮极厚,软磨硬泡,对成绩的贪婪比中国学生过分多了。

你要说中国学生都是做题家,美国人都是创新教育。咱来说说科研。有志于本科毕业后深造的理工学生大多会去找个实验室做科研,这足够体现创新能力了吧。我现在的 lab 经常会有本科生,有中国来的也有本校的美国人。我们博士生给他们的课题都不是一个级别的,美国本科生让他写个 code 跑一跑模拟算不错了,中国本科生都是直接给一个方向自己调研文献做理论跑模拟,最后很多能发出来一作文章。当然,美国最牛的学校最牛的本科生那是强,有本科就能发一作 science 的,这个我在国内是没听过的(限物理领域)。但是这是因为我国科研水平和美国差距太大,国内物理领域一年不见的有一个 ns,这边很多 lab 一年保底一篇,所以本科生的上限自然也没法比。我觉得把国内最强的本科生弄过来是有的一比。

至于为什么美国做出来的东西牛,你看看我们研究所教授的构成就知道了。我在哈佛 mit 合办的一个研究所,大概 20 个教授,其中土生土长美国人有一个半,那半个是华裔二代。但是剩下的那些虽然不在美国出生,现在却也是美国国籍了。美国科研为什么牛,明白了吗?

我们对自己的理科基础教育应该要有自信的,不能跟美国学改的越来越简单,美国科学家根本不是美国培养的。现在时代变了,科学的金字塔已经越搭越高,所有的创新必然基于对该学科现有知识的全面深入的了解,或者对实验技术精益求精的探索。一个基础不好的人很难靠所谓的创新思维发现什么了不起的东西。相反,如果学到了某领域的前沿,创新其实是水到渠成的事情,因为学科分类也越来越细,在某个领域最前沿的人全世界可能也就几个,找到可以继续发展的方向并不难。所以我们要学应该跟苏联学,短短几十年出了多少顶尖科学家,现在苏联亡了,但是在各个领域还能见到毛子的身影。而且这些人共同特点就是理论功底极好。人家怎么搞的?就是应试、奥数。全国的尖子集中在一起学,搞末位淘汰,比中国恐怖多了。

知乎用户 旭 - ASAHI​ 发表

初中生连π是无理数都不用知道了,害搁这优越呐?

如何看待教育部印发义务教育六科超标超前培训负面清单(试行)?

知乎用户 同变分之伦 发表

说俩小故事,不指望题主听懂其中的禅,因为我学了好几年数学才懂:

1. 二十世纪后半叶最伟大的数学家格罗滕迪克不会做乘法

2. 拉格朗日在发现一个叫柯西的骚年很有天赋时,让他爸爸保证柯西在十七岁前不要过多接触数学

知乎用户 知乎用户 033qRx 发表

从我的视角看,感觉是历史积淀不行,造成大学教育不行。而牛的东西起码得是在大学专业性质的去研究。

我大学是学计算机科学的,我能感觉出教软件工程、微机原理、数据库的老头、老太太们根本没怎么摸过计算机,就是会背书、特别能背书,和他们学习,只能对各种概念混个脸熟而已,做出什么东西?不可能。教 C++ 的老师是请的某公司的高级程序员,结果表达能力不行,根本听不懂他讲什么(过了这学期就被投诉到离开学校了。)。猜测美国应该有科研人员,在研究的同时在大学任教,还可以调动一些大学生作为助手(一举两得)。

高中时候别说研究东西了,除了休息、娱乐,基本就是学学学、背背背,大学也为了四六级挠头,而做研究吧,还是得相对 “专”。别说上学了,就是工作,想过个面试,也得背背背,工作 10 多年了,人家拿书本的东西问,一样能问住,很少面试官问工作中解决过什么特殊的问题,解决问题的思路等这些,因为面试官可能也没什么 “积淀”,可能都没工作几年。

知乎用户 王听玄​ 发表

因为都是华裔在做。

知乎用户 小兔​ 发表

你不能用衡水中学的水平比人家公立黑人老墨小学的水平。要比人家的私立精英小学的水平,你会发现人家抓得一点也不松,水平一点都不差。

要比,就用中国的菜场中小学水平比,你会发现中国的水平也高不到哪里去

知乎用户 呆蛙 发表

做出牛逼东西的又不是中小学生……

最后还是学士硕士博士博士后在科研前线。

蒙古人摔跤射箭马术有先天优势,民众体格普遍比中国人壮,但是解放军能轻松碾压蒙古人民军。

知乎用户 huang liu 发表

在数学上,中国古代是以实用为主,西方以研究各种脱离实际自己臆想的完美的模型各种平行线证明为主。哪个对生产力有价值不言而喻,真正学以致用才能激发积极性

现在正好是反过来的,中国数学现在把基本的一些西方的定理证明,计算演化搞得极度复杂,并以此为傲,很多人都搞烦了。西方数学恰恰真正在建模,金融,人工智能进行大规模应用,反而让人兴趣极大。

中国应该抛弃西方近代所谓数学传统,恢复自己的数学传统,以计算,解决实际问题为核心构建数学教学。现在云计算,大数据,人工智能,正是这个的好时候。

知乎用户 lu luce 发表

因为美国有钱。。

聪明的大脑是可以买的。我们俗称科技移民。

就是你如果清北毕业,数理化基本功扎实。你在美国可以有非常好的生活条件。

住大房子,开大汽车。

因此上,在 2008 年以前,全世界的聪明人都去了美国。

知乎用户 笨虎 发表

去瑞典读硕士的时候发生的事:一天在图书馆里有两个瑞典金发姑娘走过来,跟我旁边一起自习的同学搭讪。

她们问他:你是中国人吧?你能教教我们微积分嘛?

颜值决定一切,没办法。这个同学其实是我们这几个中国留学生里最爱玩、数学最差的一个,但谁让人家长的帅呢?

但是他数学不好啊,他怕露怯,但又不想失去这个艳遇的好机会。他就答应了,带着复杂的面部表情跟着她们走了。

过了一会儿,他回来了,表情很是轻松自得。

我们问:怎么样?

他说:害,什么微积分啊!就是咱初中学的二元一次方程组

瑞典大学里面教数学,是从奇数偶数开始教的。

所以二元一次方程组,对于瑞典人来说已经是很高深的数学题了。

我们读经济金融的,国内本科要学微积分、线性代数、概率论与数理统计。而且考试的时候很多题都会故意刁难。

而在瑞典呢,研究生课程把这三门合成一门,叫经济数学。而且考试非常简单,拿 VG(最高评分)轻轻松松。

这件事我问过瑞典的教授,人家说:

你学经济的又不是学数学的,数学说到底只是一门辅助研究的工具,运算过程现在计算机软件都能帮你搞定了。你要把更多的精力放在研究经济现象上,而不是解数学题上。

这就是欧美的教育,把数学作为一种工具,够用即可。

而不像我们,复杂的数学只是为了层层考试选拔,很容易把人学傻。

如果真对数学有强烈兴趣的人,自己选择深造就好了。

国内有些人说欧美这种快乐教育是为了愚民,我没觉得啊。

拿瑞典来说吧,从小学到大学教育都是完全免费的,只要你想上大学想深造就一定能上。

只不过即便你读完博士,做了大学教授,工资也不会比学校里的厕所清洁工高出太多。

全看个人志向了。

摘抄一个点赞很多的精选评论:

幸好我知道瑞典有 Mittag-Leffler 这样对数学家。 否则看了你的小故事,我就会以为瑞典人数学不如中国人…

这个评论者的阅读理解能力,满分 10 分,能打几分?

知乎用户 金头大老虎 发表

什么年代了,还有人相信那些万赞的答案?那些答案放在 80 年代有人信还好一点,现在还有人真信?答案显而易见啊,美国的经济基础决定他们的孩子可以很早的接触到各种科学技术,就是这么简单!

远出说你们看看北京那些重点小学和中学的科技馆,近处看看每个省定期组织的中学生代码编写比赛和机器人比赛?哪怕是放在我们 70、80 后身上,但凡我们接触了电脑,那点比美国的差了?以前为什么有差距?你让连电视都没看过的孩子想想一下?你让飞机都没见过的孩子想想外太空?

还什么美国教育制度?最好的环境和最多的经费吸引高级教授,然后通过奖学金吸引拥有最好基础教育水平的学生来学习,用了战后 30 年西方差不多所有的精英凝结成的结晶怎么就成了制度使然?

知乎用户 一苇奈何 发表

教育。

假设各自有 1000 个学生的话。

美国是所谓精英素质教育,美国要做的是争取把这 1000 学生里面最有天赋创造力最有发展潜力的那部分选拔出来,投入最大教育资源。至于剩下的那部分,大部分是普通职业教育了。结果就是出现一部分精英和大部分没有发展潜力的低端产业工人。成绩只是选拔的一个选项,它们更看重的是发展潜力,毕竟高中成绩在大学教育里是很基础的。比例也许是 10 个顶尖精英,990 个普通产业工人

中国更多的是量化,也就是靠成绩来选拔。 毕竟创造力和发展潜力这方面的选拔太容易滋生腐败。而成绩选拔在中国的现阶段,更接近公平。最尖端的那部分可以进清北接受最优秀的教育资源和培养。 其他大部分并没有堵死上升的通道,只是相对比较难,这就在中国学生里形成梯队,最顶尖的那部分接受了最好的教育,(但相当部分在接受教育后去了欧美,这是大形势造成的,不能全算到政策头上) 而其他部分接受的是高于美国普通大学的教育的。所以从低端到工程师就形成了中国的庞大的产业群体。 比例也许是 5 个顶尖精英,995 个涵盖了从工程师到低端产业工人。

俩者看,美国的精英倾斜的教育资源是远超过中国的。其教育产出也是比中国要高的,同时中国的优秀学生滞留美国,也增加了美国的科研潜力,这就更容易在尖端技术上形成突破。 而中国的教育相对于美国来说,拥有更多的工程师,产业工人素质普遍强于美国。

美国的移民国家的形成,全世界优秀人才的集中效应,让美国的创造力维持热度。但并不是绝对的领先的。而中国作为追赶者,说句实话,走才刚利索,跑的话慢慢来吧。我们的优势在于一个创意产生后我们的实现能力是远大于美国的。 未来,在新技术领域,中国迟早会追赶上的。

知乎用户 C Vincent 发表

  • 我们的教育是不是真的高大上呢?我中学是在我那个省最好的中学,我的中学有个习惯,上课不用课本,因为仅仅靠课本上那点习题,那点讲解的知识,恐怕是远远不够应付高考的,甚至能过会考都不容易。然而我那个中学当年一届只招收 600 个学生,国内大多数,不是重点中学的学生,学的还真就是课本上的那点知识。知乎回答问题的人,大多来自于一二线城市,自小学习也不错,回忆一下你中学有多少时间花在课本上,多少时间花在《五年高考,三年模拟》上? 为什么现在很多人来了美国以后觉得美国也不过如此?因为他们在国内的起点就很高,来了美国以后接触了美国屌丝,于是友邦惊诧;
  • 我们中学真花费了很多时间在有意义的学习上吗?回忆初中高中,初中三年,前两年抓紧时间把课上完,初三一年应付高考。高中三年,前两年把课上完,高三复习应付高考。实际学知识的时间,只有四年而已!12-18 岁,是人学习能力最强的时期,那时候的学习效率要比 25 岁以后高 2,3 倍都不止。然而就是在学习能力最强的时期,我们大量时间花费在做题,解题技巧上。真正学了多少东西?
  • 我承认应试教育是现阶段相对合理的人才选拔制度。至少以国外学校那种参考高中各科会考成绩来挑选学生,对于中国是极度不靠谱的。中学毕业的时候,同学申请国外的本科,申请材料里提供的会考成绩,不过有的科目是别人代考的。
  • 这些年在国外看到,国外的小孩,进入大学以后,学习的知识层次,深度,花费的努力程度,丝毫不亚于中国小孩。但是他们还是普遍承认中国人聪明。我想是为什么呢?早几十年,留学出国的基本都是学霸,精英。千军万马过独木桥以后的佼佼者,来到国外自然秒杀那些学渣。而如今这几年,出国留学早就不是稀罕事,留学生质量快速下降。而西方的教育产业化,使得经济(尤其是澳大利亚,加拿大两国)有极大比重依赖于中国留学生。我根本看不出来现在部分自费留学的留学生比外国学生优秀在哪里?哪门课好过,水,基本全课堂都是中国学生。有考试作弊被抓,有抽大麻。而面对留学产业这颗摇钱树,国外学校的录取难度,毕业难度,考试难度,都是逐年下降。
  • 所谓美国中小学生学习差,只是我们的错觉,就好象十年前《读者》上的鸡汤一样。早些年一群在国内没见过学渣的学霸出国以后,给歪果仁留下了深深的心理阴影,自己也见识了学渣,于是就产生了这样的谣言。
  • 中学教育本来就不该,也不可能高大上。如果按照知乎名校诸位的水准来定义中学会考。恐怕绝大部分高中生都拿不到文凭。别忘了,这个社会上,考不上大学才是主力,能上个大专已经是佼佼者了。大部分人以后的数学知识确实算个帐就足够了,英语也用不到。任何成熟的教育体系,在中学阶段都会给有升学需要的学生提供强化教育。重点中学,重点班,实验班,各种课外辅导材料,各种课外补习班,高考移民。谁敢保证靠着教育部提供的那套薄薄的中学课本,就能考上个普通本科?

知乎用户 何枫艺​ 发表

其实这是一种近乎以偏概全的说法,美国人高中的数学难度与我国初中生相当,我国初中学的都是基础得不能再基础的东西,那时的数学难度范围非常有限,甚至我国高中的数学比如三角函数,立体几何,导数都是入门级别的东西(当然是说教材里的内容),而真的尖端数学领域不是靠普通数学教育能够培养得出来人才的,所以那些高层次的领域看的并不是这个国家平均数学水平的高低,而是看这个国家最尖端的数学人才的水平,个人愚见

知乎用户 natasllik 发表

其实现实是很复杂的,产生的结果却有高有低。

美国的基础教育现在正在陷入一种潜在的危机,主要有几大原因:

1. 美国小学,中学,高中的反智倾向;

2. 美国基础教育投入有限且两极分化;

3. 基础教育的老师素质越来越参差不齐,有可能是由于妇女解放的原因(妇女解放前,很多聪明有教养的妇女会选择基础教育;而妇女解放后,同样资质的妇女会参与与男性竞争的各种高薪行业)。在某些地区,老师不恪尽职守,甚至会醉倒在课上。

U.S. Students Slide In Global Ranking On Math, Reading, Science : The Two-Way : NPR

4. 美国的文化对老师的尊重非常有限,尤其是基础教育的老师,这与教师水平越来越差是相互促进的。老师的社会地位不高一方面阻碍了老师行使教育的某些权力,另一方面阻碍了优秀人才进入这个行业。

虽然美国的高等教育是世界领先的,但是越来越普遍的现象是来自广大发展中国家(如中国,印度)或其他中等发达国家(如韩国,某些中东欧国家)的人占据着新招募的科研人才的大部分,美国本土的学生则越来越不愿意从事科研,特别是需要很多数理运算的科研。

美国本土的学生可谓一代不如一代,我有朋友在小学教书,反映说美国小孩非常不爱学习,家长一方面想让孩子更有竞争力,但却在另一方面不愿意让孩子受苦,因此会更加折磨老师。美国新一代的孩子不仅数学能力不强,而且读写能力也越来越差。

基础教育的潜在危机还没有大幅度的表现出来,然而在不久的将来可能会越发变得明显。

我们之所以认为美国人数学上很有成就,其实可能与我们所参照的年代有关。而且,美国是个移民国家,之前有德国犹太科学家和苏联科学家注入的科学动力,现在又有源源不断的广大发展中国家人才移到美国,所以短期内美国科学的优势还是存在的。但是,基础教育的烂摊子是没法从根本上拯救美国的,只靠每年为数不多的科技移民是解决不了美国最根本的教育问题的。而且,一旦移民导致国家政治结构发生变化,未来又会增加很多的不确定性。

总之,没有一个超级大国是永远强盛的,此消彼长乃世界和宇宙之规律,希望中国越来越强盛吧。

知乎用户 雪碧踢墙 发表

我在美国高中任教 22 年了。 没有觉得美国的数学简单。说简单的人一定是没有能力修高级数学的学生。 美国和中国不同,每个学生修课不是相同的。学生可以选很前卫的大学课程,也可以拿比较低级的课程。学生根据自己的兴趣和能力选课。如果一个学生要上名牌大学,显然低级的课程是不够的。如果一个学生不想学理科,那么他 / 她可以修低一点的数学。所以,一味地说美国数学简单是不全面的评价。学校每年有很多国内来的学生,英文不说,数学也很少名列前茅的。有一位上海育才中学来的,也只能修中级的数学。美国的高中 AP 教学( 提高班)接近一个国内的两本大学。

知乎用户 匿名用户 发表

美国学生学的数学比我们的简单?

那为什么我手里的几本经典微积分,线代教材都是美国人写的?

知乎用户 JCrossover 发表

以下是丘成桐先生对中、美教育发表的部分演讲:

比如,很多人认为中国学生的数理化成绩要比同龄的美国孩子好,中国学生基础知识要扎实得多,只是创新能力差一些。而丘成桐对这种观点毫不客气地泼冷水:“这都是多少年来可怕的自我麻醉!我不认为中国学生的基础知识学得有多好!”

他说:“美国最好的学生真是好得不得了。应该这样比较,不管是美国,还是中国,能进哈佛大学的学生都应该是这两个国家最好的学生。而两类最优秀的人相比,美国学生的基础知识绝对不会逊色于中国学生,相反是要强很多”

教育界和社会上还有这种说法,认为中国的中小学生要比美国的学生数理化知识学得多,比如,在某个年龄段,中国孩子加、减、乘、除的混合运算已经学得滚瓜烂熟,但美国孩子加减法还做得磕磕绊绊。

丘认为:“这也是错的,在美国比较好的中小学校里,中国学生念的功课,他们也都是要学的,而且学得很灵活,绝对不是像中国那样填鸭式地教。一些好的学校,十一、十二年级学生的微积分已经做得非常漂亮,但听说国内不是所有的高中生都学微积分。”

丘成桐还在一些演讲里反复强调:对于中小学生来说,语言、数学、写作是三门最重要的功课。

他说:

西方的教育,从小学开始就训练小孩子的表达能力,无论语言和文字的技巧都得到良好的训练。一般来讲,受过这种训练的孩子都能够毫无困难地在集会中表达自己的想法和科研的成果,因此他们在课堂上能够自由发挥自己的意见而得到老师跟同学的重视。我们常常讲,中国的学生为什么到了美国念研究院,讨论的时候比不上国外的学生,我想这是他们从小训练出来的一个结果。

除了语言以外,推理是西方教育很重要的一环,因此数学是中学和大学最受重视的一门学科。欧氏几何定理不见得对社会有直接贡献,可是它的推理方式却是最有效的逻辑训练。以前,美国主要的大学非常看重两门学科,一个是语言,一个是数学。语言和数学不能够得到高分的话,他们基本上不会考虑接受你做他的大学生。最近还加了一个写作的能力,三门,语言、数学和写作,这三点是美国所有名校最重视的训练。

很多美国中小学还加上基础的法律训练。懂得法律和遵守法律是现代国民应有的知识和操守。

丘成桐还认为,美国小孩的用功并不比中国学生差:

一般来讲,美国中小学鼓励学生交流。初中二年级以前,美国的中小学都比较鼓励小孩子发挥所长,让他尽量去博物馆走走,去运动场上玩玩。我们中国有的教育家因此以为,美国的中小学生不行,比不上中国。事实上,到了初三、高一以后,美国小孩的用功并不比中国的学生差。他们不用考试来训练学生,也很注重他们的基本能力。我们看到很多好的美国小孩,他们到了高中一年级或二年级才开始发挥热情,拼命去念书。到了大学以后,他们不会觉得学问是枯燥的。美国的名校我去过好几个,在念理科重要的学科的时候,他们很用功,花的功夫绝对不会少,往往是念书念到两三点钟才睡觉。而中国很多小孩经过小学和中学沉重的考试冲击以后,丧失了追求学问的兴趣和热情,这是很可惜的。

知乎用户 Erinwong 发表

我觉得是这样。你题目说的教育,是障眼法

首先,你错怪了美国的教育。美国的教育分精英,中产和穷人教育。精英教育和中产对孩子的要求,是跟中国一样一样的。穷人的教育才是你题中的概念。

其次,你要知道教育对一个国家的意义: human capital 的积累。劳动市场吃不下那么多人,就找个地方安放一下。什么自我成长与发现啊,这些大部分是你家庭关注的事。所以,在本意相同的情况下,中美教育之对比,不应该拿最后人才的贡献做比较,讲真,国家并没想你那么优秀的。你那么优秀,是家庭的因素,和你自己本身基因带来的性格因素。

第三,美国的人才把身份亮出来,多少是中国人,欧洲人,印度人。对吧,这就很清楚了嘛。

第四,一个国家人才的多寡是该国教育体制内容的问题,还是该国经济的问题?我认为是经济的问题,那么显而易见,如果把 19 世纪之后的中美互换,而教育体制不换,人才之战还是否呈现现在的局面? 不好说。

所以,我认为一个国家的人才并非从教育体制出来,经济环境和实力决定这个国家的人才。当然很多人说鸡生蛋蛋生鸡嘛。我的意思是,在没有严重错误产生的教育体制下,如果中国的教育是根本不重视教育,那当然不能比。现在是两种教育体制都能出人才,谁更好,根本比不出来。

我的观点 1. 题目是指着地主家的猪圈问: 猪圈那么脏,但为什么地主的别墅那么豪? 不搭噶的事。

2. 美国宽松教育和中国填鸭式教育不是形成两国人才差距的原因。

3. 美国式教育如同美国式民主一样,很多国家都在用,然而。。。你们看到碾压中国了吗?

4. 美国式教育可能教出有创造性的人才,但更多的是成了吃喝玩乐中二时期过成了一辈子的傻二楞。

大部分老师自己都活得不明白,照本宣科都懒得教,素质教育? 就是不管,任你玩呗,玩到 16 岁,可以打工了,去吧,交税养国家。

在加拿大 15 年,深受小孩 3 点就放学,一天屁都学不到,回家还得自己教之害的人的一点想法。

知乎用户 寰太逻辑学家 发表

这个问题的真相稍微有点复杂,我给大家解释一下,美国中小学的数学确实比中国的要简单的多,但是呢,由于中小学数学对普罗大众的日常生活和工作实际上没有太大的用处,所以,虽然中国的中小学数学教育让中国的普罗大众有了比美国人高的数学水平,但由于没用,中国的普罗大众并没有受益。举个例子,你高中学微积分 以后从来就不会用到,根本没有用。

另一方面,虽然中国的中小学数学教育非常先进,但是这些知识实际上是落伍的知识,大概是一二百年前人类社会数学和理科的知识总结,而真正人类社会目前尖端的理科知识,工科知识,都掌握在私人公司或者研究所里面,不可能从课本上学的,比如喷气发动机的设计。这些尖端技术大部分都掌握在西方组织手里。

所以说呢,中国的中小学教育只是让你学会了落伍的数学理科知识,而先进的尖端知识只能从工作以后去学,比如说芯片制造,高超音速空气动力学等等。这些知识大部分掌握在西方公司或者研究所手里面。西方国家只需要从人群中选出 5% 的理科尖子,进一步学习这些顶尖技术就够用了。西方普罗大众的数学水平虽然差,但是还是能挑出 5% 的理科尖子对这些技术进行掌握的。

而中国的情况是,普罗大众学了一二百年前落伍的数学以及理科知识,但是对日常生活没用! 而中国工程师和研究人员想要掌握人类社会最先进的技术,但中国的公司以及研究所没有这种技术,所以总的来说就是,高精尖的技术没有,低端落伍技术人人都会,但是又没有用。这就好比一把刀子的刀刃不够锋利,但是刀背特别厚,但是没用。

所以说题主实际上把落伍的理科知识当成了有用的理科知识。实际上据我观察发现,其他穷国如印度菲律宾的学生的数学水平都挺高的,但这并没有帮他们成功进行工业化,所以说这些落伍的理科知识啊,并没有太大用处。还有一点,这也类似于把编程技术当成了高科技,实际上编程就是一种落伍的,普罗大众都可以学的理科技术,并不是高科技,很多农村青年都可以在家里面拿本书一看就学会了编程,包括印度和菲律宾农村青年。所以说编程技术往往是农村青年在搞,你说农村青年会搞啥高科技呢?

知乎用户 扫地焚香​ 发表

私以为,有两个问题存在。

第一,国内的教育不肯承认个体的差异。凡事喜欢搞平均,反对特殊化。反应在教育难度上便是 “满足大部分人的教育需求”,也就是说,你说简单吧,没有天赋的学生学不下去。你说难吧,聪明的学生早就在课堂上睡着了。

面对这种情况,国内教育对差生说的话是:“加油,再克服一下难度,跟上来。”

对优等生说的话是:“你上课干什么我不管,别影响其他同学就行了。”

这就暴露了我们教育中平均主义的坑爹之处:能推动知识进步的人在课堂上打瞌睡,而普通人却在课堂上经历着 “这是什么,这又是什么” 的困境。

第二:大学,高中教育本质本末倒置,大学教育水平极其低下。

大学以前的教育其实都属于通识教育,是不分兴趣,不分天赋,不分男女的基础教育。国内对基础教育变态一般的关注其实并不是因为大家关心基础教育对孩子的好处,而只是关心怎么样考一个好大学。

于是,高中地狱一般的求学经历与大学轻松,自由的学习环境的强烈对比,就会让孩子一进入大学便自由散漫,丧失目标。

再加上,国内大学教育水平实在是过于低下,哪怕是重点大学,尸位素餐的老师也不在少数,更别提二,三本大学了。高中老师尚且知道如何激发和维持同学们的学习兴趣,而大部分大学老师真的就是 ppt 一开,化身朗读机器人。

手机打的,先回答这么多吧~大家想看的话晚些我再更新一部分

知乎用户 悠然听风​ 发表

曾经看过一个视频访谈类节目。

美国着重的并不是中小学生的教育,而是他的大学教育。

所以美国可以用非常优秀的大学教育,来利用英语优势,在全球收割教育类的人才。

举几个可能不恰当的例子好了。

创立了 Facebook 的扎克伯格,其实母国是南非的。

而硅谷里面的精英,很大一部分都不是美国本土出生的,只是在美国读完了大学教育之后,留在美国工作而已。

至于医护和军事领域,也有许多并不出生于美国的外籍人士,在美国工作。

所以还是重点去看看美国的大学教育吧,看看他们的教育理念和我们有什么样的不同。

知乎用户 李越阡 发表

-——Update 08/18/2015——–

已关闭评论 对知乎的下限再次刷新 我知道你们是不肯就事论事呀 那评论我收起来了 中间有几位在美国学教育或者有教学经验的朋友说的都挺好的 兼听则明 目的达到了 可以了

P.S.

对啊 我写英文就是在秀优越呀 快来咬我呀~

-——Update 7/15/2015———-

有些人在评论里面对行文中带有英文冷嘲热讽甚至谩骂,本人实在不堪其扰,完全不值一驳。详见

为什么有人讨厌别人说话时夹杂英文? - 裝腔

评论区希望只有相关讨论的留言,不喜欢的可以点反对和没有帮助,过来秀智商的狭隘喷子一律屏蔽删除。

我只想对那些认为我在秀优越的盆友们说一句话,你们啊,naive!

-————————————-

不谢邀答。

作为一个完整经历过中国正统应试教育后,大学本科继续选择数学为专业,且有一段美国大学数学教学经验的人,我相信我的见闻能解释大部分人关于中美数学教学相异的困惑。

一言以蔽之,美国的数学教育,尤其是基础教育是非常薄弱的。 但从分布来看美国大学中数学水平相较中国,样本均值偏小,样本方差偏大,极差比中国的学霸学渣更可怕。拿均值去代表生产力,在一些行业可能会被奉为圭臬,但在 innovation 和 independent research 方面就会与事实大相径庭,毕竟科研和创新甚至是人类的历史都是靠最聪明的那一小撮人推动的,尖点越细,越容易有所突破,rest assured.

这是事实,我在亲身经历之前,也会觉得惊讶。去年的春季学期,我在学校的数学系做 TA, 主要内容是 Advanced Calculus 和 Intro to Differential Equations. 我校是一所以高昂学费,中产汇聚,以理工科为主的私立大学,历史比较悠久马上要 Bicentennial Anniversary 了。 Prof. K, 一个从她的名字就能知道是东欧裔移民的女教授是我的大老板,秉承着泛毛子变态数学家的 Stereotype,确实是一个经验丰富的 Scholar,她是一个 Teaching School 升上来的,教学已经十多年了,人美且十分 nice,在我还在被这群熊孩子吓的不轻的时候,她平静的告诉我她已经身经百战见得多了,“Well, I have seen it all before.” 此为前情提要。

拿到选课名单的时候我决定得好好备课,人数居然达到了 98 人,因为理工院校的原因,数学课几乎是所有院系的通识课,数学教学任务也十分的繁重,所以名单里的专业多是 Engineering, Natural Scinence 以及少量的 Business School 和 Information System 的学生,这些学生基本是高中里面的佼佼者,录取者基本有 A - 以上,且有良好的数学成绩。我潜意识里面认为都是在高中修过大量数学 Pre-Calculus 的 geek 们,但事实上我错了。稍后我会详述,首先介绍教材,用的是经典教材 Pearson 出版社出版的 Thomas’s Calculus. 作者是 “享誉” 教材界的 MIT 已故教授,George B. Thomas.

这本书的第一感觉是细致,严谨,深入浅出,写的非常的友好平和,非常容易上手,比起国内用的高数和数分,几乎可以说是非常易读的一本。Detail-Oriented,习题丰富有趣,如果认真的读完,微积分这一块会打下非常坚实的基础。 但是,这本书过于细致的追求培养直觉和逻辑,事无巨细的推导和引例降低了使用这本书为教材的效率,而且对于直觉的过于强调缺乏对数学工具性的注重,也就是当人类直觉失灵的时候,数学是如何 dominate 的。另外,这本又厚又重 200 多刀的怪物让我都觉得讲课的时候难以携带,以至于我将其肢解为几部分……

展开来讲,我随手挖了几个坑,有时间的时候填,算是对这个难忘经历的一个纪念吧。在此之前,先放几张有意思的图来骗赞吧 LOL

Quiz Exam 奇多无比,这就是典型的改卷的日常。

  • 教学模式

总的来讲,这里对数学的教学还是非常重视的,因为是理工学校的原因,这是他们今后四年的基础和底子,对于 Natural Science 的学生来讲,属于给 Graduate School 打好底子,因为没有好的数学成绩,无法有足够的 credentials 和 independent research 的能力。 对于 Engineering 的学生更不用说,属于学好吃饭的手艺的保证。学校为了保证就业率和一贯以来工程界的好口碑,在数学教学中一贯非常的严谨,强调 rigorousness.

一周双课(约六个学时)撇去不表,每周一次的小测 Quiz Block(频率有点让我想起高中的一段一测),一月一次的 Exam(比 Quiz 考的题量大,更全面,难度更大,书上例题和课后习题的变式加强版),还有 Midterm, Final Exam(两小时完成,渐进难度,最后几题还比较 tricky), 还有一周两次的 Recitation Session(我负责的 session,每次 1 小时,主要是 recap 本周所学内容串讲,答疑讨论,并给大学霸们回答超前的问题),最后就是 Optional 的 office hour 了,每周两小时,我在学院楼等待同学过来问问题或者任何想讨论的话题。 学生们必须拿出十分的精力和时间来学不然就会在任意的一个 Quiz Block 或者 Exam 或者期中期末考试中拿低分,导致最终成绩不太好看。如此精细的设计让我想起了中国高中时期的应试教育,强度之大,阵线之长,考核之细,让我也觉得学生很累,负担很重,难以想象这门四学分的课要占用如此多的时间,非常的 intensive。相比之下,我当年学数分三学期,老师基本是放养状态,讲完课后就属于神隐状态,多数还要靠自己自学…… 中国大学的老师,绝大多数,不重视本科生教学,咱学费是交的少,但得到的 attention 也是一分钱一分货,谁让职称系统如此操蛋呢?

由此看出来,就这门课来讲,如果你认真学习,美国大学的学费还是出的很值的,我明显感觉到很多一开始基本功差(尤其是 Algebra 部分)的同学在经过最少半学期的学习后,成绩有明显的上升,由最后得出的数据分析出,期末考试相比期中考试,中位得分和均分都有约 20-30% 的提升,当然学霸依旧全满分,学渣依旧 F 的残酷真实放之四海皆准啊。

  • Prof. K 谈美中学生差异 (TBC)

对于在与同学们的接触中发现的数学基础的期望落差,我一开始是打算心里憋着没跟谁提的,毕竟我不清楚这边老师的要求和心中的想法,冒失的吐槽多半会被当做没礼貌的傲慢,所以我在和老师聊天和邮件的过程中也就吐槽下一些 typo 啊,handwriting 啊什么的。 直到有一天 Exam 测试,一位可爱的白人妹子坚持到了时间到了才交卷,然后站着开始对我啜泣并对自己交了白卷表示抱歉,真实让人顿生怜爱啊,我从来没有经历过这种阵仗╮(╯_╰)╭,所以尴尬的很,只能上前特别安慰,鼓励其在 office hour 过来找我答疑…… 经过这个之后,我找老板谈了几次关于美国数学的基础教学,表示了我的一些疑问和看法。

我是这样写的邮件

“Frankly speaking, I think the results are bad, most of the students cannot get a relatively high score. I asked myself if I am too strict with the grading rules but I am not actually, most of the students cannot even start with several hard questions, also they all had a bad time with the algebra part, not the integration technique.

I think of this just because a girl who stayed until the last minute of the exam this afternoon handled me her answer sheet and at the same time she started to weep , asking me whether the exam is focused on improper integral. She could not work out a single problem so I just told her not to worry and I can help whenever problems arise. ”

Prof.K 的回复

“I have to say that what you are feeling is normal. The students in the US can be very weak in math, and it is my impression that they have been getting progressively worse since I started teaching in 2001. Their lack of algebra skills can be shocking, indeed. It is likely that some of the students will be upset with their grades on the exam. In many cases, it serves as a message to the students that they need to start working harder.”

她的回复中值得注意的是也提到了数学教学的改革

Math wars - Wikiwand

,使得数学成绩一蹶不振,一年不如一年。同时她也表示,大多数还是不努力,因为学霸们不管多难还是考的高……

  • 学生质量与轶事

98 个学生都是 freshman, 水平参差不齐,下面说一点 totally biased 的观点。为了追求政治正确不被喷子恶心到,观点不构成群体性的结论,请自动 assume 我的样本全是片面的。

  1. 中国学生大概百分之 8-9% 左右,基本是美高或者国内重点中学过来的本科生。在班上属于最好的那一批,基本最后全拿 A 了,不得不说,国内的数学基础教育(初中或高中)的确好过这边同 percentile 同学太多。
  2. 聪明的特别聪明,捉急的特别捉急。 聪明的 WASP 话都特别少,学霸范儿中美都差不多的,简单听听课,从不提问,但基本拿满分,气场强大,目中无人的很普遍。
  3. 第三世界的贵族们。。。我学生当中有个非洲小国来的,基础较差,但很勤奋,每次上 recitation 都会来,然后也经常来办公室问问题,所以对他印象比较深。一日闲来无事,我搜了下他的 facebook,竟然发现他在本国军舰上出席活动的照片。。。其父是本国海军将领。。。
  4. 拉美同学们比较吊车尾,基本不听课也不来上课,招进来的很多都是 token 吧。。。参见 South Park 里面萌萌哒 Token。。。

先写这么多,有机会再根据评论写点别的。

知乎用户 李建 发表

因为美国是精英教育,只有垃圾的公立学校里面学的才是简单数学,这帮人最后的职业都是售货员清洁工建筑工毒贩子妓女之类的。通常我们所以为的美国的数学就是这种!而私立学校里面的课程,比我们的强度深度都大得多!这些人出来后才是决定美国未来的力量!

知乎用户 盛京剑客 发表

肯定都是在华人帮助下完成的,要不怎么可能呢?

知乎用户 王咏刚​​ 发表

来看一门真实的美国高中数学课(不是必修课)。课程的课件、源码、工具、参考资料全公开。请自行体会美国中学数学针对兴趣学生的课程到底有多简单或多难。

课程名称

最优化或数学规划(Mathematical Optimization)

课程背景

这门课程最早是由库比蒂诺高中(Cupertino High School)的数学老师 Julia Roberts(大明星茱莉娅 · 罗伯茨?Julia 编程语言?似乎无关)和斯坦福大学航空航天教授 Mykel Kochenderfer 在 2014 年联合设计的。

课程目标

  • 增加学生与应用数学领域前沿问题的接触机会,包括最优化(Optimization)方法以及相关的编程技能;
  • 增加对 STEM(科学、技术、工程、数学)感兴趣的学生数量与类型;
  • 为学生未来的 STEM 学习打下更好的基础。

直观感受

详细展开课程大纲前,看几张课件截图,大家直观感受一下这门课的难度:

课程结构

课程一共有五个单元。前三个单元只要求学生代数基础扎实,且拥有安装了 Julia 编程语言的电脑。最后两个单元需要 Calculus AB 水平的微积分知识。整个课程的教学时长估计在 23 周左右。

(按:有关 Calculus AB 水平,一位中学时就是美国学霸的朋友说,“Calculus AB 是屌丝 Calculus,我们学霸当年都瞧不起的,正经的都学 AP Calculus BC”)

课程大纲

(按:翻译仓促,外加一些专业术语在中文世界不统一,以下部分错漏难免)

第一单元:导论

  • 1.1 最优化:关于课程的概述
  • 1.2 矢量:定义和应用
  • 1.3 迭代和递归 1:定义和例子,包括斐波那契数列程序
  • 1.4 迭代和递归 2:割线法(Secant method)
  • 1.5 迭代和递归 3:寻找包含最大值 / 最小值的三点区间
  • 1.6 Julia 编程语言初步:如何获取,基本运算
  • 1.7 Julia 的 If-Else 结构,测试条件
  • 1.8 循环结构:Julia 的 for 循环和 while 循环
  • 1.9 数组:定义,命令和运算
  • 1.10 割线法:学生编写割线法求解方程的程序

第二单元:无界(Unbounded)情况下的最优化,无微积分基础

  • 2.1 导论:定义,局部最优值和全局最优值
  • 2.2 三点区间:学生编写寻找三点区间的程序
  • 2.3 局部最小值,暴力搜索法(Brute Force):学生编写暴力搜索法寻找局部最小值的程序
  • 2.4 局部最小值,黄金分割搜索法(Golden Section intervals):学生编写黄金分割搜索法寻找局部最小值的程序
  • 2.5 局部最小值,斜率法:学生编写斜率法寻找局部最小值的程序
  • 2.6 最大化和最小化:学生把寻找最小值的程序修改为寻找最大值的程序。
  • 2.7 全局最小值,测试点法:通过在一个区间上的测试点寻找全局最小值
  • 2.8 全局最大值,锯齿法(Shubert-Piyavskii 方法):函数曲线的斜率,寻找全局最大值的锯齿法
  • 2.9 三维空间的最优化问题概论:两个自变量的函数,三维作图
  • 2.10 三维曲面的最小值,暴力搜索法:两个自变量时的基本最小化技术
  • 2.11 Hooke-Jeeves 模式搜索法
  • 2.12 循环坐标(Cyclic Coordinates):截面,循环坐标搜索法(按:请参考坐标下降法 Coordinate Descent)
  • 2.13 扩展:最大化,四维空间中的三自变量问题
  • 2.14 随机方法 1:定义,收敛,蒙特卡洛方法
  • 2.15 随机方法 2:模拟退火算法,遗传算法

第三单元:线性规划

  • 3.1 约束最优化(Constrained Optimization)概论:约束(Constraints),可行域(Feasible Region),角点(Corner Points)及图示
  • 3.2 矩阵运算:基本运算,矩阵的逆
  • 3.3 高思消元法(Gaussian Elimination,Row Reduction):Julia 中的数组命令,使用高思消元法
  • 3.4 线性规划导论:单纯形法(Simplex Method)
  • 3.5 单纯形法:转轴操作(Pivoting)
  • 3.6 单纯形法:选择顶点(Pivot)
  • 3.7 两个完整问题:柠檬汁问题和农场问题
  • 3.8 通过构建对偶(Duality)来解决最小化问题:构建对偶矩阵
  • 3.9 非标准约束下的最优化问题

第四单元:无界情况下的最优化,有微积分基础

  • 4.1 基于微积分的最优化:概论,数值微分(Numerical Derivatives)
  • 4.2 辛普森积分法(Simpson’s Rule)
  • 4.3 牛顿法(Newton’s Method)求根
  • 4.4 Julia 编程语言的包,锯齿法
  • 4.5 基于微积分的函数作图(Curve Sketching):函数作图中的一阶和二阶导数
  • 4.6 用函数的凹凸性(Concavity)验证最大值 / 最小值
  • 4.7 梯度(Gradients):两个或多个自变量时的微积分,梯度的概念,寻找梯度
  • 4.8 梯度的应用:梯度的计算,梯度的含义
  • 4.9 两个或多个自变量时的二阶导数:黑塞矩阵(Hessian matrix)
  • 4.10 黑塞矩阵的应用:行列式,特征值,凹凸性
  • 4.11 基于梯度的最优化 1:最陡下降法(Steepest Descent),使用 Julia 的微积分包
  • 4.12 基于梯度的最优化 2:共轭梯度法(Conjugate Gradient)
  • 4.13 扩展:最大化,全局最大值 / 最小值,四维或更高维度

第五单元:约束最优化,有微积分基础

  • 5.1 导论:背景,为两自变量问题构建和绘制可行域
  • 5.2 非线性约束最小化:基础,下降方向与可行方向锥(Cone)
  • 5.3 非线性约束最小化,阶段 1:使用最陡下降法
  • 5.4 非线性约束最小化,阶段 2:矢量归一化(Normalization)和角部区域(Corner Regions)
  • 5.5 非线性约束最小化的完整过程
  • 5.6 惩罚函数法(Penalty Functions):利用平方损失函数(Quadratic Loss Functions)求解最优化问题
  • 5.7 内点惩罚函数法(Interior Penalty Functions):利用内惩罚函数(Barrier Functions)求解最优化问题
  • 5.8 帕雷托最优(Pareto Optimality):帕累托集(Pareto front),帕累托改进(Pareto improvements)
  • 5.9 马尔可夫决策过程(Markov Decision Process,MDPs):马尔可夫性质(Markov Property),马尔可夫决策过程,部分可观察马尔可夫决策过程(Partially Observable Markov Decision Process,POMDPs)

课程链接

课程全部详细资料都在下面的链接里:

Mathematical Optimization

知乎用户 德华 发表

我们学的是应试用的数学,他们学的是能应用的数学。

我们认为他们的数学简单是因为他们多数人用不到那么难的数学,用应付日常生活算算账即可,所以教得简单。而真正需要用到数学的数学专业或其它物理、经济学专业的人学的数学那是真难,他们是真的懂每个概念背后的原理和公式推导。

美国我不了解,但我去过德国,我大概知道点情况。我在德国读博士期间有一个奥地利小伙,算我师弟了,他说他数学好,我一开始还不相信,能有我好吗?我高数可是考过满分了!但我当时已经把学过的公式定理概念全忘记了,看到文献里有公式也赶紧略过。但这哥们就牛了,只做了一个实验,就开始各种用公式拟合推导,写出了一篇论文…… 这对我来说太不可思议了。而我当时在德国读博士的 group leader 就更牛了,他是研究软物质的,竟然在他年轻的时候写过一本全是公式的书,全是他自己一个人推导的,一举奠定他的行业大神地位,我想在国内还没有任何人可能做得到吧。我们的搞软物质或高分子人一般都在加点热点材料、测测性能就发表,最多偶尔利用别人的公式套一下,再解释一番。

任重而道远!

知乎用户 zarchitect 发表

楼上的答案,大多讲的是美国对尖子生实行精英教育。

不过比较公众教育,中国的中学数学难度确实高于美国。

但是美国人认为: 大多数人过了 18 岁的时候,大脑发育会有一个飞越性的进步,逻辑思维能力会大幅度上升,所以他们认为高难度的数学,适合在大学进行。

于是,我们在初中高中六年时间的 “高难度” 数学课程,人家安排到大学,一个学期就能补上了。

微积分课程修完的同学可以估量一下,我们在中学学了 6 年的数学内容,是不是加起来也就是一个学期的微积分课程?

所以,我认为美国的数学课程安排更合理,我们国家的教育改革确实应该借鉴一下。以前大学 5% 的录取率,只能让少数的精英进入大学, 把中学的数学安排的很难,更多的是为了提高大学的入学门槛。现在大学的录取率超过 70%,可以考虑一下降低中学的课程难度了。

================

转一段漏屋老师的文章:

美国 SAT 上那数学题让咱初中小孩做,全满分,这些小老外那叫觉得难。你说他 们不怕耽误下一代?不怕,人家明白着呢。

   看咱中国同学,恨不得幼儿园时就急着学小学课程,小学生可怜啊,早起天还没亮,下学作业写到天黑,周末还有课外学习。中学生更可怜啊,为了考上大学了,那 苦就不说了。高考,真烤啊。“啊,我中了!” 考上大学了,这回可放鸭子了。敞开玩儿吧。

再看外国同学们,上了大学了,功课突然紧了,每天学习跟打仗式的, 比中国一般大学学生的学习强度高几倍。一门高数课我们要学半年,国外恨不得几星期就学完还考试。

能行吗?当然行!!因为人到了这个年龄,记忆力,理解力都开 始快到高峰,一个月学习的收获比中学小学一年都多。(其实中国大学生也能作到,只是大多一学期都在玩儿,到最后两周一突击,好几门都过,因为能力一样嘛。 只是在国外上大学,每星期都是中国那最后两星期)。结果本科时,人家把以前中学小学被咱拉下的早都补上了,到研究生阶段一下就把咱盖过去了。到这阶段,要做研究项目,需要创造力了。结果外国同学小时候开发的智力全用上了,中国同学到此时才思枯竭,小组项目中只有给人家找资料的份,人家还经常不爱带你玩 儿。(很久以前考出去的中国留学生中有很多到了国外还是比较优秀的,但那多是人精,不能拿我们的人精跟一般外国人比。大家同级别比一下)。不说这事了。改 变教育体制是要靠大家努力的。

知乎用户 Fred Zhang​ 发表

不要再说美国高中生数学是我们初中水平什么的了

美国普遍数学基础教育比不上中国是事实。

但是!注意这个” 但是 “!

理论数学的制高点是天才们才能触及的。所以对于天才们来说,基础教育可有可无。

说说美国的天才吧。。。

高中一般在美国都是自由选课,所以虽然一般学生都是选比较容易的,

但是好学生自然都会选难度比较大的

比如 AP 或者 IB

我在 11 年级的时上的是 AP 微积分和一些 12 年级的美国人一起。大学高数的东西,国内高中应该是不学的,更不可能给高二的吧。。。

我们年级一个华裔小哥 11 年级上的也是 AP 微积分,但是我们上的是 AB,他上的 BC,进度比我们快很多。这小哥刚刚录了普林斯顿。。。简直就是大神

然后 AP Cal AB 课上的美国学生也有很多很努力很厉害的,所以并不要以为美国学生数学都很烂

还有就是美国是移民国家,所以数学好的外国人移居美国,也提升了水平

知乎用户 江先生​ 发表

如果你完全不知道一个数学公式,你如何把他推理出来,这其实是我们数学的困境,我们学了太多公式,但我们忘了最重要的东西,这些东西是怎么来的,我们会应用公式,却无法对这些公式有深刻理解,因为从一开始他们就作为公式教给你了,而不是一个思路,所以越到后面背得越多,我们知道解答数学题,却无法理解这些东西,古人常说授人以鱼不如授人以渔,我们每个从小都被教着吃鱼,却没人教我们制作鱼竿找鱼饵钓鱼

知乎用户 山鸡村的 X 先生 发表

能做题是一件事,但是科学素养又是另外一件事了。搞题海战术很容易,但是在亚美利加国有些孩子学数学就真的是兴趣,然后宽松的环境也允许他们在一个领域去研究,学校也会设置给这些孩子专门的课程,如果学校没有也可以去社区大学修课程,有多种可以定制自己教育的方式。

而且这些研究数学的艺术和体育修养也很高。所以发明创造还是集中在欧美。

我见过很多中国小留学生刚来的时候对于美帝的数学和科学教育各种鄙视,直到去了好学区 / 换了荣誉课程之后才发现自己和同龄人的差距,不仅人家数学比你好,人家还会多种乐器而且早上五点爬起来参加运动队训练。

相信我,虽然在本土大部分美国人都在吐槽教育和医疗系统,但是在这个体量级的国家能做到义务 12 年的教育已经很不容易了,更何况有些州已经开始升级到义务副学士教育了。

知乎用户 小球喵 发表

如果中国的教育放弃了偏难怪,放弃了奥数,神一样的阅读理解,全文背诵

中国的家长还知道怎么教育孩子么?那些不利于身心发展的教育模式给了多少家长安全感?

都说读书改变命运,没有一个人表示这个状态就很好啊,随遇而安啊。

小时候背诗的时候你是否问过这首诗的含义?如果你从来就没问过。那么你真的适合中国教育

另外题主你对于美国儿童的了解是来自于网络还是知心读者青年文摘啊?

知乎用户 刘一 发表

其实两件事完全不冲突,毛病在于中国的教育有问题

为什么中国人算术学的好?

因为我们小学时被迫进行了非常多的练习

而到了初中甚至高中,题目的计算量大了很多,但计算器还是被禁止

高中课本写着每星期 4 课时,但任何一个普通高中的学生每星期数学课会少于 8~10 课时?

我高中三年每天学校课程 8 课时 + 额外收费的补课每晚 2 课时 + 周六一上午 5 课时,合计每周 55 课时〉,语数外都能得到 10 课时以上。(所谓的补课也仅是要指额外付费,老师、班级完全不变,也可能上新课,所以不可能不去)

这还是全走读,没有各种早晚自习的北京一所普通的市 / 区重点

所以很多人都被迫训练出一种很强的笔算、口算和心算能力

而到了大学

中国人无论是学理工还是文科,高等数学都是必学的,而且都会涉及多元函数微积分的内容

而美国的 Calculus:

Calculus I,必修,包含基础的一元函数的极限、导数和微分、积分知识,大概是国内高数 1 比较基础和容易的部分

Calculus II,理工科必修,包括一元函数微积分中比较比较难的部分

Calculus I,选秀,包含多元函数微积分

所以看起来的确是,美国的数学学的比国内容易

但是,以上这些都是假象

有多少人能真弄明白线面积分是个什么玩意?泰勒、麦克劳林公式都是干嘛用的?

很多人都是学不懂得,只能靠大量做题来磨

所以大部分人即使学了高等数学,也是没学明白

更不要说更抽象的线性代数和概率论了

而且必须一提的是,国内数学课本编的太烂了

当年开始学线性代数时,最大的疑问就是行列式这东西到底是怎么蹦出来的,还有矩阵的各项规则是怎么确定的

相信很多人都只能不求甚解的死记硬背吧

到了美国,就不是这样子了

在第一年,学生需要学习几乎所有专业的基础知识,来塑造一个通才

普通学生只需要学习最基本的微积分知识就可以了

学有余力的学生才需要钻研更高级的数学知识

中国学生自然感觉到严重的自卑感了

因为,能学数学系的,都是在数学上真正有天赋的人

知乎用户 我很人二 发表

我是美国的(未入籍,我来说一下吧)

窝在中国蹲了 17 年,随后去了美国

中国的数学和美国数学最大的不同就是美国数学真的是有些科目最高数学要求就是可爱的三角函数就完事了,比如护士专业 business 专业基本上就没事情了,接下来学生可以自己去做想做的,为什么我的志向和数学无关我的数学强制要学这么深?国内高中数学都可以刷死一大批人(别和我争论这个,中国大学生比例 7% 醒醒了)

你要是精通数学喜欢数学的,那么好,高中有 Advanced Placement 课程,俗称 AP,你可以体会到不一样的酸爽(尤其是物理)数学 ab 还好,bc 可以把人弄拉闸,基本上统考满分五分,三分居多,ab 倒还好,也或者是我预习的透彻,。

顺便美国课程很自由的,十年级就可以 i 去上 ap 课程没问题你喜欢能接受就可以,不喜欢数学的只要三角函数 precal 就可以了。

再有喜欢数学的,你可以不修难的生物课化学课什么的,的确是这样,你只要把基础化学弄出来你就能弄到一个看似牛逼哄哄的文凭然后开开心心做你想做的事情。

对比国内很多数学优秀仅仅因为语文玄学英语玄学就活该平庸的,我能说国内很多人只要条件合适,大学转进 qs 前一百都不是大问题

知乎用户 Shannon 发表

原因是复杂的,我只提一点。

美国的教育是面向普通人的,也是极其注重个体差异的。

如果一个孩子没有在任何方面表现出任何天赋,那他会在极度简单极度舒适的氛围下完成教育,开启他极度普通的一生。

如果一个孩子在任何方面表现出哪怕一点点天赋,从家长到学校,都会给予其特别的照顾,帮助他发展。这里的天赋,不仅仅是学科类的。

所以美国大部分人尽管受教育程度很高,但是知识水平并不高。但是人群中聪明的那一部分,有天赋的那一部分,很少会被浪费被埋没。美国那些超级牛的东西,就是靠着这些人。

咱们国内的教育,更注重同质化,注重教育公平,这对大部分人来说,是绝对的好事。但是由于方方面面的同质化框框,让很多孩子的天赋被限制。我们可能也有很多能做出超级牛的东西的人,但是很大一部分在应对那些同质化的东西的过程中逐渐消失了。

举个不恰当的例子,2020 年国家游泳队冠军级别的选手因为体能不及格而无缘决赛。就算傅园慧的有用水平是冠军级别的,但是跑步不行,一样被刷。

这件事虽然很教育没多大关系,但是道理是一样的。一个孩子,就算他物理再牛逼,语文成绩不行,一样考不上好大学。

对基础教育来说,如果要求很简单,有特长的孩子就可以轻松的去发展自己的特长。如果要求很难,所有人都会内卷起来,包括有天赋有特长的孩子。

当然我说这些,不是要论孰好孰坏,只是针对这个问题而想到的一个点。毕竟,教育是百年大计,考量的标准是多方面。

知乎用户 扩散性百万辣面包​​ 发表

中学的事情我并不了解,不过身为一个在中美各读过两年本科的学生,我觉得可以从大学教育的角度来贡献一个数据点。

恕我直言,国内的选课制度和专业制度就是 ***。我在国内读的是经济统计学专业,本来志向是做一名量化研究员,可是学的课程当中有政治经济学、国民经济统计学、货币金融学… 这些课属于我的必修课程,却极大程度上妨碍了我的职业规划;且,你没有任何手段上一门其他的课去抵掉这些课的学分。

转学到了国外以后,除了高数、线代、概率论、数理统计这些通用的数学课要求必修以外,在选课层面上学生有极大地自由度,你可以选择随机微积分、偏微分方程、随机微分方程、数值分析、偏微分方程的数值方法、衍生品定价、蒙特卡洛方法、数据分析、数据挖掘、时间序列、高阶时间序列、机器学习… 基本上涵盖了所有 MFE 项目的先修课要求;你还可以辅修数学,用数学、编程和统计,填满你几乎全部的专业课和辅修!

对于能力不强的同学而言,他们确实没有必要全选数学来自虐(有的课上了两周就有 2/3 的学生 drop);不过对于那些能力强、有主见的学生而言,这样的自由度毫无疑问意味着更宽广的舞台。

而更广阔的舞台会极大程度的激发一个人的主观能动性,以使他获得可持续成功和恒久幸福。

知乎用户 木兰她哥哥 发表

这个问题很诡异,必须分开来回答。

第一,首先说,提问者描述的中小学生绝大多都是公立学校的,我可以很明确的回答,你所说的那类学生大了以后没多少能读常春藤学校,基本上八成都是垃圾,你没看错,对,垃圾,如果把你描述中的中小学生长大了抛开语言优势,拉到中国来,没老板会要他们,他们的就业优势甚至不一定有农民工高,用句形象的话来说就是 “既懒又馋又闹腾”。明白不?而你看到的优秀人才,从小到大都是很辛苦的读私立学校的,从来没有!从来没有哪个私立名校读书是很轻松的,不重视数学的。这类人才是美国出精英的人群,跟你说的那些快乐教育长大的垃圾几乎没有任何关系,几乎没什么交集。顺便给你算个账,那类精英,大概多少钱钱可以从私立幼儿园读到本科毕业,有人估算过大约在 110~150 万美元,明白吗?在美国有没有钱不是看你开什么车,是不是用爱马仕的包包,而是你有几个孩子在读私立名校,有三个孩子从幼儿园就开始读私立名校的绝对是有钱人。

第二美国的教育重点和精华在于研究生和博士培养,他有从全世界吸引本科生去读硕博,然后挑选优秀的留下来使用,你所认为的优秀人才大多数都是别人从小花钱打基础培养,到了硕博阶段美帝升华培养,这个世界只有美帝才可以这么玩。

第三曾经的我们容不下那么多的高端人才,现在的我们依然容不下太多的尖端人才。因为……“没钱,没项目”。

这问题很混搭

知乎用户 FlumenStellarum 发表

题主看看最近几年的菲尔兹奖(或者如果题主是行内人,可以把你觉得重要的成果列举出来),看看有多大比例是美国受过基础教育的学生? 中国也有人做出很牛逼的东西,只不过被题主有意无意无视了。不然就是题主自己孤陋寡闻。

其实根本不用最高票答案那么多废话。AP 很简单的,还是比国内容易。美国也是数学竞赛强国,但还是没中国这么强,跟俄罗斯、韩国一个档次。

所以,其实美国的科研大牛很大一部分根本不是美国自己培养的。是别的国家培养的,研究生阶段以后去了美国而已。

知乎用户 叶湘伦 Jay 发表

由于有考纲这种奇葩的东西存在,我们的下限比美国高得多,上限比美国低得多。另外,自欧拉以后,不是战斗民族的数学最好吗?当然中间战车和美帝都很辉煌过!

我们只是国际奥林匹克竞赛厉害。国际奥林匹克伤仲永不是我们一个国家的事情。全世界都在伤仲永。本来成材率都是低的。(就像拉玛西亚也是这几年出了梅西这一批天才,博杨不久伤仲永了)。不过 IMO 也出过菲尔兹奖。

陶哲轩

吴宝珠与佩雷尔曼、陶哲轩是当代数学三杰,三人都是第 29 届(1988)IMO 金牌得主,佩是满分,16 岁时;吴可能被扣 1 分或满分,15 或 16 岁;陶与吴一起参加一次 IMO,最后一题没做出,当然他当时不到 13 岁。当时得金牌的中国选手何宏宇(四川省彭县中学)与陈曦(复旦大学附中)好像现在没声音了 。

吴宝珠(越南)2010 菲尔兹奖。陶哲轩是华人,当时是中国人,11 岁 12 岁 13 岁分别参加了三届国际奥林匹克竞赛,分别获铜牌银牌和金牌,陶哲轩也是澳洲第一位也是唯一一位菲尔兹奖,是华人第二位菲尔兹奖。第一位是伟大的丘成桐道长。我看过各种帖子讨论陶哲轩和吴宝珠谁更牛逼的。陶教授还是 6 月份首届数学突破奖六位获奖者之一,奖金是诺贝尔奖两倍。你懂得。

至于说民国之前培养出了无数的大师,咱们就不吹水了。熊庆来、华罗庚、陈景润、陈省身、吴文俊、谷超豪……

美国是第五次科技中心无可辩驳。基础科学最近可能只有物理有像欧洲转移的趋势。不过我们国家也不弱啊。物理学方面:爱因斯坦徒弟是**束星北,束星北徒弟是李政道**。爱因斯坦学生是费米,费米学生是杨振宁。就看当今,美国最年轻女院士**庄小威(物理)就是中科大少年班的。生物学方面比如谢晓亮。化学方面汤森路透集团(Thomson Reuters)发布了 2000-2010 年全球顶尖一百化学家名人堂榜单(TOP100CHEMISTS,2000-2010),这份依据过去 10 年中所发表研究论文的平均引用次数而确定的最优秀的 100 名化学家榜单中,共有 12 位华人科学家入选,六人是中科大培养的。其中是: 斯坦福教授戴宏杰第七名,阿肯色大学教授彭笑刚第八名,伯克利的杨培东**第十名。

再看

全球顶尖

100 位材料学家榜单, 共有 15 位华人

科学家

入选, 其中前 6 名均_为华人。前五名是中科大本科,第六名是台湾清华。。。。第一名是杨培东。。。。又是这个名字。_

根据汤森路透(Thomson Reuters)近期公布的 2014 年全球高被引科学家名单,中国(含港澳)共有 134 人入选 2014 年 “高被引科学家”,排名世界第四,仅次于美国(1702 人)、

英国

(304 人)和德国(163 人) 中国科学院有 46 名研究人员入选,占全国入选人数 34%,居各科研机构及高校之首,并有 5 人同时入选两个学科领域。中国科技大学入选 6 人,在全国高校排名第一。

年度 100 个世界顶尖青年发明家 中华人科学家有 13 人榜上有名,包括

加州大学

柏克莱分校纳米技术应用研究专家崔屹博士、哈佛大学助理教授

庄晓薇

黄昱

、段镶锋、杨培东鲍哲南、刘湘军、

陈天桥

张前

等。。。我又看到了庄小威和杨培东!!!

不好意思,上面所有出现的名字只要是建国后的,全部都不是中国国籍。。。

都说世界顶尖科学家华人占 30% 而中国人口只占 20% 哦。。。至于国内教育怎么样。我就不说了。

知乎用户 知乎用户 G8C9Rx 发表

执着 与 热爱

首先 必须承认 我们的 9 年义务教育 在质量上 绝对不逊色任何一个国家和地区

但是 我们在创新上 和其他国家 有很大差距

若把学习比作一个跑步的过程 那就是马拉松

开始 我们都在前面 远远甩掉其他国家

但没多久 我们就被赶超过去

有些人 停下了

有些 慢慢走

有些 喝水休息

有些 离开比赛

而且其他国家的人 继续跑 虽然中途有些也出局

但是很多能坚持到最后

-————————–

外国 在小学时候 素质教育 甚至会带学生 去消防中队 告诉他们 怎么和消防员联系

并且每个人亲自练习实际体会

我们只会在书本上写好 119

国外是 实践是检验真理的唯一标准

我们是 书本背诵好是唯一标准

这就是差距

-————————

外国 中学 分各类学校 比如 数学 物理 外语

我们是分文理班

人家 把喜欢一类科目的学生集中起来 有针对性的提高培养

这些学生 今后读大学 也进入类似相关专业

我真见过 有植物班的

今后进入生物专业

我么要是有这样的班级 那老师就被开了

-——————-

而且现在国内 社会发展快速 导致人心浮躁 追逐名利

很难有人静心研究

在外国 我们前系主任 就是为了科研 主动让贤 自己继续教课 搞研究

因为当系主任 还得照顾系里事情 忙不过来

中国呢 为了整个教授 主任的头衔 尔虞我诈 勾心斗角 疯了

老师如此 学生咋办?

-——————–

中国是数学大国 但不是数学强国

事实面前 我们必须承认

很多时候 看到祖国强大了 挺开心的

但是 你要是内行 你当然懂 和世界顶尖差距多大

就好比 三本的第一名 和清华第一名

都是第一

你懂得

知乎用户 Hana 发表

1. 申请名校的美国学生高中阶段要学完微积分。

2. 垃圾学区的美国学生高中毕业不会解一元二次方程。

3. 不管你是哪一种学生,老师只会给你讲古德找不,额妹子印。

知乎用户 Giorno Giorgio 发表

你大概没看到过有几个美国人玩 excel 那个溜,各种函数等等信手拈来,我们这里没人比得上,他们也分的,我们看到很多那边普通学校人数学渣就感觉全民渣,其实根本不是这样的啊,就好像我如果你拿重点高中的人去看一些民办高中那种优越感之大→_→我觉得他们也只是心算口算比较烂,可能是因为没中国的乘法口诀那么好背,反正我是从来不敢小觑人家的逻辑思维能力和数学能力的。

知乎用户 知乎用户 KZISe7 发表

中小学的知识不能影响你的人生,教育却能

知乎用户 阿斯兰 发表

美国人的研发人员,早就变成印度和华裔。在知乎里居然还有人信中国的平民教育不行,简直滑天下之大稽!看来美国的素质教育➕服务生更适合他们。

知乎用户 赛先生 发表

美国给不热爱数学的学生最基础的数学教育,给热爱数学的天才最高水平的数学教育。这个观点还是很有道理的。

知乎用户 硬条​ 发表

美国的教育分两类。

一类是普及的公立教育。所谓那些数学很差的,还有啥玩意都不会的,快乐教育的,素质教育的,都可以列归此类。这些学校是给大多数家里没钱的孩子上的,确保有地方教他们识字。好批量的生产服务人员、低阶产业工人的。咱中国的素质教育,学的就是广大的美国公立教育机构。

另一类是精英教育。就是你说的那些做出超级牛东西的人才培养摇篮。大都是私立学校,少量公立学校。特点是学费贵,门槛高,入学对家庭也有要求,起步也要是技术类、金融类、管理类中产家庭往上,也吸收世界各国优秀的孩子,少量公立学校学区非常贵,能买得起这里房子的也非富即贵。在这里学习的压力很大,课业压力非常重,虽然教学模式看起来和国内不同,但内核比中国十年前的应试教育还应试教育。

美国的核心竞争力是什么?一个多元文化的开放态度,是文化输出影响世界一流人才都向美国聚集,进入美国的高等学府学习并留在美国。他在吸收世界各地的人才,所以本国那 3 亿人大部分都在垃圾的公立学校混日子没问题,并不影响美国的创造力。

中国则是依靠人口优势,自己这 14 亿多人,全民普及应试教育(这在美国叫精英教育),尤其十几年前沿用了很长时间的高中文理分科,再加上大学扩招,批量生产了大量技术性人才,充斥到了各行各业。到现在,中国的大学理工科毕业人数已经彻底吊打美帝。这些人才不问出身、不问家庭,是十几年应试教育培养的产物,当年那些学生,有多少没被老师罚站过,少部分还挨过老师的打(不是个别老师打孩子,是每个班的尤其班主任,一定会教训不好好学习的娃),那时候老师打学生,越是穷人的家长越高兴,那是老师在替你管教孩子。有多少凤凰男不就是出身于此的?

这么多人才发挥的创造性简直匪夷所思,这么多人才造成的激烈竞争也是匪夷所思。所以中国慢慢成为这个世界上唯一一个全产业链国家,这十年在高技术领域、应用技术全面飞速增长。什么超级山寨?你造出来了一个产品,中国企业看一眼就能模仿出一个便宜十倍甚至百倍但能保持你 90% 以上性能的产品?你以为这很简单就能完成?

为什么中国被称为发达国家粉碎机?正是因为我们有这么多的底层技术人才,在各行各业打破了发达国家的技术垄断,还有低价大批量生产的能力,迅速打入各国的市场,一下子把贵的要死的外国货给挤压没了。前些年去日本疯狂采购高科技马桶盖、电饭锅的,抱歉,那其实都是 made in China 的。

其实当年日本、韩国也是这么做的,可惜这些国家就是现代版的美国殖民地。国家体量小,还被美国牢牢的控制住,几个金融危机、商业打击,就把日韩的这些企业要么打击死被美资收购,要么美资重资参股控制起来了。

为什么美国这么限制中国?因为人种不同?日本不是黄皮么?当年还和美国掐的那么狠?意识形态不同?开玩笑吧,华为是国企么?核心的原因只是利益。

因为中国的发达国家粉碎机属性,还是一个不受美国控制的国家,不是美帝的现代殖民地。美帝是靠科技水平吊打他国,科技是第一生产力么,维持强大的军力,并将技术下放到民用商用,持续的垄断市场造成巨额利润。中国能把一切技术垄断打破,并低价批量生产,让这些发达国家的核心技术产业无法再持续垄断,无法持续获得超额利润。

有这么高的利润,就可以给那些产业工人开高工资,即使他们很蠢,但没关系,老爷我能从国际市场上垄断技术赚钱,赚到的非常多,给你们多一点也不妨碍老爷我多赚。产业工人手里有钱就可以去消费享乐,就刺激了服务业的发达,这样哪怕端盘子的都有钱赚都能过得很舒服。

但没有超额利润,这些国家如何去养那些蠢蛋一样的服务人员 ,产业工人?端个盘子洗个碗都能过的不错?

为什么美国也好,欧洲也好,普通人越来越穷?为什么 45 年战后到 90 年代这五十年欧美都那么多蠢蛋普通人都过得很好,新世纪这二十年突然变得很穷,还越来越差了,法国巴黎的居民家里都安不起个空调了,这是发达国家?因为这些国家的高技术垄断企业在这二十年不能垄断了,没有了巨额利润,没法给国家缴那么多税,国家没有那么多钱钱了,这些企业想活命都很不容易了,只能去国外第三世界国家建厂降低成本,本国国民的平均工资就没法持续增长,那些在车间打工的没法获得更多的钱,保住工作都很难了,也没钱用于消费,消费降了,服务业也赚不到足够多的钱了,洗碗的端盘子的自然也没钱赚了。

回到开头: 美帝学校批量生产蠢蛋和他能搞出很牛的东西并不矛盾,但现在有个国家,他的学校通过应试教育,把美帝少量生产的精英人才大批量的培养出来了,释放到了各行各业,经过几十年的努力,也开始生搞出超牛的东西了,而且未来会越来越多,而这个国家,就是我们。

知乎用户 赛宝宝 发表

中国基础教育水平超过美国也不过是近年的事情。在此之前,中国的努力的是扫盲,普及初小,高小(这两个词年轻人可能不懂 ,初小就是读到小学三年级,高小就是小学读完,高小毕业在农村一度就是知识分子),九十年代还在普及九年制义务教育,即普及初中。美国普及高中,乃至大学都不知道多少年了。

但如果中国一直能提供更多更强的基础人才,那么专业化和顶级人才的水平超过美国,只是时间问题。

很多问题其实就是这么简单。大方向不错,比的就是人力物力财力加时间积累。

lol 压倒韩国如此。18 年之后 LPL 资本的力量显示出来后,LPL 成为世界第一已经成为定局。

将来足球发展也是如此。只要中超长期大量的投入资本,中国足球崛起就是时间问题。

科研出成果,和天才灵感没关系,就是长期大规模的投入人力财力,积累到一定程度,就会出成果出大拿。

2050 年以后,中国大学能占百强的 1/5,每年能获得 1/4 左右的诺奖。

30 年转眼就过,拭目以待。

知乎用户 Stark Huang​ 发表

是教穷人的东西很简单而已,美国大学里牛逼的要么是美国中产以上的小孩要么是各国过来留学的精英

知乎用户 Reggaeboy 发表

我看到很多回答都是把一些旧闻和道听途说作为例证来论述这个观点。我在一所国际学校工作,我自己是公立学校毕业的,我来谈谈我的所见所闻。

以英美澳加新为例,他们的教育都是精英教育,也就是把精英阶层和平民阶层的教育目标区分开了。我身边做管理岗的外国同事基本都是家境比较好,从小上私立的寄宿学校。他们学术水平很一般,简单的算数都算不清。但是他们社交能力强,口才好,做事情有思路,有领导力。这些就是精英教育给他们带来的优势。而平民阶层出来的,大部分思想局限性极强,比如我们中国人基层人员很多人思想上有一种大局观,喜欢站在管理者的角度考虑问题,而外国的基层工作者就没有这个 “坏毛病” 或者说上进心。他们往往不善言辞、头脑简单、情绪化、身材管理失败,一看就不适合做管理。

那么他们是如何进行精英教育的呢?

首先是配备足够的资源,比如私立学校一个老师对应非常少量的学生、优质的设施配备、丰富的课程和课外活动。比如在我们学校,250 个学生配备了将超过 60 个老师,提供 100 门课外兴趣课,配备室内体育场、恒温泳池、击剑室和各种乐器室等等。学生的兴趣得到充分发展,学习得到个性化的指导。

其次是中小学教学上轻目标、重过程:相比于我国,英美的中小学教学目标定得很低,小学阶段以兴趣为导向,知识点很少。但是,他们每一个知识点都是先激发兴趣,再鼓励孩子自己去探索寻找答案。比如在数学方面,圆周率的学习会把大部分时间用来研究和实践割圆法,而不是把 3.14 直接告诉学生然后让他们拼命做题。这样的好处就是孩子不厌学,并且从小培养了科学精神(发展问题、探究、解决问题)

再次是个性化学习的实施:每个人的大脑是不一样的,有人对色彩敏锐,有人对数字敏锐,有人语言文字敏锐。每个人的成长曲线也不一样。有些人早熟,很小就知道要学习,但是大部分男生小时候就知道玩。因此根据每个人的特点制定符合孩子行为阶段的学习计划非常重要。另外上一点说的,知识点少一点,这样中小学阶段学生的差距不会拉开。

最后是强调高中才是学术发展的关键:当到了高中阶段学生更加接近成年,自控力有所提升,对未来的发展方向、学科的偏重也比较了解。这个时候进行分科并提升学科难度就非常适合了。英美的高中数学分难度的,学生可以选择高难度的数学来获取更高的学分。微积分这样的内容在高中就能学到。基本上学生在高中就已经想好大学要学什么专业了。

当然,这样的教育也并不意味着学生有非常强的科研能力。在美国占科研力量主力的自然是亚裔包括印度裔。白人主要强在领导力的培养上面,就是自信乐观、人缘好、口才好、体能充沛!

知乎用户 上官竹 发表

工科博士生表示,

看了一眼美帝本科一年级数学教材,感觉是:这 TM 也用学?

然后看了一眼美帝本科三年级选修数学教材,感觉是:这 TM 也是工科生能学会的?完全看不懂……

还有一点就是,国内的理工科教育由于太偏重技巧,导致学生学习重心都在那些易于手算、纯技巧难变通(简言之闭卷两小时之内能手算出来)的问题上。而对需要高级计算器甚至大型电脑才能计算的问题不感兴趣,对分析原理更不花费太多精力。

问题是等到一毕业,谁尼玛还去算手算能解决的问题?!

相关专业的主要看看中美结构力学的实际授课内容差距就懂了。。。

知乎用户 小火苗 发表

穷人孩子快乐教育将来公立高中结束继续混底层,精英的孩子直接私立走起一天只睡 5 小时,耶鲁哈弗 MIT,将来继续统治,懂了吗。

大家族的子弟不学无术考不上也没关系,小布什这样的捐捐钱照样上耶鲁,不是暗箱操作哦,这都是明明白白放在台面上的校董推荐

知乎用户 M3 小蘑菇 发表

美国学校分公立学校和私立学校两大类,公立学校用公款,教师收入低,教育的终极目的是 “让每个该上学的儿童都能上学”,注重的是公平,质量就别管了,那些所谓题目简单,上课氛围轻松的大多是指美国的公立学校

而私立学校学费高,不是谁都能近的,一般要有一定社会地位和财力的家庭才能把孩子送进去,私立学校的教育终极目的是培养精英,注重的是公平,自然不会轻松到哪里去

另外,中国学生是小学中学苦,大学轻松,美国学生是小学中学轻松,大学苦

知乎用户 湘西老土匪一名 发表

很简单的一个算术题,一个很牛逼的东西,全世界大概有可能做出来的,有十个人,美国一家就招去了 3 个,自己培养出来的有 2 个,大概 50% 概率是美国人做出来。你以为这就完了?不是,其它国家分这 5 个人当然不现实。打个比方这 5 个人分别在 5 个国家,如果是特别穷的国家,这就要折掉,如果不穷,但国家不重视这块,科研能力有限,这又要折掉。如果是有钱,科研能力又比较强,但因为偏科,需要这个牛逼货做当紧做的项目,放弃掉这个很牛逼的东西,这又要折掉一个。

全球吸精大法和海量科研投入,就这个。别扯人才机制和科研学术机制,扯多了都是水,往前面那两条看,就这么简单的核心。二战之前没这么大差距,二战以后,它本土最安全,钱最多,有问题没有?没有,所以这两招看起来简单,别人玩不转。

不是没有机会,大把人提出弯道超车,可没人把美国逼到那份上,略加逼近,就是一通大棒过来。打个比方,各国都有人才,美国说好,集中人才办大事,但中国不在可选范围内。它这样做过吗?做过,然后他集中 80% 发达国家一起搞。是的,中国也可以自已搞,但你花了大成本搞出来以后,标准不通用,你怎么大规模生产摊低成本?一个两个项目还顶得住,项目多了,光这成本开支,你顶得住?不要以为这是小问题,时间一长,你的人才天天玩屠龙术么?还是得走,走去那里?美利坚。

又说体制,呵呵,合理的体制不是天生都那么合适的,那是钱养出来的。很多人抱怨,我这项目乍就不批呢,我拿着到美国批了。因为十个可能性的项目它也投得起,你穷你得选最大可能性的一个去投,还不一定资金够。直接使它职业的精细程度、自由度、偏执和狂热都最大化的释放出来。讲个不好听的,到一定学历程度了,我们老是学术,因为找碗饭吃更重要,它可以要求先别急,把道学通,我养得起。低端的术,它直接花钱买得了,通天大道走通了,它就一直霸在最高端。

别动不动板子打到基础教育上,和这块关系不大,中国做的不错了。往大学往上,进入人才选拔通道以后,咱们差距就体现了,人才流失严重、投入不足、学术风气差等等。种一大块稻子,长势不比别人差,可到结穗时,不浇水了,然后怪种不够好,苗不够好。中国这几十年教育界一大怪现像,推锅。

学术和玩没区别,以前都说中国人和木头一样,这也不会玩,那也不会玩。我就从不喷王石,因为他打了广告,说山在那里,然后中国人就玩起来了,不就是有钱的信号呗。不急,高分试卷放在那里,能不能抄到,和要多久抄完,都是个考验,没那么容易的。这不就听说,有的专业都准备不让中国学生去学了,你说要全球吸精,他说你们中俄就是朝鲜,你要玩海量投入,他说不要和中国人做生意,专指着你动脉放血。

知乎用户 崔丁 发表

来来来,我再换一个说法,为什么美国中小学生学的数学比我们简单,中国人却能在产业上追美国人越追越紧,甚至还现实部分超越。

看来数学越简单越不行啊!

结论有问题,手动狗头

知乎用户 交易如人生 发表

很多人会说国家体制导致的个体差异,不是很多几乎所有的媒体都在这么宣传,资本主义的个人主义容易诞生天才。

但有了足够的理智后才会发现,这个说法还没有被证明,人们只是从结果推到出这个以为的原因。这是缺乏依据的。

比较符合逻辑的原因是,西方国家几百年的知识积累,和较为稳定的社会环境肯定是一大助力。对比我们国家,近代这一百多年没法说了,能吃饱就不错了,能少被欺负点就不错了,哪里有这个条件产生天才,产生超级牛的东西。

要想真正做出一个对比,只要再给中国一百年稳定就可以,到时候可以横向对比一下,是否西方更容易诞生天才诞生很牛的东西。

知乎用户 陈必红​ 发表

中国已经做出超级牛的东西了。美国人制造的东西,中国或早或晩都会造,或者说跟进,中国起步晩。中国造出的高铁,我的预言是,美国生生世世永永远远都不会造。中国的盾构机世界第一,美国永远无法超过。老干妈辣酱,美国人想破脑袋也发明不了,且已经成了美国监狱中犯人私下使用的货币。不信你去百度上搜。

知乎用户 张戎 发表

希望我昨天(2017 年 8 月 22 日)第一次参加美国中学家长会的体验能提供回答题主问题的素材。哎呦喂我又把我的第一次献给了知乎。

特别声明,以下结论基于美国中西部一所中等偏上的公立学校(R)和北京首都机场附近一所私立学校(N)的对比,数据量不支持普遍中美对比。

N 校与 R 校对比:

  • 家长:R 校父母双方参与更多,在孩子教育上没有明显的父亲缺席
  • 老师:R 校更重视孩子综合能力的培养,虽然数学进度落后于 N 校,但素质教育更超前。

首先,家长会的形式是:17:30 到 17:36,70 多个家庭 100 多名家长在大厅集合,然后 17:40 开始根据自家孩子每天 07:30-14:50 共 7 节课加 1 节浮动课的课表,与每一节课的任课老师在该课教室见面,每场 10 分钟,转场间隔 3 分钟。下图是昨晚日程表,大家体会一下 R 校老师和家长的守时程度。我 17:32 进场,大厅会议已经开始。

![](data:image/svg+xml;utf8,)

我在 N 校时非常喜欢参加家长会。参加家长会的以美妈为主,约占 9 成以上,或是祖父母一人来参加。我喜欢的主要原因是老师讲得好。而 R 校这里大约一半的家庭是父母一起来的,另外一半或父或母(美妈比帅爸稍多一点但不明显)。

每位老师首先会介绍自己的家乡、家庭、宠物、学历、经历、爱好,然后会讲自己对课程的思路、计划和要求。回想 N 校的好老师们开家长会更多谈课程,很少谈自己的私生活。下面是我的笔记。

  1. 科技工程和数学 (STEM):老师强调五大教学目标,包括创造性、沟通、批判性思维、团队协作、和解题。有没有商学院和咨询公司既视感?
  2. 戏剧:艾玛,女老师长得挺像爱乐之城女主!她在纽约百老汇演过戏,佛罗里达大学的戏剧学位。她本学期的首要目标是帮助每个孩子克服美国人的第一恐惧—当众讲话。她还说对死亡的恐惧排在后面。(为什么没提 “床总连任”? 偏执的我课下检索发现数据来源是 1973 年 R.H.Bruskin Associates 的调研结果。好吧,跑偏结束。)为了帮助孩子们练习当众讲话,她要营造一个安全的氛围,让孩子们首先摸清彼此的脾气秉性,在这样的安全氛围里孩子们即使摔倒了也可以很快爬起来。她还说在舞台上可能真的会摔倒。她会让孩子们从静止造型练起,逐渐熟悉戏剧的 C.O.R.E.,即人物、目标、关系、和环境。她期末的大作业打算让孩子们组织一次山寨版 “周六夜现场 Saturday Night Live”。最后她播送了一条通知,说学校正在排练《丑小鸭》,鼓励感兴趣的孩子报名试镜。散会后我上前握手,说孩子们有她教真幸运。握手时间小于李梅亭握孙小姐。
  3. 社会学:这门课相当于国内的历史地理合体,外加政治经济。上年主要讲欧美,本年主要讲东方,中国是重点。他们会专程去城里的博物馆学习中国历史,观看介绍中国的 IMAX 电影。班里华人很少,女老师目光投向我的方向,说欢迎去过中国了解中国的家长来助讲。看来会后我又要去握手了。另外,该课程还会涉及 “个人未来财务规划”“全球化”
  4. 数学:数学老师的介绍比较偏课程内容,分普通班和提高班。提高班本学年主要教学目标是:线性方程和函数、角度和相似形、平方根和勾股定理、指数和科学记数法。根据以上信息,你猜猜这是几年级?
  5. 科学:有点像咱们的自然课。老师强调实验课的重要性,说所有缺席的实验课都要补做。
  6. 语言艺术:语言老师的目标是,不仅要教会孩子们写作,还要写得 “有趣”!我差点举手问要不要同时保证 “有料、有种”?还没等我想出来怎么把这两个词翻译成英文,老师果然提出“有趣” 以外的其它要求,大致就是“有料、有种”。“有料” 包括:写作要有清晰的目的、要做广泛的文献检索并记录出处、要了解多种不同视角、要领会不同作者的风格特点。“有种” 指的是鼓励孩子表达并捍卫自己的想法,如果对学校和老师做法有不同的意见,一定要书面或口头沟通。
  7. 健康和体育:大致相当于体育和生理卫生合体,外加心理卫生。体育成绩评定不是跑多快跳多高,而是努力程度、运动员道德、和个人进步。健康主要内容包括:精神情绪和群体健康、营养学、急救与安全、毒品预防、生殖系统和艾滋病。

希望这份简短的 R 校家长会笔记作为一个切片对大家了解美国教育有帮助。

其实我感觉正面回答题主问题比较困难。题主问出 “为什么” 的隐含假定是”中小学数学水平应该是能否做出超级牛东西的主要因素并正相关”,但这不一定成立。首先,数学教育只是两国教育中的一小部分,虽然中美教育目前还有区别,但是中国现在也并不是只重视数学这类基础教育,在素质教育方面,我观察到 N 校的教学理念已经比我小时候在北京上的一流的公立学校当年的教学理念有很大进步。其次,中国人也在做出超级牛的东西,比如微信、共享单车等。即使在 “做超级牛的东西” 方面,我们可以量化比较两个国家,我相信中国的超级牛度在飞速提升。将来可能发现中美素质教育旗鼓相当中国数学又强,再加上 5000 年的文化底蕴,结果中国的超级牛度全球第一。

对了,孩子今年上 7 年级,相当于国内的初一。

… 更多回答请看张戎

参考文献:

  • Karen Kangas Dwyer & Marlina M. Davidson (2012): Is Public Speaking Really MoreFeared Than Death?, Communication Research Reports, 29:2, 99-10

知乎用户 巨佬 RD 发表

就说上海,那教材是个啥啊

高中三年我就根本没看过数学书、物理书、化学书。因为里面包含 0 信息量

几乎没看过语文书、英语书、政治书。因为语文政治书太无聊了 英语书太简单了

正常看过历史书、生物书、地理书 就这里面的正常一点。

知乎用户 醉里挑灯看剑 2018 发表

这个问题可能跟思维方式有关系, 西方人是管状思维, 如果决定做某个研究领域的话, 会有一种要做到极致的心态。中国学生灵活性其实好于美国学生,基础知识也扎实,缺乏的可能是攻坚的精神。因为中国文化里的趋利避害部分太强大了,还有所谓的中庸之道,这种文化能让普通人容易求得个人利益最大化,但对团队合作能力及个人创造能力阻碍极大。

如果每个人都想做轻松,收益大的工作,那很多基础研究根本无从做起。美国人并不是做一件事成一件事,平均智商也决不比中国人高,只是因为在对研究的执着程度上强很多。当然,不是所有美国人,大多数美国人也是平常人。

这种执着,用中国人的眼光看,其实就是傻,因为他们做的很多事情都不能马上出效益,甚至在很长一段时间都不能出效益。所以,研究水平的高低无关于智商,关于心态。

知乎用户 薛凯纸​ 发表

因为在美国,不同阶层数学教学的目的不一样。

“为啥美国人的数学 / 大部分理科这么差,他们的科研还这么优秀??” 曾是我很大的一个疑惑。后来我发现,在美国,基础数学和高等数学的差别对待,是为了数学这门学科对于不同人群有着不同实际操作价值。

一. 考试专用数字?

高中的我痴迷于 AP(美国大学预科)考试。我在高二的时候考了 AP(微积分 BC。AP 微积分分为微积分 AB 和微积分 BC,微积分 BC 比 AB 难,微积分 BC 满分相当于同时也考了一个微积分 AB 的满分,成绩单上写为两门微积分。我还考了 AP 物理 1 和 AP 物理 2。高三的那年我考了 AP 化学,AP 统计,AP 物理 C - 微积分力学,AP 物理 C - 微积分电磁学,和 AP 生物。非常惊险的是,当时考 AP 微积分 BC 快要结束的时候,我的计算器突然死机了!

在经历了手动六次方,手动开根号,手动算对数等惨绝人寰的操作以后,走出考场的我内心宛如有一条脱缰的野狗:为什么!他妈的!要给我这么多神他妈诡异的数字??整数!我的亲人!你在哪里??

在我考的 14 门 AP 考试的理科中,除了环境科学,其余的学科都是需要使用计算器的。回想起我还在高考体系的时光,那时候的数学物理化学都是伊甸园式的理想化存在—我们见到的基本都是整数,分母分子可以约分到整洁美丽的样子,三角形全是直角三角形,除了直角以外的另外俩角要不是 30+60 就是 45+45,再不然这个三角形的三边比例一定是 3:4:5 这种友好又贴心的样子。在数学或者物理计算角度的时候,我们最后的值永远带着π这个调皮的小妖精—我们从来不用把 3.14159265359… 带进结果里给一个不精确的小数值近似值奇怪的小数,分母分子都很大看着很凶的分数,或者你的根号里面带了一个 20 以上的数,往往是初高中数学考试中错误答案的指示器—我依旧清楚的记得走出中考数学考场的我嚎啕大哭,因为我最后一道大题解出来的答案是根号 42 这个诡谲的数字—众所周知,在中学数学考试中,根号 42 = 你个垃圾,这题零分

刚刚上大学的我打算学精算。但是因为我的 AP 微积分成绩已经满足了很多门大学基础数学的必修要求,我直接从比较高级的数学课(大部分中国留学生的第一门数学课)开始上。我第一学期修了两门数学,其中有一门甚至是数学系推荐大三才修的高阶课。但是我并没有感受到什么特别困难的地方,因为这两门数学课其实和我在高考体系的时候接触到的数学的形式很像—虽然是数学系的高阶数学课,但是我们更注重的是数学公式的理解和纯数学理论的应用我们高阶数学课的考试甚至并不需要带计算器—这和国内的数学教育是一样的。答案也是以精确值而不是近似值的形式存在,很多涉及三角函数的题目答案里也带着π。

下图为节选的数学课本习题答案:

香港大学交换学习一年以后,我确定将神经科学作为专业以及终身的事业。回到 UCLA 后我开始上一些数学外的理科课程,包括生命科学物理,生命科学化学,生命科学(生物),以及科研统计。我发现当年 AP 的噩梦又回来了:计算器成了立身之本,所有的答案都要用小数或者是规定了有效数字个数的科学数表示。完美的 30-60-90 或者 45-45-90 三角形从我生活中消失不见,所有题目 里要数字都感觉像是教授在键盘上瞎敲上去的。举个例子,最简单的物理的力的合成:

这种已知直角三角形两直角边长求斜边长的题,竟然不是 3:4:5 或者 1:1:√2 或者 1:√3:2??当时的我简直不敢相信自己的眼睛,这感觉酸爽的就像你凑上去想亲自己的媳妇,结果发现认错人了然后被甩了一个大耳刮子似的。

这题的解是这样的:

我简直气的两眼昏花,这算了半天发现这个角是 29 度??就是不给我 30 度角这究竟是什么仇什么怨??

不仅仅是物理,就连化学在计算方面也非常不友善,比如说下题 d 族金属复合物形成常数的计算:

:)

二. 计算器究竟是礼物还是毒药

美国人的数学差到世界闻名,这一点确实是空穴来风。计算器的使用带来了计算器的依赖—美国人的口算笔算能力就和他们的心算能力一样令人心酸。我亲眼见过一个美国同学列竖式计算 7 乘以 9(这个因为他们没有九九乘法表我觉得其实可以原谅,但他们列这个竖式干什么这个问题至今成谜),也因为脱口而出根号二约等于 1.414 而赢得满堂喝采(…)在我因为无法继续厚着脸皮面对他们崇拜的眼神而承认并不是我当场心算根号二而是因为我把根号二的值背下来了的时候,他们崇拜的眼神换成了看数学怪胎的眼神,不可思议的问我:“你为什么要背根号二的值?”

当然是因为中国人都是小天才没事就喜欢背平方数根号数啊!(强颜欢笑

当我在口中默念九九乘法表算一个两位数乘以两位数的竖式的时候,我的小组成员们围着我:“Kelly 你念的是来自中国的数学咒语吗??”

那么,如果计算器百害而无一益,为什么全世界最出色的美国科学教育,要如此广泛地普及计算器的使用呢

三. 不完美的数字—真实的世界

这种特殊数字的规律性带来的计算捷径的好处在我的心中根深蒂固—物理实验课上我给我们组搭造的斜坡是 60 度的,弹簧选择的是整数长度的,棱镜选的是好计算的焦距的。在我得意洋洋的跟我的组员解释这些数字的特殊性的时候,我的组员丝毫不领情的质疑:“那我们用特殊的数据,测出来的值真的能够代表通行于真实,非实验室设定中全部情况的规律吗?

如果你已经知道了你所做实验将要探究的公式,这个问题无疑是一个愚蠢的问题。但是,实验的本身就是模拟科学发现以及科学探究的过程,是模拟一个我们不了解理论,从而去通过真实的实验自己发现规律的过程。因此,我们的实验课内容总是先于比我们的课程进度—在不了解一个概念的情况下做实验,才能最为逼真的模拟从 “未知” 到“已知”的发现过程。而在这个过程中,数据选择的随机性至关重要的。如果我们选择的实验数据只是真实世界中的一种极端情况,就算这样的的实验数据能带来看着漂亮工整的实验数据和推导过程,它在科学领域的价值依旧为零—这也就是统计学中所说的样本偏差:当选择的样本并不能完美的代表样本背后的整体,样本中发现的规律并不能代表实验所探究的整体的普遍规律

确实,我们必须承认的是,我们的生活中不仅仅只有 30 度,45 度的直角三角形和整数—自然界中数字的无序性才是世界本身的面貌。当我们过度沉溺于被简化得易于计算的数学模型,往往会造成我们对自然的扭曲理解以及错误期待:如果你在做实验之前就假定了一个你预想会出现的实验结果,你的实验过程将更难保证其科学性和准确性。作为人类,我们很难不被这种内心中的隐性期待误导。**计算器的使用以及长期对于这种非特殊值的数据处理,为自然科学中数学的应用打下了良好的基础。**科学的意义,正是在杂乱无章的世界中找寻规律。我们也许并不能和我们想象中那样在 “做题模式” 和“科研模式”中自由切换 – 长期只处理特殊数字(整数,最简分数等)的局限性,造成了杏仁体内的 associative learning—就像中考时的我,脑子里根深蒂固的是非整数与失分的错误隐性链接,在看到这种 “不完美数” 的时候自动激活的就是恐惧反应,这样的错误链接(也就是偏见,或者说刻板印象),在科研领域无异于毒药

专业数学,则属于一种不同的情况。专业的数学更加侧重的是数论,追求的是建立纯数学的纯理性关系式,探究的是数学本身,而不是数学在其他领域的运用。在这种情况下,精确的,无误差的值则变为了首要目标—代数作为一种纯数学模型,抹去了具体数值的限制以专注于数学关系的探究,正是纯数学模型的一种最好体现

四. 执着于完美数据可能关乎人命

过度理论化的数学根深蒂固后可能带来哪些不良后果呢?因为个人的学术兴趣主要在于医药科研,下面我举的这个例子就是 “沉迷理论而不注重现实” 这一现象在医药行业可能造成的可悲后果。

新药物的推出或者上市,通常起源于在实验皿或者大鼠。然而,药物机理在实验室的细胞上或者在大鼠身上发挥的作用,并不一定和在人类身上时一模一样。要是一样的话,那么药物动力学这个高深学科的存在就成了一个哲学问题。一种药物在实验室检验后,还要通过在人类志愿者身上试验,确认无害后才能推向市场。这是一个劳命伤财,耗费金钱时间人力物力的巨大项目。想象一下你在做完了漫长的实验室阶段以后,终于拿到了审批,可以在人类志愿者身上试验了。结果,你发现这种药物他妈居然没有用?甚至有的志愿者死掉了?这咋整??我们在实验室的细菌上说好的能够有效减轻症状的呢??

那他妈一定是我记录错了数据,数学公式摆在那里,完美的挑不出一点儿差错,数学万岁,生物物理万岁,公式万岁,一定是试验错了

于是有些(其实是很多)丧尽天良的药物试验人员(业界耻辱,大部分高风亮节的人(比如我)完全不能比)就(有时候在医药公司的威逼利诱下)转向了篡改实验数据,只保留能偶证明这种药物有正面疗效的实验数据,抹去那些不支持他自己假设的实验数据—那些一定是记录的错误或者是偶然情况;或者把年老体弱的受试体从样本中排除,只研究药物对青壮年的效果;还有的试验对中途退出试验的受试体数据不予考虑,对这些中途退出药物试验的人是不是因为副反应太强甚至死了漠不关心—然后拿着完美的试验报告喜滋滋的发表,然后药物被推广上市,然后毫不知情的患者惨死于副作用(那场非常经典的悲剧:用于治疗心血管疾病的药物导致了突发心脏病,数以万计的病人因此丧命)。

过去,全世界只有 17% 的药物试验是在非美国的地区进行的,然而,在经济利益的吸引和医药行业的影响下,现在这些在美国以外进行的药物试验数量已经超过了半数。药物试验数量在印度的年增长率时 20%,在中国是 47%。但是由于药物监管力度美国和英国则分别开始年均 6%,年均 10% 的药物试验数量下降。主流学界对被隐藏起来药物副作用以及数据的真实,完整性表示担忧。1998 年一项对于中国医药的荟萃研究发现,中国医药科研杂志上没有任何,哪怕一项药物试验的结果是说明该药物的副反应大于正反应,从而不应该被推向市场

真的是因为所有的药物都这么完美吗?这是一个值得深思的问题。

世界真的和数学 / 物理考试中的模型一样美好吗?这是一个值得深思的问题。

那些在实际药物试验中原本证明对人体副作用大于正作用而却被推向市场的药物,现在到了谁的手上?这是一个很值得深思的问题。

然而,事情一般都不是非黑即白的,我也不是崇洋媚外的。作为一个中国人,我时刻为我们民族世界闻名的出色数学能力感到骄傲。心算口算速算能力对数学应用的灵活性至关重要,对数学以及物理公式的背诵则对帮助概念的理解不可或缺。偏执于哪一侧都会有相应的缺点,在这个连续体中找一个合适的值似乎才是最好的做法。具体应该怎么操作,这个问题留给教育界的朋友们研究,我还是回去安安静静的继续研究脑子吧。

:)

知乎用户 速冻皮 发表

这个水平指的是普遍的。并不意味着美国缺乏数学方面的尖端人才。

与中国不一样的地方在于

如果这个美国孩子对数学感兴趣,喜欢数学,那么他将得到支持,钻研数学以此来证明自己的天赋或者收获成就感。

如果一个中国孩子对数学感兴趣,喜欢数学,只喜欢数学,那么他每天都会挣扎在自己的兴趣特长与 “你看你英语怎么学的啊!”“你分数学一半给语文行么!”“数学好怎么了!高考就考数学么?”“你数学这么好,就少浪费些精力在数学上,有时间多背背英语单词!” 等等老师家长的口水之中。

所以如果说美国的教育是取长的话,那么中国教育大概是补短。

知乎用户 抠脚袒胸老宫酱​ 发表

因为中小学的数学只能称作是算术,算术学的好不好跟数学没关系。

中小学数学学这么好又有什么用呢,即使大学选了理工科,工厂工地二选一。

中国汽车工人数学好又有什么用呢,

美国汽车工人人均 12 万美元年薪,中国汽车工人两班倒一年十来万人民币。

北大数学黄金一地啊四个大佬全在美帝,许晨阳国内干了几年又归海。

美国数学比中国强很奇怪么

知乎用户 王若枫 发表

为什么美国中小学生学的数学比我们简单却还能做出很牛逼的东西?

因为很牛逼的东西不是美国中小学生靠课本的数学知识做出来的。

人家中小学数学不复杂,大学知识的复杂程度可比我们有过之而无不及。

知乎用户 GloriousShadow 发表

1、教育的真正作用是 “提供通往不同领域、不同阶层的梯子”,而不是 “育才”:

人才是 “发掘” 出来的,而不是 “培养” 出来的。

种瓜得瓜、种豆得豆,豆再 “培养” 也变不成瓜,然而 “豆有豆的可取之处,瓜有瓜的可取之处,让不同特质的人自由选择符合他们特质的道路” 这才是教育的真谛。

2、古今中外,“资本” 都是决定性的力量:

干啥不要本钱?有了本钱,才有余力琢磨 “日常用度” 以外的东西,才有资格谈“发展”。

重赏之下,何愁 “没有勇夫”?

美国什么时候开始 “执全世界牛耳”?发完战争财以后,而不是 “校园扩建、扩招以后”。

“资本”是国家腾飞的基石——一个国家,假如长期处于 “资本力量薄弱” 甚至 “反复打压资本” 的状态,往往源于“对经济规律的非理性认知”,这种国家不可能形成什么“竞争力”。

没有大财团,人民生活会更好吗?恐怕,那时候,光是 “生活” 两个字就已经足够奢侈了。

知乎用户 万象肥朝 发表

| 时间有早晚,谁在前面走,谁在后面走,谁都明白。都说第一个吃螃蟹的人是勇敢的人,勇敢的人会面临困难,同时会是利益的最大获得者。中国从一穷二白开始,我们能打破一个旧中国,同时必须善于建造一个新中国。教育从农民群众集资建校开始,到现在有很好师资校园建设,还有很多学校一个班 40、50、60 人不等,全国考试多考一分,就可能超越千人,各种情形,参差不齐,不拼不行。生活中每一种模式的形成,不是哪一个人,或者哪一只巨大的手能轻易改变的,教育亦然。同一个学校甚或同一个班级,相处三五年,学习好的,也许会更好,差生依然差,这能怪谁,没有对错,也不必怪谁的谁,只不过如此,老天安排,循路而行。

知乎用户 胡丢丢 发表

简单的一个比较,美国 GDP 约为中国的两倍,但是美国人口只占中国的 1/4,中美国土面积相当。

换算知: 美国人均 GDP 大约是中国的 6–8 倍,人均国土占有面积大约是中国的 4 倍。

加上身为全球霸主的各种 buff,美国的精英教育投入是你难以想象的。这也是美国为什么身为资本主义国家但是高福利社会待遇高于社会主义。更多的资本加高了壁垒也创造了技术和教育压制,强者愈强,弱者愈弱。

历史经验告诉我们,投入高产出大。妄想靠压榨自己用别人 1/4 之一的投入产出别人 2 倍以上的成果是行不通的。

知乎用户 茶哥反鸡娃​ 发表

雇印度人华人去做基础运算,拿电脑完成任务就好了。

不想展开说教育的区别了。

说一个别的,如果你的工作可以轻松被一个电脑替代,或者一个教育水平很低的人替代,就别得意洋洋了。

知乎用户 执悲今厄 发表

你认为,所有美国人接受『公平』的教育,即,美国政府规定一条明确的教育之路、所有学生都按照这条路统一前进,并且美国中小学数学非常简单。

你错了。你看到中国是这样,并且你没见过不是这样的,于是你认为世界都是这样,但你的推理不成立。

如果美国人接受『公平』的教育、却中小学数学简单而精英人才多,那么那不合理。

如果美国人不接受『公平』的教育、且中小学数学简单而精英人才多,那么那合理。

知乎用户 湖海散人​ 发表

其实挺好回答的,因为他们的科研主力军是从中国这样的国家过去得

知乎用户 Misha Gabria 发表

最近在自学编程。

之前没有任何基础,唯一在大学上过一学期的课(连 C 语言都不是的 B 姓课程,出来后才发现居然国内的重点大学如此落后!)当时就是各种算算算加背背背过的考试,考试还是笔试,连计算机都不碰的那种…

Anyway, 虽然和工作不搭界,但是能感受到现在编程语言的普及性,将来等我们这辈年纪大了,不会编程大概就和现在的老人不会玩电脑上网一样。

于是开始搜课程。很棒的是,老美已经做了很多类似于非盈利的编程学习网站,并且为美国本土的人提供社区学校课程。

我随手注册了一个,零基础课程,适合 3-18 岁的(虽然早过了年级)。用了几次以后,居然学习动力十足并且觉得不是很难。

没有死记硬背,没有枯燥的解释,甚至一开始连打字都没有!它所做的,是先训练你的逻辑思维。通过游戏(僵尸要去吃花,蜜蜂要采蜜酿蜂蜜,愤怒小鸟要找到野猪等),你要找到相关的模块(简单的前进,转弯,跳跃)拖入编程框来完成任务。慢慢地,难度增加,任务变成,前进 / 后退多少单位,转左 / 右多少度,重复行为,条件行为(如果… 则…)你做出来的东西,在屏幕上要顺利运行,你必须像玩乐高一样,把一块块散件功能拼凑起来。

让你学的很有成就感的部分就是,你觉得你在完成一件作品,你很清晰每一步是为什么。就好比你学钢琴,虽然手还不熟练,但能弹出好听的自己想要的旋律了;而不是每天练和弦,到最后技艺精湛却没有灵魂。

这大概就是你觉得美国初级教育很简单的原因。它重在兴趣和可能性,重在培养一种使用工具的思维,而非成为工具。

到后来用代码画画部分,涉及到角度和线长的计算部分,有一段视频让我印象深刻:

如果我要画一个长是宽 2 倍的长方形,我可以设置 “重复 2 次(前进 50 右转 90 度,前进 100 右转 90 度)”,但是,为什么不把 50 部分设制成“边长”,“100” 部分设置成 “边长 x2” 呢?虽然 50X2 很简单,但计算对于计算机来说太容易了, 为什么不留给它去做呢(the computer is so good at it, why just let it do the math for you?)

然后的练习里,比如无论画 N 边形,周长都是固定的 A,那么编程模版里就拖入一条 “设置边长为 A/N”; 比如一个 N 变形要重复很多次来完成一个圆形,那么每个移动的角度就是 “360/N”。

之后跳出各种奇怪的图案来让你自己完成代码模版里的格子组合,在玩得很有乐趣的同时,你在完成一次次编程的构建。

是的,相对于以前认为代码是一行行写的,现在学到的方式简直是 3D 式的。课程训练的是你得到一个题目后的构建能力,逻辑思维,简化能力,以及如何应用各种工具。

我是基础入门者,这个课也很明显是给初级教育群体的。学到目前为止觉得非常简单有趣,你让我去写代码,我恐怕一行完整的代码都还写不出来(因为我只学会拖模块啊)。但是,对于一个实际应用问题而言,我已经开始有如何拆解 / 组合 / 简化 的惯性思维了,并且非常清晰。

我不能说如果继续坚持下去我会成为编程高手,但是我越来越明白,为什么美国的 coding 人员如此牛逼,为什么各行业牛人云集:

他们的教育方式是:设计和创造一个 Machine 的视野和思维, 简化,并利用可利用的一切现成的工具。而不是努力成为 Machine 里一颗质量不错的螺丝钉。

知乎用户 蜗牛跑的比我快 发表

一说到说中国的各个领域不出成果,就有专家和各种精英跳出来说是中小学的学习负担太重扼杀了学生的创造力。呵呵!敢情义务教育搞得好了也是原罪。一棵树长不高原来是怪它扎根扎的太深了。

一说到中国出不了顶尖人才,就有教育界的专家跳出来说是义务教育的锅,说的多了大家好像真的信了。但仔细想想又有哪里不太对,这些能称为专家的人都是搞高等教育的,他们掌握着话语权。广大中小学教师只好一边给学生批改作业一边背上扼杀学生创造力与想象力的黑锅。

科研出不了成果,我们能理解。主要还是国家底子太薄,高等教育投入不足。那你们努力把高等教育搞好啊!为了甩锅你们这些搞高等教育的众口一词的指责义务教育干什么。为了自己的脸面去刨教育的根基。拜托了,咱们国家教育就剩下义务教育还搞得不错了。你说基础教育阶段失败,那那么多的在中国读完小学初中高中甚至大学研究生以后出国留学的为什么能成才?

甩锅一时爽了,等整个教育体系一块烂掉了,又会有专家跳出来说这是因为基础教育阶段管的太松导致学生基础差造成的后果。

反正话语权在手,反正官字两张口。

知乎用户 低调雾雨霏霏​​ 发表

其实这是一个非常错误的观点。

之前有幸参加过北美某世界前一百学校的交换,上过一些课,个人感觉在北美本科生的数学水平还是比中国学生差的,基本上本科一年级的内容还属于中国高中内容。

后来又参观访问了部分澳洲学校,发现他们那边的教授(澳洲跟英国很像,教授在那边非常难以取得)纯正的美国人或澳洲人非常有限,大部分来自中国(包括香港、台湾),其他地区华人(新加坡、马来西亚),伊朗(中东地区独苗苗),南亚大陆(印巴孟)以及前苏联地区,核心原因估计就是因为基础教育上的差距。

之所以美国能够做出世界领先的科技,我认为核心原因是因为美国强大的吸引力。美国自己教育水平很差,但愣是靠无战乱、和平、繁荣让大批优秀的科学家前赴后继的移居美国做出高端科技,比如爱因斯坦、钱学森、杨振宁、李政道等等。直到现在,美国也是清华北大学生首选的留学目的地,仍然对于中国最尖端的人才有这巨大的吸引力。

尤其在信息学领域,美国除了吸引了大量的优秀人才以外还有最为完善的工业环境,人才提出想法以后就能很快变现。即使东欧有更好的方案,但受制于产业能力不足,最后还是老美方案占了上风。这样在外人看来就好像是美国人搞出来的。

还有一个值得注意的问题,北美大学相对于中国大学来说在课程教学上更加扎实,这在一定程度上让他们弯道超车。具体而言,中国许多大学在本科生教学上用心程度其实还不够,许多专业课程的老师念念 PPT 就了事,课后作业布置了就完事,基本不讲解作业中集中出现的错题(甚至作业批个 “阅” 字就完事),更别说根据学生水平调整教学计划了。学生平时不做,但只要最后考试完成了抄一下别人都可以。

然而,北美大学,特别是稍微好点的大学就不是这样。

每次作业都有截止时间,无论任何原因晚交就扣分,积累一定次数就取消考试资格直接挂。一旦发现抄袭,这个学生基本就完了。大部分专业课除了正常上课以外,还有专门的助教上习题课、实验课,作业肯定全部批改,错题集中的时候老师还会在正常上课、习题课中予以讲解。细致一点的老师甚至还会统计每次作业中每道题的得分,让大家各自都熟悉自己掌握水平。

知乎用户 还是不注名好 发表

因为美国有钱,有武器,谁做出了超级牛的东西,拿钱买就行了。你不卖?上航母,重新说一遍,你卖还是不卖?

例子有:辉瑞疫苗 (德国),deepmind(英国),阿尔斯通(法国)、东芝(日本)、三星(韩国)、ASML(荷兰),台积电

这些东西都不是美国的,但美国都对他们有绝对的控制权。超级牛的东西,谁做出来,不都是一样的么?不都是美国的吗?

知乎用户 Heterogen 发表

这种偏颇的认识大概源于自身所处环境的蒙蔽吧…

帮 institute 组织过美国高中生的 Science Fair,当过评委,看这帮小孩儿写成的数学 paper 里多繁难的推证都有。有些数学素养格外好,能把思路表述的特别明晰。他们每个人的发展节奏非常自由形态,而且不设上限。国内中学的数学教育只是被那几次统考高度统一了下儿步调罢了。

以及,做系统跟数学关系不大的吧。

上了大学后一起跟美国佬泡实验室发现,论及一些美国娃们的实操、实验设计能力,还真不是盖的。

顺便跟着一起吐槽下国内编的高数教材…. 比较之下,简直了。

知乎用户 倪伟 发表

那些超级牛的东西本来也不是美国的普通中小学生做出来的呀!

美国的顶尖中学生也是很玩命很优秀的,而且到大学研究生阶段还有世界各地的优秀的学生加入。

知乎用户 晨沐云 发表

科研是站在巨人肩膀上的登高远眺,美国的巨人很高,所以可以看的更远,中国人还在建造巨人… 你说的是巨人头顶的风景,我们还在打造巨人,所以看不见…

知乎用户 Tony 发表

因为我们大部分人的知识巅峰期在高三?

在本科 下面这些也是常态

不考不学

不好找工作不学

不好拿高分的课不学

那么最后到金字塔尖去做研究的时候 我们并没有什么特别的优势 还是各国极少数热爱那个领域的人在比拼

具体到问题中的美国 如果一个美国学生热爱一个领域去钻研学习 那么他能够获得的教育资源是很丰富的(比如高水平的教授 导师) 和中国学生比可能还有优势

知乎用户 大忽悠 发表

中美教育差距在于

中国人能培养出一百个学霸的同时,美国人能培养出一个一两个学神和九十多个学渣。

在中国,社会分级是在你高考结束、乃至大学毕业之后才开始,而美国则是在你一出生就开始

知乎用户 赛博铁犁 发表

因为美国一小部分做出超级牛的东西的人是超级天才,生在中国美国都可以。剩下的,读的私立学校,比中国学的难多了。

所谓的 “简单,快乐” 的基础教育。

无非是某些团体,希望洗脑中国民众,永久性的固化阶层罢了。

如果你是富二代,那确实会希望中国变成美国。

如果你是普通人,学美国的话,你的孩子小时候大概率崇拜运动员,打爱学习的人或者因为喜欢学习被打。长大了喝消毒水。

知乎用户 phk189 发表

你的前提就是错的。

我朋友在明尼苏达大学工作,她儿子在明大附近公立中学上 6 年级。孩子数学极好,AMC 获奖,明大专门给这样的孩子提供大学数学的课程,我朋友说第一天她儿子从明大数学班回来,拿着一套 calculus 的教材,宛如回到了我们自己的大学时代,翻翻书,就是我们大学学的内容。自嘲搞不定了,要辅导的话要她老公,一个物理系博士后出马。

而且美国学校从小就有天才班。

人家优秀的孩子获得的教育远远优于普通孩子。科研成果不还是靠优秀的人做出来吗?

再说,你自己拿本美国初中教材,和中国的做对比,同等时期学的内容并无差别。国内喜欢题海战术,做的练习题多些倒是可能,各种变态的变形多些是有可能的,其他的知识点并无太大差异。

所以,别再相信道听途说的东西。

知乎用户 倍思校库网 发表

不管喜不喜欢数学,当今中国的现实是,数学成绩好不好,对于升学、高考,甚至留学,至关重要;原因很简单-数学特别能拉开考分差距。不信你去看,文科高考成绩好的人,数学成绩一定很不错。

为什么我们的中小学数学这样难,而能利用学到的数学知识进行创造发明的那样少?为什么孩子们耗费 12 年学数学,最后绝大多数人除了简单加减乘除以外的东西都忘光了?

而在美国,初中生学的数学是咱们国家小学生学的水平,高中生学的数学是咱们国家初中的水平。可人家就能做出很牛的东西!我们的差距在哪里?

长久以来,中国人的迷思就是,为何 “美国人数学这么差,还能出这么多牛逼科学家?” 这个问题的答案已经被答烂了,我结合自身的经历,系统地给大家科普一下!

美国给不爱数学的学生 提供基础数学教育

美国的每个地区,对于高中毕业的学生,应该有何种的数学修养,大多是有硬性规定的。

比如麻省,每年对特定几个年级的学生有统考,这个统考的主要内容之一就是数学,数学不过关的,高中是不允许你毕业的。

所以基本上每间正规高中,也都对数学水平有最基本的要求才准毕业。据我所知,这个标准大概在会运算简单的三角函数就可以了。

这个水平,以咱们中国的标准来看,确实不算高,而且很多人还都是勉强混过去的(高中课很松)。

这种低要求的直接结果就是,美帝 99% 的学生(99% 这个数字并不夸张,可能更高)的数学都停留在生活勉强可以自理的水平上。

给热爱数学的学生 最好水平的数学教育

以高中为例,对于一些数学比较好的不安分分子,为了安抚他们,不给社会添麻烦,学校(不是每间学校都有这个条件)会提供 Advanced Placement 课程,也就是宅男们喜闻乐见的 AP 课程。

以数学为例,高中最高级的 AP 课程,叫做 BC 微积分,2005 年我上高中的时候,课本是下面这个:

我们的任务则是把这本书,从头到尾学得通通透透。

值得注意的是,只要你前面的课程成绩都好,你几年级修这个课是没有限制的,我上学时班上最年轻的同学是一位俄裔美国人,他修此门课程时才上十年级(相当于我们高一),他最后这门课的成绩是 A+ ,在我们学校,意味着每次考试的成绩,都在 95 分以上,实在是学校公害。(这位大哥后来去了宾夕法尼亚大学读 Material Science —材料科学)

这本书的内容,大家都可以查到,人家高一就学这个,你们各位自诩为学霸的,颤抖了吗?基本上到学矢量微积分之前,学完了这本书,你做微积分已经应如四则运算般自如。

我作为一名合格的宅男,还选修了 AP 物理,我们的课本是这个:

有兴趣的同学,也可以查查这本书的深度如何。

教材不是统一的,AP 的任课老师可根据自己喜好选择教材。另外老师会推荐一些课外读物,供不安分的宅男们消耗能量。

AP 课修了一整年以后,就可以报考全国的 AP 统考了。AP Exam 统考的难度,个人觉得数学比较简单,物理则很难,当然也可能因为我抽象思维能力太差了。即便这样,小弟我全考了个 5 分也是没有压力的(满分 5 分)。

说到这就差不多了,中心思想和开始提到的一样,可能论数学物理的平均水平,美国学生确实远远不如中国学生,但是谈到「给天才 / 有兴趣的人的教育」时,中国的教育制度,则还停留在解放前。

而到了大学,这个差距就越拉越大,到了研究生阶段,没有鄙视国内读研读博的盆友们的意思,但与美国的 “高高等” 教育相比,真的无法相提并论。国内的院校、资金条件、师资条件和科研硬件条件,都比发达国家相差太远了。

这样的制度,从侧面看,很大一个好处就是,年轻人可以把多余的时间和精力,专注到自己喜欢做的事情上。

很多比较聪明的宅男,高中最后一年基本没有什么数理课程了(全提前学完了),学学哲学,历史,艺术,玩玩乐团,体育,对成长都有帮助。前面那位俄裔学霸,还是该地区的长跑冠军,真是令人厌恶。

另外一个好处就是,学霸们可以和学霸们在一起玩一起上课一起耍,不要小看这些宅男们,学霸们聚在一起的能量们是很大的,你没发现,在你们现在打工的地方,老板们年轻时都是学霸吗?

我们的小学数学和 美国小学数学的差别

说起来,我们想到的可能会是那个经典 “一个水池,进水出水,以什么样的速度来保证水池怎样怎样之类的” 大概这些吧,也就是我们所说的应用题。

或是前段时间在微博上看到的一个小女孩背乘法口诀表里面的 3X5=15,背到哭了。真是让人觉得小萝莉可爱的同时也不得不说,都是过来人,看谁笑话呢。哈哈。

不过,言归正传,我们所学的数学,很多都是通过套用公式来一遍又一遍的做题来证明公式的正确性。

可是美国小学的数学不一样。

他们通常都是学数学为了来解决问题吧,会套用在实际的生活中去学习。

比方说,今天学习数字 3。一个消防员站在一棵高高的树下,树上有两只小猫被困在上面下不来,消防员要去救小猫,怎么救。

首先选一下可以使用的工具,灭火器?捕蝴蝶的网?还是梯子?因为树高要用梯子,好,几个梯子才能够到?一个不行,两个,三个呢?大家大可不必去纠结这三个梯子拼起来怎么安全,不是消防车都是自动的等等问题。

我们的 “小明和小朋从两个地方出发,以同样的速度,走多多路,后面 balabala” 的情景这些,也未见得就不是童话故事,然后我们长大后发现童话里都是骗人的,因为没人会这样无聊到去做这事。

再引用一个朋友孩子的例子。他儿子上 3 年级,以我们的眼光粗粗来看,他们学的是:

1.10 以内的加减法

2.1000 以内数字的读写

这是什么级别?好像现在幼儿园中班就已经教这个了 !

但我朋友花了 3 小时教材,越看越汗,还特地总结了一下他们教的我们没教的:

1. 同样从 1 数到 10 再 11,我们是单纯的数,数到 10,100,1000;但他们数到 11 后,开始讲进位、十进制,开始引入二进制、五进制等;
2. 讲 10 以内的数字,区分数字的用途,同样是数字 3,可以是 3 个房间,第 3 间房,房间长 3 米,他们有什么不同?
3. 在数字的用途一节后需要写 paper:我们说华盛顿有人口 3,454,456(数字是我随便编的),这个数字是精确的嘛?还是估算的?为什么?怎么证明?
4. 有一些逻辑题目,类似于 “教授的隔壁是医生,医生喜欢蓝色” 这种问题,我们孩子是从小作为智力题目来做的,他们则介绍了只需要用到 1-10 这几个数字的表格,介绍如何将这些条件填入表格,最后如何出答案。

其实还有更多……

总体感觉这个教材:

1. 强调数学的基本概念

2. 强调逻辑思维

同样是逻辑问题,我们一直强调的是用脑子解决,会的就真是个聪明的孩子,不会就你怎么这么笨。而他们从小就拿出来,教孩子用工具解决。

国人一向号称自己是最聪明的民族,但**创新实力却很弱,**我相信我观察到的这一点是原因之一。

3. 细致

我们长大后,看老外的文档,事无巨细,是不是特别佩服?可为什么我们从来没有耐心看下去,写出来呢?

我想,这就是原因,我们都没有耐心为孩子细致的解剖这个世界,孩子长大后怎么能细心?

总之,美国的教育更注重的是在大学前开阔视野,找你所喜欢的、感兴趣的领域和学科,所以他们一直到高中都有很多自己选修的课程,很多很多课,看似很轻松,学自己喜欢的嘛。

而当我们经历过那不想回去的高考之后,全都瞬间解压,觉得没什么事做了,开始玩乐。

但美国大学生却非常辛苦,学习努力,这不光是因为他们的大学不好毕业,而是,大学时期其实才是真正学习的好时光,术业专攻,之前都在寻找培养兴趣,拓展视野,找到自己的兴趣所在。

所以,可见的,他们的数学简单,其实不是简单。而我们学过的很多数学公式,现在生活中也都不会用到了。

所以说 “美国学生学的数学比我们简单” 这就是个伪命题,不存在这样的事情。

原文自留学字典(本文转载网络,作者、原始出处不详,信息仅供分享。)

知乎用户 悉尼煮夫 发表

认为美国数学简单,这可能是对美国的教育制度不是很了解。我不在美国在澳洲,我以澳洲为例,通过讲解澳洲的教育模式,来解读美国为什么数学看起来简单,但是却能取得研究成果,因为澳洲和美国的教育理念及方法是非常类似的。

1. 公立教育分为普通教育和精英教育。

新州的学校构成

(以上信息总结自 Wikipedia:Education in Australia)

澳大利亚的新州,从 Y5 开始进行精英教育,直到高中毕业 Y12. 基础教育是向每个人提供并为一切人所共有的最低限度的知识、观点、社会准则和经验的教育。但是精英教育,目标就是之上超群的天才。天才的定义,是什么?

新州教育部对于 OC 和精英高中的描述是这样的:

Opportunity classes and selective high schools form part of the NSW Department of Education’s commitment to identify and provide a range of opportunities for gifted and talented students in NSW.

简单来讲,OC 和精英高中就是为了天才学生设立的。

(以上信息来自 NSW Department of Education)

In line with the ability classification provided with the WISC-V IQ test, selective high school and opportunity class placement is most appropriate for those students classified as having very high to extremely high academic merit.,

(以上信息来自 NSW Department of Education)

意思是:精英教育,只取 6.7%+2.2%=8.9% 这部分的天才孩子。实际上,在新州只有 6% 的学生能够进入精英中学接受精英教育。

2. 教育是进阶制的。

澳洲的教育,是进阶制度,也就是从低年级就开始接触数学的各个分支,比如代数,分数,平面几何,立体几何等等。三年级开始学习分数,只学最基本的分数的意义,到了高中,仍然学习分数,开始学习约分,乘除法等复杂的应用。所以,在高中阶段看的的知识内容,是广而杂的。但是,难度是逐步加深的。很可能你看到的某些内容,中国在初中就学过的,但是在澳洲,由于循序渐进式的教育方法,会让人在某些阶段觉得学习的内容很简单。

3. 精英教育的难度,真不简单

我曾亲眼在图书馆见过,旁边精英中学的高中生,作业里包括微积分和导数等内容。着实吃惊,甚至默默在其身后驻足一会观看。这些内容我是在大学才学习的,这里,高中就会教授。

4. 精英接受的教育,能力和应用大于知识和习题。这可能更接近或者说本身,就是我们口号一直在喊的素质教育

以下内容节选自某论坛一位家长的文章,该家长女儿在悉尼著名的悉尼女子高中 Sydney Girl High School, 是悉尼一所老牌精英女校。

个人认为,孩子在求学阶段,要广泛接触各种活动,以便开拓眼界并且有机会尝试并且找到真正的兴趣所在。我们在小学已经参加过公立系统的大部分活动,但还是有很多小学阶段没有接触到的活动,这时候要参加这些活动,起步就比别的同学低了,有的 selective 的活动可能就轮不上了。
我们的小学学校小,没有辩论队,也没有达芬奇和脑力竞赛,辩论 7 年级时参加的人很多,老师可能会偏爱以前有经验的人。女儿到 8 年级才努力入选年级队。达芬奇(Da Vinci Decathlon) 因为学业要求,8 年级也入选了。脑力竞赛(Tournament of Minds) 是不分年级的,一开始没有能参与,以后只能参加临时组队的比赛了。7 年级开学时,最好 “削尖脑袋” 打听,积极报名,否则懵懂中会错过机会。如果一开始错过了,又很有兴趣,后来想要加入的话一方面自己要学习相关知识技能,另一方面要让负责的老师知道你的兴趣。
华人来讲,演讲和辩论普遍是弱项,我女儿也不是那种天生能说会道的,我觉得中学阶段能得到一定的锻炼,对于今后在洋人圈子里活动,职场突破天花板会是很有用的。而且因为参加比赛和相关的活动(比如少年议会,青年议会等),能结识别的精英中学和私校的精英分子,能取长补短,很有益处。
学校的音乐艺术教育很强,乐队的活动也很重要。一般华人孩子学钢琴,这时候可以选一个便携的乐器以便加入乐队。sgh 经常会有音乐奖学金提供给学习冷门乐器的新同学,不要错过。我们当时选了圆号和打击乐,结果给了圆号奖学金,但没有学下去;打击乐倒一直玩着。学校有老师上门教授乐器一对一课,但会占用主课时间,如果选择在学校学习乐器需要自觉安排补课。
如果不加入乐队,还可以参加合唱团,舞蹈团,剧团,值得注意的是,这些活动都需要自己报名,参加选拔,也需要合理安排时间,下面会讲到自我管理的问题再详细讲。
大部分的兴趣活动在课前课后,周末会有体育比赛,最有挑战的是划船。我知道南区家长每周三天 4 点起床送孩子去训练,不过家长们也成立自己的划船队,也玩得欢。正因为划船队是巨大挑战,藤校队员划船队员很是偏爱。我还知道一个小窍门,不爱运动的孩子也能参加划船队,可以做那个喊号子的工作。论坛小莲家的有才做击剑运动裁判也是异曲同工吧。
所有这些活动,sgh 的传统是家长出大力组织维护,相比较而言,华人家长参与度同学生的占比很不相称,真心希望多一点华人家长参与活动。
澳洲中学课程设置很有意思,有网友曾经指出,中国力在灌输知识,澳洲讲究实用,有一定的道理。我的理解,公立教育在于培养有一定知识技能的合格公民,同时分流部分学生参与职业培训,筛选出精英接受高等教育。后两个目的应该说公立系统还是不错的,但是,合格公民显然比例不算最高,数量上有很大的提升空间,质量上也差距较大。不过,这个同我们华人大致没关系,同本主题更不相干,是题外话。
7-8 年级是过度期,主要让孩子适应小学到中学的过度,开课的内容非常广泛,煮饭做裁缝做木工都学。男校女校可能开设的课程略有不同,但在 sgh 是有机会学到木工,烹饪,缝纫,IT 这四种 TAS(Technology and Applied Studies) 课程的。八年级还用戏剧表演替代了一节英语课,帮助学生了解戏剧的同时让学生体验表演课,以便决定要不要选修。不过即使不要选修,每年英语课学莎士比亚时还是需要表演短剧的。这个很符合多元智能的理论,精英中学的大部分孩子必修阶段过了就不再继续学这些了,但是,hsc 也是有相关科目的(同样,不同学校开设的 hsc 科目也是不同,擅长的科目也不同),这意味着动手能力强的孩子能凭这些课考高分呢。
轻松涉猎各个学科让学生找到兴趣也是初中阶段课程设置的目的之一。遗憾的是,由于时间限制和公立学校的资源限制,孩子们可选的课程却很有限,这时候,家长的阅历和经验能够帮助孩子取舍。
我的观点是,因为时间有限,学校主要学自学不容易的课程和需要积累的课程。这里面首选外语课。十几岁学外语同双语养育不一样了,系统学习需要专业指点,语言学习需要环境氛围,所以,在学校
选修语言课
很合算。sgh 共有四种外语可供选择,中文,日语,法语,拉丁语。7 年级必须要学两门外语(七年级的分班基于外语的选择),8 年级如果再选两个外语的话,一个在课表时间内,另一个需要在早上和中午的课外时间学习;9,10 年级外语就是选修课,挺大一部分学生 9,10 年级依然学习两门外语。如果要学外语的话,7 年级就要选好,再不行 8 年级也要选定了,因为语言是要积累的,前面没学,后来再要学就赶不上了。
9,10 年级的选修课,如果孩子已经有强烈兴趣,选自己感兴趣的。如果像我们一样,兴趣广泛,则还是选自学不容易学的(我们选了音乐,拉丁语,法语)。比如历史课,必修有的,思路都一样,选修的深入一些,如非必要自己看书研究一样达到目的。
个人很喜欢贯穿 12 年的 pdhpe 课,帮助孩子们认识自己,找到个人在组织,群体,社会,自然中的位置。包括香蕉课和有关性病的研究,在同伴中以开放的态度没有忌讳地学习性知识,都会让女孩子们受益。
sgh 很强调自我管理,对自己身体和情绪的管理,也包括对时间和资源的管理。学校的各种公益活动也提供安全的探索空间,帮助孩子们建立思辨能力。

由于引用部分内容较多,我把重点部分标注黑体,便于直接找重点。

通过这个家长的描述可以看到在精英中学,学生能够参与的活动之丰富,同中国的高中不可同日而语。中国的高中,绝大部分时间,都用来准备高考。不能否认澳洲也有书呆子,但是,书呆子在澳洲会被同学看不起的。

结论:

无论澳洲,还是美国:

  • 精英学生可以被筛选出来,接受精英教育;
  • 学生学习的知识绝不简单;
  • 教育的方式更接近或者代表了素质教育;

因此,单从基础教育部分而言,其精英学生的知识及能力,都很强。

至于高等教育部分,不在本文讨论范围,不予置评。

本文部分观点和内容来自公众号 悉尼煮夫

知乎用户 鹏鹏​ 发表

1 美国大学,尤其是名牌大学学习压力不必中国高中低,其学术水平很高。

2 美国吸引全球优秀人才为之服务,搞数理化的有很多是亚裔。

知乎用户 大卫 发表

因为世界各国数学教育出来的精英,被高薪吸引到美国去啦。

本国公立教育出来的,呐,就是红脖子呢。

知乎用户 Xi Yang 发表

因为尖端创新直接依赖的是少数精英,而不是平均水平。美国只是平均水平差,但是人家学制相对灵活,学霸可以在很小的时候就接触高级数学工具。

国内大概中科大少年班可以破之。

知乎用户 QED 发表

更新了几个论点:

  1. 首先是美国的课程和教授的教学水平怎么样?这个问题要从两面来看待:美国的基础课程_非常简单_,水平很低,主要原因我觉得是学生太差所以稍微难点的就听不懂。而且美国学生特别爱自信装逼然后稍有不顺心反手就给 teaching staff 一个举报。进阶课程开设的是不错的,但是由于大量学生水平实在不咋地所以最后只能以**放水收场**。比如说我当年作为非专业人士学研究生级别的 ML,竟然还拿了 A,期末考试 mean&medium 都不到 60%,我能比 mean 高出一个半 SD(按正态来算比 87% 的人要强),和我一起去学的哥们直接超了俩 SD 接近最高分了。这还是期中之后一堆菜鸡退课了的结果。我这里强调的是这门课非常不错,但是学生实在太差。最后他们也没办法从中学到太多东西,混个成绩混个学位罢了。
  2. 存不存在厉害的美国人?存在太少。举个例子的话,我们组十几个学生 / postdoc,统共四个美国人,包含一个拉美裔铁废物,一个非常左的女生水平也很一般,剩下两个水平凑活但是也就组里平均的样子。华人(包括湾湾)算上刚刚毕业加起来的有七个,五个的水平都远强于这几个美国人,剩下俩刚来我也说不好什么水平。然而从我们组出去的两个美国人(分别是 15-17 年的博后和 04-10 年的博士)很强。虽然说相比于我们组毕业的人数来说出货率不是很高但仍然说明存在厉害的美国人的。
  3. 这个现象一直有吗?其实也不是。最近比较明显而已。就比如说我们老板最近的美国学生都很菜,5-10 年前的就有点实力一样。美国教育水平是逐步拉垮的。不然我们领域也不会有好几个纯正美国人先驱 (比如 Norman Ramsey,David Wineland 等)。评论区有说是教育改革失败了的问题,问题发生在 80 年代并一路存续变本加厉下来。我觉得这是有一定道理的但是我也不是历史学家就不多评论了。

原答案:

好多答案都在尬吹美国精英教育,看的我脊背发凉这就是意淫家嘛?

quantum engineer 的答案实际上和我本人的观察最为接近,

为什么美国中小学生学的数学比我们简单,美国人却还能做出超级牛的东西?

最重要的就是美国吸收别国猛人来壮自己的科研水平,而且这个传统已经很久了。

在这里补充个其它两个论点吧。一个是我自己在 MIT 做 TA 的经历,另一个是我的小学弟从北大 transfer 到 MIT 的经历(某种程度上算是二手吧)。

我做 TA 的是一门研究生课,但是大部分研究生都是我们自己这个方向的,学习起来很有 motivation,大部分时候没有太多问题问我,作业也做得很好。而本科生就参差不齐了。最差的本科生每次作业都要给我发几封邮件,每推导一步就要给我发个照片问下一步怎么做。这种不放弃的努力精神虽然值得鼓励但是水平是在也是太差了。反正一个学期过去我感觉这个本科生的水平大概介于我高二到高三之间。还有一个倒是不太问我问题,但是作业基本上只有 20% 是靠谱的,后面就开始瞎写。一个正弦函数近似成线性然后整道题就开始胡扯八道,然后我指出来还义正言辞的说自己的近似贼合理(甭管最后反推回来是不是参量已经好几百个 pi 了)。这些就是 MIT 这种千里挑一的学校录取的美国本科生。

而我的学弟来的时候英语贼菜所以还蛮累得,但是很快就秒杀基本上所有他们这届的本科生了。他先来我们 lab 做了一个学期的实验,期间从 0 开始学了电路并改进了一个控制系统,也已投入使用了。后来对凝聚态理论感兴趣就去了别的组,一个学期下来已经有一作文章了。他的同学们除了和他一起从北大来 MIT 的本科生(俩人都是 IPhO 金牌),大部分都还在学课或者划水。

总体来说就是美国本土的顶尖学府本科生的平均水平是真的菜,学的内容也是真的水。除了靠特例之外,主要还是吸血。非常无奈的事实。

知乎用户 南民北居​ 发表

这是很久以前的问题了吧?

一个链接镇楼。我娃心中圣地,布朗克斯科技高中,7 个诺贝尔物理奖,1 个诺贝尔化学奖。

List of Nobel laureates by secondary school affiliation

https://hr2.wiki/wiki/List_of_Nobel_laureates_by_secondary_school_affiliation

作为一个近距离观察娃基础教育(坐标美国纽约)的家长,有很多感触,不请自来。但这个话题涉及内容太多,有时间再详细分析。这里先简单说一下。

1、问题两句话的逻辑很有意思。前半句说的是整体平均,后半句说的是极少数。所以 “却” 字的转折是不需要的。因为本来就是两件没有必然逻辑联系的事情。就如:我们村都不富,为什么村长那么有钱?

2、问题来了。美国做出数学超级牛的人,都是美国基础教育培养出来的吗?答案是不。美国是个移民国家。大学相当比例的教授都是国外移民。

3、即便如此,美国大部分数学大家是美国的基础教育模式下培养出来的。这种模式有利于培养数学牛人吗?

从某种角度来说,是的。美国基础教育其实是因材施教、分层教育。

4、美国基础教育中数学学的都比中国简单吗?

注意,我这个问题加了一个 “都” 字。既然数学大家不需要很多人,那么,一个国家数学基础教育就不需要 “都” 很难。

美国数学基础教育中,哪些人学的难呢?

公立系统中,美国有相当一部分基础教育很强的天才学校、天才班,磁力学校、宪章学校;私立系统中也有很多牛校。这些学校学的数学不简单,进度比其他学校快,高中很多学生学了不少大学预科课程。

更进一步,还有天才计划。美国约翰霍普金斯大学的天才计划,培养了二十万天才学生,这些学生 13 岁以前就已经在美国高考 SAT 取得了高于 1300 分或单科 700 分成绩。脸谱扎克伯格,谷歌布林、甚至 Lady Gaga(小时候作为极客天才参加该计划数学科学夏令营)都是这个计划校友。

所以,美国基础教育是因材施教。这种模式下,少数天才走特别通道。剩下的,也有公立学校九分校十分校的牛娃,其余大部分人放飞自我。

国人听说的,主要是后面部分。

美国这种模式,是现代教育学研究成果和美国宗教文化以及历史因素等多种因素复杂影响的结果。

前面这部分才是美国牛娃集中地。仅仅纽约市五所重点高中(纽约市共八所特殊高中,但三所是艺术类),历史上共培养出了五十多名诺贝尔科学类奖项的获得者和一名文学奖。

另外有位高赞回答中以 PISA 成绩作为中国基础教育质量好的论据,这个情况我曾经认真研究过,直说结论。不同意该答主。PISA 结果是样本刻意选择的结果。只需两点:请下载 PISA 官方各国样本说明文件详细阅读;请搜索国内研究某年中国 PISA 成绩下降原因的专业教育论文。你就明白了。

知乎用户 碎雪 发表

因为数学比我们简单的学校称为公立学校。

公立学校是给美国的穷人上学的。

这批人完成学业之后,将来是要做售货员。做汽车修理工,做洗车工之类体力劳动的。所以他们不需要学那么多知识。

而美国的有钱人精英阶层,实行的是精英教育。他们的教材和公立学校的完全不一样,师资力量更是天差地别,更加别提其他的设备和环境了。

而里面的学生,学习的时候,也非常的辛苦,甚至比我们的衡水中学学习的还要辛苦。

你看见他美国那些超牛的东西,就是这一群人发明和生产的。

知乎用户 落地雄心 发表

嗨,这事儿吧,任哪个国家也不能指着中小学生去做出来 “超级牛的东西” 吧。

其实这是一个非常大的话题,试着说两句吧。

首先,当然是美国已经是一个很长时间的发达国家了,在相当多的领域内有着丰富悠久的积累,不光是美国,很多其他的欧美发达国家都有着自己特别擅长的领域,也都能做出很多很多超级牛的东西。

而中国真正的发展其实也就是建国以后的事情,高速发展则是在改开以后了,说一穷二白完全不是夸张。

第二,是社会制度的刺激,其实严格来讲不能说是制度,而是美国整个社会的生存形态的刺激。

在这个社会里,对私有财产的保护,对法律的尊重,对个人权利和公权力的划分,做的都非常好,这样的话人的创造力会被极大地激发出来。

也就是 “一分耕耘一分收获”。

这不是美国的独特之处,世界上任何一个国家,只要这几点做得好,国家都会迎来很好的发展,反之则会把国家和民族推向深渊,没有例外。

说起来很简单,但是实际上能做到的国家不是很多。

第三,说到教育问题。

西方的学业安排更合理,小时候主要是长身体,让孩子们有一个快乐的童年,随着年龄增长,学业啊、压力啊一点点提高,等到了大学,二十郎当岁,正是精力、求知欲、脑力最旺盛的时候,这时候咱们老师啊、家长啊帮着一块给孩子参谋参谋,先找准人生的方向,然后再好好学习,高标准严要求,宽进严出。

十几岁时一秒钟就能学会的东西,不需要在七八岁费尽心思去学习。

反观咱们,基础教育固然非常扎实,但是也透支了孩子对学习的兴趣和热情。

而且咱们教育的理念非常有问题,咱们对孩子的教育的目的,不在于提高孩子自身,而是着眼于于扫平孩子的同学们,在竞争中脱颖而出,是为了在独木桥上把别人挤下去。

我认为这个其实是非常不好的,是会毒害孩子的精神基础的,这种教育理念下走出来的孩子,是很难具备一个科研人员的素质的。

第四,咱们国家承担科研任务的还是以 “机关单位” 为多,企业负责的相对很少,这样的话官僚主义的危害、僵化低效的流程,以及学术腐败就会成为阻碍科技发展的重要因素。

其实这个部分至今仍然处于一片灰色地带,讨论的人不少,但是谁也不能为体制拿出既不伤筋动骨又能提高效率效益的可行办法。

说到底这套机制的运行逻辑是自洽的,虽然令人鄙夷,但是在没有外部冲击的前提下,它可以永远地运行下去。

知乎用户 胡子胖叔 发表

作为一年级小学生的爸,我也许能说几句:因为在欧美,学生是没有真正意义上的课本的,这说明了什么?假设,在国内,教学大纲如果说一年级孩子需要掌握 10 以内的加减法,那老师就只会教 10 以内的加减法,考试也是相对应的,孩子做全对就 100 分,就是班里面的 “学霸”,但是他们真的都是“学霸” 吗?这就是为啥我记得我小时候一大堆学生满分,98 分可能都得挨批评,只有随着年级增长学的东西越来越难,100 分才会越来越少人能达到。

欧美呢?体系完全不一样,在公校甚至都没有统一的课本,老师教学也非常 “佛系”,基本就是快乐教育和鼓励为主。有不满足的家长就需要把孩子送到课后补习班,在补习班,孩子理论上是没有上限的,只有做到自己做不出了才会停止。以数数为例,如果国内要求的是一年级由 1 数到 10,欧美的要求就是 1 数到 10 为一级,然后 10 到 20,然后到 30,直到 100,孩子数到 10 就数不出了就一直练习直到数得出才可以晋级下一级别,而不是根据年龄。加减也是一样,能算出 10 以内,就不会停止直接到下一级,直到能算一百以内加减法。

这个体系在我看来有几个好处:

  1. 让真正有天赋的孩子更快成长,而不是去 “等” 平庸的孩子,对有天赋的孩子来说就是真的学无止境,做出题主说的超级牛的东西关键还得靠有天赋的孩子。
  2. 让真正有天赋的孩子能够跟其他同样有天赋的孩子组成小团体,互相竞争促进进步,良性发展。

这个系统能够最大限度让拔尖的人成长更快,我觉得这才是根本。

知乎用户 弯刀先生 发表

其实西方是阶级式教育,我们平常所鼓吹的西方 “快乐教育”,实际上是他们普通民众所接受的 “低端教育”,就是书本写得很浅显易懂,书里面的干货知识也不是很多。好处就是,没有老师教学生自己也能看下去,学习也十分容易,但是结果就是,学不到太多东西。

所以现在你看美国人,很多基层民众的常识是很差的,科学素养也比较低,不少人认为地球是平的,不少人对美国有多少个州、美国多大面积、人口多少,没有正确概念。

因为老外知道,权贵阶级和普通阶级就是不一样的,以后治国理政、叱咤风云,轮不到这群普通阶级民众的子孙来干,所以保障他们受到基本教育,然后留着给他们投票就行了。

但你看欧美顶级学校,最典型的就是英国的 “九大公学”,绝对不是玩儿快乐教育的,比中国课本还难,地理课甚至还有测绘等实践课程。伊顿公学不仅学费高昂,想入学还得有名人推荐信。这种专门培养精英的学校,不是玩儿快乐教育的。所以人家的精英,本来就受了不错的教育,能够做出比咱们强的东西,很正常。

还有就是高等教育,不像咱们目前水课这么多,大学课程比较难实践、科研也占很大比例,对学生的塑造是完全不一样的。

所以我们现在教育界,鼓吹快乐教育,是绝对错误的,那是老外用来培养垃圾票蛆的教学模式和教学质量。我们就是要坚持目前的基础教育,然后重点应该关注高校的教育质量提升,这点确实差太多了

知乎用户 David Dong 发表

1,美国的教育是严重分化且资本导向的,你有钱就有好的教育,你没钱就没有,跟我国的整体基础教育的根本原则就并不一样。

2,教育的好坏难易是很难简单的量化衡量的,就数学而言,良好的计算解题能力跟数学思维的培养也是两个纬度上的概念。中国的基础教育总体来说是个智商筛选器,跟优秀的教育还是有一点差别的。

3,教育本质上是个高度消耗资源的东西,中国过去的资源不足,所以给出的教育不能算十分优秀,现在资源更充沛了,但是教育的成果还需要一段时间才能慢慢显现出来。

4,美国即使大部分基础教育都是一坨翔,因为资本强悍和国家整体的开放政策(所谓的美国梦),还是可以吸引到全世界人才。就拿数学来说事,以前俄罗斯,东欧,亚洲很多的数学人才一转身就变成了美国人。人都是现实的,你看国内多少 CS 的人心心念念的就是去美国找份工作,去了之后再回来的概率是很低的,然后就变成熬出身份变身美国人了。

知乎用户 刘文硕 发表

中国教育才回复多少年啊???从回复高考到现在过了有一代(60 年)么???

装什么糊涂啊!!!现在所谓的科技带头人,那些国内知名期刊杂志的主笔还有学术论文发表的第一作者都是什么文化水平?看没看过刚恢复高考时候的高考试卷,放在现在 2020 年,小学生差不多都能考上大学吧!!!!

所以你问为什么美国人能做出来那么牛的技术……. 对不起美国人也不是建国就牛的,美国人牛前期真的是因为当年德国反犹导致大批量真才实学的犹太籍科学家加入的,就是这样,美国本土的学术水平真正显著提高也是二次世界大战之后了!!!!至于中国,你有什么机遇???别忘了,两弹一星的元勋可都不是中国的大学培养出来的!当年邓老等的徒弟可都是现在理工科里面的泰山北斗,你说说看,这上层建筑你拿什么跟人家竞争?

远的不说,我们提提邻国,是我们在高新技术的研究上比不上日本了还是比不上韩国了还是比不上俄罗斯了,宁这么膨胀还有跟美国掰掰手腕子?

理智点行么?美国现在有几个新人(30 岁)一下的拿出来了世界级的发明创造的,这不说明问题么?中国呢?好好看看高级期刊上面文章的第三作者和通信作者行么,国内行规,大概率这些人才是真正的第一作者!。

知乎用户 Feng Henry​ 发表

1、美国数学并没有比国内简单太多,至少我不觉得拉开代差。

2、美国注重精英的培养,普通人 VS 普通人或许美国要差一些,但精英 VS 精英美国并不差。然鹅,超级牛的东西往往都只关乎学术精英的事,跟普通人也没太大关系。

3、数学在工程方面很重要,但并非唯一重要。理、化、工程、管理等方面同样重要。美国人说,憋说数学啦,咱们来聊聊综合实力吧。

4、美国高校及科研机构吸纳来自全球各地的人材。在 TOP 团队中,总是比较有机会看到数学成绩比较好的亚裔身影,在很大程度上弥补了数学不足的短板。

5、数学不那么简单的中国,近些年来也有不少超级牛的东西。

知乎用户 新年快乐​ 发表

应试教育的标准,就是所有问题,都有一个统一的答案。你看到的是人家学生学的简单,你看不到的是,他所教授的是解决问题的方法,是思路上的启迪,和方法上的多元化,创意化,他重视的是思路,是创意,而不是一个统一的答案。

每一个环境,总有每个环境里的利弊,我们的教育体系结构,是存在着某种需要改进的不足之处。所以,我们也不能,轻易地把其他国家否定了之,这是不利于,社会进步的一种表现。

在地球村里,领居家种地的方式比我们有,长项优势时,我们不应该鄙视人家,更不能斜视忽视人家的这种长处。一棍子打死,或者说是已一种不平衡心理,情绪化的东西,搞杵一些不必要的对抗,无意义的敌视,这些不可取的态度,本身是一种带有某种自我封闭性的,自我保护。

文明自身的碰撞,交互性,才是人类文明,可以有这样的成就的真正意义所在。

建国二百多年历史的美国,可以成为一个超级大国,甚至是世界警察,帝国主义国家的领袖主导,这本身就是一个奇迹。

他的发家史,就是建立在工业革命后,西方资本主义国家的肩膀上的,他吸收融合融化了的就是,一个世界文明进程养料的过程!

移民国家的最初建立,就是西方航海路上,所向无敌的漂泊迁徙的勇气。这个国家建立,就建立在他的吸收与博纳广储上,从他对于世界各民族人才的善待,与其先进性的大踏步前进路,是相提并论的,相互作用的。

人才辈出的前提就是,你为这些人才建立提供了,他愿意贡献他力量的场所!这是一个起码的条件,我们所看到的美国的牛,归根到底是一种人才辈出的贡献问题,是人与人之间环境竞争的结果。

这种多民族的融合!所碰撞出来的东西,就是人类文明各种各样的精粹,之间的各种复杂角逐出来的智慧结果。

我们只能是去吸收利用,这些文明成果,并且善于融合在自己发展的道路中。而不是排斥抵制,做一些无意义的比较。

也只有我们越来越快的吸收,并融合融化,而且成为我们国家自身的创意,创造性的精神,这种转化,才会有回报。

科技时代,谁能走在其他民族前面,谁就会占领市场。我们的教育体系,肯定会逐步的有一个结构调整的过程的。

四十年的改革创新,让世界看到了不一样的中国,也让中国老百姓越来越意气风发走向未来,同样的是这样一种进程,会越来越应用到,我们这个社会的各个方面的。

在我们国家的政务大厅,全面推广后,政府效率提高的不是一个台阶问题,而整个是一种全面的开放,透明化的问题。

毕竟社会进步确实是,有一个结构过程的转变,所以我们拭目以待,期待我们自己的社会方方面面,都越来越好起来,也期待着我们的国家越来越强大起来!

知乎用户 歌者​ 发表

做出超级牛的东西不光需要数学,简单来讲需要一大群有创造力的人,外加一小撮有创造力且数学很好的人。

美国教育没接受过,不知道,不瞎说。中国教育并不培养人的创造力,更侧重培养人的学习能力,更精确的说法是考察人的学习能力,重考察轻培养。以考试成绩作为评判能力的依据,除学习能力外还有数理思维能力、阅读理解能力、英文听说读写能力等,从而分配对应的后续教育机会或者对应的就业岗位。培养能力只是一个考察内容、评判依据、量化依据而已。

更何况已经说了,这其中还基本没有创造力。

“超级牛的东西” 在我看来指的是美国在互联网产业、设计行业、航空航天、音乐产业、体育产业、电影传媒工业等领域生产、制造出的世界领先产品,比如 Facebook、波音飞机、阿波罗登月计划、好莱坞电影。美国在上述领域取得了毫无疑问的巨大成就,取得这些成就的关键无一不是创造力。除了创造力,数学确实在其中起到了一定的作用,但完全不是关键。

中国当下的问题是,数学好的人很多,或者说培养出数学好的人很容易,或者说培养出中学数学好的人很容易,但是想培养出有创造力的人、允许有创造力的人通过创造过上好生活、有创造力的人受到尊重而非受到各方面的打压、否定,在当前国情下,存在相当大的困难。

知乎用户 李艾蓝 发表

因为美国靠吸收全世界的高端人才搞科研搞创新,美国人从来没指望仅靠自己土生土长的老乡去搞高科技。而会这么想的恐怕也就只有东亚儒家圈子。

知乎用户 晨韦 发表

再听到这么说的,你可以直接怼他,读者文摘看多了吧!

知乎用户 王飞猫 发表

具体情况不太清楚,但宏观教育理念,我们不同。

美国更崇尚 “自由意志”,我们更崇尚 “圣人意志”。

简单点来说,美国培养小孩,他们会花最多的时间去悉心说服,而我们则会以最便捷的手段去解决问题。记得以前看过一个文章,说一个外国妈妈的见闻:

小孩不吃鸡蛋,美国教育机构怎么办?-> 讲道理,什么有营养啊,好味道啊,等等,但绝对不会塞进小孩嘴里,因为这样做会收到律师信的可能性很高。

我们怎么办?-> 我认为小孩不懂得这是食物,所以直接塞嘴里去就行了,要是吐出来,要不就骂,要不就打,要不就群体嘲讽(批斗、羞辱)。

而价值观塑造时期又是怎么样的?

我国是批评教育,打骂教育,群嘲教育。

美国则是鼓励教育,据说比上述的教育法要好 30%。

这里面就出现这么一个问题了:奖励机制的塑造,参考巴普洛夫的狗 - 摇铃实验。

美国的方针是让学生自主地、感兴趣地走上专研知识的道路。

我们的方针往往就是让学生必须听话去学,逼着去学,打着去学。

当然,我说的是我的时代的我所觉得 “是这样” 的普遍做法,因此我不谈细节,我还代表不了那么多人。所以就给你参考一下。

这样的教育下就有了这么一个结果:

美国,学生看到书本就心情愉悦。

我们,学生看到书本就烦躁不安。

可以说,在厌恶且高压的背景下成长,就是我们的方针,因为我会要赶超这个,赶超那个,所以必须要服从 “圣人意志” 前进。为了工薪,你的确需要再不喜也要硬着头皮去做,但这种情绪会导致人失去效率,失去专注。

实际上,你读过大学就知道,大学安排,一学期 17-18 节课上完 2 本高数,一本 200-300 页纸。课堂上,真的那么重要吗?答案是否定的,基本就是跳着章节,选着重点来教,你想学?自己会宿舍努力吧!运用你当年中学的知识去学吧。

西方呢?早前那个首相还是什么大佬说,他连 9x9 乘法都背不出,因为他的专业不需要,所以他不管这块,即使管,那拿出你的计算机,没有?没关系,拿出你的手机,然后打开计算 app 去算就行了。

很有趣吧?按这实例的逻辑来说,中美的教育属于一种相反的形式。

美国属于那种前松后紧型,我们则属于前紧后松型,或者一直紧下去。

我记得我以前的高学数学老师说过一句话:数学嘛?在各个年级(小中高)的定义都不同,具体忘了,主要是 “极限”,“虚数” 等问题,这个教学边界的不同是学历决定的,所以会出现 “认死理” 的现象,所以你小学极值是 0 到 10,初中可能是 - 10 到 10,高中就正负无限,然后正负无限其实是邻居。属于不断地安插新的理念进去修修补补,人家美国由于前松的结果,可能就是一个最终定理丢出来,不给你修复了,毕竟都大学了,对不对?

再就是,美国估计是分级教育了,才出现我们看到的不同,例如你这个班最终目的不是进科研机构,那你的数学就是实用数学,结果一截图发过来,我们就看到了怎么那么简单啊?

因为人间是面向实用的,能不简单吗?就像,加减乘除,会计出入记录。

那么科研班呢?那就不是实用性了,而是科研型的东西了,如各种算法等等。

你可以摸心自问,如果不是职业需求,你在小中高所学的数学,特别是几何三角,数列,矩阵,你现在用得上吗?估计教孩子的时候,才用得上吧?毕竟,我们是大杂烩教育。

我们的教学往往到最后数学不是最重要的,最重要的是从那堆杂乱无章的大杂烩中学会《逻辑学》,但我们并没有直接开课教授。而是设立了非实用数学。正如我大学读经济学的,高数也就是经济应用型的高数了,不带几何三角,因为日后用不上。

但中学就成了为大学打地基的模式,假设你能大学,假设你建筑行业,假设你林林总总,所以瞬间大杂烩,让你缩短大学时所需要的支出时间。毕竟,只有小孩子是最好控制的。

知乎用户 哈欠 发表

最近十几年的物理诺贝尔奖,大多数都是微观粒子的成果获得者,而这个成果获得的前提又是欧洲的粒子加速对撞机的建设。

后来国内中科院也要建设对撞机,结果被否了,因为投入太大。

科研就是个烧钱的东西,而且是持续不断的烧钱,欧美有钱,烧得起,那些出国不回来的中国科学家,很多也是因为在国内没有钱给他烧,没有先进的设备仪器,没有足够的科研经费,只能去国外搞研究。

知乎用户 bohmwu 发表

老外,主要指欧美,数学在差不多最后一年 (大学之前) 都会把跟国内一样的内容学完,当然前提是你选修了。所以老外通常不选,而不像国人出国后都拥在数学课里。

当然因为乱七八糟可以选修的课比较多,什么神学,心理学等等。所以课堂生活还是很丰富的。

但是很多例如大学的经济学需要上数学课,但是难保学生没学过。所以通常会统一再学一把,当然速度更快,难度更高。特别说明下,这是在英国的情况,不过我估计美国应该差不多的。

那么回到题主的问题,本来想说,其实也不是应试害得,现在想改一改,应该说不光是应试教育害得。

想想,孕妇穿所谓的防辐射服,都是多久的事情了,为啥还是有人信?想想,牛奶问题都这么久了,为啥还是没法解决?再想想,事情安全,道路安全?

如果说是我们这个人种的属性,我想说,至少,对岸貌似还好嘛?

所以,环境还是一个大问题,为啥有人出国留学?为啥学艺术设计要去欧美,学新闻要来美国。我想还是有原因的。

再者,抄袭了这么多年的我们有没有思考过,抄的没法再抄的时候该怎么办呢?

知乎用户 雨落长川​ 发表

我们的数学教得难,并不是因为中国需要那么多的数学家或强数学要求的工程人才。

普通人暂且不论,就算对于数学要求比较高的工程人才来说,中小学数学教育也有很多内容是溢出的,就算知识不溢出,解题的方法和复杂度也是溢出的。

那为什么还要这么难呢?

因为我们人多啊,上一级的优质教育资源少啊,搞这么难是因为要大家相互竞争啊。不难怎么撕出一二三四名呢?

知乎用户 清渊​ 发表

SAT 考试的数学(毕业必须)难度约为初二水准

AP 考试(申请顶尖大学必须)的数学(也就是 AP 微积分 BC)的难度大约在大二高数水准。

最低要求和最高要求差的太大了…. 哦对,这个最高要求还真不是给顶尖天才的,一所随便好一点的国际学校或者美高,目标专业是理科的如果高二还没考完微积分 BC 那是要被歧视的…

知乎用户 海盗船清洁工 - 8503 发表

我一直觉得问这些问题的人本身对问题没有很好的理解,所有中国人都一样的吗?当然不是的。 那凭什么我们把所有的美国人当一样的呢?再者,美国的课程也有层次,我们所谓的初中水平的美国高中数学是最低档的,难滴我们的高材生去考要照样要复习刷题才能考的好呢。

至于为啥有成就的数学家都在那边,你想想,我们是不管你喜欢不喜欢,高中你得学很难的数学去高考,人家是你不喜欢就学简单可以了,喜欢数学可以学的很深很难的,自然就把有天赋的人给显现出来重点栽培咯。

兴趣是奋斗的最大动力来源,有了动力成功就是看天。。。至于有些回复计算与数学之间的问题,10 多年前就有美国有名数学家联名信至美国教育局,说美国中学学生就应该提高计算能力,毕竟算得慢就没兴趣的。计算是 arithmetic,确实是讲求技巧,但不是旁门左道;真正旁门左道是把数学(mathematics)的解题做成 “套路”,然后教给学生们应试,考完全部忘光。对不起,高中学数学老师教我的情景还历历在目。数学是要思考的,思考过做出来才是学到了。做成套路就变得想电脑算数一样,公式化,这样可以毁了很多代学生。

中国的教育到底问题在哪?是不是马云说的问题在校长身上?我觉得校长只是大部分问题,不是全部问题,等到教育策略有效被解决,这个问题才会有效被解决。

发发牢骚,说说事,勿喷。

知乎用户 名字什么的不重要啦 发表

为什么中国有几百万伪军,各种汉奸,日本在中国战场上还有伤亡?

因为,汉奸伪军和造成日本军队伤亡的中国军队,不是同一批人。

你如果要把日军伤亡数字记在伪军汉奸头上,就会得出 “汉奸伪军给日军造成了直接伤亡” 的结论。

知乎用户 GUNANA​ 发表

因为美国可以掠夺世界上所有的人才

你看看美国科学家里面亚裔有多少

知乎用户 肥叔 发表

看答案一个个真的以为美国教育很牛逼。

各种分析头头是道。

还能扯上让孩子多运动能提高大脑能力,能提升学习知识的能力。

真服了,不知道运动队的,大部分成绩都不好么?

难道你们不知道美国是移民国家嘛?

掐尖全球的尖子,进入他们的国家,一直到特朗普时期,美国人都认为自己的工作被外国移民抢走了,于是开始 ban 绿卡,甚至连大学和科技企业需要的高端人才,也一并 ban 掉,弄的科技企业一直抗议。

你们真的看不见美国移民局掐尖这个事儿么?

知乎用户 入不忘兮​ 发表

因为算数是算数,数学是数学。

正如普通人是普通人,天才是天才。

知乎用户 水陌轻寒​ 发表

如果假设中美的教育下的学生都服从正态分布,中国的教育,平均值比较高,方差却很小,所以蠢货很少,天才也很少,美国的教育,平均值要低一些,但方差很大,所以蠢货很多,天才也很多。但是,数学的顶尖问题,是需要天才去解决的,一般的人才插不上手,所以美国占优势。当然,不能因此说明中国的教育就不好,美国的教育中国学不来,底子没人家雄厚,学了画虎不成反类犬。

知乎用户 早鸟 发表

分两个点回答这个问题:第一个点是美国是否每个学校数学都那么渣。第二个点是牛逼东西是什么教育体系教育出来的。

第一点

大部分美国教育就是很渣,所以硅谷的高薪工作大部分是中国人和印度人在做。美国快乐教育愚民出来的本土娃脑回路是不够的。精英娃又不屑于做。但是美国有很猛的精英教育。美国的精英教育比我们难太多了,你知道这边好的高中很多 AP 课程吗,高中把微积分学完的在好高中太正常了,这还是在好的公立。顶尖的私立学校更好很多,情商智商艺术体育全面发展,中学时代有的就已经跟大学教授合作做研究了。

第二点,美国允许小孩很早就偏科,类似于国内竞赛保送,但是他们的资源太丰富了。好好偏科可以初中就开始沿着自己的兴趣爱好研究,等到大学毕业都有 7-8 年的经历了,还有就是牛逼东西很多也是其他国家移民研究出来的,AI 领域美国顶级专家里面中国大陆人比重最高。还有美国的大学体制,他们允许很长的科研周期,还有其他很多硬件软件方面的优势。

题外话,最近 trump 要卡中国留学生,美国学术界慌的一 b。没学费了要倒闭一群学校,还有就是缺了中国人才不够啊。

知乎用户 蚍蜉 发表

个人不才,想抛砖引玉。你们都在讲教育,实际上我觉得跟教育无关或者说关系不大。美国同样的教育方式移到发展中国家很可能培养不出人才。不是发展中国家的人不聪明,也不是老师不够优秀。而是整个社会发展与科研环境不匹配造成的。就拿中国来说,每年申请的专利或者论文很多,但这些专利或者论文与中国发展阶段不匹配,以致于无法产业化或商品化。无法产业化的后果就是研究人员的动力和热情就会减弱,减弱的后果就会促使人们从 “短平快”(周期短、见效快、效益高)入手,所以你会看到一些以捞钱为目的研究(当然,肯定也有一些做的好的,但这种比例一定很低。做的好的科研工作者肯定对自己的研究给予了非常高的愿望才促使其家坚持不懈)。中国的情况也适用于其他发展中国家。那怎么才能好转呢?就是进一步发展各种产业,当研发能力的企业越来越多并且越来越赚钱的时候,整个社会科研环境才会好转。企业强大,说明市场大,又说明资金集约能力强,又说明产品转化能力强。就拿人工智能来说,若不是 Google、IBM、微软等公司的强大,就不会促进人工智能的发展,因为只有那些大公司才能将人工智能产业化。由于人工智能的发展又促进人们的学习热情和知识的储备,从而又促进了学习机算机科学与数学的动力。当大公司无法消化那么多人才的时候,各种厉害的小企业就会诞生(因为知识储备已经够了)。不只是 IT 业,还有很多产业。当有更多的宝洁、可口可乐、沃尔玛、通用、波音等等的时候,整个社会发展环境已经不一样了,各行各的发展促使社会对理论研究的需求,有了这些,才是数学、物理、化学、生物的最好的时代。

中国怎么才能促使数学像美国一样牛呢?就是要有更多的像华为一样的公司,当每个行业都有类似于华为这样公司的时候,才是中国科研环境最好的时候。

你们说的数学难不难的问题只是人才筛选的问题,这些问题还没有达到问题的核心。既使一个人在学生时代从来没做过难题一样不影响他们未来的发展,当各种科研平台很好的情况下,学生毕业进入这个平台也会调动他的潜力。

有人可能会问,为什么人才经常外流?那是因为社会的发展与这些人才的愿望不匹配。当这些人没有资金、没有实验室,甚至连家庭都照顾不了的时候,你说他们不移民吗?他们肯定会移民到能更好提供平台!这也是发展中国家人才容易外流的原因。

知乎用户 Timothy 发表

美国初中生学的数学是咱们国家小学生学的水平

这个勉强算对。尤其是 6-7 年级时的数学是很简单。

美国高中生学的数学是咱们国家初中的水平

这个属于单纯的意淫而已。

如果一个美国高中生想学数学并且能跟得上课程进度,在拿到学校的许可后,可以去当地的社区大学或者四年制大学上大学提供的数学课。此外还有 AP 微积分与统计学可选。我们那儿(弗吉尼亚州林奇堡市,人口不超过十万)还有 Central Virginia Governor’s School of Science and Technology 专门给数理化优秀的学生提供高等级的数学与理科课程。只要跟得上,向量微积分也可以学。其他城市,尤其是大一点的城市,基本上都有在提供这样的教育体制。

我们从高中开始,尤其是 10-12 年级时【9 年级算是过渡期】,就开始每门课的难度任由学生选择了。语文好,就选 AP English Language and Composition【更倾向于写作】或者 AP English Literature and Composition【更倾向于文学鉴赏】;语文一般,就选 Advanced English 就行了;语文不好,就选 Regular English 就是了。其他科目同理。艺术,历史人文,经济学,心理学,数学,电脑科学,物理,化学,生物,环境科学,外语等,哪科好,就修哪科的 AP 课程,哪科不好就修哪科的 regular 课程。对了,如果体育好,只要其他学术课程及格,专心地把自己的那项运动做好,虽然去不了 D1 的大学,D3 的还是基本没有问题。

只要不是太差的学校,美国这边都可以做到相对地因材施教

这道问题有超过三万多的关注者,就问一句,有多少人在高中毕业后,还会日常地使用微积分?

知乎用户 卢振奇​ 发表

看了一下,好像是 16 年的问题。

到现在六年了。

那个时候如鲠在喉,却不得反驳。

前两天有个新闻,美国某高超音速飞行器今年第三次试射失败了,问为什么。

是啊,为什么?

这问题里面有个回答很好,如果你不爱学数学,那么就给你快乐教育。如果你是天才,就把你变成美国人。

冷战之后,优秀白人被送去学金融法律医生,科研几乎就是靠东亚、印度和俄罗斯人顶着。又由于政治原因,中国人和俄罗斯人被排除在军工研发之外了,于是军工研发只剩印度人了。

那你们猜,为什么导弹试射会失败三次呢?

知乎用户 happy12345 发表

说句实话,我们的中考,高考制度,优化掉了一大批的瘸腿尖子生,我们说什么都要讲究德智体美劳全面发展,却忘记了很大一批术业有专攻的瘸腿生。

有的人理科真的拔尖,思维跳跃性非常强,但是文科就是不行,英语就是拉跨,但是他们对数字,对计算却十分的有天赋。

有的人文化造诣非常有天赋,言语能力非常强,但是理科就是不行。

(不用拿奥林匹克,或者国际顶级比赛加分或者特例录取作比较,这类人的起步成本本身就很高,我上面说的都是普通出生的平民子弟,也有特例,别杠我。)

这些人将来真的不是人才吗?真的没有发展前景吗?显然不是,这类人大有可能变成专业性的顶尖人才,因为他们的大脑就是为单科而生的,但是却被目前这种考试制度优化掉了一大批,好苗子荒废了。

说真的,挺可惜的,也许有人会说,就算没考上重点,没被认可,以后也可以努力啊,其实现实中一旦路被限制了,路上的资源就被限制了,在黄金年龄被限制了发展,以后想跳出来真的很难很难,不是没有,凤毛麟角。

===========================

看了一下评论,大概大家都没明白我说的意思,我说的瘸腿是指单科拔尖的,并不是仅仅的优秀而已,就是单科非常强,比如满分级别的。

但并不是每个学校,每个家庭,每个学生,都有能力,师资,金钱和时间去参加那些所谓的培训和竞赛,然而这些学生的思维,思考能力却远超身边的其他人,最终可能就荒废掉了。

也许就像有的人说的,反正我们人多,不差这几个。

还是那句话,中高考目前仅仅是对国内而言,不公平中的公平,想通过这个来筛选人才,是肯定不够的。

知乎用户 起名难 发表

LZ 看看美国大学阶段数学的公开课,非数学专业的数学就难得要死!我看过 MIT 的线代和普林斯顿的高数,远远超过国内大学的深度和广度 (难得一 B)。所以说,中国是大学前教育世界第一,高等教育就一般了。

-————————————– 看到一些别的答案,补充下 ———————————

美国的基础教育有点类似于 “因材施教”,你要是对数学感兴趣,可以自行选修一些难度很大的数学课,而一般人对数学要求没有那么高。到申请大学的时候,很多专业(理工类中的一部分)需要你学过一些课程,要是没学过很难申请。就算有的学校大二才选专业,你的数学基础太差,课程跟不上(总体来说美国大学课程又多又难),你也会自行放弃的。

知乎用户 阿典​ 发表

因为人家能课外选修和变相跳级,制度上吃小灶对 “天才” 比较友好。公立教育就是菜小,养工人、给打工人子弟数学扫盲的,高中毕业万岁。

八十年代华侨们回国,美籍华人科学家建议下,中科大搞了少年班。

知乎用户 天芒星​ 发表

基础教育和高等教育是两码事,人家出发明成果靠的是高等教育。

美国人普及版基础教育不如我们,但它们的高等教育累积的本钱雄厚,我们现在暂时还比不上人家。

另外,这几年,咱们的高等教育水平正在慢慢追平甚至超越美国。同时,美国佬的基础教育分化成普及版和精英版,它们的精英版基础教育水平并不比我们差,不过它们现代教育起步比我们早一百多年,至少累积的经验还是比我们雄厚的。

知乎用户 郭斯特 发表

首先美国是一个很大的分布式系统,没有统一的教学大纲或者要求,老师自己决定教材和授课进度,所以任何美国人如何如何,美国的 XX 如何如何的论点一律无法严格成立,最多只能说有一定的统计意义而已。

也就是说,数学教学水平的高低,各个地区差异很大。比如我住的地区的公立高中,学生的数学水平不但不及天朝高中,其实是不及重点小学的。不过问题是,不是所有人都喜欢数学,适合学数学,三百六十行,能安身立命的行当多了,有几个是要求数学素养的?美国文化的优势本来就在(自己打自己脸一下)沟通,交流,演讲,忽悠,商业。

不过这不影响那 1% 有数学素质的人能继续深造。关键就是那 1% 的人有条件上大学,读硕博。光看看美国公立大学里多如牛毛的奖学金项目就知道了,你高中学习成绩优秀有奖学金,当过兵有奖学金,少数民族有奖学金,残疾人有奖学金,父母做某种职业,住在某个地方都有奖学金,恨不得随便找个理由就给钱。私立大学的奖学金更多。有钱就是任性,人才自然能得到发挥。说到这就不禁想起不争气的天朝,教育财政分配占 GDP 的比例,往事不堪回首,简直就是耻辱。忍辱负重发展经济,现在终于有钱了,终于死活达到 4% 了,下面就是看怎么花这个钱了。

知乎用户 海蓝 发表

以我通过网络报刊以及各种文章的对美国教育的理解,

问题的答案不复杂:学的粗浅的和搞出很 nb 成果的,他们不是一个群体啊

类比:北京人均收入那么高,为何作为个体我挣得都不到

因为挣得多的是别人

知乎用户 wentao523 发表

很简单 我们中国人好比五年级一班, 数学平均分是 80 分

美国人是五年级二班,数学平均分是 75 分

我们五年级一班的最高分是 85 分,最低分是 75 分。

五年级二班最高分 100 分,最低分 50 分。。

统计学叫做美国人这种方差大, 就是有一些天才,有一些奇思妙想 精力以及智商及其特异的人。。。

我们虽然平均 甚至高,但是我们都是普通人,清华工科生比普通 211 的强,但是也就强了 5%。。。。

我们的人都太聪明但是有没有特别聪明的。

美国人有一些特别傻 但是又有一些特别聪明的 特别少,但是精力旺盛 各种冒险 各种运动 各种探险和不要命。。 往往社会就是由这类人来带领和开拓

这其实是根本原因,从我们的小孩子就可以看出来

都说有十个儿子,乖乖宝将来不惹事 但是也很难有成就,有的坏小子,精力旺盛 探险精神强,雄性激素旺盛,女人们还缠上来啥都不要就想跟他谈恋爱。 我们老家都说这样的坏小子 将来有可能有成就。。。

美国人就是这样的坏小子多, 我们就是乖宝宝多。 并非后天 而是先天的精力 外在和内在的不同

后天文化又加强了这一点。。

再说 我们华人在美国二代三代四代 其实也没做出什么成绩啊 ,我们出国那么多清华北大的 ,在美国被阿三碾压。。 出名的没几个。几个出名的 也是在别人的体系里面安排好的工作,做出了应该的成绩。

所以我的结论就是我们智商都不低 但是没什么特别的人 但是科技的突破 重大项目的突破靠的是这些特别的人 即使数量很少。

我们的专家水平极其有限,当然还是比普通人高,但是就高了那么一丢丢。。。 跟世界的高水平比起来,没有竞争优势

所以复旦北大教授说俄乌不会开站,开战后又说三天结束战斗。。 当时伊拉克也是这么说,他们其实说的是真心话,唯一问题就是他们的水平不高,(但是还是比我们普通老百姓高那么一点点)

知乎用户 vivi 发表

中国的教育基本上属于数理逻辑智能的教育,而美国的学校和家庭对孩子的教育是全方位的,包括有运动智能;音乐智能;空间智能;自我认知智能;人际智能;自然认知智能;语言智能和数理逻辑智能八个方面。通过大学招收怎样的学生,告诉大家对孩子的选拔和培养方向,可能不知所以然,能够做得所以然就好。数理逻辑智能是需要通过大量的练习和训练才能完成,这也是为什么一般受西方教育的孩子和中式教育的孩子相比,数学的难度和深度会比较差一些。而独创性和创造力则和空间智能 音乐智能和自然认知智能密切相关,也就是说一个有创造力的人才是一个多方位智能都发展不错的人,而我们的教育内容缺失太多太大,没有经过大范围的培养基数的人自我成长的几率很小很小。

知乎用户 清平安乐公​ 发表

最近看了个纪录片,是说英国请了一队中国的教师,到一个学校,教一个初三的班级一个月。校长会在一个月后进行一次全校的考试,想研究中式教育和西式教育哪个会更好。

随后,这个班级被抽调去了另个教学楼,没有刻意筛选学生,然后这个班的学生需要像中国国内的初中生一样安排课程,早上来到要做广播体操,每周一要举行升旗仪式,然后还要比其他班级多考 2 门课程(中文和体育)。

一个月后,大家完成了考试,校长亲自宣布成绩,先说的中式班的全科平均分 64.8,在国内那肯定是差生中的典型了,只能算刚及格。镜头给到了中国的老师们脸上,估计这个班是这些优秀教师带过最差的一届,当我们以为中式可能要输的时候,校长说了西式班级的平均分,57.4 分。

我心里大写的牛逼。一个学校,全科平均分不及格,这还是在没有中文和体育两科拖后腿的情况下。

虽然最后中式是获胜了,但是我看得出来,英国的校长和老师们对中式并不欢迎。连中式班的学生也表示,他们不喜欢竞争,不想在班级里因为低分而沮丧。

nice!这个观点提出来的时候,我们中国人肯定无法理解,特么你这个成绩你告诉我你打算心安理得还表示快乐?

本来我觉得就他们这种宽松的教育,指定完蛋的时候。我发现了他们教数学的一个模式。

在英式班,平常上课,他们是不教几何的,在初中,几何必然是最难的,他们选择在平常大课上放弃,而是在挑选出有这方面天赋和兴趣的同学,进行课后辅导,我们俗称,上小课。

也就是说,他们其实并没有放弃有难度的教育,只是因材施教,那么一伙进度相同的学生在一起上课,效率会更高。而其他没兴趣没天分的学生,你教了也是白搭。

所以,我们就是没有看到这些。美国的公立学校教育,那就是垃圾啊,美国人自己都知道,如果读的是公立,这辈子就算完了,所以要削尖脑袋去私立学校,连大学,也是私立的强于公立的。而我国教育体系则完全相反,我们会认为,读私立的学生是家里条件好,在公立成绩差,才去读的私立。而大学,不读公立,那就是混了个野鸡大学啊。

所以这是相反的一个东西,导致了我们老去拿我们优势的公立学校去和美国薄弱的公立学校比。在他们私立学校,学生的课业不比中国少,精英教育,针对教育,会更厉害,这里出来的才是支撑美国的精英。而随着美国公立和私立的差距逐渐加大,美国公立学校出来的学生,能有成就的就越来越少了。

在美剧初来乍到里,从台湾移民的黄氏一家人给孩子读的都是公立学校,妈妈给大儿子(也是故事原作者)的未来规划是,高中肄业后加入帮派,锒铛入狱,出狱后吃国家低保。而小儿子学习能力很强,他们一度尝试让邻居推荐去私立名校,规划则是,先当律师或者医生,然后出任美国总统(这里我觉得这个妈妈的想法不是服务美国,而是给孩子定了一个 NO.1 的目标,在中国估计目标就是当皇帝了)。这就是明显的差距。而无论哪个儿子,妈妈都觉得读公立学校学的东西简直没意义,帮他们报了中式补习班。这就是比较直观的东方人看美国教育的视角了。他们都已经移民拿了国籍,不用纠结他们是不是中国人了。

从他们小学允许孩子在课堂上随便说话这点,我就表示,这种课堂纪律,根本没法听课,能学到啥?

知乎用户 RoseofVersailles​ 发表

不要用中国的思维去类比美国…… 美国是个移民国家,中国不是。

有很多高赞回答写的计算机领域获奖大牛,全是中国名字。如果有心,可以再去查一下他们的通讯单位,你就会发现大部分都是美国科研机构。说这个什么意思?意思就是美国的顶尖人才本来很多就不是自己培养的…… 题主问 “为什么美国中小学生学的数学比我们简单,美国人却还能做出超级牛的东西?”,答案就是美国顶尖人才根本就不是在美国上的中学好么……

知乎用户 东南枝上有甜瓜​ 发表

大家答得其实都有道理,但是,忽略一些东西。中国的数学基础教育,为什么要怎么难。似乎认为数学基础教育不应该怎么难,而应该差异化。

个人认为此言差矣,数学基础教育必须要上难度。个人觉得当前的数学难度和内容没有问题,问题是教育方式,不应该是题海战术,应该要有研究性学习和创造性学习,辅助以个性化教育。

为什么我会这么说?这是因为我国的产业结构啊。我国产业结构,自然是制造业为主,而且越发的从轻工业转向机械制造,精密产品制造,另外建筑业,高尖端基础建设业发展迅猛,it 产业,互联网产业方兴未艾。而这些产业背后是需要一定素质劳动力支撑的,想想要是没有一定数理基础能行吗?无论是装备制造业的产业工人,还是各大科技园的 it 民工和码农,抑或建筑工,如果就是美国基础数学的难度,咱们的产业根本无法支撑起来,经济转型从何谈起。

而美国呢?人家根本也没多少制造产业,除了在华尔街坐办公室的以外,其他人 wasp,欧洲人的赚钱方式就是从事各种价格坑爹的服务业。至于 nasa 和硅谷,有老中和老印怕什么,而黑人兄弟们主要负责演戏唱歌打球,底层工作有墨西哥非法移民担着。因此,产业结构决定了美国人可以这么差异化教育,一般人勉强自理,牛 x 者一日千里。更何况,美国不少成果都是人才引进干出来的。(当然,人家确实很有吸引人才的一套)

总之,因材施教这个孔子留下的教训没有错。但是,现实的产业发展,和国家定位,以及人口现状决定了,我们不能因此而把数理教育变水,事实上,我们有学数学的语言便利优势,为什么不多投入一些呢?而且,过分强调精英教育的话,我们也许不会是美国,而是印度,是啊你也有 13 亿的人口红利,但是你怎么发掘不出来呢?就是基础教育差啊,直接就是制造业,工业水平上不去,一直原地踏步吧。

所以,我们的数学难度定位没问题,而是方式问题。另外,为什么牛的研究成果?那就是高等教育体制,研究性学习教育的缺失,创新思维引导不够这些老问题了。当然,中国需要创新,我也必须支持创新。但是,我们既要看重从 0 到 1 的创新,但是也要正式我们从 1 后加 0 的创新。所说,我们很多时候是借(chao)鉴(xi)发达国家,但是,我们由于有高素质的基础劳动力在,我们可以把他们的成果在规模上千百倍的放大,甚至进行极为精妙的改进。这也是我们的优势。总之,一切的分析需要基于我们国家的现实发展需要,而不是,光看别人。

知乎用户 cheby chev 发表

又是一个明显有指向性的问题。如果这个问题改成 “为什么美国中小学生学的数学不如中国难,但诞生的世界五百强企业却比中国多” 是不是就没有那么定体问了?

总而言之是个投入与回报的问题。很多人觉得人聪明就能做出好的科研。其实这只是一个必要条件,说白了教育和科研都是一门产业,可以看作工业化的一个门类。越来越多的科研需要越来越多的仪器,小的仪器几千,大的动辄百万,美元。仪器还有上下游配套、维护,这又是卷入各个精细工业门类上百亿的产业链。还有大科学装置更是烧钱机器。

就算数学、计算机、理论物理这些不太依赖于仪器的行业。人力成本也是天文数字啊。科研工作也不仅仅是埋头苦干,需要国内国际大同行小同行开会交流吧,需要一定程度的奖励吧。科研工作者有家庭吧,或是自己高薪,或是安排家属,让他们无后顾之忧才能安心科研吧?需要培养学生吧?要积累一定量的研究生,需要更大批量的高等、初等教育择优选拔吧?为了选拔优秀人才,社会上还有足够的工业岗位分流那些淘汰的稍微没那么优秀的人吧?这一整套产学研的开销又是个天文数字。

再远一点,科研评价体系、奖惩制度的建立,也是靠人一层层打磨出来的。二十年前国内都是任人唯亲,学术圈乌烟瘴气,那是因为制定和执行规则的人是庸碌之辈;现在为什么好些了?大量的千人、青千回国,自立门户,也刺激了本土精英奋发图强,良币源源不断的流动,劣币也就没了市场。要等宇内澄清,需要积累多少代的科研管理人才,这些人的培养选拔,跟上面说的一线科研人员一样是个天文数字。

总之,中国才富了几年啊?欠下的账太多。需要在科研的各个方面持续投入、加大投入,才有可能追赶甚至超过美国两百余年的积累。任重而道远啊

知乎用户 超越者 HY 发表

想想为什么河南山东人同样这么说北京学生,但是北京学生还能获得很好的社会地位和客观回报。一回事。

知乎用户 Xuyihui 发表

说句难听点的话 国外大学(我只针对理工科 商科这种不算)真的比国内大部分大学压力大好多 学的也多很多 你说的那些初中高中的数理知识差距在大一上学期就抹平了(而且我们高中花很大力气学的几何知识在大学的知识体系里根本没啥用 比如说 你研究计算机算法需要几何吗?国外高中大学的教育体系直接把几何知识给跳过了。。。)

我是在国外念得大学 我老婆国内 985 来这里念得研究生,以上纯粹是我的经验之谈。

知乎用户 啊梓从小就很可爱 发表

你搞混了初等教育和高等教育哪个对人才的专业需求大,初等教育好比让大家识字会写,高中及之前的所以教育都是普及很简单的概念为主,你觉得很难的数理化生数不过是学数学前先把十个数字认好了而已

不会真有人觉得我小时候算数比你快所以我我以后一定是博士后而你是带专人吧。

知乎用户 逗比是相互的 发表

那是课堂教学,美国的数学精英教育比中国有过之而无不及。

为什么中国不搞?大城市一直在搞,小地方搞不了,因为补课要钱啊,跟老美一个搞法穷人家孩子永远出不了头。

做出东西的永远是少数人,大多数老美数学很差。我看美国的 TAR,里面有口算一个值来找对应的地点,题目是 33 减 4,你就知道基础教育啥样了。

别说老美教育好,那有点愚民政策。老美统计学家一个平均值就能忽悠广大民众,这里看来还不如中国百姓。

知乎用户 知幾 发表

都不知道你想说什么。你咋不说美国人认为地球是平的(地平协会),还是登陆月球了,出了那么多科研人员??

举个我们国家早期的说法,搞核弹的不如买茶叶蛋的。但是觉得一个国家命运的是搞核弹的,肯定不是卖茶叶蛋的。同样的道理,美国那些天天快乐教育的公办学校的人,你认为能够觉得国家的命运吗?这些人连 100 以内加减乘除都拎不清。这样的事情,也就是好莱坞大片忽悠一下底层美国人而已。不然,在目前这个万分艰难的时刻,为什么没有人来救他们??

很简单的道理,你就是一文盲的话,人家随便忽悠你一下,你自己压根没有分辨的能力。然后不停的催你,你最后只能跟着别人套路走。

美国的核心价值在于顶层那一群人的取舍。基本上绝大多数选举的政治献金都是这些人出钱的。而且这些政治献金在美国是合法的。说白了,谁出钱,你还不是得听说的。满足人家的要求和利益。即使如特朗普当初所说自己不差钱,还是寻求政治献金。说白了,拿了钱就跟这些人一伙的,不拿就是看不起人家。政治主张彼此分配不均。你觉得你有机会当选??

可能吗?

知乎用户 魔法少女小爱 发表

因为你口中的这些做出超级牛的东西的美国人,和你口中的美国中小学生,压根不是一批人。绝大部分美国人,是压根不上大学的。他们的大学生,或者更高层次学历的人,都是从全世界筛选出的精英。美国科技发达,给他们奠基的科学家却大部分都是外籍的。像奥本海默这种虽然出生在美国,但是父亲是德国移民的移二代,都算根正苗红的了。

————————————————————

补充点内容。如果有人非要吹美国的教育的话,建议他们看看湾湾。湾湾就是跟着美国学,结果瘸了。因为湾湾只能把美国中小学的模式学过来,大学的学不到。美国大学能够从全世界筛选精英,那是建立在他们雄厚的经济实力上。湾湾虽然经济数据看着挺好,但是考虑到他们要给美国交大笔的保护费,实际上并不富裕。湾湾这样强行学美国教育模式的结果就是,他们的年轻人水平都很菜。湾湾的例子充分说明了,美国的教育模式一点都不牛,美国牛的是他们可以从全世界筛选精英的钞能力。

知乎用户 Doug 发表

要通过存量看本质。

就好比思聪可以攒一台超牛的服务器,全世界排第四。

不在于他的电子信息工程专业课学得比你好,而在于:

有钱,

有时间。

知乎用户 黄亮 anthony​ 发表

那些超级牛的东西又不是用中学数学完成的

知乎用户 kun yu 发表

简单的回答是,做出超级牛的东西用不着中小学数学的东西。

小学数学和中学数学连基础都不是。

中国学生在国外继续出色和强大的学习能力有关,和勤奋刻苦有关,和数学基础…… 我只能说这关系被夸大了……

因为天朝数学难并不是因为基础,事实上说打基础的话,教纲覆盖的范围部分还不如很多西方国家。

天朝数学难是因为有很多会被看成奇技淫巧的东西在天朝都常规化了。

当然,中国学生在中学里的数学的确可以完爆大部分外国学生,这点没说错。

知乎用户 录音机 发表

核心问题没有提到啊。

说我们国家重基础,平民教育,数学水平高。这些都是无可非议的东西。但是,我们忽略一个有工作经历的社畜都知道的基本事实:从事一线研究的,都是金字塔的底端。我们知道乔布斯,扎克伯格,比尔盖茨,马云,桑德伯格,孙正义。他们哪个是技术顶尖的大牛?

所以教育有两种,一种是培养引领者的,一种是培养技术宅,哦不理工大师的。至于中考分流出去的,没发现哪里对他们的态度都是一样的吗?

知乎用户 匿名用户 发表

反对部分回答。

美国人给给爱学习的人提供的数学教育是最好的吗?

众所周知,哈佛大学作为世界排名第一的大学,其招生要求极高。数学 55– 荣誉抽象代数(Math 55 - Honors Abstract Algebra),哈佛大学数学系提供的一门课程,被哈佛大学数学系这样描述:“This is probably the most difficult undergraduate math class in the country” (这门课也许是全美国本科数学课程中最难的)。

那么这门堪称传奇的数学课程究竟有多难呢?

哈佛大学的『数学 55』课程是什么样的? — 李星河 的回答 - 知乎

http://www.zhihu.com/question/20953190/answer/66890187

也就是说,这门传说中美国最难的数学课,也就相当于北京大学高等代数大一水平。

美国顶尖学生学 AP 物理 C 到电磁学乃至核物理的水平很深吗?美国顶尖学生学 AP 微积分比中国学生强很多吗?

Too young, too simple.

以下为国际奥林匹克数学竞赛获奖历届总分第一(

https://zh.m.wikipedia.org/zh/%E5%9B%BD%E9%99%85%E5%A5%A5%E6%9E%97%E5%8C%B9%E5%85%8B%E6%95%B0%E5%AD%A6%E7%AB%9E%E8%B5%9B

  • 2000 年:中國 8
  • 2001 年:中國 9
  • 2002 年:中國 10
  • 2003 年:保加利亚
  • 2004 年:中国 11
  • 2005 年:中国 12
  • 2006 年:中国 13
  • 2007 年:俄羅斯 2
  • 2008 年:中國 14
  • 2009 年:中国 15
  • 2010 年:中国 16
  • 2011 年:中国 17
  • 2012 年: 韩国 1
  • 2013 年:中国 18
  • 2014 年:中国 19

嗯,不用仔细找了,我替你看过了,这里面并没有美国。自 1977 年以来,美国一共就拿过三次第一。事实上,中国截止 2000 年获第一的次数就是美国的两倍了。

上面的这些案例告诉我们了什么呢?

美国中小学生的数学教育与中国根本不在同一等级上。

甚至于说,美国顶尖学校本科数学专业学生的数学能力并不能比北京大学生命科学专业 / 地球物理专业学生更高。

-——————- 有赞再更 ——————–

知乎用户 我凌​ 发表

可能是因为哪怕爱因斯坦这种大牛最后也得去美国做工作吧?

知乎用户 badfatraccoon 发表

其实美国人学的并不是简单,只是思路不一样而已

中国是继承苏式教育了,苏联人觉得反正数学发展到现在,不需要再按照人类认识数学的步骤重来一遍了,可以从基础开始一层一层直接往上盖,具有比较强的严谨性。美式的话会还是按照数学被人类慢慢认识的过程去学习,对数学概念认识相对直观。这两种方法算是各有利弊吧,但是没有必要产生什么优越感。

另外美国高中提供不少选修性质的进阶课程,AP bio, AP calculus 啥的,这些都是对标大学课程的,中国的数学是面向大学统一的必修,自然和选修不能平行比较。

还有个问题就是,不要片面根据电影电视内容来判断美式教育的水平

知乎用户 Burgundian 发表

教育及社会管理理念,中美两国完全不同,中国是把所有人推上平均水平,美国是因才而定,不是那块料就不强迫。

所以,我们有非常好的普通劳动力,美国有很好的领军人物,两国贸易战前的分工,就是教育分野的结果。

现在美国社会动乱,很大程度上跟教育也有关,很多底层人士,实际上早已无法满足现代化制造业的要求,某些族群早就废了,这个教育理念着实值得反思。

至于天才,我觉得天才与教育关系不大,给予足够的条件,天才总能出头,而烂学生真的要花很大力气。

知乎用户 无名医师 发表

很简单。

美国已经比我们先发展了 100 年了。

100 年前,1910 年,我们在抗日,美国已经发展了 100 年了。期间虽然打了几次贵族内战,但实际上无伤大雅。书上写得热闹,实际上还是贵族内战。

1949 年,我们终于成立了新中国。在这之前,中国的家底儿早就被列强掏了一遍了,剩的都是断壁残垣。

我们用 70 年时间,从只有美国经济体量的 10%,到现在无限趋近美国,是通过艰苦卓绝的努力,以及现在良好的教育实现的。

美国能做出超级牛的东西,是因为美国用了 170 年,我们只用了 70 年。而且咱也有很多超级牛的东西,现在未见得就输人家太多。

中国的教育制度事实上是先进的,之所以现在看起来有很多人鼓吹素质教育,就是因为前浪们想要人为塑造阶级壁垒,让新的前浪慢点儿、困难点儿产生,让自己的后浪能浪得更顺当一点儿。

中国做出卓越科研贡献的人,大多数跟家境优渥没有太大关系,都是普通家庭出身,付出比别人多得多的时间,扑在科研工作上,筚路蓝缕,互相扶持,才有了我们现在的科研成就。

所有攻击现在教育制度的,想要在应试教育上撕开口子的,多半都是既得利益者。素质教育的结局就是阶层真正固化,跟所谓的国力强盛没有一毛钱关系,跟无产阶级没有一毛钱关系,跟大多数普通人没有一毛钱关系。

它的存在,它的壮大,只会不断挤占你的教育资源,不断掠夺你的优质资源,不断吞噬国家财政。

你想让富人拿钱出来,给大家一起花,那你真是想瞎了心了。不能考验人性,不能期待人性,还是需要坚定不移地,脚踏实地地向前才可以。

真正的素质教育,应当独立于学历教育。即进行职业技能认证,而不是给简历镀金。

知乎用户 老汉​​ 发表

我弟在美国读博,通过和他的交流,我大致说说我的看法。

机制不同。

用一个比喻吧。

把这件事比喻为爬梯子。

国内是群体一起爬一样的梯子。

国内的家长给孩子们能做的事情就是提升爬到梯子顶的速度。缺点是勉强了一部分人,也限制了一部分人。优点是强制的降低了文盲率。

在美国那边则是梯子是不同的。

既有高的梯子,也有小的梯子。

明码标价,所以那边的教育与投入是对等转化的。

通过不同的投入,让不同的教育资源与人进行适配。

缺点是有相当部分的美国人文化程度很低,优点是大量 Nerds 可以通过奖学金和贷款等形式获得非常好的教育资源,并且后续也有非常成熟的机制转化成收入。

要不然为什么我弟在美读博呢,的确是教育资源强大,这一点应当肯定。

知乎用户 HaloCap 发表

两个原因

一是因为美国吸引了全球的人才,不用自己从头培养。全球那些有天赋的人,不小的概率最终还是为美国服务。

二是因为中小学里学是初等数学。真正有用的高等数学,在美国不管是教材质量还是授课老师的水平,目前为止都高于中国。虽然太平洋两边大部分大学生的高等数学学习能力都还是挺差的,但好的教材和老师能提高学生水平很多很多。中国高等教育完全是用学生人数基数大来掩盖了各种短处。

知乎用户 jinseng 发表

能做出牛逼东西的又不是普通人。

普通人中小学就是数学学得再牛逼也是普通人。

知乎用户 知乎用户 Ww6v7m 发表

注意,做出最多牛逼東西的是 40506070 年代的美國人

而那個時候美國教育並不是今日

而現在美國人才尤其是科技領域已經出很大問題了,年輕人不願意接

都去搞金融了

知乎用户 Arondes​ 发表

人家的天才儿童

也是接受超常教育的

数学竞赛也一直都有 难度并不低

还有一点 就是美国高校在博士阶段 吸收了很多其他国家的顶尖人才 付出的只是一点奖学金 这个我国比不了

知乎用户 Boris​ 发表

这属于风马牛不相及的事。

真正做出超级牛的东西的绝大部分不是美国基础教育培养出来的,而美国基础教育培养出来的庸才比比皆是。

注意,美国是以全球吸血的方式攫取天下财富,再豢养从全球各处搜罗来的学者文人,其研究成果跟美国的基础教育没啥关系。

而美国的基础教育做为资本垄断的牺牲品,早就建立起财富划分的篱笆,把贵族和平民清晰的划分开来,穷的永远穷,富的永远富。

知乎用户 王一点寒​ 发表

史记《吕不韦列传》有一句名言,“此奇货可居”。

吕不韦是中国历史上最成功的商人。在最成功的商人眼里,最能盈利的商品不是商品,而是人。

我至今清楚地记得,当阿甘跑过橄榄球赛场时球探那两眼放光的眼神。

题主只拿数学作了比较,其实在许多方面,美国人都善于人尽其用。这在演艺界,在体育界表现得更为突出。

美国人是把人才当做可居的奇货来对待的。

在遴选人才方面,美国人做的确实不错,可以说是不拘一格降人才了。就拿 NBA 来说,看看对选秀对象的各种分析,简直跟选种马一样。很多都是多年跟踪的。再加上 NCAA 以及发展联盟等各种平台,基本上对一个人从身体到技术,从心理到性格会有一份完备的报告。为什么美国人在寻找,考察,评价人才上会花那么多心思,投入那么多?因为人才能给他带来收益。这种以盈利为目的的选拔最直接,最纯粹。反观中国体育,对人才的选拔范围非常小。中国足球人说的最多的就是中国足球人口少,注册人数还不如人家几百万人口的小国。而中国球迷发的最多的牢骚就是十四亿人竟然找不出十一个会踢球的。这就是没有人愿意去投入,去搭建人才选拔机制。资金下不去,就吸引不了人来从事足球相关的基础工作,人口基数就上不来。大家都在做终端,没人搞开发,这就导致下面根基浅薄,而上面却工资虚高。再加上人情社会,真有能耐的不一定给机会,关系户却可以获得资源抛头露脸,水平就更上不去了。

说回数学,首先美国人能做出超牛的东西还是因为美国的科研环境本身就领先。有一批学术大拿,有充足的资金,有成熟的市场,可以让人才更好地成长,做事,以及成果转化。这是一整套产业链。美国人之所以选拔人才的原因是有一批可以利用人才赚钱的产业存在。比如华尔街,比如各种科技公司。数学是基础科学,无论是金融模型还是最近比较火的 matlab 等专业软件,想赚钱都离不开数学人才。在美国的资本家们眼中,人才是宝贝,是钱。这让他们如饥似渴地寻找人才,并给他们提供好的环境来出成果,最终目的是给自己赚钱。我记得任正非也曾给俄罗斯的一个数学人才提供过帮助。包括阿里巴巴蒋凡,华为早期的李一男。这些人才如果没有平台,出不了这么大的成绩,甚至许多人才还不善于融入社会,比如《雨人》中的雷蒙,艾伦图灵以及上文说的阿甘。但在资本家眼里,他们所需要的不是人才的社交属性,只是他们的专业属性。资本家不在乎人才会不会拍马屁,或者有什么后台,他们要的就是能出成果,像谢尔顿一样。所以,真正促使人才能够脱颖而出的还是最终的平台。有了平台作为需求方,自然有人去给平台寻找和培养人才,因为人才,真的是奇货可居。人家可以满世界淘宝一样找人才。

美国能做出超牛的东西可不止全靠美国的人才。我们清华北大的人才不都跑人家那里去了吗。我们的好学校是掐尖一个地区的好学生,美国人是掐尖全世界的人才。所以,美国人有牛的成果靠的不只是美国的基础教育

高赞已经说了美国针对特殊人才有特殊教育通道了。我就不再多说了。这就是美国的精英教育。不同于我们全国教材大致相同,中高考大致相当的统一化教育,美国是有精英教育和平民教育之分的。我们的许多专科人才都因为英语被耽误了。这一点我是非常不赞成的。英语作为一门工具性的语言,不应该在中高考中占这么大比重,语言天赋和理科思维有时候是相悖的,许多人都有这样的感受,学起文科来顺风顺水,但学起物理来怎么也听不懂,比如韩寒那样的。同样有许多人,物理数学很厉害,可语文英语就是很差。在我们国家,前者还好一些,后者基本进不了好大学,因为三大科有两门语言学。这在人才遴选上就造成了大量浪费。英语这东西作为语言,作为交流用具,连最傻的人都能学会使用,无非就是个环境问题,只要人才需要,搞几个集训,实在不行放国外进修两年也就会用了。可我们却把它作为一门学科来研究和考试,完全违背听说读写的语言学习规矩,上来就是读写,听说却占比很少,十几年时间也不能让人学会应用,反而造成了人才的大量流失,实在很失败。

这是我们的数学人才和美国的数学人才所面临的环境不同。

说了美国教育对人才选拔的动力和环境,还得交代一下问题中所说的中小学生数学比我们简单。美国因为是奇货可居式的人才选拔机制,只对对自己有利的人进行投入,其他人则自生自灭。因为搞科研成果,一小部分人才就足够了,其他人,没必要学那么多,我也没义务教你,反而你懂得越少越好。这也造就了许多人说的,美国普通人数学不太灵光的现象。而我们的普通人基本运算都非常灵活高效。

总的来说,美国对人才的选拔更功利,目的性更强,导致旱的旱死涝的涝死,而我们则一刀切,长得慢的我使劲往上拔你,长得快的我各种条件限你。各有各的好处。我是真的希望在人才选拔上我们能学习一下美国的这点长处,看到人才的可居性,看到投资人才不仅不会亏本,还是回报率最高的商品。把人力真的当做资源来对待,去挖掘,开采,运输,提炼,生产,销售人才。这才是商人的最高境界。反观我们那些身居高位和掌控资源的人,则更倾向于把资源和权力粗暴地传承给下一代,实在是资产增值的最懒,效率最低的方式了。

知乎用户 郭昊 发表

中国只有搞竞赛的一批人完整的学习了数学思想和现代数学,所谓一般的 985 学霸,中学学的都是远古数学,和现代数学有一条数理逻辑的鸿沟,加上中国语文教育的祸害,在大学数学学习上其实是磕磕绊绊的,浪费了很多精力走了不少岔路,相当一部分理工科学生都是硕士博士甚至毕业多年以后才搞懂一些现代数学基础知识。

知乎用户 Sarah 发表

“美国人却还能做出超级牛的东西?”

不知道这个问题里说的那些做出超级牛东西的美国人,里面有多少是移民二代,又或者是读到研究生阶段才去美国的人?这些人的中学小学根本就不是在美国读的。

很多移民是非常重视教育的,既会给孩子挑学校,有条件的还会给孩子开小灶。

至于读到研究生阶段才去美国的人,基础教育,大学教育,都是在其它国家。

知乎用户 枇杷李​ 发表

能做出牛逼的东西,不仅仅是依靠教育就能成就的,它是良好的学术环境,工业基础,创业环境,可用资源,试错容忍,上下游生态等等多重因素叠加的结果。

美国教育牛逼的本质,除了顶尖的确实牛逼之外,更在于它能吸引全球最牛逼的人。美国教育的强大,更是因为全世界教育强大的结果。不要小看它整合全球资源和人才的能力。

国内数学这么牛逼的这些学生,你能肯定他将来不是在美国发展?

知乎用户 往事随风 发表

说一个和数学不相关的回答。我专业体育学,2016 年我有幸跟随学校参加一个国际体能峰会,峰会规格较高,北体,武体,上体,国内顶尖的体育院校都派了代表。

台上主讲的是一个博士,教授老外,老外兴高采烈的和我们分享了最新的训练成果,多功能身体训练。

老外先和我们分享了,经过训练后受试人员身体的各项指标的变化,然后分享了受试者的体型变化,以及他们对该训练方法的看法,推广的可行性。其中有很多涉及专业名词和实验把控方面的东西我就不细说了。

老外讲完后,示意台下的听众可以提问了。接下来一个专家,问了一个让我无比尴尬的问题。我至今无法忘记,作为知名体育大学的教授她问的问题不是实验的漏洞,实验的疑惑而是,你为什么要把这个训练叫做多功能身体训练… 真的…… 台上的老外看着这个教授…… 满脸无语。人家致力于如何提高人体极限,如何更简单有效的给人类带来健康。而我们还在为了名词解释争论不休。

知乎用户 五角场喵叔 发表

衡量一个国家的国民生活水平,应该花大气力提高人民的下限。

衡量一个国家的科技水平,应该话大气力提高精英的上限。

再穷不能穷教育,道理有,但是这个资源怎么分配是个问题,我们要面临一个大的基本前提,任何资源不可能无限提供,教育资源也一样。

而且我们也不要去任意的猜想,普通人的对科技研究本身的需求和渴望,很多老百姓只不过是想通过教育而找到一个合适的饭碗而已,在这个前提下,一味的训练做题,还不如能创造出更多普通人的岗位 - 这样来的有意义。

所以说我感觉在人民生活上,要拿出提高教育群体下限的劲头,来提高每个人的生活质量。

在教育上,要拿出生活中制造顶级富豪的劲头来,制造有个性的顶级学霸,并且能留住人才,能为我国家所用,这样比起普通孩子玩命做题,结果社会上也就多一些外卖员的工作。。。。。不是更加有实际意义嘛?!

知乎用户 贾森​ 发表

原因很多,但是咱们的基础教育复杂,其实很大程度上源于经常要求学生一种用 “过时的” 技巧去死磕题目。

人为什么是生态链顶端?因为会发明使用工具。

可中国基础教育里,初中一元二次闭着眼睛级别能做的,非让你拿着小学一元一次的工具,耗费高中函数级别的推算把题给解了,美其名曰锻炼逻辑思维。

这就很不人道,也没啥太大的用处。

知乎用户 华商韬略​ 发表

文 | 华商韬略 斯钦

近年来,中国芯片屡遭 “卡脖子” 的严峻现实,让国内掀起了投资半导体产业的热潮。

面对热潮,华为任正非却冷静地指出:中国芯片产业要想发展,光砸钱不行,得砸数学家、物理学家、化学家。要踏踏实实在数学、物理、化学、神经学、脑科学各个方面努力去改变。

在这段对话里,数学,被任正非放到了绝对第一的关键位置,被提及了多次。

数学对于华为而言,无比重要——华为的 5G 标准,就来源于 10 多年前一位土耳其教授的数学论文。

与任正非一样重视数学的科技大佬,还有马云。

虽然马云第一次高考,数学只得了一分,但是马云多次在公开场合强调:“数学非常重要”、“数学是一切的基础”。并亲自举办并主持阿里巴巴数学竞赛,设置了最高达 2 万美元的单项奖金,用以激励人们对数学的重视。

中国拿了那么多奥数冠军,却没有一个人获得数学界最顶尖的菲尔兹奖。中国人在奥数上的手下败将美国人,却有 13 人获得过菲尔兹奖。甚至于近邻越南,都能在 2010 年获得一个菲尔兹奖。

这与 “中国学生奥数能力完胜美国学生” 的报道,形成了强烈反差。

数学大师丘成桐认为,如果不重视数学等基础教育,不重视学风建设,“中国科技的发展至少退后 20 年。”

英特尔创始人、“硅谷之父” 罗伯特 · 诺伊斯,拥有数学、物理双学位;苹果创始人之一的史蒂夫 · 沃兹尼亚克是众所周知的数学天才;操刀苹果 M1 芯片的总设计师 Johny Srouji,高中数学成绩一直是满分……

美国人在二战时期就已认识到,一个一流数学家的作用,堪比 10 个师的部队。从 1920 年开始,美国就成为了世界数学中心。直至现在,全球前十的数学学科顶级大学,美国就占了六个。

正是中国与世界领先国家在基础学科上的巨大差距,让中国在 2020 年年初提出了 “强基计划”,从国家层面正式确立了数学及其他基础学科的关键地位。

2020 年 12 月 4 日,中科大向世界宣布:潘建伟等人成功构建了一台 76 个光子的量子计算原型机 “九章”,让中国成为了全球第二个实现 “量子优越性” 的国家。

“取名‘九章’,是为了纪念中国古代著名数学专著《九章算术》。” 潘建伟说。

因此将这台量子计算机取名 “九章”,就有了唤醒国人对数学的学习热情、重回数学巅峰的积极意义。

达芬奇有句名言:“数学是一切科学的基础。”

对于中国来说,想要在科技领域有所突破,解决 “卡脖子” 的现实问题,发力数学等基础研究,已刻不容缓。

——END——

欢迎关注【华商韬略】,识风云人物,读韬略传奇。

版权所有,禁止私自转载!

部分图片来源于网络

如涉及侵权,请联系删除

知乎用户 三有苹果 发表

美国大学之前的教育,如果单纯理解成快乐散养的公立教育,以及死命刷题的私立教育,那是有点片面的。在我眼里只有高质量的教育以及低质量的教育,而前者,很昂贵。所以但凡有中低层收入者负担的起的相对高质量的教育机构,很多人趋之若鹜。有些人把美国的牛逼初高中想像成衡水那样的了,但其实人家在刷题和快乐之间找到了一个平衡点。

我举一个不算最恰当的例子。

“贵族 “学校 VS 普通公立学校,化学课;就算两者完全使用一样的教材,” 贵族“学校也有更多资源能去满足学生的实验需求。能做更多实验,是不是化学课也变得更快乐更有趣一点了呢。

再举个例子,不少美国 “贵族” 学校用的老师都是博士(我注意到国内也有老师高学历化的趋势),讲课先不说,人家的知识储备已经超过大量普通公立老师了。而只要钱给的足够多,完全可以找到讲课风趣幽默的博士。这样,是不是上课也更快乐了呢。

类似这样的情况还有很多,你觉得中小学生数学简单,一方面很多人他就是来自低质量的教学机构,出身限制了他们更进一步;另一方面,虽然美国科技发达的原因还有很多,但接受优质教育的美国人,掌握的知识的比你只多不少(取决于题主的学校)。

希望中国除了校规校纪的管理方面以外,大学之前的教育体系 + 政策不要学美国。


推荐两本书

《特权:圣保罗中学精英教育的幕后》讲了一个贵族学校的故事,有中文版所以我放的中文标题

还有 Diane Tavenner 的《Prepared: What Kids Need for a Fulfilled Life》。讲的是作者创立一个质量比较好的特许学校联盟的故事。就是我上面说的 “中低层收入者负担的起的相对高质量的教育机构。”

知乎用户 一天到晚游泳的鱼 发表

中学不知道,小学数学不简单的。最近因为众所周知的原因学校改成了网课,有机会听了几次,觉得更多的是教的方式不一样。举个例子,三年级数学,教几何的基本概念,老师从等边三角形入手引入角和边的概念,问学生们加一条和原来的边一样长的边后是什么形状、有几个角,逐渐引入四边形、五边形、六边形、七边形、八边形的概念,最后问小朋友们如果持续这样加下去会出现什么,小朋友们很自然地就想到了是个圆。老师很有耐心,小朋友们的参与度也很高,效果很好。

知乎用户 格陵兰帆船 发表

研究不是靠中小学知识做出的。

中小学知识不过是对普罗大众的教育,未来的研究者不能指望这个东西提高自己的知识水平。

举个例子,USA 的 IMO 代表队可并不弱。

知乎用户 水烛 发表

我看回答的都差不多了,我个人觉得题主的问题有点不对。

影响 “制作出很牛的东西” 原因有太多太多,文化、政策、学术氛围、法律、个人意识、教育理念等等其实都有关联,一个数学水平,只能说是其中一点,并不能得出 “由于数学 XXX,所以 XXX” 的结论。

我贴点美国人自己看法!!

他直接跳开了教育,而直接指向了其他方面。

他提出第一个就是人口,美国人口在欧美来说算是比较庞大的,所以虽然平均起来教育程度不算高,但依然有大量人才。而虽然中国和印度人口更多,但他们受制于下面两个方面:钱和国际影响。

他提出的第二个就是钱,好的科学是要大量金钱的,而美国不仅有钱,而且肯投入,并且基础研究设施都已经完善,钱也能用在刀刃上。

第三点:国际影响。

他认为这点非常重要的就是美国是英语国家,而英语又是通用语言。

这就造成全世界优秀人才都能在美国大学学习。

(而在中国的大学,首先要学中文这一点就难倒了很多外国人。)

美国作为世界中心,有良好的开端,只要一开始有学术成就,就会吸引全世界的人才聚集,就像滚雪球一样。

所以,他说了一句话 “这种方式完全独立于他们的教育体系”。

总的来说,这位博士认为,美国是全世界唯一一个在人口、金钱、国际影响力全占的国家,造成了科技的发达。

当然,我这个回答可能有些偏题,纯粹只是想换个角度告诉大家影响我国创新能力的原因很多,数学只是其中一条而已。

尤其是我看到许多私立教育机构在后面回答的,简直就是在打广告。。。。。。

知乎用户 敌方炸弹小飞机 发表

有一件反国内常识的事儿是,很多美国人的理科基础其实根本不差,而且概念理解的特别透彻,能活学活用。至少我身边的美国同学是这样的。 当时刚到美国颇为震惊。

至于那些初高中就毕业工作的美国人,他学很难的科学也没用啊。为什么要强迫他们学难的数学。

我想趁机夹带一点自己的抱怨,可能是国内的教材就是愿意把简单的东西写的很难理解,这可能让你觉得国内学的难。

—————

典型的错误想法是,认为美国牛的原因是国内的人才都跑出去贡献了。(错)

人们抱怨人才流出,但是从未想过原因。这也解释了,为什么厉害的人愿意出去深造,因为外面真的能给他们深造的条件和提升的机会。

事实是这部分人本来就很聪明,国内的教育体系在某些程度上甚至限制了他们的发展。 这些人的履历一般都是国内最好的大学 + 国外博士学历,这段国外学习的经历尤为重要。假如他们一开始就在外面学习,说不定会成才更快。

———————9-13 更新————-

我发现个问题,美国人的教学理念是学会够用就行,有些美国教授不介意给每个学生 A。 中国人的理念是,学会还不够,解题技巧和片难怪得上去拉开档次。 这也是为啥国内学生会有优越感,说我们学的难,美国人都是傻子,海外 GPA 水。 还会比较说中国学生考试过 90 的很少,但是美国却有一堆 GPA4.0,以此推出美国是水的,傻的,低级的。

另一个区别就是计算器的使用,美国人认为我能两秒按计算器,或者用编程,为啥要手算? 中国人认为,我能手算还不够,为了显得我聪明我得心算!不仅要心算,我还要比谁算得快!

对于学时这件事儿,美国学生很喜欢说,我在学习上花了很多时间,真的很难很累。 中国 “学霸” 会说:啊 我不学,反正就是会,或者夸自己同学太牛逼反衬自己也不差。

归根结底,是国人特别喜欢当人中龙凤的人中龙凤了,这会有种比较虚的优越感,感觉别人都是傻子。别的都简单,别的都低级。 但是人生呢?是不必把力气都花在真的没用的事儿上的。

知乎用户 茕茕孑立 发表

美国人长期贸易逆差,也就是其国际贸易的经常账户是日常赤字的,如果换一个国家,这个国家的货币早就崩了。但是,美国没有,那么美国是如何平衡他的货币账户的呢?

货币霸权?甭扯了,美国如果真的平衡不了他的货币账户,他就是拿枪顶在别人脑袋上,维持一时可以,维持二十年三十年,货币也得成废纸。

真实的情况是什么?移民啊!一个人取得美国身份,什么概念?这个人的全部家庭资产就从一个国家的资产账户转到美国的资产账户了。就每年这些中国、印度移民去美国带过去的资产,最后就把美元的贸易逆差平衡了,你说恐怖不恐怖?

数学家也一样。那些中国最聪明的脑袋,还有印度、越南等等,全都捐给美国了。

最后说句招封的话,希望有生之年能看到,算了不说了,被封了不值当。

知乎用户 A 小姐姐 发表

辅导美国小学材料 go math 了好几年,G1-G8 都有,说点我的看法

美国小学的数学教育在初期非常注重理解,引发学习兴趣以及建立学习信心。

课程的安排是由浅入深的,而不是我小时候的一个板块在一个固定时间要从简单到难在很短的时间(比如 1 个月)学习完毕,我至今都记得我初中几何那几个章节学完之后的沮丧感。。感觉这么有趣的东西就要这样离我而去了。。美国课程是一个概念在 g1-g3 这 3 年不断的出现,由浅入深,随着孩子的年龄增长,孩子的理解能力加强,对这个板块的概念就会理解的非常扎实已经容易变通。

相关联的所有课程都会发散性的伴随其中,而不是单独的一个一个板块像任务一样完成。

美国小学数学对数学的理解建立的有多么扎实,举个例子,g1 学习每个数字的价值,建立 ones,tens 等观念,由最简单的数方块,通过触摸摆弄现实存在的东西对每个数字的理解,再加深到对个位和十位的理解,再到加减法中个位和十位的关系,再到 2 位数加减法中个位和十位的关系;g2 会有两位数,三位数中每个数字代表的价值,3 个数,4 个数,5 个数相加对个位和十位的变化来理解个位十位的关系,加减法中因为个位数的变化导致十位数增长了,或者因为减法的关系十位数不得不变成个位数,再代入到百位数和千位数去学习;g3 为了加强学生的数学逻辑感会用简单几何进行乘法运算,让孩子对面积和乘法的逻辑变的十分清楚。。穿插金钱的利用,不同的钱之间的关系,不同国家的金钱的简单换算,让孩子明白钱是一个抽象的数学概念,不仅仅是手里的两张纸;通过找图上的点来寻找不同坐标的点之间的关系,以及数字的抽象和几何概念。。等等等等

我小学的时候老师就说如果 a+b=c,那么 c-a 就等于 b,为什么,没有为什么,记住,代入。

也有老师会花心思说一下为什么,但是也就是一节课的时间,老师要完成这个概念,孩子懂 不懂不好说,更重要的是以后就再也米有这方面的说明了。

我就记得我辅导 2 个不同的 G2 小孩,一个孩子真的说完之间的关系很快就理解了,另外一个孩子用了 3 堂课,数方块,数饼干,换不同的数字进行计算,还是不明白,教比较小的孩子理解抽象的概念是需要非常耐心的,有的孩子抽象概念非常容易建立,有的就是很难。最后终于理解并且多次验证不会出错,她变的十分开心,而我也是一头汗。。觉得好心累。

但是理解的快的那个孩子非常粗心,而且不在乎结果,做错了就做错了;理解很慢的那个孩子非常喜欢抄别人的答案因为非常在乎结果,而且十分细心(美国小学经常组队完成课堂任务,这孩子每次作业都完成的十分好,但是单独的作业 80% 都是关于理解的,这孩子的缺点就暴露出来了)

我觉得那个理解很慢又细心追求结果的孩子,如果在中国学校,可能要到很大了数学才会暴露问题,但是那个时候再补课孩子的学习压力就很大了,而且也会对数学有难度恐惧,幸运的是她很早就被发现对数学的理解能力不好,美国数学又不会有一堆难题专门去为难孩子,训练孩子,这孩子的数学信心慢慢被建立起来了,变的敢问敢自己尝试之后错了,找出问题再改。

而理解快的那个孩子我觉得在国内学校可能也会被弄的比较惨,因为答案出错率非常高,而且不能理解的就没有耐心变的乱写,幸运的是美国小学数学作业每次就那么一点,每个部分都照顾得到,她可以从她熟悉的部分得到快乐和信心,又有足够的时间可以多练习(因为每天的作业少)。

看到这里有人会问(我的朋友就问过)你看美国小学的教育方式孩子就是学不好。我想说 1 是补课的孩子肯定是有些问题的,有的家里父母不具备英文水平所以孩子从小有不会的就没办法问,后来才找的补课,有的是爸妈没空孩子学习习惯很差,这些都需要培养,但是问题早暴露,保持学习兴趣和学习信心,都比小学的时候每次考试得 95-100 分永远看不出有问题来的好(个人意见哈)

我们从小学习数学就是各种公式的带入,简单的时候完全不需要理解也从未想过要去理解,数学次次 100 分,老师和家长都说如果错了就是你不够细心;初中有简单几何的时候其实我本人是开始对数学第一次有了那么点儿兴趣,觉得真的非常有趣,小学代入公式的全盘送分题我就是那个经常拿 98,99 的,初中的简单几何次次满分,真的非常有信心 – 但是很不幸很快这个信心就被打破了,因为初中开始数学老师开始培养数学的缜密思维,不停的强调数学是一门缜密的科学,一个点的错误就会导致全盘皆输,每次拿到题目就如临大敌,做完还要不断的检查,每次考试都感觉像我这样偶尔犯错的孩子数学简直是灾难;高中开始这种灾难变成了灾难之海,大量的练习感觉对着门课在被慢慢培养成一个不能出错的机器人,而一门不能出错又没有意思不太能理解还有大量题海的课程,我只求赶紧度过这个阶段;大学一开始学微分我是十分 high 的,次次做第一排,我还记得数学老师一开始很 nice 的跟我说,不懂就问孩子,然后我真的什么不懂就举手,往往每节课老师都因为我不停的提问和课后问题感到很苦恼,最后我终于懂了,在高中我就没有理解过我所学的数学,到了大学这样的理解对我来说变得更加难。。。我有时候感觉数学老师看着我的眼神都带着悲哀和怜悯。。你怎么就那么跟自己过不去呢。。

还曾经有个 g5 的孩子,虽然她补课过程中很多问题我都轻易的能用我从小的数学学习方法很快给她提供办法,但是理解这个办法每次都是她和我速度差不多,她每次课堂上学的理解知识都 80% 可以理解,我只需要解答她不理解的地方,再辅助一些练习加深印象即可,我经常对她的数学能力感到十分惊喜,并且她虽然数学不是最出色的学科,但是一直对数学学习表现出了巨大的兴趣,对于她完全不理解的知识也很少觉得恐惧,只需要大人引导一步一步去解决。

并且她的外国父母每次也都说,你看她对数学一点都没有难度,她只是需要去面对这个并且解决啦,我们对她的数学很有信心。然后就丢给我了。。这点我也觉得很赞。

这样的孩子如果之后接触到新的数学知识,我觉对她的理解能力会有非常大的信心。

如果这样以理解为主的学习下去,我也相信一旦接触到更深层的数学她会具备更高的数学学习能力,培养学习能力才是最重要的。但是我小时候感觉这方面学校给予的很不够。

再加上 g8 学完没多久就可以接受 ap 课程,提前接触大学的一些课程,我觉得对大学数学课程的理解和学习会有非常好的辅助作用。。说实在的我都有点羡慕这种学数学。。

探索,创新的学习是真的有乐趣,试想从小学学一些有趣的课程都直接告诉你答案,然后让你背下来,往里代,真的很枯燥哇。。到大学了突然让孩子变成可以自己去探索,然后最后能自己来一篇大大的论文。。

ps 美国小学虽然数学不会一遍一遍的要求孩子做题,不能出错,保持细心,但是在其他课程中注重讨论,各抒己见,团队协作,g3-g5 一支培养孩子探索学习,g5 开始每 2 天一定会有一篇调查研究/讨论/描述的作文,很多课程都是大家讨论学习而不是直接给答案的,答案甚至可以都不是唯一的,对他们之后学习数学所必要的论证性方法,发散性思维,严谨性态度,其实都有非常重要的影响。。反观我小时候反正只听各种老师强调来强调去。。要发散,要仔细,要严谨,要论证。。但是实际上答案都是唯一的,结果都是重要的,过程都是算分的,不按照模式来都是扣分的。。到了该自己探索的时候我往往略迷茫,真的不知道从哪里开始。

辅导了这么久我确定,思维是可以被训练的,能力是可以被培养的,越早开始越好。没有人的各项指数是一成不变或者很难改变的,你可以想象成每个技能都是一个技能条,不断的练习技能条从 1% 走到 100% 的过程不是很有趣么,可是如果只有做题一种方式能锻炼技能条,其实锻炼的也只是一个技能条而已。

个人看法啦。。并不是说国内就不好,如果同样的学习压力之下国内的学习肯定是最高效的,这是真的。

知乎用户 小刘 发表

美国人给你一年 20w 刀乐你去不去

你数学肯定比大部分美国人好

这不就解决问题了吗

所以这个问题的根源是美国的钱是哪来的

神神会告诉你,是美国对资本家管理到位,所以工资标准高 // 安然和高盛先后点了赞

兔兔会告诉你是美国用美元霸权收割全世界 //08 年次贷危机表示小意思

当然具体信谁看你自己了

知乎用户 愚不追日 发表

你女朋友拿一段美女视频给你看,性感火辣。

然后你女朋友问你,这个美女穿的什么颜色的鞋子。

你没回答上来,于是你女朋友一哭二闹三上吊,因此你给自己打了预防针,要记住鞋子的颜色。同时你被惩罚睡几天地板不能碰她。

过了几天惩罚期到了,你老婆又拿来一个美女的视频给你看,性感火辣!

你第一时间就看完鞋子准备答题了。

果然,你女朋友又把手机拿开,但是她并没有问鞋子的颜色,而是问美女穿的上衣编号是多少,你懵逼了,你女朋友又一次一哭二闹三上吊,你决定以后不看美女了,专门去记这些稀奇古怪的东西就行了,虽然再次被惩罚几天不能碰她。

但是你很生气,明明你很爱她的,她怎么就搞些莫名其妙的事情难为你呢?

这次你并没有期待下一次的考验,而是思考这些毫不相关的考验难度与爱的深度有多大的关联。

后来你明白了,你只是其中一个备胎。

能成为名义上的男朋友,也只是考得更近而已。

你想象的爱人,你们之间互相依赖,永不抛弃对方,眼中只有彼此,当她犯错的时候,你会像中国所有的直男癌一样,认真的告诉她该怎样才能获得进步,当你有难题的时候她如果不能给你帮助也能给你鼓励!于是你两同舟共济,一路偕行!

**实际上你的爱人,你与他们终日向她朝贡,她永远不会说 “咱们没可能”,只是说 “你还差点,努力吧!”。**当她犯错的时候,那问题一定是你承担,当你犯错的时候还是你承担,可是你的世界只有她,而她永远都不会说 “咱们没可能” 就是钓着你,虽然最后她还是只会挑一个人,当然更大的可能是找一个更新的人!

知乎用户 runner time 发表

归根揭底就是制度问题。

如果一个国家的天才少年无法被公平公正公开机会均等地发现和培养,并最终成为这个国家最尖端的生产力,引领这个国家和这个国家的人民继续前行,这个国家的制度一定是有问题的。而且,天才少年无法正确完好归位的时间越久,国家就越动荡不安,将直接导致政权的瓦解。

这个道理同样适用于其他阶层的人民,各尽其能、各司其职、各归其政,才能保证国家和社会的稳定以及长足发展。

我史书看得不多,如果有什么说的不对的地方,还望谅解。

知乎用户 楠楠 发表

在大学,某同学出国交流,回来之后谈到在美国上课的经验,说道:美国大学讲的化学,即使是没学过化学的人也能听懂。让我很有感触,个人认为,美国教学,知识非常基础,更注重学生自主学习能力,学生们上课时间少,上课内容浅,但是课下作业常常是开放性的,需要花费更多时间,阅读大量资料,经过自己思想的碰撞才能完成,他们完成作业的过程中锻炼了自己的综合能力

知乎用户 張子佾 发表

高中开始到的美国。

说实话 “美国人” 不都等同于 native。必须说仅就数学来说,相当一部分厉害的人也来自于半路移民或留学生,特别是中国、印度和巴基斯坦人。本地美国人里数学天赋高的人也有,但不算特别多,美国的教育方针是过滤掉对数学没有兴趣和天赋的人,所以大学前课程安排的确相当,简易。

但这并表明大家和数学就没有关系了。因为美国更注重逻辑,这哪怕很多社会学科也是如此。我们在写 essya 和 thesis 时,被反复强调思维和表述的逻辑性。最后能进入数学系 (或天文学系) 的的确是少数,但他们将来如果走团队开发,会需要周围人沟通时启用到最基本的逻辑,这个比数学模型更重要。甚至,最简单的,对话 (dialogue) 要复合逻辑性 (logical)、多信息性(informative)、连贯性(coherent) 等。

最后,其实大部分的人的确不会在将来的生活中深一步接触到数学,但美国有一种尊重数学、尊重科学的氛围。银行的汇率浮动、政府福利的申请条件、加油站的计量转换、甚至机场的航班排班,一般的美国人不会随便对专业人士指手画脚 (仅仅基于外行人的感觉或直觉),而更愿意乖乖安静,享受数学和科学带来的成果。我个人觉得,这比运算飞快来的更重要。

知乎用户 半枝莲 发表

这方面可参考一位在中美 top 大学都学过,还亲自在美国大学任教的人对其相关教育经历的分享及探讨,他正好是数学方向。

在美国 7 年求学执教的见闻和感受
本人对理科教育一直很感兴趣,多次就此话题发帖。很多朋友对教育以及中美体系比较 甚感兴趣观点众多。现在我想简述一下我在美国七年求学执教的一些第一手见闻和感受。一是希望起到抛砖引玉之效,二是给自己总结一下,为自己回国工作后和人侃教育整理一下材料。
几点说明:
1. 我在 21 世纪初在北京某大学 A 学习,之后来美留学读博士(涉及学校 B C),再往后在美国执教(大学 D)。我对国内本科硕士,国外本科博士等阶段的基础科学教育(包括学习和教学)都有一定体会,在我的系列中也会对中美的大学做一些对比。
2. 我的专业方向属于
纯数学
(以及相当程度的理论物理)。我对实验类学科或工科了解很少,故无法涉及这些方向。但我教过不少数学课(大学里学生最杂的课),这使得我能大范围接触全校各个专业学生并由此感知他们的数学水平。
3 . 我求学的学校各自代表了中美的最高水平,我执教的也是老牌藤校。一方面我感觉到了一流大学确有过人之处,另一方面我也看到了一些令我大跌眼镜的现象。我感到这两方面的很多东西我似乎没有在中文媒体网站上见到较多的提及(虽然谈论建立世界一流大学的文章很多很多)。
在 B 和 C 的经历:本科
我是大学 B 的学生(研究生),但我的导师在大学 C 导致我常到 C 校去,故我对两校均有了解。
本科新生数学基础弱于中国同地位大学
B 有美国数一数二的理科生源并有很多顶尖国际生源支援,但我感觉他们的平均数理水平仍不及我在中国的母校 A 的(规模更大的)入校生源的水平(当然 A 的生源在中国也是数一数二的)。我认为这是中国基础教育的一个胜利。当然 B 的新生水准也是不错的,少有我认为明显配不上 B 的。
题海 / 考试海 / 教辅 / 补课 体系
尽管 B 有很好的生源,他却毫不懈怠,反倒响鼓偏用重锤敲。B 高度重视数学基础课并有自己一套方案:
除了通用的教材,还有两三套自编的教学讲义“教辅材料”;教学讲义和 “教辅材料” 各有侧重,相互补充,时有更新;大量布置作业习题并对习题精心组织分类;除去正课外,要求学生必须上一周两次的小班习题课(由助教上),而由于有超强的研究生生源这些助教(在读博士研究生)大都具备独立主讲全课程的能力(相比之下中国的大学 A 里甚至很多博士后都不能做到这一点);由于**具有极高的师生比例以及博士生本科生比例,B 可以为每一门大课组织庞大的教授 / 助教 / 批改作业者体系;教授和助教均提供很多课外答疑时间;相关院系,学生宿舍,学生内部均会组织 “学习班”“解题班” 等,有的学生甚至自己出钱雇家教教自己。
此外 小班习题课上 常有各种小测验(计入最后成绩);全体的大考试 则每学期(12 周)有 3 到 4 次(相比之下中国的大学 A 通常 16 周内只大考两次)。
开始我还没太在意,后来我突然意识到 B 的这套体系不是很像中国高中盛行的 题海 / 考试海 / 教辅 / 补课体系么?
那么这套体系运行情况如何呢?至少从出口看,效果很好:学生考试成绩不错,我教高年级课时也感觉学生的低年级课的基础打得好。学生的负担很重吗?那是很显然的。有时我都觉得没有必要搞这么多任务。但可能是学生已成年以及本身上进心强等方面的因素我没有听说过什么被压出严重心理问题的事(也可能我孤陋寡闻了)。
B 的这套本科基础课教学体系是 B 在北美诸校中脱颖而出的一大法宝,这一点我在到了大学 D 后有极深体会。
B 是让很多中国大学流口水 很多知识分子常挂嘴边的榜样,可是有多少人知道 B 的基础课教学体系是这样严酷(和类似中国高中)呢?我曾试图和来 B“考察高等教育”的中国高校人员 介绍 B 的经验, 可他不感兴趣。能调动起兴趣的是各种雕像 Logo 和特色建筑带来的 “人文气息” 和 学生活动招贴,草地休闲等体现的“自由学术氛围”。。。。。。
在 B 和 C 的经历:研究生
现在我讲讲 B 和 C 大学的 研究生教育情况。我只了解数学这样的基础理论科学,工科和实验科学的情况可能不大一样,敬请注意。
研究生学术水平远超中国同地位大学
这当然是来美之前我就预计到的,但来了之后我才知道究竟强到何种程度。
雄厚的学术背景
不少研究生在
刚入学时或入学后一年内就已经掌握了不少较深的理论
。两年以后,我见过的国内来作访问的一些 985 学校教授已经无法和我们讨论问题了因为有太多我们懂而他们不懂的东西。
B 精神原子弹
一个突出的感受 就是关于人的精神面貌。这里是我见过的人的精神力爆发性最强的地方。有一批为学术痴狂的学生 时不时处于唯恐自己学得太少太慢的精神状态。我也是其中之一。在来到这里之前,我觉得自己长期处于一种与周边格格不入的学术化偏执状态,来了之后我觉得自己不孤独了。我甚至很担心我的某些同学会过劳死。
惊人的效率
强大的背景和只争朝夕的精神造就了惊人的学习效率。和中国的 A 大学做个对比。比如在 A 大学 五六个好学生进行任务分解 折腾十二三个星期的讨论班(每周讨论一次) 可以理解清楚某一复杂前沿学术文章。在那时候我感觉良好得很,因为若把这文章交得我手里我一个人便能在相同时间内轻松搞定。而到了在 C 大学里 一个研究生中的猛将靠单干两三个星期就能拿下。当然我没多久也成了这样的人。但如果不是 B 和 C 的环境我恐怕还在沾沾自喜自己鹤立鸡群一个顶五六个而没有意识到自己的效率还有提高至少五倍的潜力。
D 本科生入学水平远弱于 BC
有些人在我看来需要重修高中乃至初中的一些数学课。个别人连小学数学(如分数的四则运算)都不熟!这绝不是我个人的偏激看法。比如学校出的关于新生数学能力的诊断性考试题竟然要考察自然数的四则运算, 分数的四则运算,一元一次方程等(当然后面会考到高中的复数 极限导数等)。而多数新生在这样简单的考题上竟只能拿到一半分。更可怕的是,他们中很多竟然敢于免修第一门微积分课(直接上第二门)。。。
教学困境
显然,在这样的情况下想让学生知其然也知其所以然已成了几乎不可能的任务。一些刚从象牙塔毕业的青年教师(我是其中一员)对此没有思想准备。我试图保住一条底线:最起码我要解释教科书上写了的东西让他们在学完后能看懂教科书。但即便这样低的目标我也根本达不到而以惨败结束。比如我一直耿耿于怀的一件事是:我教的微积分学生可能几乎没有人明白积分的定义。。。直到第二年 我才从更有经验的人那儿了解到:不要试图准确描述基本概念,也不要花较多时间解释主要思路动机或多问为什么(因为这对学生来说实在太难了。。。而且会遭到学生的强烈反对)。
那么是不是说 在所谓 “素质 / 启发性 / 创造性 / 实质性” 教学完全走不通后 只有 “填鸭” 教育一条路让学生会算一些题就行了呢?事实上由于没有大学 B 那样强的(类似于中国高中的)管控体系 我们连 “填鸭” 的能力也没有。
最终的结果是 通常已经有所放水的期末考试 常常是多数人无法及格。而且低年级的劣质情况积累影响中高年级,甚至导致系里不得不把一些 有多年历史的 中高年级的课程内容砍掉 1/3 以上(但之后仍是大多数人考不及格。。。)
我刚到学校时曾与系里一位老教授攀谈,当谈及基础教育失败导致学生数学能力极差且有越来越差之势时,这位曾上过战场的老头竟然控制不住 在我面前哭起来。我在教了好几门课后开始明白他的痛苦了。
学生有自由但反受耽误
另一件让我印象深的事是:D 没有让我觉得学术上拔尖的本科学生。到了高年级 确有一些功课学得很好也有心向学的人。我给过他们一些进一步发展的意见指导,但无一例外的发现他们学过的东西非常非常少 对学科的理解则基本还停留在新生和科普水准。在了解了他们在本科三四年中的学习经历后 我发现他们基本上都有几个问题:
1 在中低年级并没有完全想清楚自己未来要干什么因此花了不少时间去体验其他的专业以及很多 “通识课”。
2 以为自己所有的专业课都学得很好就行了。
这样做有啥问题呢? 问题在于他们被真正拔尖的孩子(其他学校的和外国的)甩开得太多太多了:三四年前大家的起点差不多,而现在别人 “轻舟已过万重山” 高出他们好几个境界去了。怎么会这样呢?这是因为最拔尖的学生具有很强的 目的性 (我一定要当科学家),自我指导性(给他足够的书,他自己就能钻研出来) 和 自我激发性(学得越好就越不满足甚至越焦虑)。
D 大学的好学生没有这么强 但是如果他们从大一开始 就坚持贯彻明确的专业培养计划他们和最顶尖学生的差距不至于这么大(起码学的专业课会多得多)。而现在我只能指出他们的学术基础太薄弱(很多基础专业课没学)并开出长长的课单书单。但我知道他们没时间了:毕业期申请期快到了,一些孩子也痛苦地意识到:自己看似没有什么问题的学术之路实际上导致自己已经输在起跑线附近(如果以成为一流学者为目标的话)。
坚持贯彻明确的专业培养计划 这其实是中国大学的办法。中国大学给学生转专业和定专业的自由远不及美国学校。这当然导致了众所周知的 学生不得不学不喜欢的专业的问题但也保证了次好的学生在愿意走此专业道路时 不至于落后先进太多。这些次好的学生(假设在中国最顶尖学校)的学术竞争力强于大学 D 的上述好学生(虽然 D 的科研和教授实力目前肯定强于中国最顶尖学校)。
我知道中国一些大学 近年来喜欢搞所谓 “宽基础” 的综合性强的 “XX 班”并赋予这些班的学生广泛的专业选择权。但我也了解到很多这样的尝试产生了不小的负面后果(比如事实上削弱了学生的竞争力),在有些学校甚至导致了公开的矛盾冲突。D 大学和中国的实践提醒我们 “宽基础”“通识”培养体制存在的弱点:最顶尖学生不会因此更厉害但次顶尖学生可能会严重落后(并在某个时候遭遇信心重挫)。
在 D 的经历和思考:死亡行军
大学 D 的新生基础太差,导致教学质量很差。这其实对学生的伤害最大。最直接的一个表现就是
数学科学死亡行军(math-science death march)
这是纽约时报一篇长篇文章中强调的一个词。这篇文章主要讲的是:为什么 有很多爱好科学有志于学习理工科的孩子 到了大学学习两三年后纷纷转专业?—- 因为理工科实在是太难了(so darn hard),特别是孩子们无法战胜低年级的数学科学死亡行军(math-science death march)。
But, it turns out, middle and high school students are having most of the fun,building their erector sets and dropping eggs into water to test the first lawof motion. The excitement quickly fades as students brush up against thereality of what David E. Goldberg, an emeritus engineering professor, calls “the math-science death march.” Freshmen incollege wade through a blizzard of calculus, physics and chemistry in lecturehalls with hundreds of other students. And then many wash out.
这篇文章本身没什么大不了,最有价值的其实是后面的 1000 多条读者评论。其中有大量认真撰写的长评论,我强烈推荐英文好的人看一看(顺便说一下,英文主流媒体常有读者评论比正文有意思的情况)。除了一些有些离题的抨击外国人或华尔街抢饭碗外,评论中出现了两派:一派有很多学生,指责大学要负极大责任:如教授高高在上,所学内容过于晦涩,教学方式过于呆板等等;另一派有很多大学教师和老工程师(很多人上来先亮身份:我做 XX / 我教 XX 已经多少年了。。。),这一派认为主要是学生水平太次,不如自己这一代人。我虽然比学生大不了多少,但我属第二派(屁股决定脑袋?)。大学教学固然可以挑出毛病,但在学生的极薄弱理科基础(尤其是数学)面前都属于小问题了。
我对这件事(数学科学死亡行军) 有直接的体会, 亲见一些有梦想 上进心强 人其实也算聪明的学生由于搞不定我讲的课(也许还有其他课) 而基本上丧失掉了追求梦想(探索科学或者做工程师)的可能。我观察过一些常来问问题的学生发现糟糕的中小学教育 导致他们脑子里的知识点处于一团乱麻的状态(可能比没学过还糟糕)而这种状态又进一步导致他们有条理的梳理组织思路的能力极为欠缺。即使我问一些他们中学学过的东西他们往往都会很快暴露出这些问题来。
把问题带到下一个阶段
也许有读者会说 我这个学究要求太高 很多学生准备做应用技术或商业金融不需要把基础课那么好。但我强调的其实是 即使是只要求不求甚解照虎花猫地使用 不少学生也有不小困难。如果要打一个比方我觉得可能类似于要一个只认识 1000 个汉字的人读报纸。以我们输出的学生水平看,很多人要想在实际工作中 (比方说)考虑优化问题分析处理信号 或者分析稍复杂的统计数据 势必感到吃力而且缺乏变通能力(这点从他们做应用题就能看出)。
一些对中学教育很愤怒的大学教授 用 “garbage in, garbage out” 来描述这种情况:高中送给我们基础太差的学生导致应用专业毕业的也是基础很差的学生而这又进一步影响到工业界。我可以告诉河里一些在工业界工作的河友一个秘密:除非你的工程师来自第一梯队大学(最有名的那六七所)否则你得假设他的成绩可能水分很大特别是数学基础课(因为数学系通常在学校里较弱势不敢得罪人)。比如如果他的成绩为 B 这其实可能意味着他实际成绩不及格。。。。
研究生教育不算拔尖
再简要说一下研究生教育。D 大学的教授虽不像 B C 那样强势但也基本都是功力深厚的悍将。可是 D 的研究生精神面貌 就远远不如 BC 的研究生,反而更像中国最顶尖学校的研究生:没有强烈的主人翁意识,没有精神原子弹,(最令我窝火的毛病)不敢学习自己方向上最难的学术工作。。。
我感到中国国内的最顶尖大学在研究生教育上 和 D 大学这样的学校比 虽然仍然落后但已经可以望其项背了。比如如果未来几年能够保持现在的吸收海龟的势头,那么在我熟悉的学科(数学)上十年后达到美国 10-15 名学校的研究生教育水平 是完全有机会的。
第二梯队基础科学教育形势不容乐观
如果连 D 大学这样的学校生源都大成问题,我就不得不怀疑:除了个位数的最好学校外,美国其他的学校的美国理工科生源质量都有大问题。我没有专门调查其他学校情况,但我零星接触的情况同我的怀疑是吻合的。
我回想了一下我在国内的高中情况:中国的第一梯队学校和其后的学校理工科生源质量远没有这么恐怖的巨大落差。
不要再猜测学校
顺便说一下,因为我说了很多 D 大的毛病,所以希望河友不要猜测 D 大是哪所学校(其实如果仔细看帖又对北美大学熟悉的话范围很容易缩小到两三所)以免在网上流传后引起不必要的麻烦。ABC 也不要猜了(虽然可能比猜 D 还容易)。
我的一些思考:美国顶尖大学的优势
注意:1 我在这篇里只讲大学的基础科学研究和教育。2 我说的很多是对理论科学的观察,实验科学因为依赖于大量实验设备分析起来可能会有所不同。不过我认为我讲的大体上恐怕还是成立的。如其他领域的河友有不同意见 欢迎提出。
美国最顶尖大学在基础理论科学研究和教育上拥有巨大优势
简单的讲 美国拥有的第一流教授和学者 占世界的份额实在是太大了。按我的粗糙印象这类似于美军在世界军事格局上的地位。其他的表现 比如吸引全世界最好学生从而拥有最好研究生院等都是建立在这一基础上的。(虽然我提过美国大学本科生源的严重问题,但最顶尖大学的还不错而且作为象牙塔里的东西基础科学的研究拼的是教授和研究生生源(不限于最顶尖学校)。)
不错 随着美国经济地位的下降它维持这一地位的能力会下降。但是这并非一个突变的过程(假定不爆发世界大战或者苏联式经济社会崩溃)。除了美国的经济地位在中短期内大概不会暴跌式下滑外,具体到学界有三个特殊因素是很值得注意的:1 除了中国美国在经济上没有威胁大的赶超型的竞争者(欧洲日本经济更惨),而目前中国仍未在本土攒出哪怕一个理科世界一流的大学;2 一个第一流学者成熟后往往有 30 年 “一流水准保质期”,这是很难被政治经济条件改变的(当然做实验的人会面临经费问题)。这意味着即使现在美国繁殖一流学者能力受到了严重打击仅凭已有人才也能维持一些时日,更何况这件事还没发生;3 最顶尖的学校本身处在经济社会金字塔顶端 抗经济打击能力很强。
如果你认为中国有在 15 年内基础科学赶上美国的机会不妨打开美国最强的 10 所大学查看相关领域的 50 岁以下非华人学者的名单。15 年后这些人极有可能仍在美国。然后你再想想中国如何能在 15 年内制造出一个超出这些人的群体来。我的看法是:即使所有华人立马回国中国一流学生也不再赴美留学 也未必做得到。原因很简单:时间不够来不及繁殖那么多国际一流学者。
美国获得第一流地位的 “经验” 对中国借鉴意义不大
那么美国是如何做到这么强的呢?我的看法是
1 美国的人口对任何其他发达工业国都具有压倒优势。从科学圈的角度看 这人口不仅指美国的三亿人也包括加澳两国和部分英国的。
2 二战导致了人类近现代史上最大规模的科学人才转移。转移目的地是美国。其实虽然美国在 19 世纪末就是第一工业国直到 20 世纪 30 年代初世界科学中心仍在欧洲 特别是受一战重创的德国法国。然而仅用短短 10 多年希特勒就重画了世界科学版图。我愿举一个很有代表性的例子。国际数学家大会是 4 年一次的全世界所有分支数学均参与的大会,具有很广的代表性(其他学科没有这么全面的大规模的全行业大集会)。1932 年国际数学家大会 20 个全会报告只有两个是用英语作的(读者可以由此想想当时美英在国际学术界的地位)。之后希特勒上台,二战来临,在 1936 之后就 14 年没开会。到再下一次(1950)时, 22 个全会报告有 20 个是用英语作的,英语也从此有了国际学界的绝对统治地位。
3 苏联解体导致了人类近现代史上第二大规模的科学人才转移。苏联人才转移对生化可能影响小一些,但对数理科学来说可是极为壮观:这是从 20 出头的天才大学生到 80 多岁的泰山北斗的全体系的迁移。转移的人少数被欧盟吃了,大部分被美国吞了。
4 在美国的基础科学 于二战后取得西方阵营的独大地位 以及冷战后取得全世界的独大地位后有能力输出第一流后备人才的国家(如中印等)的一流学生蜂拥而至更使美国如虎添翼。
其他的因素(比如很多人津津乐道的 “体制”“创新精神”“自由” 等)即使是有利因素其影响力也和上述因素不在一个量级上。
对中国而言:因素 1 天然具备,在经济赶上来自然就会发挥作用;因素 23 可遇而不可求(多半是不可遇);因素 4 属于已经成第一科学强国后的锦上添花,对如何做到第一没什么用。
美国大学的基础科学体制宏观上是一个容易学的顺其自然的体制
虽然美国获得第一流地位的 “经验” 对中国借鉴意义不大,但仍可考察一下其维持第一流地位的“经验”。比如说大家都爱谈的“体制问题”。
在我看来 美国的基础科学体制 在最宏观的层面上 其实很简单:用大把的钱 养大把的 “学霸”(对大学者的昵称),学痴,顶尖学生; 然后让这些人 自己折腾自己治理 就行了。中国有了大把的钱后也可以学当然学了以后还有等足够的时间才能见效。
现在人们抱怨的种种中国学界问题 基本都可以用这套体制来化解。缺乏创新精神?“学霸”和顶尖学生就是创新的标杆;学术抄袭腐败?“学霸”,学痴等很少会这样对此容忍度也很低;心思不在学术上?在中产之后还心思不在学术上的人会被边缘化;学术上近亲繁殖,派系山头斗争?嘿嘿,“学霸”/ 大学者基本都是有态度的人因为学术上总要面临有所为有所不为的问题。为了贯彻自己的学术判断,“学霸”分派系抢话语权是常态。那美国学界为啥没乱套呢?学霸太多自然地就形成制衡了,派系之间的 “多不管” 地区也很多为标新立异的人提供了空间。
顺便说一句:我认为真正最难学的是苏联体制,而苏联体制的效率明显高于美国体制(至少在数理科学上是这样的)。
是不是说 中国从美国除了最简单的烧钱 “顺其自然” 外 就没什么可学的了呢?不然。美国体制的一些具体举措仍有可借鉴之处。
美国体制的一些具体举措有可借鉴之处(举例)
我举例讲一个我觉得值得借鉴的东西:tenure track 体系(终身教职轨道体系)。研究性大学选择自己认为比较有前途的做了一段时间博士后的青年学者 将其置于此终身教职轨道上考察数年。期间较好地保障其研究条件。数年后若发现此人确实不错则委任为终身教授若觉得没有那么优秀则让其走人(即所谓的 “不升即走”)。这是一种竞争性很强的制度,对最强者很有利(快速上升有效地避免论资排辈),对次强者有一定风险但也可以强烈激发其动力。
对中国来说 这种制度不仅有利于增强国内学术环境里 一直偏弱的学者的斗志,而且也是中国在有钱后同美国抢夺一流人才的需要(毕竟绝大多数海外华人科学人才是在美国)。10 年前我就听说过有引进此种制度的传言但可能因为客观条件所限(钱少?)没有施行。最近几年,几所国内最好大学开始在基础科学院系逐步开始使用这套 高门槛高风险高回报的制度。据我所知 这在海外一流青年学者中引起了很大关注很多人都动心了,不少已决定回国或正在筹划回国。。。对这种 “高门槛高风险高回报” 而且显然对非海龟不利的制度当然有不少不同意见(科学网前不久曾集中讨论过这话题)但看来上层下了决心要和美国正面直接竞争。
我的一些思考:美国理科基础教育的薄弱
美国理科基础教育质量差而且积重难返
我前面的几篇已描述了 我认为美国理科基础教育质量差的理由这里就不再多说了。那么有没有办法改变呢?在 D 大学让我大跌眼镜后我决定做一下调查。当然很快我发现早有受不了了的大学教授作了调查并常年奔走呼号。(Berkely 的伍鸿熙就是这样的一个热心人,他收集了大量资料和例 教育爱好者可以去看看)
总的说来 我认为明显改善的希望很小。
1 中小学教师素质太弱 本身也是糟糕理科基础教育的受害者
比如伍鸿熙和他的同事调查了很多中小学数学老师,竟然罕有人能解释 类似于 “除以一个分数等于乘以它的倒数” 这类简单事实的原因!(可惜我一时找不到有多高比例教师做不到这些事的原始数据,不过绝对是骇人听闻的高)。最后作调查的大学教授们发现中小学数学老师在上中小学时 他们的数学老师就未能给他们解释 这些简单事实的原因。等他们长大了,逐渐见怪不怪,也能顺利做计算,可是他们始终没有理解为什么!他们虽然上了大学但往往是大学里学得较吃力的学生 被微积分线性代数折磨得够呛,而且最重要的是:大学也不会教这些东西呀。这时我就深刻感觉到了中国这套师范学校体系的重要性。
2 教材和教学标准体系有严重质量问题
看了愤怒的大学教授和一些家长的控诉后,我才了解到原来很多流行的数学教科书内容有严重漏洞,歧义或错误。感兴趣的河友可以去看上面链接中伍鸿熙找出来的各种奇葩例子。我只半剧透一个。一些美国学生不懂分数的乘法:为啥 2/3 * 1/5=2/15? 就是不懂!甚至还专门造了一个词(fractionphobia– 分数恐惧症)来描述这种不懂的恐惧。(中国人想不通)这事儿怎麽就能这么难呢?美国的教科书害的。。。
教科书的问题其实是教学大纲和教学指导思想的产物。那么教学大纲和教学指导思想出了啥问题?大致说来美国有两派。第一派由中小学教师和教育学专家组成主张初等教育要实行 “启发式”“探索式” 教学多从实际从动手出发 用计算器等代替传统计算教学等;第二派由大学教师和一些家长组成主张传统教学法 强调记忆和练习 强调严格的书本知识学习。两派争执不下 矛盾公开化在 90 年代曾酿成著名的“数学战争”。从现在的情况看第二派在掌握教育主导权上失败了。前述的问题是在第一派的思想下产生的。
​3 教育改革阻力极大
那么美国能不能回归一些传统或者直接向中国之类的体系学习呢?不少人就是这么主张的(当然他们中的很多不说学中国而说学新加坡之类)。但是这类改革会受到强大的教师工会阻挠。家长总是分散的,大学教授人数少而且 外国人多 不接地气。他们是无力和人数众多高度组织的中小学教师争权的。更不用说 把握相当一部分话语权的教育学专家们往往是第一派。。。。。。
4 经济困难
宏观经济困难导致美国政府捉襟见肘。各利益集团都要争夺缩水的经费蛋糕。在金融军工医疗等巨人面前公立教育是弱势。。。
5 种族因素
非洲裔和拉美裔的理科成绩弱 在好大学的理工科里所占比例极低。而他们占人口的比例在不断上升。。。
6 教育标准在全国范围内不统一
在联邦制下 地方有很大的自主权。我询问过我的学生,感觉不同地区的学生学的课的难度 知识点甚至顺序都会有很多不一样, 情形十分混乱。更麻烦的是美国人迁移性大。我听说有很多孩子在搬家后会出现学习的课程无法衔接的麻烦。我甚至知道有的人在中学里没学过物理(连牛顿三定律都没学过)。。。
7 反智主义的干扰
有至少两类反智主义。一类基于保守宗教思想。这类思想对知识分子没大的影响力 但有一定群众基础,有民粹式的政治影响力。其对某些地区出身的人 有洗脑作用。比如曾有学生告诉我他因为对上帝的信仰和家庭教育因素长期以来对学习物理和数学有难以克服的感情排斥性。。。
另一类则以 相对主义 多元化 极端环保主义 女性主义等面貌出现 或者表现为一些受过良好高等教育但理科学得不好的人 质疑学习理科的必要性。这些人虽不如第一种多但往往更难缠。我可以举一例管中窥豹。纽约时报曾刊登一文,鼓吹 “数学作为必修课 会阻挠我们发现和发展年轻天才”(Making mathematics mandatory preventsus from discovering and developing young talent.)并建议 大多数人不用学中学的代数了。注意,这是中学的代数:是教你用 xyz 代表变量 教你了解多项式教你解一元一次方程的代数课!他的提议固然受到了多数读者的抨击 但值得注意的是也有相当一些人支持他。。。
美国的理科基础教育质量差 是美国人自己也意识到了的一大问题 各种抱怨充斥媒体 但始终不见什么好办法。我在做了一些调查后 也感到问题积重难返 解决希望渺茫。
我的一些思考:依赖外国人才输入的科研模式
美国的基础科学研究高度依赖外国人才输入
这是大家都熟知的事实。那么依赖程度有多高呢?最好的办法是看各学科的行业学会的年度报告。我查看了美国数学会的多年报告发现美国的数学博士中 出生在美国的所占比例常年维持在四成多。另外在其所统计的美国顶级大学数学系高引用率学者中外国人占六成。所以在数学界可以说至少半壁江山靠外国人。其他的学科也许会略有不同 但美国的基础科学研究高度依赖外国人才输入应该是个准确的判断。
美国的这种情况是非常奇特而且前所未有的。历史上曾出现过的其他顶级科学强国(老欧洲三强英法德以及后来的苏联) 在巅峰期其科学事业都是基本上完全由本族人支撑起来的。
基础科学研究高度依赖外国人才输入 是一把双刃剑。
先说其好处:1 它使美国科学在世界上所占份额大大超出其经济总量占世界的份额(美国 GDP 约为世界的两成多 但顶级学者显然不只这一比例)。2 它使美国的潜在竞争者追赶美国的难度大增。
但它也有明显的风险:当国家相对衰落到一定程度后 可能会导致科学水平大幅度下滑难以坚守住一个高位。
我仔细解释一下。
以数理科学为例。外国人才的最大两个来源是 前苏联人(或者稍广一点 – 前华约人)和中国。苏联(和华约)靠着具有惊人培养人才效率的苏联体制在冷战结束前夕积累了群星璀璨后浪推前浪的 全年龄段顶尖人才梯队。苏联崩溃后人才梯队几乎全体系地离开苏联多数去了美国。此后苏联地区的优质科学研究和教育体系基本崩溃。现如今不仅在一流学术竞争方面基本出局(Perelman 等极个别人例外,而且 Perelman 是脱离学术体系的异类)前苏联地区甚至连向美国输出一流后备人才(顶尖大学博士生)的能力都已非常薄弱(我留意过好几年 B 的研究生来源,我想不起一例 B 大学从前苏联地区的大学招研究生的例子而同期 B 从中国招了一大批学生)。简单的说解体后的苏联不仅金蛋被抢光 会下金蛋的母鸡也被杀死了。
苏联解体 20 年了。未来 15 年内美国从苏联获得的人才战利品将过期而且无法补充。
中国的情况则是最有意思的。自中国对外开放以来中国理科的最顶尖学生就持续不断的流向美国。这一方面强化了美国的领先地位 另一方面导致中国长期失血始终难以完成本土一流人才的原始积累。可以说中国近二三十年的状态对美国维持其地位来说是非常理想的:中国基础和本科教育足够强可以大规模长期为美国输入后备人才但研究生教育和基础研究足够弱以至于无法大规模吸引顶尖人才回流。
然而中国的很多事业都有这样的特点:格外漫长, 挫折失误连连,长期看不到希望(或者说只能将希望寄托于人多), 眼看着别人有很多 “捷径” 但学不到手,最后走了一条异常艰苦一个台阶都绕不开的路,然后逐渐收获果实并能望见难以限量之光明前途但很多人还没意识到最难的阶段已过去。。。
我感觉中国的基础科学研究大概也是这个路子:顶尖理科人才的长期外流纯粹从国家角度看,在三十年尺度上是吃大亏的事,但从五六十年尺度看却可能是最终盈大利的。中国通过美国的体系在海外顺利完成了一流人才原始积累和建设全年龄段人才梯队。由于中国经济实力的急速提高,美国的令本国人民和精英都始料不及的地位下滑以及中国青年有了美国可被中国 “取而代之” 的“皇帝轮流做之贼心”,最近几年来已经有未来 10 年内还会有成批第一流中国青年人才回流。在我看来 中国基础科学在本土完成一流人才原始积累 就如中国经济总量超过美国一样是没有什么悬念的事。各种人们抱怨的国内不利因素只会影响早几年还是晚几年实现的问题。当然从在本土完成一流人才原始积累到基础科学赶上美国还需要一段不短的时间(学术后代繁殖需要时间)。
一个中国顶尖学生 20 多岁到美国学习 30 多岁回国工作这是一种新模式。六年前(08 美国金融危机前)这样的模式凤毛麟角, 但最近几年呈现滚雪球之势。我周边(包括我)有越来越多这样的人而且相互影响相互呼应。这和更早时期的顶尖人才的 20 多岁到美国学习 之后在美定居的模式 大不同。后者对中国也是很有意义的(回国讲学组织会议,保持华人在世界学术中心一席之地等)但对美国的直接贡献更大。而前者则显然对中国贡献远大于对美国的而且做贡献的能力是在美国获得。如果说美国在一流基础科学人才这个领域从中国抽了 30 年血的话,现在中国要开始反过来从美国抽血了。美国无法通过刻意压缩从中国招研究生阻止中国抽血:如果它这样做 则不仅会导致生源质量下降而且会导致顶尖学生不得不在中国深造从而帮助中国更快的进行一流人才的繁殖。更可怕的是中国学生在 21 世纪以来在美国后备科学人才中占的比例似乎越来越高 近年来更是迅速扩散到本科阶段(我亲身体验似有爆发趋势不知是否和美国高中生的理科水平进一步下滑有关)。这意味着中国的潜在抽血能力还在增强。。。
美国能不能找到弥补 “苏联空洞” 和“中国抽血”的办法呢? 由于本土理科教育质量积重难返靠本国人是极为困难的。这件事和美国高度依赖外国人才输入的现状是相互促进的。这事情有点类似于美国要将已经丢掉的制造业从海外拉回来一样不靠谱。更麻烦的是,在制造业方面 美国总还可以将希望寄托于新的高科技,而在基础科学方面 按照定义 已经没有更 “高端” 的东西了。
依靠其他外国也不太可行。印度或有在未来向美国输入更多科学人才的能力但我很难想象他能弥补 “苏联空洞” 加“中国抽血”。欧洲有人才 但只要法德荷比瑞等几个老欧洲核心经济不崩溃美国捞不到多少。非洲 大中东拉美 东南亚等 人口众多 但理科基础教育体系太弱 输出后备人才到美国一流学校搞基础科学的能力几近于零。
从更长远角度看 如果美国的经济地位持续下滑(哪怕仅仅相对于中国)美国的白人人才也存在外流中国的可能。我以前觉得不大可能,但我近年来已经看到了一些国内最有钱学校用高薪招到美国中档藤校正教授水平的白人。顶尖学者由于职业特点国际主义倾向较重而且美国和西方已有一代人以上学者是在高度推崇全球化的大学环境中成长的。。。
日本在经济出现 “失去的 20 年后” 基本保住了本土的一流科学人才(当然这也说明了日本的封闭性)。在中美经济力量对比易位后的 20 年内美国的科学界只怕“欲为日本而不得”。
教育问题显现效应有较长滞后期 美国精英可能准备不足
我以前提过 一个第一流学者成熟后往往有 30 年 “一流水准保质期”。另一方面一个中国学生可能现在刚到美国来读书离他回国可能还有 10 年离他在国内开始有效的繁殖学术后代可能还有 15 年;这意味着虽然他最终属于“从美国抽血” 的人但这事有 10 几年滞后期。
从主流媒体看 美国的精英阶层 对教育问题显现效应有较长滞后期这件事似乎没有足够警觉性。至少没有警觉到认为 就算现在集中资源死保基础科学地位都有可能动手晚了点。不错有很多指出美国基础教育差的声音,但对于基础科研和高等教育(研究生教育)舆论似乎还是普遍很乐观 认为这是中国差距甚大美国不必太忧虑的领域。对于美国高度依赖外国人才很多人单纯觉得这是美国的优势:外国人来说明我们厉害嘛。更有甚者各种有浓重意识形态色彩的肤浅言论还大有市场似乎喊几句中共体制压制自由就能推出中国创新赶不上美国。。。
美国例外论
根植于上述许多问题深处的精英思想根源 是 “美国例外论”。我认为
1 美国冷战后的经济地位与其人口比例高度不匹配
2 美国冷战后的基础科学地位与其经济比例不匹配(与其人口比例更是超高度不匹配)。美国例外论者大概也同意这两点。
我们的分歧在于:我认为从长的历史尺度看 美国的上述地位的获得有很大的偶然性而且是不可长时间持续的(即使美国没有犯大的错误),更不用说美国还犯了一些大错误;而例外论者认为美国是 “例外的” 从而即使相对地位有所削弱,也必然是居第一位的领导者,不会被中国取代。很多诚实的例外论者也承认既成或几乎既成事实(比如从中国出口超美到中国制造业超美到中国经济超美),但是他们往往在下一个里程碑处继续持“例外论”。
结语
上述历史观的差异是很重要的。对很多美国精英来说 美国的基础科学和高等教育具有中国在值得预测的未来没有机会哪怕接近的地位 从而不仅成为他们自信的源泉而且可能为其他领域(如制造业)提供美国 “王者回归” 的机会。对我来说在基础科学和理科高等教育方面 “中国速胜论”和 “中国无法超美论” 都站不住脚 。但中国在跨越我一生主要学术期的未来 30-40 年的 “持久战” 颇有胜算。在认识到这一点之后我就知道我一定要回国:不仅因为我是中国人也因为我是希望参与到新的世界科学中心形成进程中去的“国际人”。另一方面我衷心希望美国人能平稳过渡到世界第二并和欧洲等力量
一起实现基础科学不过度集中于一国(中国)。**这种长期存在的平衡和竞争力量对中国和人类的基础科学的长远发展都会是有利的,有长久历史经验积累的中国人当能体会。

上述来源:西西河中文网,作者:changshou


相关问题特别是中国传统教育及现代教改讨论,还可参考

如何评价中国现在的教育?汉语适合当学术用语吗?

因为近代教育误导及影视的扭曲宣传,现代中国人对中国传统文化的很多方面了解都是来自二手的以讹传讹或别有用心的宣传信息,而非古籍文献记载与考古出土文物互参的历史。

科技方面先参考

现代科学真的是西方文明发展的吗?西數中化 1.0有没有什么平常人以为古代没有但实际上已经有了的东西?

发现很多人这方面都被误导科普,有兴趣中国数学史的可以参考

中算选摘

現代生活方方面面都可以考慮傳承創新,比如以前設計過的簡中式婚禮

中式婚礼和西式婚礼你选哪一个?

中化英語詞典

有哪些只有既懂中文又懂英文的人才能听懂的笑话?

![](data:image/svg+xml;utf8,)

知乎用户 张一片​​ 发表

时间问题。

这个世界其实是一小撮顶尖的工程师、科学家,带领全人类突飞猛飚的世界。

抛开后天培养因素,全人类的智商呈正态分布,但拥有最顶尖智商的天才,是按概率随机出现的。

只要有足够的人口基数,公平的选拔机制,必然会源源不断的诞生天才。

这些天才是做出超级牛的东西的根本,只要给他们机会和平台。

现在中国庞大的人口基数,相对公平的选拔机制(哪怕最近负面屡屡爆出),必然源源不断的诞生很多顶尖天才,他们在中国已经不缺机会了,缺平台。

现在很多前沿科技的研发机构还在美国。。。等再过十年二十年,就不一定了。

我们就坐等天才拯救中国吧。


对比看下一两年前这个问题下的答案,全是在妄自菲薄,中国教育各种不好,国外教育各种开明。

现在大家应该知道了吧,开明教育是培养底层人民的,人家顶尖的正规私立教育,可是很严的。当然这十多年混子也很多。

譬如特朗普家里,前多年还有叔叔是顶尖科学家,这多年没听说他哪个亲戚是科学家了,全去搞金融政治去了。

所以美国很多前沿的研究机构,美国本土教育出来的研究员,占比已经很低很低了。。。。。。

千万不要低估我国的基础教育

比如清华姚班、智班那些天才,如果不去搞金融,将来出几个超杨振宁的顶尖级科学家,也许人类历史进程都改变了。

譬如新能源,核聚变,储能技术,人工智能,机器人,物联网,基因医疗技术等科技,再往前突破一步,人类就不一样了。

(印度的人口基数也很大,一亿中产天龙人也能源源不断出天才,但从这一点看,未来不容小觑。)

知乎用户 韩韩 发表

能吸引优秀的移民。

很多伟大的人都是移民,包括了乔布斯(其父亲约翰 · 扬达利是叙利亚移民。)

google 的创始人,拉里 · 佩奇是犹太移民后裔,谢尔盖 · 布林是前苏联是犹太移民后裔

知乎用户 friday pp 发表

因为做出超级牛的东西的人,不是美国人,不在美国受中小学教育

知乎用户 iBankSavage 发表

几万清华学子都去帮人家打工了呗。

知乎用户 vitu 发表

这个问题的本质在于:做出超级牛的东西只需要万分之一最顶尖的那几个人

国内目前的教育体制对天才学生的创造力,学习数学的热情,科研的能力整体环境其实是压抑的

所以你会看到在封闭体系知识结构的平均水平中国学生是高于美国学生的,但是在开放体系,需要创造力和研究能力的领域,美国的环境的确优于国内

不过世界也在不断的变化中,10 年后还不一定呢

知乎用户 黄小立 发表

学数学是为了证明自己牛逼,还是因为数学的确有价值?如果是前者,按性价比来说,随便翻开一页,记住个名词然后显摆给小白听,远好于花三年五年证明一个猜想。

投机者可不喜欢套牢。

知乎用户 ChinaTerran​ 发表

大学之前,中国的课程难度都是比其他国家难的。但这个难的意义并不是培养人才,而是摊派人才。中国考生太多了,谁去清华、谁去 985/211、谁去二本、谁去三本、谁去大专?谁说的算呢?你自己的 “分数” 说的算。这个分数要是区分度低,就无法将这么大数量的学生摊派到中国的教育机构,为了保证高区分度就要提高考试内容的广度和深度。

至于为什么美国能开发出高实用性的产品(题主暗示中国不能)的问题。无论是过去的美国高产还是现在中国的迎头追上,都是资本和体制问题。

首先,现在美国、中国的基础设施建设覆盖率已经覆盖了主要的大城市。这就为科技人才提供了足够的生活支持。然后,多余的资本从建设基础设施项目转移到科技研发项目。而此时,中国的制度也开始为这一资本行为提供方便和监管。在利益的趋势下,大学之后的人才有意识地开始追逐利润,而此时的资本已经在科研阵营向人才们招手。

过去,中国在高科技产品方面也真的不落后。建国之后,我们聚全国之力研发并制造原子弹、氢弹,还发射卫星。从那之后,中国又开始研发人工燃气的研发,石油勘探、水上钻井技术、高铁技术等研发,今日自主设计的航母也下水了。可以这么说,从一开始,中国在最顶尖科技方面都没有太大的落后。只不过上述项目都是国家级的,这些科技大部分都和军事、资源有关,老百姓不能直观的体验。但是如今,中国的科技水平也开始走平民化了,比如说微信支付。这可能是大家最直观感受到的,出门都不带大把现金了是不是?如果说国家级高科技研发是为了民族的生死存亡,那么民生级的高科技应用就是方便老百姓(赚老百姓的钱),这时候资本和制度就成为了主要原因。这里的制度不仅仅是对市场的监管,也有教育机构、研发机构人才选拔和晋升的制度管理。过去的中国,研发人才都聚集在国家科研机关。如今,市场被打开了,各大公司也成立了研发部门,这也是为啥中国高科技开始走向民间。而事实上,美国这样的制度比我们要早,所以美国民间高科技产品研发就比我国早,老百姓普遍觉得美国的科技更多(核心科技还不是武器、航天,大家都一样啦)。

现在我国的民用高科技产品已经有了强大的发展势头,这和中小学的教育难易没啥太大关系。中国人数学看似强,这种强不过是 “熟练” 罢了,用来解决已经被解决过的问题中国教育真的是一流,因为大学之前的教育都是基础教育,也就是能解释解释周围的现象,但是真正搞研发的领域都是没思考过或者还没有被解决的领域,是需要大量的试验和理论推导的过程,可不是你用海量题库就能解决的。

知乎用户 月上西楼 发表

你听说的内容不真实,首先得出这个结论的人不是脑残就是别有用心。

教育的目的是什么?

我如果把所有的目前已知的科研成果,科学成就,原理等等都教给你,你以为你就能成为科学家了? 你以为把所有的诗词、小说、散文等等都背下来,你就能诗人、作家了?

为什么受过这么多的教育,学了这么多的知识,却成不了那样的人呢?

计算机可以存储所有的东西,为什么不能成为科学家、诗人、作家、艺术家?

因为 “思想” 才是最关键的,如果你只是把所有的东西都记下来,跟计算机的区别也只是数量等级上的差别罢了。

知乎用户 小雨飞龙​ 发表

不是特别清楚,随便说两句

据说中国的数学教育是教给所有人同等的,高级的知识,不管你喜不喜欢数学

但是美国的数学教育是教给不喜欢数学的人最基础的数学,而交给那些喜欢数学的人最顶尖的数学知识

没有优劣之分,体制的差异而已

(听说的,不保证对)

知乎用户 猪头猪 发表

我要说两国教育的目的是不一样的。

中国的教育本质是社会分层,教育到最后争夺的是社会分配权。

为什么?因为中国有大量的国有企业。这些企业和事业单位不能家族遗传,只能从社会招人,而这些往往又占据重要的国民经济比重,往往待遇还不错,那么不同学校对应不同国有企业的层级就逐渐形成了。

所以高考很重要,从而使得中国的竞争集中在大学之前,从而倒逼小学初中高中的教育越来越难。

美国的教育不是为了社会分层,因为社会基本已经固化了。这样如何保证固化才是重要的,那么普通的民众教育水平越差,对阶级固化越有利,中小学自然教育就不会很好。

但是上层人因为资源掌握的多,获得的教育资源多,大部分学习不是为了社会分层,找个好工作,而是因为太有钱了,没事干,不如做点研究。而且美国的优势主要还是吸引全世界优秀的人才,所以美国的尖端的确很厉害,不过这个尖端里面其实也有很多中国背景的学者。

中国的教育其实也能培养出很多的大牛,但因为高考分流作用和大学老师待遇一般,使得很多大牛流向高薪企业从而沦为社会螺丝钉。

但我相信再有 20 年估计应该就能见到中国做出超级牛的东西了,因为第一代大神给自己孩子解决好吃饭问题后,他们的下一代基本就不会为了钱去干一些并不牛的事情,还有国家科研经费够了,为了钱离开学校的大牛也会少了。

知乎用户 文刀叉点 发表

美国的教育是玩儿真的。。。

虽然他们自己人也经常吐槽他们教育很挫

但是他们要求的各种课外小项目,小论文等等东西,是真要做出个东西来的

斯坦福的公开课应该有不少人看过。其中期末的时候老师有可能会喊提交一些分组项目作为期末作业。这些作业那些学生都是很认真对待的,甚至其中一些还可以商业化

国内,大家都懂的。什么期末作业,随便网上找个抄一下就行了。

大学上完了你究竟学到啥了,自己都不知道

至于说初高中的数学水平。这个其实和未来搞科研的关系不一定很大,中国初高中,至少我那个时候没怎么普及计算机的教学。也就是走个形式而已,但是美国那边现在也搞得启蒙越来越早了。

=====

至于题主说到的 gap 问题

我觉得其中一个区别在于:美国的学生可能更知道自己需要什么,而不是像国内这样,是迫于周围的压力而选择高校

美国的学生,高中毕业去工作的也有不少。那些选择要读大学的,都是明确了自己未来目标的,当然这其中也有兴趣所向的原因

中国很多学生,知道自己为啥要读大学吗?知道自己为啥要读这个专业吗?知道自己这个专业都要干啥吗?

=====

另外还有一点,中国出过多少大数学家?国外又出过多少?美国计算机界出过多少专家?

感觉中国的学生数学能力确实比国外的强,但是这个强未必是在创造性上的强,而是技能上的强

简单来说体现出来的就是更会做题。但实际上,这仅仅只是因为美国那边教的东西和我们不匹配而已,和智商没有任何关系

但是,我们却缺乏他们的一些对问题的思索,以及一些设法解决问题的行动力。所以当他们已经有工具(数学或其他)的时候,他们实际就已经领先我们了

=====

当然,事情也不是绝对的

国内现在的大学生创业也越来越多了,这也说明中国人要是想做,也是可以做的。

这件事情,可能和一些制度,以及教育的差异有关

知乎用户 凉心四十八天​​ 发表

因为美国不只有中学,还有大学。

前二十强,美国占了十席。

知乎用户 许广新​ 发表

简单说,中国学生普遍是 50-85 分

美国学生就是 20-95 分

换句话说,美国的精英目前来说比中国的精英还是有一定优势的

而美国的蠢蛋比中国的蠢蛋还蠢

教育中,美国的理念是我 3 亿人口出几十几百万个精英科学家就足够了

剩下的只会唱跳 RAP 的文盲都无所谓

但是中国的理念是 13 亿尽可能多的出科学家,但是尽量别有文盲

就好像这次疫情

中国是牺牲经济也要保证尽可能控制疫情

不管你是博士学历还是小学文化

不管你身家百万还是吃着低保

感染了就全力救治

而美国是经济面前,人命算个 P

反正高层感染的不多

底层感染就感染被。。。

所以就目前来说,美国的科技水平比我们还是有优势

我们也正视这种差距

但是中华民族最大的特点就是厚积薄发。。。

知乎用户 百答通 发表

原因很简单啊,

因为中国数学最牛的那帮人,可能有一半最终都去了美帝。

再加上印度以及其他无数发展中国家,最牛那帮人,至少 8 成去了美帝。

日韩欧这些看起来的发达国家,最牛的人里,也至少有三分之一去了美帝。

(不要以为西欧福利好就能留住人才,一个大学教授扣完税的工资,和一个生了 9 个娃的非法移民到手的补贴差不了多少,换你你也跑美帝)

比如被誉为数学界的诺贝尔奖——菲尔兹奖,最近几届美国获奖者是:

玛利亚姆 · 米尔扎哈尼 Maryam Mirzakhani 美国(伊朗裔)

曼纽尔 · 巴尔加瓦 Manjul Bhargava 美国(印度裔)

安德烈 · 奥昆科夫 Андрей Окуньков美国(俄罗斯裔)

82 年还有个美籍华裔丘成桐 获奖。

再加上美国的私立教育,的确很牛逼,真正的为了培养精英而定制,里面的学生压力也很大。

然而,美国公立基础教育(注意我说的是公立基础,别扯什么公立大学),就是稀巴烂,洗都别洗了,美帝自己都拍了很多片子来黑自己的教育体系,参考【等待超人】。

知乎用户 饼叔​ 发表

实际情况是美国学生中小学的时候数学就很好,有图为证。

知乎用户 Enzo Jiang​ 发表

中国人只是算数好,而不是数学好。切莫坐井观天。我刚来法国的时候听数学课简直感觉自己是文盲。

知乎用户 Elmer 发表

你的问题已经说出了答案。

因为人家高等教育牛啊。

知乎用户 喜欢夕阳 发表

首先美国数学是突然变难得。很多大学的数学选修课会非常难。

另外美国的中学数学知识点学的很快

换句话说,中国的学渣去了美国也是学渣

中国的学霸,去了美国,如果没有很强的自律

也会跟不上

所以说美国人不卷

学得了就学,学不了拉倒

知乎用户 杨树森​​ 发表

截止到目前,美国吸引了全世界最多的数学人才,这是毋庸置疑的,也是美国数学教育的成就。他们的数学教育就算有再多的缺点,也否定不了他们的优势。我们希望中国的数学教育能够更好,是因为中国同样是世界的一部分,中国好就是世界好。

也许让你感到意外了,我赞成买菜用不到的就不必学的观点,只不过其中的买菜是广义的。首先不讨论复杂的问题,我就问问你,买菜还需要手算小数乘法吗?哪个超市里没有电子秤呢?即便没有也会有计算器,再不成你有手机。所以说,具体的四则运算怎么算并不重要。但是,你不能不知道什么叫加法、乘法,否则就算是按计算器你也不知道该按什么。

接下来跳出买菜这个简单的事,说点进一步的。现如今连人文学科都依赖统计学了,而统计学的基础是简单的概率论,这样的概率论源于微积分和线性代数,所以为了理解这些学科,不懂点微积分和线性代数是不行的。这样的懂,是怎么个懂法呢?

当然也不是做计算题。即便你把有理函数的积分、线性方程组的求解算得再快再准,也没什么用。根据因式分解理论,利用多项式方程的解,有理函数的积分总是可以求的;为了探讨线性方程组的解的结构,就去研究向量空间及其上的变换。这样的思想才是值得学的东西。

最近我在做全国大学生数学竞赛,今天做了这样一道题:设

上的可积函数,在

处可导,且

证明

比起写出这道题的完整解答,我认为从直观上理解为什么会有这个结果是更重要的。在学术或应用场合下,我们不会提前知道一个问题的结果是什么样的,在论证之前必定要提出合理的猜想。考虑积分序列

你能否直接看出它是个无穷小,甚至是个二阶无穷小?这样的洞察力,不论你将来做什么,都显得更有用。

另外,虽说追求实用,你却不能过于依赖看得见摸得着的应用。数学也好,别的领域也好,有大量问题的解决依靠着看起来无关的工具。可想而知,在经典物理学诞生以前,人们也会认为用数学解决物理问题是不可思议的。到了现代,我们采用几何分析这样的 “硬” 方法解决庞加莱猜想这样的 “软” 问题,在一开始也可以说是意料之外的。

我们不去探讨美国数学教育失败的那一方面,仅仅说成功的部分。他们能够顺应时代的发展,认识到哪些内容是对于学生来说有用的,而不是机械地要一个分数,甚至以无用但是提高成绩为荣。发达国家更懂得教育是最不可能浪费人才的岗位,至少他们的政策更加有利于这个观点,作为结果,他们的精英教育是更成功的。

再来反思我们的教育。中小学到底做了多少不用于提高水平的事?甚至当我指出很多人成绩好但是能力不足时,他们会说这表示他们很机智,而不是费大力气非要使自己达到教学要求。本着良心,有这样想法的人,与社会的关系到底是共生还是寄生?我的理想是让更多的人能更有良心一点,在此基础上,学习更多更有现代意义的数学,形成更好的审美。

知乎用户 杨谢谢谢谢谢​ 发表

你觉得的简单,是美国给穷人的教育。

美国的资产阶级子弟的教育可一点也不简单。中国的中小学交易,主要是普及,让每个人都有一定的科学素养,美国的资产阶级的教育,是为了培养统治阶级的人才、是为了维护资产阶级专政的。这些接受精英教育的人和我们接受普及教育的人一相比当然相差巨大。

知乎用户 雍和​ 发表

19 世纪英国教育界大 V、跨界哲学家、社会学家 Herbert Spencer 曾说:

孩子在快乐的状态下学习最有效。

很多人其实都会有个误区。

就是觉得学习知识、或者做练习册肯定是痛苦的,想要快乐那就玩玩手机、看看电视。

但这种想法完全是错误的。

比如下面某入学考试的四则混合运算,你让孩子拼命刷这种题他又做不出来时,他的确是非常痛苦。

但让他画一画流程图,帮忙找到海盗藏起来的宝藏,孩子觉得很有趣,玩完之后再再回来做练习册,那这种类似游戏通关的学习方式,也是快乐的。

上面这种做法,其实就类似于 “学习的金字塔” 理论所引申出来的 “玩中学” 的理念**。**

**

**

不是说鼓励让孩子一边看电视一边写作业,而是采用有趣的教学方法,为孩子营造愉快的学习状态。

当孩子能够以享受和快乐的状态去学习、感受数学的魅力时,其水平自然也会狂甩同龄孩子几条街。

快乐和学习,从来都不是割裂的。

事实上,在美国幼儿园、小学数学教材中,从来不缺少趣味性,老师按照 “让孩子玩中学、学中玩” 的原则给孩子上课,结合游戏的同时也能潜移默化的让孩子吸收数学知识。

今天推荐的这套《数学天才玩出来》,正是严格贯彻了 “玩中学” 理念,全美小学生几乎人手一本的**《Mathmania》中文版。**

这套书非常牛的一点是,正是他们非常善用一个脑洞大开的游戏场景,去将数学里几个知识点串联起来,这样一边释放孩子天性,一边让孩子在在游戏中,不知不觉地消化那些晦涩难懂的数学知识, 加深他们对数学知识的认知!

远比单纯机械性的刷题效果,要好 100 倍!还能拓展他们的思维,增加他们的分析能力!

很多人可能不知道,就连一些欧洲发达国家的学校和幼儿园,也都让他们的学生使用《Mathmania》,在全球已经热销 10 亿!

但这样一个没有枯燥的说教、也不搞填海战术的游戏书,原版价格也并不低,49.98 美元 8 本,折合下来大概要 360 多块。

为了能让更多孩子,也能感受到这套书的魅力,我们死磨硬泡,终于是找来了**《Mathmania》**中文版!

也就是今天推荐的这套《数学天才玩出来》!同样是熟悉的题型熟悉的趣味熟悉的烧脑价格却低了一倍不止!

**

**

甚至《Mathmania》的唯一问题——知识点稍显庞杂的情况,这套中文版,也特地根据国内小学数学课标,把原版的内容做了重新的编排和分级!

将 1-6 年级的小学知识点循序渐进的分成三个级别,分别是适合幼小衔接小学低年级孩子使用的入门版、适合小学低、中年级孩子使用的提高级版、以及小学中、高年级孩子使用的挑战版。

**

**

3 套图书,共 30 册,完全能够满足整个小学阶段的数学游戏需求

每周做几个数学游戏,坚持下去,就一定能见证孩子的数学能力成长和数学思维的进步。

此外,这套书还特地邀请名师,对难度较大、思路较为复杂的题目做分析,提供给孩子多种解题思路,拓宽思维!

好书难得,即使是低龄的孩子,也建议入一套。因为知识是永远不会过期的,有书在,才有看的可能。

【数学天才玩出来】

风靡全美的重磅数学游戏书原版引进

几乎囊括 1-6 年级的所有数学知识点

边玩边学更好理解数学概念帮助提升孩子数学能力

[

数学天才玩出来 入门级 (1-10) 综合读物 少儿 天地出版

天猫

¥84.32

去购买​

](https://uland.taobao.com/ccoupon/edetail?e=jhha7f31GbylhHvvyUNXZQst4RUgB%2BQM1dI5raycii4uynEbLfMGNpuwdvbsxuix2yZugn64OL7yMW3eIAWKRa6LeGhgJY%2B%2FS%2BreFeRsyJbXf9vlJpT9pag26qABpOhLL0K5wP1X3W%2BIA6jjcweSve7Je37Eb9sh3IUUB4fliO1DgALVjBcq0cU1eELcThDu0H2i7N0oQCiKnpmVD2ZOOwWpRuBNPWNrOxCh%2Ft7Cums%2BMX%2Bn%2FRCbolxZL6bdMGUdNVAeL7MOcQlK1Ctf8F4HBS05U98g2UZ6guhEtL8BJnlyD2riaLmgN0DcDNbDhKcAoCHCUosWNEKHoL2TTpnYROzB1bY1M7mGFl9Bz75lHa1rhaqfExWhzMlMTjSQsK4p5bgGAUYJbG0%3D&traceId=213fca2516492039062632848e29f4&union_lens=lensId:TAPI@1649203906@2108a52a_08e6_17ffc36892a_51af@01)

[

数学天才玩出来 提高级 (1-10) 综合读物 少儿 天地出版

天猫

¥84.32

去购买​

](https://uland.taobao.com/ccoupon/edetail?e=BPc2jK8ms7alhHvvyUNXZQst4RUgB%2BQM1dI5raycii4uynEbLfMGNpuwdvbsxuix2yZugn64OL7yMW3eIAWKRa6LeGhgJY%2B%2FS%2BreFeRsyJbXf9vlJpT9pVm7HB6K5jcGD3oQqrg8jRWIA6jjcweSve7Je37Eb9sh3IUUB4fliO1DgALVjBcq0cU1eELcThDu0H2i7N0oQCiKnpmVD2ZOOwWpRuBNPWNrOxCh%2Ft7Cums%2BMX%2Bn%2FRCbolxZL6bdMGUdNVAeL7MOcQlK1Ctf8F4HBS05U98g2UZ6guhEtL8BJnlyD2riaLmgN0DcDNbDhKcAtc2EzoWE3EK57D7%2BXmSnRlBjNzOWlt1yFl9Bz75lHa1rhaqfExWhzMlMTjSQsK4p5bgGAUYJbG0%3D&traceId=213fca2516492039062632848e29f4&union_lens=lensId:TAPI@1649203906@2108a52a_08e6_17ffc36892a_51ad@01)

[

数学天才玩出来 挑战级 (1-10) 综合读物 少儿 天地出版

天猫

¥84.32

去购买​

](https://uland.taobao.com/ccoupon/edetail?e=3KbY7L7nxNKlhHvvyUNXZQst4RUgB%2BQM1dI5raycii4uynEbLfMGNpuwdvbsxuix2yZugn64OL7yMW3eIAWKRa6LeGhgJY%2B%2FS%2BreFeRsyJbXf9vlJpT9pVSob3gbVoobt3YlKERssJSIA6jjcweSve7Je37Eb9sh3IUUB4fliO1DgALVjBcq0cU1eELcThDu0H2i7N0oQCiKnpmVD2ZOOwWpRuBNPWNrOxCh%2Ft7Cums%2BMX%2Bn%2FRCbolxZL6bdMGUdNVAeL7MOcQlK1Ctf8F4HBS05U98g2UZ6guhEtL8BJnlyD2riaLmgN0DcDNbDhKcAN3tvOzPB2l3dK6dZkK31ClBjNzOWlt1yFl9Bz75lHa1rhaqfExWhzMlMTjSQsK4p5bgGAUYJbG0%3D&traceId=2132d91616491171653033027eea33&union_lens=lensId:TAPI@1649117165@213ff2e0_1447_17ff70af891_1c88@01)

Part 1

这套书,我觉得最出彩的地方在于——玩中学,它通过游戏的形式让孩子学数学,大大降低孩子对数学的畏惧心理。

别小看里面的数学游戏,这些题目编排都非常用心,而且难度循序渐进。

我们就拿 “数字连线” 这个游戏来说,它的题目编排都遵循着孩子的认知规律。

下面这张图是 “入门级” 的数字连线,它一开始是从 “顺序数数” 开始,这是 “认识数字” 这一知识点的第一步

接下去,“入门级” 的连线游戏出现了针对 “偶数” 的连线训练。而了解奇偶数概念是 “认识数字” 这一知识点的第二步

(来自入门级第 1 册)

紧接着到了 “提高级”,依然存在连线游戏,但它是进行 “等差数列” 的计算,这是 “认识数字” 这一知识点的第三步

(来自提高级第 2 册)

再比如 “坐标定位” 这一知识点。

下面这是 “入门级” 的题目,用拼图的形式,让孩子初步理解坐标的概念。

(来自入门级第 2 册)

而下面这是 “挑战级” 的题目,是通过十分复杂的画图游戏,帮助孩子更深入地理解坐标知识。

(来自挑战级第 10 册)

Part 2

这套游戏书之所以能这么受欢迎,除了它的游戏形式以外,游戏设计里非常注重数学思维的培养也是重要的一个原因。

数学是一种思维方式。数学学习的目的并不是为了简单地掌握数学知识,而是要在数学学习的过程中,发展孩子的思维,提高数学素养,用数学去分析、解决实际问题。

1

注重多种思维培养

小学阶段常见的数学思维,比如空间思维、逻辑思维、等量代换思维等,这套书都会训练到。

比如,下面这个游戏是训练集合的概念,什么交集、并集、包含…… 这些集合里的概念这里都有涉及。

(来自挑战级第 9 册)

下面这个 “动物跷跷板” 的游戏训练的是**等量代换的思维方法,它也是孩子未来学习代数的基础。**

**

**

(来自提高级第 6 册)

2

多个方法引导孩子建立数学思维

书中的引导方式,常常给人一种豁然开朗的感觉,特别有意思。

比如这个 “数字神探” 游戏,就将数字与英文字母巧妙地结合起来,孩子需要经过一番逻辑推理,找到每个数字对应的英文字母,最终将其拼凑成一封重要的、有破案意义的信件。

(来自提高级第 1 册)

比如这个剪纸游戏,通过重新拼组,找到最终的答案。

(挑战级第 2 册)

3

解题思路来辅助

此外,每本书专门设置的 “解题思路” 版块,还是特地邀请名师,对难度较大、思路较为复杂的题目保驾护航。

事实上,这种有过奥数经验的名师参与,他们的解析往往会为了拓宽孩子思维,而提供多种解题思路。

比如超经典的 “鸡兔同笼” 问题,就在提高版中,出现了更复杂的 “鸡牛同栏”。

(来自提高级第 7 册)

而答案,就分别从假设法、列表法这两个方面,去引发孩子思考。

挑战级” 出现了 “制造车辆”, 它依然属于 “鸡兔同笼问题”。

(来自挑战级第 6 册)

这里依然提供了多种思维角度,除了之前接触的假设法之外,还有方程

除了讲解解题步骤之外,书里也都配备 “参考答案”,方便孩子检查。

【数学天才玩出来】

风靡全美的重磅数学游戏书原版引进

几乎囊括 1-6 年级的所有数学知识点

边玩边学更好理解数学概念帮助提升孩子数学能力

Part 3

这套书的游戏非常注重生活互动性和实操性,我们来看看书里的游戏和普通习题的区别。

这套书里的游戏,和市面上其他普通习题的区别之一,是因为这套书的游戏,重点关注了生活的实操性和互动性。

比如说,一般练习题,在锻炼孩子的加减运算,是这样的,孩子看的就头大,更别说做题了。

这套书,却将小数的加减运算,融入了 “收支平衡” 这个游戏中,生活应用性非常强!

(来自挑战级第 9 册)

书中非常多的游戏,都是以生活运用作为场景,让孩子看似是在解决日常生活问题,实际上已经把数学知识点摸索透了。

这种在日常生活与数学知识之中,建立联系的方式,不仅能帮助孩子树立解决日常生活问题的信心,还能进一步提高孩子数学能力!

球队拍照,要按照高矮顺序排队。孩子是否能运用加减法完成这次排队呢?

(来自入门级第 3 册)

帮妈妈买东西,需要精打细算,怎么运用小数的计算,来选择购物券,让价格更优惠呢?

(来自挑战级第 1 册)

值得一提的是,这套书和那些动不动就厚厚一本的练习册不同,它主要是采用了小薄本的设计,孩子边玩边学,压力不大,短时间内就可以做完一本。

在坐火车、乘飞机的旅途中,都可以拿一两本出来当游戏一样玩,不仅能缓解旅途的烦闷无聊,还可以趁这个时间充分学习。

也方便拿到学校,和同学一起挑战书中内容,增进同学间的感情!

此外,本书是大 16 开,开本较大,这样孩子在游戏时涂画都比较方便。内文采用 100 克国学经典纸张,既方便书写,又保护眼睛。

Part 4 分册介绍

这是入门级 10 册:

它包含了近 200 个趣味性超强的数学游戏,涉及奇偶数概念、数字顺序、数量对应,加减运算、平面图形、立体图形、简单逻辑推理等小学 1—2 年级的数学知识。

适合幼小衔接和小学低年级孩子阅读使用。

下面这个是知识点列表:

这是提高级 10 册,包含了近 200 个趣味性超强的数学游戏,涉及跳跃计数、四则运算、运算符号、平均数的计算、时间计算、直尺测量和空间方位等小学 2—4 年级的数学知识。

适合小学低、中年级孩子阅读使用。

这是知识点列表:

这是挑战级 10 册,包含了近 200 个趣味性超强的数学游戏,涉及四则运算、小数的加减法、分数的概念、小数与分数的转化、百分数概念、周长的概念、角的度数、最小公倍数小学 3—6 年级的数学知识。

适合小学中、高年级孩子阅读使用。

这是知识点列表:

【数学天才玩出来】

风靡全美的重磅数学游戏书原版引进

几乎囊括 1-6 年级的所有数学知识点

边玩边学更好理解数学概念帮助提升孩子数学能力

[

数学天才玩出来 30 册 挑选 Highlights 入门级提挑战数学

京东

¥288.00

去购买​

](https://union-click.jd.com/jdc?e=jdext-1487392165785001984-0-1&p=JF8BARQJK1olXgAEVl9cC0sRBV8IGloQXQUAXFlfCUgWAF9MRANLAjZbERscSkAJHTdNTwcKBlMdBgABFksWAmoIGFkdWgQDV19eFxJSXzI4RQwTIlAAAR09CBcfdWZaGFNTGWV8NFJROEseA20AGloVWgUyVF9cCE8SAGwOGmslXQMyFTBdCUMWBGYOEmsUbQYFUVhYCUsfAGgOHV4lXQ4GZIjznJ6fvLa4vo2_89OP5IvUo5-phV84K1glBlNWHRtBCE4eBW0AGFsSWQALUF9cCEMeBnMJB1slXQcDV1xfDU8eAQEJdVgUXgIAV1ZYCksnA18LKwV7XFUCV1lcC0J5Xm5NGgFIFkBsUVlaD0sTAV8KGloXXzY)

知乎用户 小七 发表

因为人家的精英学生比我们学得更苦更难啊。

顶级成就只看精英,不看平均。

问题来了:

既然如此,那为什么要让我们的平凡小学生要学得比他们难?

我问你,为啥一般公司里的专业技术人员比老板要更懂技术,挣得却比老板少那么多?

因为你只能卖脑力卖苦力,而人家手里有资源啊!

人美国五六十年前就把全球大部分资源握在手里了,发点货币,全球都给他缴通胀税,商品哗哗的来。而你父辈祖辈没攒下什么基业,没能控制全球啥资源,也做不到凌驾全球的武力威慑。那你现在就只能在国际市场上打工挣钱。不把各方面技能学好,你能挣到个毛线的钱

知乎用户 悠然南山见 发表

因为做出超级牛的东西的人,是里面的天才,也有可能是外国移民。所有资源向你倾斜,成才几率大高。但你回头时发现,这个社会 80% 的人连高中数学都没学好,也就只会四则运算,但足够用了。公立学校大家快乐学习就可以了,你换成想以名校为目标的美国私高试试,比中国还要激烈。

我国是资源有限,竞争激烈,你需要拼过大多数人,普通难度区分不了人时就加大难度,内卷化。等你上大学后发现,其实很多是没必要的,毕业后发现很多压根没用。

知乎用户 徐义尧​ 发表

付云皓大神的文章很令人很感动。为了切题,对那个答案需要稍加补充的是:竞赛出成绩和做出好成果,还有很长的一段距离。不过我觉得题主原问题的根本原因基本还是涵盖在那个答案之中了,只是作为答案为了简练,还可以提炼一下。

个人感觉最重要的还是中国在教育资源,尤其是优质的高等教育资源上和美国的差距,导致中国的学生必须要承受很大的压力,做很多的重复低效的训练,并且因此被消磨了兴趣,从而逐渐输掉了竞争。IMO 成绩大抵是我们从整个故事前半段某个时间点上捕捉的一个剪影,不过确实反映了问题。另外像人才的流失,庸师的误导等等其实也都是相同的根源。 如果教育蛋糕的总量不增加,真的很难找到最佳的办法让最优秀的人都如愿分到一块好的。

那篇文章对高中阶段的种种乱象都有所提及。其实在大学教育阶段,又何尝不是如此。据我了解,很多学校的优秀本科生,整个本科生涯就是围绕着考上好学校的研究生。然而考上好的研究生的标准常常并不十分科学,尤其是国内考研的英语和政治简直让人吃了屎一样难受。这样度过的本科四年们,和那些盲目地刷竞赛度过的高中三年们,不知毁掉了多少创造力和价值。我们无法过于责怪应试制度,因为它还至少保障了公平。我们只好感叹,要是中国有 20 所清北,200 所 985,那一切都会不同。这样的假设下,埋葬了不知多少人的青春和激情。

于是乎,莘莘学子们蜂拥逃往国外。或者说,追随着梦想奔赴他乡。然后,他们又会蒙受指责,什么精致的利己主义之类,全都扣到头上。不过我相信,当一批批理想主义者从美帝和其他帝国主义国家载誉归来时,或者在国内潜心研究之余时,他们是会为祖国提供更多更为优质的教育资源的。我觉得我们正在见证这一变化。

知乎用户 吴湘 发表

我觉得美国的中小学公立教育的数学师资就有问题。老大上三年级的时候,拿回家一道类似于鸡兔同笼的问题, 具体题目忘记了, 反正是我们小学做的同类问题的简单版。然后她告诉我,老师带着他们做了一节课, 始终做不出书本最后的那个标准答案,最后老师自己放弃了。我用一分钟把题目解出来给她讲了一下, 她明白了。

老大四年级的时候的班主任老师是一位非常优秀的男老师, 给他们培养了良好的学习习惯, 尤其是英语教学非常有特色,孩子四年级以后变得非常爱读书和写作了, 就是这位老师的功劳。但是这位老师教数学只能用惨不忍睹来形容。他只允许孩子用某一种固定方式来做除法和分数,我怀疑是别的方法他都不会, 孩子始终没有搞懂为什么只能这么做。

我们这里的公立小学, 还是本地比较好的小学, 在 http://greatschools.net 上评分是9分(最高为10分),比这差的学校就不可想象了。所以看到这种情况, 我只好果断把孩子送数学补习班了。

为什么能做出很牛逼的东西?因为该行业最优秀的人才都集中在美国, 比如数学补习班的老师, 清一色的中国名校本科毕业, 在美国读的硕士或者博士。比如美国奥数训练营的带队老师, 就是中国人。如果你真的对数学感兴趣,这里有世界上最丰富的资源和最优秀的老师, 只不过他们都不在公立教育系统。记得给孩子买过一本小学课外数学书, 是一个MIT的教授编的, 讲得深入浅出, 孩子数学一般, 但读了以后就明白什么是二进制了, 很快6进制,8进制也能按理推算了。

知乎用户 邪来烦恼风​ 发表

为什么总有人觉得是愚民政策呢?凭良心说你高考那些东西之后有什么用?我一个数学作业的,对我都没用,那有什么必要每个人都要学?一套考核标准,既不公平,又不科学,还没有用,我们真的有资格批评人家吗?

普通人,只要干一份简单的工作,会简单的加减乘除,就已经够了。教育的目的不是让你产生学的越多越牛逼的错觉,而是让你保持兴趣,让人找到自己人生的方向。就这个意义而言,我们的教育是失败的。不管是哪个国家,对数学感兴趣的人都一定是少数。那么教育就应该让这些人的兴趣留下来,而不是让他们厌恶数学。人生是没有固定答案的。

不过结构性失业,一般人又能有什么办法呢?

知乎用户 王旭东​​ 发表

我们的数学教学思维出现问题了。而且是很严重的问题。我看过苏联那套绿皮的微积分,美国的斯图亚特微积分以及我们经典的同济六版。算是可以稍微比较一下几个教材间的优缺点。苏联人的教材就是深,基础理论非常深刻,苏联人是从戴德金划分开始的,一开始就用非常严密的数理逻辑建立起实数域的理论,然后再一步步扩展微积分的理论。看苏联人的书如同看推理书,非常严密,非常深入,也非常具体。缺点就是枯燥。而美国人的书呢?是从初等数学开始的,一开始光是直线公式就反复导。美国人更看重的是数学建模思想,之所以在一些很 “简单” 的事情上反复做出各种数学推导,就是为了让数学建模能够融入到生活中去。其实很厚的一本书,大部分就是反复在打基础。而且讲真,美国的书印刷真的精美,图文详实,就是把学生当傻子一般生怕搞不懂,所以从最基础、最简单入手。

那么我最想评(吐)论(槽)的就是同济六版了。具体内容大家都学过,我就不具体教学的内容了。这本书给我的感觉,就是为了完成教纲任务。可以看,但是自学就比较困难。没有特点,报流水账一般。唯一优点就是没啥错误(如果看过谭浩强的书的话,会觉得这书写的还算不错)。

通过教材,我们可以看出一个问题,就是对于教学的方式方法和思想的问题。苏联没经历过所以不清楚。美国的看过公开课,那是在上课传授知识。那么中国是什么情况呢?中国的老师就是在完成任务指标。这仅仅是一项工作,能糊弄过去就行了。当然也有责任心强的希望学生多学点。但是大部分老师就像来顶个岗一般。有人说这是大学情况,初高中不是这样的呀!那么大家都经历过小初高,是什么情况呢?老师更强掉的是猜测题型、强调解题技巧,从头到尾就是解题、解题、解题。数学思想呢?知识体系呢?实际应用呢?这不是老师们的重点,重点是你能考多少分。

我们重视的是教育改变命运,或者说教育的目的异化了,通过拼解题来获得社会上升渠道。而忘了教育的根本目的是为了培养人才、传授知识、启蒙民智。应试就如同一个竞技场,大家在里面拼命,愿赌服输,输的没啥好埋怨的,赢的自然就有更好的机会。这样就是把教育变成了斗兽场,就是为了 “社会公平” 这样一个 zz 正确而已。所有的人都能看到教育的问题,但是所有的人都不愿意去轻易放弃这样的渠道和更改这样的规则。不仅小初高激烈竞争,大大小小的幼儿培训班难道不是这种竞争的延续?我甚至看到过幼儿园开始的编程教育。这些孩子真的能学到系统的开发知识吗?我乎有很多搞 EECS 的,真的学一门语言或者学习一个机械结构需要多少底层知识?就那些与或非、分支、循环,没有一定的逻辑心智,四五岁的小孩能搞明白?培训机构贩卖焦虑的同时,不仅在收割智商税,还在真正拔苗助长、误人子弟。但是没办法,培训机构就是教育竞争的延续,资本肯大笔投入蜂拥而至,本身就说明了这方面的刚需。而家长们就像是群体性对拼内力一般,谁都不肯轻易撒手,只有无止境的投入。

所以,我很困惑,我们的教育真的是在教育吗?

知乎用户 西塞狼烟​ 发表

是说美国的普通教育的数学比中国的简单,精英教育其实比中国要求更高。西方是将普通教育和精英教育分离了的,由于精英教育数量相对较少,没有引起大家的注意。美国的普通教育主要是培养劳动力人口,精英教育培养高端人才。

在纽约和加州的精英学校,在数学教育上不仅不比中国学校简单,而且学习时间也超过中国。许多学生一天只睡四个小时,加州精英学校常有学生跳楼自杀的报道。我们判断一个事情不能被表面现象迷惑。看看这个吧!https://3g.163.com/dy/article/EEB5160E05198035.html?spss=adap_pc

知乎用户 老查留学老查老师​​ 发表

“美国中小学生的数学比我们简单” 题主可能说的可能是 SAT 和 PSAT 考试当中的数学测试部分会比中国高考简单。

但是美国高考并不是美国选拔人才的全部考察标准,这跟中国一刀切的教学模式有本质的区别。美国高中生有选择自己课程难度的权利。

如果你的目标是美国的顶尖大学,不仅要参加 SAT 的学科考试,还要学习 AP 考试里面的微积分。和小伙伴一起参加建模竞赛,提前学习编程,向大学展示你对学科的兴趣和探索深度。难道你还会说美国高中的数学比中国教科书数学简单吗?

如果你的目标是文科或者只是普通大学、社区大学,你只需要对数学有一定层次的了解就可以,那么你说美国高中数学比中国高中简单那是肯定的。

美国高中教育是把学生分层,因材施教,每个学生的选择得到充分的尊重。

有任何美国本科留学问题,欢迎添加小助手微信(laocha103)咨询。

知乎用户 赵山河 发表

不要相信美国所谓的快乐教育,以为美国人都是憨憨。美国的快乐教育是针对普通家庭的,这些家庭里的孩子最后可能会上社区大学,这批人最终多数都服务于那些精英阶层。

美国的孩子补课什么都跟亚洲人差不多,也就是精英阶层的孩子,会接受精英阶层的教育,他们起点就比普通人高。

另外,很多发展中国家向美国输送了大量人才,这也是非常严重的问题,中国、印度等等国家培养出来的骨干,都去美国当了专家学者。

最后就是内部精英子弟掌权,再引入出色外援供给美帝服务,所以美国能如此牛 X。

知乎用户 Gavin​ 发表

美国的情况和中国有比较大的差别。他们公立教育很差,但是私立教育狠严格。所以不能一概而论说美国中小学学的简单。反正美国两极分化比中国严重。并不是所有的中学的教育都是快乐简单的。

知乎用户 yunfeng ding 发表

美国教育系统是分化的,精英教育和快乐教育,我们国内看到的大多是美国快乐教育的一面,其实快乐教育的另一个叫法是穷人教育,快乐教育只做了很粗浅的教育和认知,水平如何可想而知。

但美国的精英教育强度一点不比国内的教育强度低,甚至更有挑战性。美国精英教育重视启发,注重培养学生的能力特长,毕竟不是所有人都擅长所有学科,这一点和中国像流水线生产产品一样的标准化培养模式不同。

美国精英教育的特点是能培养出更多具有创造性的人才来。再加上美国每年会吸引全世界大量的精英人才,所以美国的科研创新一直保持世界领先。

中国的教育特点是培养成本低,可以短时间内提高国民的素质,毕竟中国刚刚消灭文盲也没多少年。现在国民素质已经得到了大幅度提升,也开始注重培养创造性。但人才选拔制度还是没怎么变,而且到了高中还需要学习多门学科,且各学科必须平衡发展,不然高考就非常吃亏。偏科的学生就非常吃亏,哪怕在某方面是非常有潜力的,所以我觉得国内高考制度需要改革。

知乎用户 逗逼宫 发表

在课本中的当然是必修内容,中国认为,大部分学生不具有自己选择适合自己的教育的能力(事实确实如此),而在欧美,很多家长能够给子女更好的引导以选择自己喜欢学习的东西,换句话说,虽然欧美的高中课本可能只到中国初中水平,但是真心喜欢数学的孩子们因为不需要必修语文英语理化生,所以很可能已经自学到大学水平了呢。

最后,毕竟从小学起,他们不需要学思想品德教育、初中政治、高中政治、思修、毛概、马哲……

知乎用户 麦克信田 发表

因为中国不肯给可以做出超级牛的东西的人开高工资,反而从各个方面给他们设置障碍

知乎用户 山栀​ 发表

美国通过二战,冷战,和后来的全球霸权,积累了巨量的财富,搜罗了全世界的顶尖科学家。然后美国的资本家们设置了一道墙,你只要足够优秀,你就可以翻过墙来一起享受荣华富贵,很少人能抵挡得住这种诱惑,绿卡就像是吊在磨驴眼前的萝卜一样诱人。同时美国的资本家们还要圈养一群猪,这群猪是自己的基本盘,保持自己权力的合法性,同时还要有阶级剥削的合理结构。

从自己圈养的猪里养出精英过于费时费力,而且猪太聪明终归是不好的,不然万一比我懂王更懂,那我懂王怎么续合同。而世界上其他国家想要羡慕美国的强盛国力,就要大力投资基础教育,疯狂内卷养蛊人才,然而培养出的人才还没等用起来,就自然而然投入了美国的怀抱。所以美国选择白嫖全世界的精英,然后只提供高质量的教育给精英的后代,剩下的猪自生自灭奶头乐就好。

想想美国公司的主要战斗力组成,从二战之后的西欧精英,到冷战后的东欧精英,到现在全球化时代的中印精英,是不是一茬接一茬,绵绵无绝期?

然而现在出了一个变数,就是中国的精英美国白嫖起来确实爽,但是中国的精英有一部分不甘心当拉磨的驴,问题就大了起来。

知乎用户 雨路青松 发表

小学学的好,是因为对就是对,错就是错,

天王老子来了,1+1 也等于 2,平面三角形内角和就是 180 度。

研究搞不好,因为 “说不对就不对对也不对”

思想不包容,则科研无发展。

知乎用户 桓大司马​ 发表

目前的顶尖科学家,绝大多数年纪大概在 50 岁到 60 岁之间,一般在上个世纪 60 到 70 年代上中小学。美国现在的顶尖科学家比中国多,说明美国在上世纪 60,70 年代基础教育比中国文革时期做得好。

2000 年以后中小学培养的学生,要到 2040 年以后才有可能成为顶尖科学家。

知乎用户 刘宇波​ 发表

你说的都是小学、初中、高中阶段,但是大学阶段就截然不同了。

而我窃以为,大学才是学习知识;尤其是专业知识最最重要的时期。

P.S. 这个问题我以前还经常会从上面这个教育现象本身找原因,最近才发现,我们还要尊重美国是一个移民国家这个现实。不断有全球最聪明的科学家、工程师涌入美国,而他们是没有接受美国教育的。这些人对美国科技作出了很大的推动;但他们和美国教育一点儿关系都没有。

知乎用户 套路​ 发表

![](data:image/svg+xml;utf8,)

其他我不知道,我只知道像苏妈老黄这样的人,未来可见日子还会源源不断的去美国创业创新了。更何况清北等名校源源不断的输送炮弹呢。印度送的相信也不会少。至于你说那些磕药逛街的黑叔叔黑姐姐?他们开心就好啦。

知乎用户 亲爱的会员 发表

美国人和日本人在太平洋上打航母大战的时候,中国连像样的步枪都造不出来。

半个世纪之后,中国人在操心我们为什么还没有干过美国,为什么还不是世界第一。

答主是不是应该问,美国教育为什么这么差劲,以前那么大的优势,怎么搞成现在这样子

知乎用户 阿观 发表

核心关键:热爱。

美国人和我们,对 18 岁之前的学习的有不一样的看法。

美国人认为的学习,教会孩子学习的能力,以及生活的能力,以及有去工作的能力,以及兴趣,而不是死磕知识点。

我们认为的学习,是考试,是成绩,无关生活和工作。

很多人以为美国人散漫,这也侧面反映了,美国人的这种态度,给了孩子足够的空间和自由度。正是有了这样的自由度,美国人的教材,和整个社会的观念,并不让孩子死磕知识点、考试、成绩。

所以,很多孩子,该散漫的依旧会散漫,成人后可能成了社会底层,但这种所谓底层,并不见得是一种我们以为的那种悲哀的底层;

但最关键的,也就是回答题主所问的:另外有一部分人因为拥有了自由度,会发现自己的热爱,投入热爱,去持续钻研,便会在大学阶段、研究生阶段,持续的深挖学科,做出真正切合实际的尖端应用。

所以,给孩子自由度是最关键的。
18 岁之前,慢一点,给孩子多一点的时间和空间,让孩子在 18 岁之前发现人生乐趣,发现自己感兴趣的方向。
然后,才会在 18 岁之后发光发热,才能创造出 “超级牛的东西”。

所以,关键还是,让我们的孩子慢一点,让孩子发现自己的 “热爱”~

知乎用户 公子千瑶 发表

因为是精英带着一群普通人在跑,普通人只要听话跟着就行。

智商,家境,父母,从小注定就分流了呗。精英培养,当然比一大群人撵着往前赶的轻松又更见成效啦。

说白了,就是讲先天公平,而不是 “教育公平”。

知乎用户 放浪者​ 发表

我只讲一件事:在美国大学抄作业是不可能的!

不可能的!教授和 TA 真的要检查!

当年我读 phd 的时候,算法课上就把作业给一个女同学抄,仅仅是 homework,然后教授差点给我们报到学校,幸好女同学开动泪水攻势在 Office 大哭起来,教授可能没见过这场面就算了… 现在那同学也是某著名无人车驾驶公司的核心成员了…

总而言之,万事就怕认真二字。不要说什么基础教育高等教育什么平民精英,我们的高等教育认真吗?严谨吗?平时作业不互相抄吗?代考的有打击吗?

先谈认真再说其它

知乎用户 Tango 发表

首先,直接原因是:做出超牛逼的东西并不依赖于中小学数学。

次要原因是:人家之后的教育没那么拉跨,看看人家写的某些东西的源码你就知道了。

玄学原因是:人家的教育看起来简单但更接近本质(这一条你选择性相信,只能说计算机方向几乎所有大佬都在推荐把书扔了用人家的教材。。。)

最后,人家的教育应该是真的不简单,他们可能只是刷题没这么凶。

知乎用户 王庚 发表

因为你从小就信了这句话!

知乎用户 山顶洞人 发表

你搞反了,是做出超级牛的东西的人变成了美国人。

知乎用户 天色 发表

因为美国没有厚重的历史拖累。喜欢数学的人基本上就只学数学,喜欢物理的人基本上就只学物理。但是中国的就是啥都要学,而且学的很难,而且中国还有一门英语课,英语要背诵各种单词句子语法还要有一定的阅读量,难度不亚于语文,我的很多同学都是被英语给卡死了,他们一天到晚疯狂的背单词,但是由于没有英语环境所以提分很难,几乎一半的时间都浪费在了英语上。

而且这还是高中的表象,最关键的就是初中。初中是不分文理科的,所以还要学政史地,这三门课都是需要花很多时间背的,而且很多人就是被这三门课给卡了,导致很多理科极好的同学读不了好高中。当然,有极少一部分在以前管的比较松的时候,靠着极好的理科能力去外地参加外地中学的招生考试,但是现在不行了,只能在户籍地读高中,这样又有一大部分的同学被刷下去。

而且大部分中国人一辈子的心力就在高考被耗尽了,到了大学就疯狂的玩。

知乎用户 爱宕洋榎 发表

别吹了…

去看看美国各大公司的研发部门有多少外国人就行了.

不用算印度裔的高管, 不用算华裔的工程师, 不用算日裔的设计师. 要知道很多你分辨不出的白人面孔里, 相当一部分人也都是来自东欧, 北欧, 伊比利亚半岛的外来人才.

美国真正牛叉的是什么? 它的娱乐业, 它的法学, 它的学术自由和严谨性, 以及优越的, 成规章的制度管理. 更重要的还是它庞大的资本支持和金融体系.

牛叉的不是美国的教育. 是美国的资本…

知乎用户 蒙蒙 发表

因为数学差的美国人和做出牛逼东西的美国人,不是一拨人。

知乎用户 Tuckers 发表

书是人写的,课时人教的,学是人上的,试是人考的,卷是人评的,事是人干的,成绩是人评价的。

教育的本质,是师傅带徒弟,徒弟有兴趣好好学,然后迭代。

但是我们发现普及教育的时候,并没有那么多好师傅,不好的师傅该如何带徒弟呢?于是我们发明了大规模教学,教材,课纲,然后说:如果你不行,你还要教书,你就用这套。转身对好师傅,好学生说,你们要是能行,你们不用信这套,你们可以私塾,当然也可以竞赛,特长班,特别优秀的可以试试少年班

题主问,为什么美国的差老师用的差教材的数学,比我们的平均线老师的平均线教材的数学简单,但是他们的好学生却很牛逼?

所以并不是参与过教育的就等于理解教育。就如同吃过饭不等于会做饭,会做饭就懂餐饮行业。

这只是其中一个角度。

从这个问题引申开去,公立教育的第一属性是公立带来的,是普适和公平,不是效率和因材施教。

就如同不能要求公费医疗体系拥有超越私人医生的效率一样,我们不应期望公立教育提供个体的过度收益。尽管总有少数人不必须公立体系的社会产品,尽管必须公立体系的我们在观察结果时可能认为这有点儿不公平,但是如果我们可以横向比较各国的教育等公共服务系统,目前没有人做的比我们更好了。

当然,另一个角度,美国是一个移民国家。很多我们认识的牛人不是在美国完成的基础教育 ,甚至高等教育。北京的著名美国大学预科们每年提供那么多科技民工,他们都是在中国完成的基础教育。所以,仅从名人一个指标,对比各国基础教育是毫无意义的。看看五大联赛各个球队,你就可以知道,青训并不能稳定顶级人才给本队,优秀的青训和他们的主队成绩没有线性相关,球探也并不只去看青训,但是优秀的青训对球队的成绩是有帮助的——球队成绩好坏拿不拿成绩看的是职业队阶段的策略,跟青训已经隔了好几层影响因素了。看球的,拿青训说成绩,要么不懂,要么想赚钱。

做教育的也如此。幸亏双减了。下一步希望尽快以《未成年人保护法》附件的形式将 “鸡娃” 和家庭内虐待儿童联系起来,Z 时代学生应该还有救。

WHY?

我不是元宇宙的拥趸,但是疫情影响下的一线高等教育上 VR 是想干就干马上就能干的事儿。再让家长瞎胡闹,用 “迷信”“愚昧” 代替儿童科学自然成长,逆人类社会发展大势而行,形成新时代的文盲阶层就不远了。

假设今天 2021 年有一个 8 岁二年级的孩子要学数学。我们用 10 年时间,形成了不可磨灭的纸笔学习习惯,上了大学再接触当代计算机辅助下的各类应用。那可是 2031 年了。孩子交废了,开不了无人机,做不了虚拟现实里的客服,连搬东西的机器狗都不会操纵,只能做抛弃的人类社会的干电池,你负责我负责还是他家长负责?

他家长要不要来这里提问?为什么他小时候数学那么难,数学又那么有用,长大了却找不到工作要饿死?

这当然是极端的比喻了,不过如果您不抬杠的话,能理解以上道理。

那么我们应该怎么做呢?先完善相关法律执行。谁虐待儿童就抓起来。谁让老师加班不给钱就抓起来。谁在教育领域搞迷信用保健品的思路卖课就抓起来。媒体谁天天连调查都没有就瞎吹工资率就按造谣抓起来。媒体要是还敢反过来散步恐慌,拍都市丽人后宫剧一帮人天天啥也不干就敢设定年薪百万就抓起来。哪个什么小四什么的出生的时候就给他抓起来。乱世用重典,愚昧的事情会很快消失,大家会好起来。

不同意呀?觉得太不人道啦?那我们继续 55 分流呗。反正是二选一。

知乎用户 酱肉包 发表

哎~ 怎么又是这种问题啊,不要以结果来倒推原因,因为这样最后倒推的结果往往都是错误的。

看到很多答案都在说美国基础教育怎么厉害,能筛选天才,似乎美国现在技术牛逼是从立国开始就由这些教育制度形成了,这样说你把一战前真正世界数学中心的欧洲放哪里了?

可以说在 1894 年美国 GDP 超过英国成为世界第一的时候,那时美国的数学水平还很一般,远不能和欧洲相比,别说英,法,德,就连和当时的沙俄比都有一定差距;要知道美国 GDP 都世界第一了,怎么那个时候美国所谓的基础教育制度没有发挥作用?

当然有些人会说为啥天才都往美国跑,为啥我们现在很多吸引人才的制度都吸引不了几个人才?

笑话,至今很多天才都往美国跑,那是因为安全,是因为发达,你看看一战前多少天才往美国跑?那个时候美国 GDP 都世界第一了,结果大多数跑美国的还是类似杰克那种靠打牌才能赢到船票的混混。

可以说美国现在科技这么牛逼的原因,地理环境起的作用远比所谓制度起的作用大,比如题主说的计算机图形学领域,要知道 “计算机科学之父” 图灵可不是接受美国基础教育长大的,人家是英国人,可为什么后来美国计算机科学那么牛逼,就是因为二战让欧洲大多数相关的科学家都跑过去了,管美国基础教学什么关系啊!

就美国那种自由散漫的制度,那种州可以叫板联邦政府的制度,也就是因为在北美这么安全的地方才能扬长避短,如果美国这种制度出现在类似波兰那种强国环绕的地方,现在美国能不能存在都是问号,何来牛逼之说啊!

知乎用户 ourenhyou 发表

不矛盾啊,

本土的精英玩金融,政治,农业,法律。

移民的精英才搞科研,医疗,工业。

看看美国大学里真正搞学术的大部分都是留校的留学生,搞行政的都是本土的能人。

懂了吧!!

知乎用户 forcode 发表

我感觉中国式教育,经常是拔苗助长,超前学习,我儿子的幼儿园大班作业,每晚回家要做 1~2 小时,里面有些内容难度明显超过了 6 岁孩子的理解能力,让孩子的学习过程充满了挫折感,小小年纪就开始厌学。

美国式教育,可能更重视学习兴趣的培养,好奇心的呵护,培养兴趣引导的学习、探索、自主阅读能力,才是教育的本来涵义,教师和家长扮演的是学习引路人的角色,而非强迫学习的督工。

另外,老外的动手能力超强,看看他们 DIY 的各种器械工具玩具,叹为观止。这跟外国爸爸普遍有一个自己的车库工具间很有关系,里面各种设备齐全,从小耳濡目染,就培养了动手的兴趣和能力。

中国中小学生一天到晚在解题,就算有兴趣有工具间,也没时间去玩,甚至很多家长都不让孩子做一丁点家务,怕浪费宝贵的复习时间,长到三十岁,自己都不会给自己做点吃的。

当然,中国的土地招拍挂土地财政,根本没考虑要让中产家庭拥有一个工具间,花二三十万,只能买到地下车库里画了一个方块的停车位,而非一个可以存放很多私人物品、工具的大工具间……

知乎用户 孤独理论家​ 发表

中国文化奉行木秀于林风必摧之。中国家长看到别人家孩子数学竞赛好,就一定得逼着自家孩子也去学。自家孩子学不好,就刷题给刷出成绩来。刷得累死了还不行,那就联合起来要求教育部把数学竞赛给废了。反正不能给别人家孩子超过自家孩子的机会。这种环境下,有数学天才也得给你废掉啊。

知乎用户 维禹浚川​ 发表

中国需要一大批兴趣为导向的人才,而不是功利性的人才。

知乎用户 无人机中的城堡​ 发表

制度有关,历史有关,教育方法有关。我提一个新的:信仰方面的不同也会导致对创新的态度不同。

美国总体上而言是基督教国家,相信神是创造万有的,因此我们现在所有的不算什么,可以创造的太多了。

我们是唯物主义,眼前的实物就是真实的,别谈什么形而上的,别谈还没出现的,都假的。

哪一种更容易激发创造性,我也不好评论。请自行斟酌。

知乎用户 dzqwym 发表

也真是日了狗了啊。

承认米帝的教育体系有重大问题,很难吗?

米帝的领先,以我的观察,还是发展得早。发展得早,一是有钱,二是积淀。

基础科学,还是需要积淀的,要靠时间来换取人才。

基础科学,其实也需要钱,或者说更需要钱。米帝早已解决生活问题,在米帝做个普通的教职,日子都是过得香喷喷的,在国内类似岗位教职人员的日子好过也就是最近 10 余年。很简单,看国内各个大学的车子、房子情况就知道了。

所以啊,要比,要比同龄人,比如说 30 岁的人 PK30 岁的人。

其实啊,最好还是要等到 20-30 年以后,也就是 2050 年碳中和的时间点再说,以高考状元不再优选管院、经院为转折点。

总之,不太看好米帝的发展模式,我们的发展路径更为乐观一些。

知乎用户 筷子​ 发表

因为美国能做出超级牛的东西的那波人跟现在中小学数学简单的不是一拨人。这波数学简单的人长大了不一定能做出超级牛的东西。

以苏联解体为分界线,美国基础教育是以肉眼可见速度衰落的。在以前美国的基础教育不这样。

知乎用户 姬博士​ 发表

中国最近几十年的发展总量几乎赶得上包括美国在内的几乎所有发达国家从建国以来的发展,里面离不开各个阶层科学家的艰苦奋斗以及他们的汗水甚至是鲜血带来的卓越成就。中国能实现跨越式发展无非和几个原因有关:1. 中国制度好,2. 中国教育好,3. 中国人种好。

你猜猜中国的发展和中国的教育有多大关系。

知乎用户 六爷 发表

用进废退

要么是这种风气没有持续下去

要么是没什么用, 当然也持续不下去……

知乎用户 山路哥​​ 发表

数学是一门有规律的科学。

但创造不仅仅需要计算,更多需要的是创造能力等……。

知乎用户 netkiller​ 发表

美国人学的是数学,我们学的是算数。

我们可以快速做加减乘除的算数题,但是无法解决数学问题。

知乎用户 kimi sun 发表

正好这几天正好和几个老美做 project。对此有点感触,组里三个老美,一个本科心理学的,一个本科学供应链管理的,一个本科学金融、会计双学位的。后两个老美数学功底还是相当强的,用数据库软件比我还熟,而第一个老美就明显不懂。虽然我没有具体研究过美国基础教育阶段不同档次学校、不同档次学生的数学基础。但是从我同组的组员来看,老美在基础教育阶段两极分化还是比较厉害的。(也许我这样表达欠妥,应该说 “术业有专攻”?)数学底子好的老美那绝对不比老中、老印差,数学底子不好的大概也没想过去硅谷写代码

知乎用户 就混吃等死啊 发表

都在回答基础教育和素质教育的问题,但我其实想谈的是另一方面的内容,我认为这远比基础教育和素质教育的争论来得更为重要,也是我认为中国科研不及美国的最根本的原因:钱和回报周期。

清华北大,他们的资金再多,相比于 MIT,Stanford 等世界著名高校,也是难以望其项背的。当然,进入 21 世纪以来,特别是这些年,国家和民间增大了对大学和科研机构的投资,但一是这些投资仍然不够,二是加大力度投资这也才仅仅几年而已。

这便要说到第二个问题了,回报周期。一项科研成就,从发表到落地实践生产并普及,或从发表到成为行业准则和新领域的起点,是需要相当长的一段时间的,美国那些超级牛的东西,起理论的发表和提出,可能都要追溯到三四十年前。因此,我们大概要等这么长的回报周期,到本世纪中叶才能看得出 21 世纪初期中国科研的实力究竟怎么样……

其实还有第三点:市场活性,即如何让科研变现。市场活性越大,科研变现速度和规模就会越大,这样科研工作者的积极性就会越高。在这一点上,美国市场的活力,规则的严谨,分配的合理度等等方面都是远超中国的。

知乎用户 何谐 发表

因为小学数学简单

所以成绩考的好的都是有天分的

资源分配给筛选出来少数人

成果当然好了

知乎用户 小薇 发表

回归到本源,能做出超级牛的东西的原因无非是:努力 + 坚持 + 兴趣,那么这几个要素中,兴趣要牵引着另外两个要素形成正循环,兴趣有了,自然努力学,有正反馈了自然会坚持。

好,回到问题本身,美国中小学生学的数学确实很简单,不要说中小学,答主记得刚去美国读大一,学的数学也是很简(bai)单(chi),答主记得当年因为每次考试都满分,教授叫我 Final 不用来了,省点纸多好~ (真不是装 bi)

那么为什么美国中小学的数学会这么简单?因为要勾起**兴趣。兴趣是小孩的第一生产力啊。**小孩不厌恶一件事并且发现到乐趣,这是第一步。至于为什么美国人能做出超级牛的东西?因为到了当兴趣已经不足够,必须到拼努力,坚持,甚至天赋的时候,留下来的就真的是真心热爱,并且有天赋的人,还有就是,美国大学的制度是大一大二学的是基础课,到了大三,你才需要选你真正要学的专业(就算选了专业也很容易转别的专业),所以到后来,经过一段时间的筛选,到大三选专业的时候,如果是选数学专业的人,基本就是真心喜欢数学 + 有天赋 + 努力 + 坚持的人,这样的人,能不做出超级牛的东西么?

想了解更多美国资讯请戳:

美国留学租房 WeHousing- 知乎专栏

知乎用户 月笼沙 发表

我们在个人进步,学科进步中有个最大的问题,就是一直在事实上片面鼓励:战术上勤奋,战略上懒惰。

这造成了我们长期以来,认为我们的基础教育傲视群雄,长达几十年没有认真反思我们教育得失,反而在孤芳自赏中越走越远。

我们直到 2020 年仍然用 “简单,快乐” 等浅显而武断的词汇形容发达国家的教育,虽然也有一些反对的声音,但也往往会用 “欧美的精英教育也是很苦” 等词汇为我国基础教育的拼命三郎作出支撑而已。很显然,并没有从文化基础,思维习惯,制度建设和教育核心等各个方面对我们的教育工作作出深刻的反思。

快乐教育是不是就是放羊,接触最新的知识领域是不是就不快乐,努力进取让自己所学来解决实际问题是不是就不算快乐?思考最新的数学方法解决问题是不是就不快乐? 知识的掌握是不是必须通过一个字写 100 遍才叫扎实?外语学习是不是引入了最新的语言学习理论?一年级就学三年级的课程是不是就叫雄厚的基本功?

很多问题,我们太缺乏深入的思考,并且总沉醉于我们的解题速度,背诵功底,而忽略了真正贯穿一生的方法论的探索和掌握。广大读过大学的人应该也能真实感受到,绝大多数大学实际上只是一个超级高中而已,有多少课程是实际考察你的运用能力和辩证能力的?

所以我们要认真思索的是,他们简单背后的原理是什么?大量的时间都去做了什么?是不是我们理解的简单和对方的简单是不同纬度的东西? 除了基本知识的简单,剩下的教学工作都做了什么?对于学有余力的学生,是不是加深的办法就是把高年级课程和题目出难就是美国教育的方法? 是不是有些听上去跟阴谋论一样的说法 “美国的精英阶层就是故意把题目弄简单,让绝大多数人浑浑噩噩,以便不动摇他们的根基”? 拜托,作为发达国家领头羊的地位就是靠你们这种一眼就能看穿的阴谋论建设起来的?

改开 40 来年,我们竟然还没有认真反思这个问题,大家难道不应该自责,不应该感到浪费了太多的时间吗?总想着对方国家成天就是吃吃喝喝,学习简单,考试简单,迟早有一天被我们超过。

能不能成熟一点,能不能用成年人的思维考虑问题。

知乎用户 水木山城 发表

批判性思维、创新与创造性思维与能力的培养与训练,更关键是教育中的自由、开放、多样性原则。

中国式教育,更多强调是知识的记忆与强化,典型即是标准化考试,特别强调刷题。包括英文对语法强调特多,对听力、阅读,特别是写作反而不重视。

而教育最大的目的可能不是教授知识,现成的书需要时可随时去查,背它作甚,

而是教授学习的方法、思考的方法、解决问题的方法。

知乎用户 来来往往 414​ 发表

跟跑步一样,刚开始使劲跑的人不一定就能更早到达终点,真正的跑步会合理安排自己的节奏,才能更好地更快地跑步。

教育是个复杂的工程。用 10 几年时间学好怎么培养更多科学家,军人,工程师并不是多做题就能做到。认认真真给学校修体育馆,实验室,让孩子参观博物馆,学着和别人合作,去森林里面探险,同样是青少年成长中的教育内容。绝对不是靠认认真真做 10 年题就能成为成功的教育。

知乎用户 池龙 发表

很简单,因为那些 “超级牛的东西” 都不是这些学 “超简单的数学” 的中小学生长大以后做出来的。

知乎用户 Sam Chen 发表

因为美国的教育与刻板印象恰恰相反。是比较的。

在美国上了四年高中来给大家带点私货。

文章较长,赶时间请下拉。谢谢

_(:3」∠)__(:3」∠)__(:3」∠)_

刚到美国的时候我也很粗线。美帝的教材真的是简单爆了好吗。除去咱们的英文问题。其他的简直不是问题。

英语课每天就超简单的学单词,超简单的阅读。老师说话特清楚。生怕咱们听不懂。超简单的啊

科学课就是抄笔记。我的科学老师更是屌炸天自己印了厚厚笔记给我们。然后给我讲课的时候填空。其实就照抄啊我说。超简单的啊

历史老师的知识量稍微大一点,但是木有关系啊!老师特和蔼可亲特 nice 特别细心教啊。超简单的啊

数学课?我算是个例,因为在国内上到初三数学都是偏上的。然而课堂作业太简单。咱两分钟写完了。然后睡觉。妈妈再也不用担心我的睡眠不足了。因为超简单的啊

_(:3」∠)_可是错的很离谱啊

后来我才发现我的大部分的课很简单是因为英文水平不够。我的课就分到很简单的。

你以为这只是英文的问题吗?吐翔突拿衣服!

等我还没跳出 ESL(English Second Language) 就认识到了 AP(Advance Placement)课的存在。

所谓 AP 课教的大概就是大学该科目的一二年级课程(记住哦!美帝的大学都是很难的哦)

高三的时候我们装逼党(排名靠前的)有了选择 AP 课的权利。一般一天七节课。我选了三节。hentai 的人七节 AP 都有(虽然后来换掉了)。

然后萌萌哒的我们的课和同年级的别人就相差甚远了。

在我们学微积分和前微积分的时候同年级 “好学的” 的大大们还在学着代数和几何(认真脸)

在我们读着荷兰人用英文写的古典文学的时候他们还在背单词。

在我们出去做 presentation 还拿奖回来的时候他们在学打字。

因为美国的教材(州和州可能不一样,请不要拿我和你比)是因材施教

你只能学到这里,你就只拿到简单的课。神们不止步于此。你是不会懂的。(我也曾是学神一枚,让我自恋下,请不要反驳我)

所以当人们浑浑噩噩的度过了高中四年然后得出来美国高中很简单这个结论。

是毫无依据的

结论。美国的高中教育可简单。也可以很难。但我更喜欢这样。不擅长学习的大大学到了基本的工作基础。擅长学习的神们都提前学习大学课程了。

_(:3」∠)__(:3」∠)__(:3」∠)_

讲到最后,再来科普一下所谓的 AP 课。

AP 课分为 AB 和 CD 两部分。分别为大学一年级和二年级的难度。每年的五月有 AP 考试。满分五分。

那 AP 考了五分是什么概念呢?

你被大学录取后的该课程不用上了。直接给你学分,让你学下一级别的。也就是在高中把大学的课给上了。

然后,想象下两年满 AP 课的神们(不是我)你还能说他们的课很简单嘛???

以上。

感谢阅读。

知乎用户 王华​ 发表

美国精英式教育,人家高中精英班都去学高等数学啦

知乎用户 硕鼠 发表

亲眼见一对土博士夫妻,用 “外国都是愉快教育” 的理念,把自己儿子培养进了:垃圾中学。美国高考,数学分两个级别。还有大学先修课。四年制高中学校里,基本把大学一年级的通用数学给解决了。——– 上海高考,也是被你们喊简单【喊着喊着,自己都信了】结果一拿上海户口,看着自己孩子渺茫的前途,哭天抢地,如丧考妣。

知乎用户 沉钧 发表

人力资源贵在各就其位,明白吗?

不是我们所说的什么样的人就干什么样事儿,而是说你愿意努力就继续读,读得越高接触的东西就是越尖端。

反之,你不愿意读,在中小学阶段修够学分毕业就行了,素质教育为主,不强求你学得多深,但求理解并能实际应用。 同时,他们也强调人与人之间的权利平衡,所以即便你数学学得不够好,但只要你愿意干活,人们就愿意给你付一份让你过得体面的工资。而不必人人都把数学公式记牢,人人都必须像驴一样从小就蒙眼转圈。

而在另一种模式下,比如缅甸之类的国家里,教育的实质目的是为了筛选出最努力的那些人。怎么样才能筛选出这些最努力的人呢?那就得出题考,难的题筛选不出来,就增难度继续筛选。但光努力是不够的,因为架不住别人比你更努力。所以,严格来说这都不算筛选系统,更应该是一种残酷的末位淘汰制

当然,筛选的目的还不仅是看你是否努力,还得看你是否听话,所以除了常规的文理课外,还是加很多思想文化课,以保证被筛选上去的人,符合总体意志。

这不是现代缅甸人才想出来的高招,事实上在一千多年前这套制度就确立下来了。你看当年那些读书人,为什么被骂球经不懂?因为他光顾着读书考试了,书上讲的是什么,他甚至都不会关心。

你以为把书本上的题解会了就懂了数学了吗?沈南鹏曾是奥数冠军,他以为他很喜欢研究数学,但到美国读博士才发现,原来真正搞研究的人居然这么苦,所以就转行搞金融了。搞数学也好,搞科研也好,不在你懂多少,而在你能投入多少精力和勇气去未知的领域头铁试错。

回到题目,为什么美国中小学生的数学这么简单,美国人却还能做出这么多超级牛的东西?因为他们搞尖端科学的人,统一在大学培养,而不是在中小学。

知乎用户 shadow Ma 发表

一个浅显一点儿的说法是:那些牛逼的东西都不是中小学生做的。

知乎用户 近代中国研究​ 发表

--作者:吴苾雯

![](data:image/svg+xml;utf8,)

陈景润(1933 年 5 月~ 1996 年 3 月):当代数学家。1973 年发表了 (1+2) 的详细证明《大偶数表为一个素数与不超过两个素数乘积之和》,把哥德巴赫猜想的证明推进了一大步,被国际学术界推为“陈氏定理”。1981 年 3 月当选为中国科学院学部委员(院士)。曾任国家科委数学学科组成员。1992 年任《数学学报》主编。

![](data:image/svg+xml;utf8,)

吴苾雯,高级记者,中国作家协会会员,中国报告文学学会会员。

著有《一个女记者的梦》、《滚滚红尘中的中国女人》、《逃离大学》、《哪把椅子是我的?》、《是谁杀了我?》、《向中国大学说不》、《两个人的故事》、《父亲档案——革命年代的私人记忆》等十余部作品。《离哥德巴赫猜想最近的人》、《漫漫征程》获全国青年读物优秀奖;《高三家长》被改编为 20 集电视连续剧《爸妈不容易》在全国 20 余家电视台展播。

陈景润这样一个数学怪人,与正常的社会都格格不入,与那个反常的时代就更加倍抵触。

1973 年的春天姗姗来迟,2 月底了,水面上仍结着薄薄的冰。

陈景润裹着棉大衣去医院看病,在路上遇见数学所原业务处处长罗声雄。罗声雄 50 年代末从北大数学系毕业,背着铺盖卷就直接到了中科院数学研究所,因爱仗义直言,没少得罪人,也没少挨过整,文革中被下放到湖北沙洋 “五七” 干校。

也许是担心自己不久于人世,陈景润将自己的秘密悄悄告诉了罗声雄:“我做出了‘1+2’,我想拿出来发表,又怕挨批判。”

罗声雄说:“只要你的证明是对的,就不要怕。”

可是陈景润仍不敢将论文拿出来。这些年来,挨打、跳楼、被专政,就是因为专心业务研究。他不能不心有余悸。

没过多久,中科院一位军代表来数学所视察工作。军代表姓王,是经历过南征北战的将军。罗声雄跟他谈起了陈景润,说陈景润将 “哥德巴赫的一个著名的猜想” 推进到了 “1+2”。

将军不知道哥德巴赫是谁,也不知道他的那个猜想有什么意义,又有多重要。

罗声雄告诉他,哥德巴赫是德国的一位数学家,他在 1742 年提出了 “任何一个大于 2 的偶数都是两个素数之和” 的猜想,这个猜想如果被证明了,将会极大地推动数论研究的发展。200 多年来,一代又一代数学家都在梦想证明它,它被列入 20 世纪最重要的数学问题之一。陈景润将它推进到 “1+2” 是一个了不起的进步,这个成果如果公布出去,将会在国际数学界产生巨大的影响。

将军听了很激动,问:“他的论文写出来了,为什么不拿出来发表?”

“他不敢拿出来,怕受批判。” 罗声雄说。

“他住哪里?你带我去看看他。” 将军说着快步出了门。

罗声雄将将军领到 88 号楼,他拍了拍小屋的门,过了好久,才听到陈景润细若游丝的声音:“是谁呀?”

“是我,罗声雄。”

门 “吱呀” 一声开了,见罗声雄后面跟着一位军人,陈景润惊愕地睁大了眼睛。

将军爽朗地笑了,他拍了拍陈景润的肩膀说:“小伙子,听说你做出了一个很了不起的研究成果,你别怕,大胆地拿出来。”

陈景润不置可否地连连说:“谢谢,谢谢……”。

将军走了,他关上门沉思,虽说这位老同志是院里的军代表,支持他将论文拿出来发表,但是如果以后军代表走了,有人秋后算帐怎么办?想到这里,他又不寒而栗。陈景润仍不敢将论文拿出来。

几天后,主持中科院党组工作的武衡来到数学所,他神情严肃地对所党委书记赵蔚山说:“听说你们这里有个青年做出了一个很了不起的研究,却不敢将论文拿出来发表,这很严重,为什么不敢拿出来?这么重要的研究成果应该直接向周总理汇报。”

原来,军代表回去后将陈景润的情况报告给了院党组。

![](data:image/svg+xml;utf8,)

1977 年,陈景润 (右) 和博士生张明若在中国科学院数学所图书馆。

不久,中科院召开全院党员干部大会,传达贯彻周恩来总理 “要加强理论研究” 的指示。武衡在会上说:“数学所有一位青年研究人员,做出了一项很重要的研究成果,将哥德巴赫猜想的研究大大向前推进了一步……”

虽然武衡在会上没点名,但不久,科学院上上下下都知道了武衡说的那位做出重要研究成果的青年研究人员是陈景润。

武衡的讲话在中科院引起哗然大波。有人说,怎么宣传起陈景润这样的 “白专” 典型来了,这不是否定文化大革命么?也有的人酸溜溜地说:“有什么了不起,如果我们像他一样不关心政治,我们也能做出来。”

陈景润将他的论文拿出来了。可是围绕着论文能否发表又引起了一场争议。有人很激动地说:“陈景润的论文研究的是古人洋人的东西,没有实际意义,不能发表,要发表,必须全所讨论通过。” 他们上纲上线,说:“陈景润的论文绝对不能发表出来,这是关系到走什么路线,树什么旗帜的大是大非问题。”

也有人挺身而出:“哥德巴赫猜想是世界难题,陈景润的研究结果意义重大,论文应尽快发表。” 他们气愤地说:“你们不是每天都在喊要解放全人类吗,连陈景润都不敢解放,你们还解放谁?”

![](data:image/svg+xml;utf8,)

陈氏定理是中国数学家陈景润于 1966 年发表的数论定理,1973 年公布详细证明方法。

1973 年 4 月,中国科学院主办的《中国科学》杂志,顶着压力,公开发表了陈景润的论文《大偶数表为一个素数及一个不超过两个素数的乘积之和》。

接着,中国科学院《科学工作简报》第七期发表了题为《数学基础理论研究的一项成就》一文,概括地介绍了陈景润的这项研究成果。这份简报被中央一位领导看到了,他要求中科院将陈景润的论文写一详细摘要。4 月 20 日,中科院将陈景润的 “1+2” 论文放大印制在八开纸上,一同报送毛主席、周总理。

陈景润证明了 “1+2” 的消息震惊了中国数学界,也震惊了国际数学界。在此之前,数论专家们普遍认为,要想沿用已有的方法(包括筛法)来证明 “1+2” 是不可能的。而陈景润居然对筛法“敲骨吸髓”,加以改进,创造出了加权筛法,使筛法的效力发挥得淋漓尽致。

![](data:image/svg+xml;utf8,)

陈景润 1966 年发表的陈氏定理(也称 “1+2”),成为“哥德巴赫猜想” 研究上的里程碑;研究成果至今保持世界领先地位。

有外国数学家写信给他:“你移动了群山!”

有数论专家惊叹:“陈氏定理是筛法的光辉顶点!”

消息传到英国,英国著名数学家哈伯斯特听了为之一震。哈伯斯特与李希特合作撰写的《筛法》一书正在付印。他马上托人从香港找到了陈景润论文的复印件,给《筛法》一书又增加了新的一章--《陈氏定理》。他在这一章的首页写道:“我们本章的目的是为了证明陈景润下面的惊人定理,我们是在前十章已经付印时才注意到这一结果的;从筛法的任何方面来说,它都是光辉的顶点。”

科学院召开党员干部大会那天,新华社女记者顾迈南刚好也在场,听武衡讲有一位青年研究人员取得一项世界水平的科研成果,她的心为之一动。几天后,她与摄影记者钟巨治一起到中科院,准备采访陈景润。

听说是来采访陈景润,有人说:“他可是个怪人,除了搞数学,什么也不知道,什么也不关心,而且是有名的‘白专典型’,虽然他在科研上成就很突出,将哥德巴赫猜想推进到‘1+2’的水平,但是他这个人政治上不可靠,是个有争议的人物,武主任在报告中也只是不提名地提到了他。”

“既然是‘白专典型’,陈景润有没有反党反社会主义的言论?” 顾迈南问。

“好像没有什么反动言行,只是不太关心政治。” 并举出一事说明。那是中美建交后,有一次他所在的五学科研究室召开讨论会,因为规定人人都得发言,陈景润没啥好讲的,就批判美帝国主义,说美帝国主义狼子野心不死云云。一位同事悄悄拉了拉他的衣角,凑在他耳边说:“中国跟美国建交了,毛主席还接见了美国总统尼克松。” 陈景润电击般地呆住了,嘴里喃喃着:“真有这样的事……。”

顾迈南与钟巨治又来到数学所。他们找到业务处处长罗声雄。罗声雄详细介绍了 “1+2” 这项成果的重大意义,同时也向他们证实,陈景润不关心政治,不参加任何活动,但是并没有反动言行。最后,他还向两位记者介绍了陈景润的身体情况,说他病得很重,并说中关村医院的医生曾几次告诉数学所,不要让陈景润死在屋里没人知道。

两位记者采访后讨论起了陈景润是不是 “白专” 的问题,他们认为,陈景润是 “专” 的典型,毫无疑义,他的一系列研究成果就是证明;至于是不是 “白” 的典型,值得商榷,因为陈景润做出了领先世界水平的研究成果,为国家争了光。他们决定将了解到的情况写成内参,据实向党和国家汇报。

当天晚上,他们分别赶写出两篇内参,一篇是《青年数学家陈景润取得一项具有世界领先水平的科研成果》,一篇是《关于陈景润的一些情况》,在这篇 “情况” 里,他们反映了陈景润的处境和身体情况,说他病情危险,急需抢救。文中并引用了一段被采访者的话:“××× 说,如何对待陈景润这样的知识分子,如何对待陈景润这类理论工作,请中央表个态。”

陈景润的故事,在文革之后,被描绘成多种形式的连环画,广为传播。

这两篇内参受到了中央的高度重视。毛泽东主席看了后画了一个圈,并批示:要抢救。请文元同志办。凌晨 2 点,几辆小轿车悄悄停在中科院 88 号楼前,从车上急匆匆走下一群人,走在前面的,一位是武衡,另一位是数学所负责人赵蔚山。他们径直走上三楼,叩响了小屋的门。

陈景润仍在灯下工作,听见叩门声,心里一阵紧张,自从论文发表后,他就一直提心吊胆着。他将数学书和稿纸飞快地藏起来,然后才开门。见门口站着许多人,陈景润神情紧张地连连说:“我…… 我没干什么,我在听英语广播,听新闻……”

武衡走上来,微笑着说:“你别怕,我们是来带你去检查身体的。”“带我去检查身体?” 陈景润吃惊地张大了嘴。他怀疑自己听错了,这么多年来,他一直受歧视、受排挤、受打击,很少有人真正关心过他。他警惕地打量着这些半夜来客,心里充满了不安和疑虑。

吴苾雯的报告文学作品《离歌德巴赫猜想最近的人》,从陈景润贫寒的少年时代开始,写到他两进厦大、流放东北、攻克哥德巴赫猜想、跳楼自杀、远渡重洋、迟来的爱情等人生历程,是一部完整翔实的陈景润传记。

转自《我的历史》

知乎用户 明通端悫 发表

西方走的是 1% 精英加 99% 平民路线

99% 的人只需要初中数学足够应付成年的 99.99% 的生活工作用了 大部分成年人的经验其实是记忆和归纳总结。

中国甚至东亚都是拼命死学的路线

精英比例还是 1% 但是 80% 的人死学 记忆 应付初高中 大学 还可以 。整体科学素质高过黑墨绿白混杂的欧美,但

想发现新理论, 不靠专注后的顿悟和洞察力, 只靠记忆和试错的模式, 几乎不可能。

精英的 1% 又受制于老师的学术水平低 有不少老师才能不足以做教授老师的 他们是那 80% 靠勤奋和记忆拼得老师资格 精神可嘉 学术卵用 这让聪明学生空耗多年精力智力在小课题 或者大课题的外围和迷途,等明白难易了 自己老了。

这段如果有人不服 参考大学靠吸引亚非拉垃圾学生提高大学排名 足够反映中国大学的老师学术水平之低(有些老师什么都知道 可没有创新没有洞察力 一辈子都是跟屁虫或者开源的翻译者)

如何判断哪个老师水平高低呢?法国的群论发明人伽罗瓦 19 岁时早指明了

为什么国人总是怀疑本国科技人员,真的是科研能力这么差吗?

张益唐其实就是受制于没有好导师,要不他会有成就得更早。

详细说明参考

如何理解 (高中阶段) 数学的本质?

知乎用户 kimi​ 发表

并不简单啊

让你觉得简单的,被各种网络营销的只有算数这一个东西

其实主要是乘法口诀表 + 汉字的数字都是单音节

知乎用户 老书​ 发表

我们普及小学初中九年教育才二十年,美国已经接近一百年了。我们大学扩招到同龄人口的百分之十,才二十年,而美国百分之三四十已经七八十年了。一个国家做出什么样的东西不是一俩个天才能解决的,而是要靠整个社会体系不断提升,分工合作的。教育是这种分工合作的基础,我们才刚刚开始,再过俩代人,等到我们的就业人口素质整体和美国差不多的时候,你再看!

知乎用户 Null 发表

因为美国没有早读,晚自习,卫生检查,跑操这些能够增加集体凝聚力的活动,导致了个人的自由散漫,不懂规矩,虽然极个别能出做好的成果,但都是单干户,远远没有衡水中学批量培养出的科学人才有用。

知乎用户 Mark 带你进外企​ 发表

我说一个亲身经历的现象。

我在美国玉米地(UIUC)读本科的时候,很多工科和数学的课程往往在第一个阶段考试的时候中国学生大部分能拿高分,在第二阶段考试时中国学生拿高分的就少了,最后 final 的时候再少几个中国学生拿 A,美国学生拿 A 的比例越往后越高。

跟他们接触后我发现优秀的美国学生有比较强的自学和适应能力,比较善于搜索和利用资源,虽然有文化因素和地利原因在其中。

当然拿 D 和拿 F 的依然是那些美国混子;

韩国人依然沙壁,因为他们大部分都是上课睡觉,交作业前发邮件求援的,拿 B 和拿 C 的不少。感谢韩国欧巴和韩国姐们成为了分母。

还有一个重要的因素就是美国和中国大学课程设置和排课的问题:

1. 美国大学一学期一般 4-6 门课,大部分学生修 5 门刚好,6 门属于 full load,超过 6 门属 overload 要向学院提出申请才能批准上超过 6 门的课程。我国的大学一学期一般 10 + 门课,课程量是两倍有多,不知道这么多年后有没有变化。

2. 在排课方面美国大学一般 40 分钟一节,每周 3 节;或者 60 分钟一节,每周 2 节;晚上很少排课。我在川大读书的时候一般 90 分钟一节课,每周两节;或者 3 小时一节,每周一节;晚上 7 点 - 10 点也是排课的。这两者的区别就是学生到底能专注地听多久,正常来讲人是很难一直保持高度专注的,尤其大学讲课又没有互动,那些一周一次 3 小时的课,基本上 80% 以上的时间都在走神。

最麻烦的时这些课不听不去上课的人也能通过期末突击或者老师考前划重点考到高分,反过来就更加刺激学生不听的欲望,前有课程设置和考核,后有考研,这些一起都让学生未能在大学里摆脱做题机器的束缚。

只能说美国精英的那群人很精英,混子的那群人很混子。

知乎用户 文才​ 发表

楼主去看看美帝的私校吧,他们努力程度也是拼命的,至于怎么衔接,大学 AP 课程啊……

知乎用户 荤素搭配 发表

按照马克思理论,人在被资本家剥削和压迫下,劳动会出现异化。类似于给你一个敲 10 分钟图章的工作,我相信大多数人在 5 分钟内就会走样。除非你热爱敲图章。

依照尼采的说法,一切贱民对世界都是负面影响,他们为了生存,会不停的要求这,要求那,要平权,要平等,要福利,摆工,要斗争,要政变等等等等。而对为社会发展,基本没有正面作用。比如缺少平民科学家和平民哲学家。

结合起来,金字塔一定是一群狂热热爱这份工作的 “人” 建造起来的,所以他极精确坚固。让奴隶造是根本造不出来的,会异化走样。

中美的差异,在于社会阶段的不同。我们是初级阶段。人家富了几百年了,很多人甚至在还没有美国起就一直富有。

  • 所以,我们以打砸抢起家的国家,正在表演弯道超车,大家正在一切向钱看。做事首先想的是 “有没有钱赚”。

文学家不为理想就不会写 “飘”,而是写地摊文学,铁血军事,黄色小说,因为来钱快。

技术行业为了钱,就不会造就 google 这样的企业,而是山寨芯片,医疗保险,莆田系。来钱快。

哲学家从政,不保持独立性,不为人性代言,转为 “专家” 等等职业。

这些人热爱自己的职业吗?除了变态外,一定不。他们只是被新的符号给统治和压迫了。这个符号叫 “钱”,过去可能叫“孔孟”“皇权”“十字架” 等等等等,毫无意义的符号,正和钱一样的无意义。

当你不会为钱发愁了,你才能做自己热爱的事,有自己热爱的事业,不然你只是一个打工的,为别人热爱或不热爱的事忙碌,做着一切卑鄙我搓的事情让自己向上爬。在头领面前卑躬屈膝。对弱小的人狼性。这是一个零和的游戏。

  • 那我们为什么要有狼性而放弃创造性呢?因为我们要让下一代做一个真正的人,做喜欢的事。让他们不像我们一样做一匹恶狼,让他们画画,做诗,思考,发明。这些需要物质基础。这也是多数贪官的想法吧。

在我有限的智商下,我只能找出这样的解释来。除非某变态跟我说,他就是热爱着初级阶段下的零和博弈。或者某位哲学,艺术,科学大卡对我言传身教,坚决表明自己就是热爱自己的事业。我只是个普通人。

所以,初级阶段必定是没有哲学和科学思想的。知礼仪先要仓什么足。

居然还有人学艺术,算了,艺术家可以给淘宝店做装修嘛,

居然还有人搞科研?算了,这种人材山寨个芯片骗点经费吧。

什么居然有人学哲学,我 tmd 要你何用啊。。。我想想,你去当官吧,一套套的当官不错。

这个就是现实,穷吊的现实。且穷吊在劳动中一定会异化,也就是说,你山寨个芯片,也山寨不好,除非你热爱山寨芯片,这基本不太可能,不能实现自我价值,就一定消极怠工。这是马列主义的精髓。

其实国家是马列起家的,国家一切都清楚,所以反复写到初级阶段这个字眼。但这不是写给普通人看的,要看明白,一定要有理论知识。

知乎用户 少年强则爹强 发表

普通人有生存空间,天才有上升空间。

普通男人不用好好学,只要愿意工作,哪怕去麦当劳沃尔玛做钟点工,也能够娶一个菲律宾美女,老婆全职在家生娃养娃。

大部分学生的学习很拉垮,因为不需要那么卷,将来做快递,盖房子,卖房子,开 uber 之类的,本来就不要什么技术含量。

少数天才,即便父母从来不管不顾,哪怕最边远的中小学,他们也能接触到临近大学生的帮扶带团体。哪怕临近高中才开始发奋努力,也还是有上升渠道的。

知乎用户 dbodb​ 发表

因为学习这些简单数学的人和做出来很牛的东西的人不是一批人。

以下不是我的亲身经历,是之前听我的几位老师说的。

老师在美国读的 PhD,了解到美国知名高校的理工学生的数理基础要比我们好一些。举个例子,我们的工科学生在本科时期对物理的要求仅限于大学物理,理论物理基础,学海洋的还要学一些流体力学,而 mit 的类似的本科生掌握的还有看似几乎无用的大量的量子力学的知识等等等等。

这位老师在和我们交流的时候表示,我们学生的数理基础还是偏弱,所以在建模和算法开发上(这位老师是做海洋信息的)都做得不足。

我们是平民教育,而美国是精英教育。他们对这部分精英的教育的要求极高,我看过 MIT 的类似专业的培养方案,我自认为学习的压力会很大,甚至可能出现挂科之类。

这大概就是部分原因吧。

知乎用户 安静的人 发表

美国做出超级牛的东西的人上的初中和高中比我们还卷

PS:当然哪怕是私立小学比我们也是舒服的

简单来说就是欧美小学比我们轻松太多,初中比我们要舒服很多,但是好的初中已经和我们差不多了,高中要上好大学的和我们这边要上好大学的人的卷度差不多了

上大学之后我们这边是放羊,美国的大学年年赛高考,考砸了直接送走

知乎用户 阳玉 发表

米国是分层次教育,穷人的孩子上烂公立学校,小学毕业会数手指头,高中毕业会四则运算。只要会说英语,会开车,会用计算器,不影响年入 4 万美元。

5% 的精英学霸,上高级私立学校,比华人更吃苦,睡得更少,作业做到 0 点,5 点起床,还要练两个小时体育。科学是少数人推动的事业,其他人 1 美元炸鸡可乐管饱。

知乎用户 燃冰飞雪之魂 发表

美国的成就,肯定不是只学了初中水平数学的美国人做出来的,

而是具备自学能力十分肯学的书呆子和学力超强的留学的外国移民实现的。

知乎用户 Jabberwocky​ 发表

先问是不是,再问为什么

随便到网上找几个数学比赛的获奖队伍,你就会发现华裔和俄裔的比率高的惊人。

美国的教育体系并不比中国更能培养数学人才,美国的优势在于可以把世界上别的国家的人才用高薪吸引过来。二战后美国吸收了欧洲许多优秀的科学家,这帮助美国塑造了科技霸权。

这问题下所有直接无脑吹美国教育体系,教材的人都没有关注统计学意义上的教育体系和人才产出之间的因果关系,事实上美国的教材确实更能让人入门,但是这也是相对于有数理天赋的人来说,事实上,如果你能独立阅读教科书而不依赖视频讲解,你已经是属于我说的有数理天赋的人了,对于这些人来说,国内的教科书学习曲线更为陡峭,但并不会影响结果。

如果你要说明美国的体系简单,但是更能培养数理人才,那你首先要拿出美国培养出来的数理人才的比率证据,事实却是,大部分在美国做出数理贡献的人并不是美国基础教育体系培养的,只是在美国的科研体系里拿工资而已。

知乎用户 下辈子再洗澡 发表

这么多答案,怎么没一个提从二战起,扎根吸了全世界多少人才?

知乎用户 黑炭 发表

我是数学系的,全系只有几个学生,白人只有 1 个,但是这人很牛。比我们牛很多,他可能会成为做出超级牛的东西的人。剩下一帮中国人,说牛也还行,但也没那么牛,不顶尖,不是执着的喜欢数学,而是拿文凭来的。

平时很多白人点菜连账单都算不清,大多数人数学也不好,但他们不是做出很牛东西的人。那样的人有几个就够了。

数学这东西是需要天赋的。有这种天赋的人需要被养起来,让他不要因为生计去干别的。我们培养的是工程师那一类的人。就是能把现有的知识给做出来。

知乎用户 TTTTT 发表

因为美国有我们的留学生!

知乎用户 流星不灭 发表

中国人存在几大无知:对哲学的无知和误解,对科学的无知和误解,对心理学的无知和误解,对法律的无知。

所以才要科普,普法,普及哲学和社会科学知识。

一个普通的中国人,在他受教育的阶段里,他能学到什么?

首先,无法学到哲学知识。学到的不过是政治理论,还是被灌输的。

因此许多人不懂哲学,认为哲学没有用。所以他们没有树立起正确的世界观,对世界、社会、人生无法有一个相对深刻的理解。

其次,无法学到逻辑学知识。许多人的思维、说话、写作都容易犯逻辑错误,不严谨,不要靠太多,完全凭感觉。批评别人的时候也就随意随心,不需要逻辑和证据,只需要一味地骂人家笨蛋,白吃,这样来显示自己的高明。思维走进误区也不知道,诡辩了也不知道,反逻辑错误了也不知道,还往往自以为是,自以为正确!

无法学到对社会的知识。对社会的了解全都是片面的,要么就是谎言。被谎言充斥着大脑。

无法学到对人性和人心的知识。不了解人类本身。但又十分好奇,因为自身有烦恼。觉得人心很复杂,人心隔肚皮。对人类的行为和心理觉得不可思议,无法理解。只靠依靠自己的想象或思维,但又不系统。

该学的都没学到,对人生有益的东西都没学到。

只是学到谎言、被灌输知识,想象力和创造力被扼杀。个性得不到张扬,特长得不到发挥。全是一个模子倒出来。

只是学到了偏颇,学到了等级观念,学到了争强好胜,你死我活。学到了要当老大,要掌握权力,要让自己最牛鼻。

学到民主吗?不!学到开明的心态吗?不!

一切都是相反!!黑白颠倒。

随便扯了,先从教育扯起吧。

主要是谈当代中国,是改革开放后的中国社会,太久远的历史就不要扯了,不现实。

看看那些 70,80,90 后的人,也就是上学时代是在改革开放时期的人们,他们到底在学校里学到了什么??

一个人的成长历程中,会接受家庭教育、学校教育、社会教育。而最最关键的是家庭教育和学校教育,因为这段时间是一个人的人格与思想等完善的时期,一旦成年之后,人就相对定型了,就难改了。

最容易出问题的就是在上学的这个时期,最容易进行洗脑的也是这个时期。

家庭教育就先不谈了,毕竟每个人的家庭情况不一样,虽然家庭教育很重要(甚至是决定性的),但我们只能谈一下学校教育。毕竟在同一国家的同一种教育体制下,中国各地的情况大同小异。

作为一个没有成年、还没有出社会的孩子,他的生活中所接收到的信息有哪些?信息来源是什么?这些信息就构成了他对这个世界、社会、他人的认识,影响他形成世界观。

信息来源无非是:家庭、学校的老师、同学和书本、大众传播媒体、为数不多的一点社会经验。

那这些信息源所传达的信息都有什么特点呢?是真实的吗?全面深刻吗?

大家好好想一想吧!你们在学校里都学到什么?书本上都说些什么?老师教给你的是什么?父母教育你的又是什么?你从报纸、电视、杂志小说里看到的又是什么?

说实话,说真话,反思一下吧!

继续瞎扯。

中国人非黑即白、非此即彼的思维方式特点是怎么被培养出来的呢?

而且很多人还认为任何事情都有一个 “标准答案”,一个问题又一个唯一的正确答案,而且这个正确答案还掌握在自己的手里。

看看我们上学的时候,老师总是这么叫我们的,考试也不过就是为了找到那个标准答案,不允许离经叛道,不允许想象力和创造力。从数学到政治,都要求背出标准答案!

背书背书在背书,背单词。一切都为了灌输,为了考高分,“学好数理化,走遍天下都不怕”,成绩就是一切,决定你的未来,决定你的地位,决定你在老师心中受不受宠,决定你是一个乖孩子还是坏孩子。

要权力、要先进。要当班干部、少先队员、三好学生、奖学金等等。这样才能显示你是一个精英分子,等级的观念在许多人幼小的心灵中就埋藏了。“我比你强,我比你牛,我比你正确!我是高手,我是优等生!”     

要当官,当官好!官本位思想也埋藏在潜意识里了。

看看书本上和电视里的东西,英雄和好人的形象总是高大全的,完美无缺的、不食人间烟火的圣人!而坏人、反动分子总是猥琐的、肮脏的。好人做什么事、说什么话都是对的、正确的!坏人做什么事、说什么话一概是错的。

因此,经常有人看一部电影或电视剧,由于事先未了解剧情,面对一个角色时他们总会问:“这个人是好人还是坏人?”

这就是单纯的二分法,非此即彼非黑即白。人一概进行脸谱化,要么是好人,好么是坏人。

好人完美无缺,坏人是恶不赦。但是,什么是好人?什么是坏人?这个标准没有人搞得清楚,因此就只好听编剧的、听组织的、听自己肤浅感觉的了。。。

面上的话总是冠冕堂皇,私底下的事情总是赤裸裸。

很多人从小时候开始就耳濡目染,或者听大人的话、观察外部世界。他们得知这个世界是分成两部分的:一个是表面上的,一个是被掩盖的真实的。

很多人还学会了城府,“逢人只说三分话,不可全抛一片心。”“见人说人话,见鬼说鬼话”“害人之心不可有,防人之心不可无”。。。所有这个格言与俗语都伸进了中国人的灵魂。

中国人知道,在社会上,“做人处事”是非常重要的,也是一门技巧和艺术。必须 “成熟” 起来!这个 “成熟” 指的就是世故的标准。

会不会做人??这才是最关键的东西。其他的品质一概先撇到一边。

原则?原则这个东西是没有的。见风使舵、有奶便是娘,才是正道。

但是原则或者说高尚的道德这种东西,往往中国人有很喜欢用来评判别人、苛求别人,仿佛自己是圣人,别人都是缺点,全都不可原谅。

中国经历了几千年的封建专制统治,很多东西已经渗入中国人的骨髓和集体潜意识了。文化的力量也是巨大的。

中国是二元制国家,由于户口政策,城市与乡村二元对立。因此要分开来分析。

中国幅员辽阔,有很强的地域文化特性,各地方的人性格不同、处世方式不同,心理特征不同,打上了很强的地域文化了烙印。

人治,而不是法治。尽管法治进程在不断推进,但从根本上还是人治。江湖中国,关系社会。面子问题是很重要的!

我们不是一个契约的社会,没有诚信可言。我们是一个关系社会、江湖的体系。要按照江湖规则办事!

很多事情是很微妙的,要细心领悟,因此才会出现 “老江湖”“老油条” 和初涉社会的 “愣头青” 的区别。

混不混得开,这是一个问题。马克思说人的本质是所有社会关系的总和,这句话放在中国,的确很合适!!

个人主义和集体主义

西方的价值观是所谓的个人主义,往往被我们屈解为自私自利、道德低下。

而我们所谓的集体主义则是占据道德的高地。

是不是这样的呢?

中国轻个人、重集体的特点,说得好听点就是 “泛集体主义”,说得不好听点就是 “极端集体主义”。

实际上,个人利益与社会公众利益要找到一个平衡,可偏偏我们片面的强调了 “集体” 的利益,“人民”的利益,个人的一切都无足轻重,都可以牺牲!

但实际上,谁代表 “人民”,难道每一个普普通通的公民的利益不算人民的利益吗?不值得尊重吗?

人民这个字眼只是一个旗号,说到底所谓集体的利益就是掌权者的利益。集体利益高于个人利益的真实含义是:牺牲和无条件奉献每一个个人的利益来成就既得利益集团(掌权者)的利益。

一旦玩文字游戏,许多人就搞不清楚了,迷糊了。

中国人其实有时候非常无情,为了所谓的 “道德” 而不顾个人的权益,这一点在天涯上有很多反映,比如人肉搜索等。已经有网友指出这是犯法的了,就算不犯法,也是不道德的!

其实中国人不要怪这怪那,也不要怪体制不好!

要怪,就怪自己的灵魂!!

制度是被选择的,什么样的文化和国民就适应什么样的制度。中国人骨头里就这个鸟样!

不要在发什么牢骚或者高喊什么民主口号了。

你们懂什么是真正的民主吗?

中国人的灵魂深处就存在问题的!!!一切都是咎由自取。

中国人务虚不务实。

东西方哲学有何不同?简单点说

西方人喜欢 “钻牛角尖”,喜欢较真,演绎和分析的思维。

中国人喜欢 “难得糊涂”,中庸之道,整体思维。还喜欢摆出一副大智若愚的样子,显示自己很牛很智慧!

愚倒是真的够愚了,但是有没有 “大智” 我就不清楚了,哈哈!

中国很多人除了发牢骚之外什么都不会做。

中国改革开放三十年,取得很大成就,但问题还很多,现在社会处在转型期,面临的挑战太多了。

要解决实际问题是一项复杂的系统工程,有难度,要试验。要拿出切实的解决方案来,而不是抱怨和发牢骚。

中国人有 “好黄帝情结” 和“清官情结”,太天真了!

以为只要有好的荒蒂和官员就能一劳永逸的解决所有问题,就能够建立起完美的人间天堂和幸福生活!可笑阿,太幼稚了。

解决问题和创造美好生活是靠全体中国人的努力的,体制和文化还是最重要的。

而不是一两个长官能够决定的。

看看你们都干了什么?只会发牢骚。

在公共汽车上你会给老人和孕妇让座吗?

连这点事情都作不了,还谈救国救民

中国的问题,说上三年都说不完。

中国人不懂哲学和逻辑学,这是非常严重的!

另外许多人都喜欢打倒别人以显示自己的高明,好为人师。

心态不正!

说话和思考很多人都喜欢片面和偏颇,思考不够周密和全面,流于表面化,看不到深层本质。

回复批评楼主,很多网友也不过是喜欢乱骂别人傻比,满足一种快感。但是却拿不出事实证据和理由,没有严谨的逻辑,完全是无理取闹。

这种种毛病不是偶然!是缺乏逻辑学和哲学教育。

中国人————迷信权威,没有独立思考能力,没有想象力和创造力。

知识多的那些人,也不过是个知识容器和现代化生产流水线工人。

中国缺乏真正的思想家和哲学家!!

以德治国,还是以法治国?

中国以德治国、人治的传统,其思想根源在于人性的性善论!“人之初,性本善”。

而西方则不同,西方坚持 “性恶论”。于是形成契约社会,从制度和结构入手,制造了许多制约和监督,让权力无法滥用。形成了法律社会,以法治国。法制化传统。

中国呢?因为隐含假定是人性是善的,再加上金字塔式的权力结构,我们中国人唯有依靠一个十全十美的 “有德者” 来治理天下。这就是中国人的 “好黄帝情结” 和“清官情结”,希望有一个出色的、贤明的统治者能够带领整个国家走向辉煌,为此,我们把所有的权力都集中与他的身上!这个领袖必须是有高度的智慧、圣人般的德行、全心全意为人民服务的宗旨,就好像远古的尧舜,大禹那样。

而西方却不同,因为隐含假定人性是恶的,是贪婪的,一旦掌握了权力就会腐化堕落,因此在制度的设计上就对权力进行了重重制约。

也就是说,就算你想贪、想腐败,都不给你机会,从根本上就限制你。

西方的基督教文化强调原罪,人生来而有罪,人活着就是为了赎罪。是罪感文化!

中国的伦理文化强调的是 “羞耻”,是耻感文化。不守妇道是羞耻的,非处是羞耻的,通奸是羞耻的,不孝是羞耻的,不道德的。。。

性善论还是性恶论,这就是区别!

哲学上的差异造成了文化与社会景观的迥然不同。这就是哲学的力量!

哲学决定一切,哲学是世界之王!!

数学和逻辑有联系,但两者是不同的,千万不要混淆。

逻辑学是需要学习的,这是一门基础学科。

逻辑学的教材和一些普及性的书籍在书店、图书馆都会有的,你找找看。

普通逻辑学(形式逻辑)并不难,一般人都能学懂的。

比如我随便说一些术语:

概念的内涵和外延及其关系,命题逻辑,词项逻辑,矛盾关系,上反对关系,下反对关系,合取,析取,蕴涵,重言式,二难推理,悖论,联言推理,假言推理,选言推理,同一律,排中律,矛盾律,充足理由率。。。。等等

中国人其实是缺乏一种 “逻辑” 的意识,在说话和思考的时候,很少去考虑 “严谨性” 这个问题。总是很随心、很随意,信口雌黄,胡说八道。还自以为是。

你们要是有兴趣,还可以了解一下分析哲学,哲学的语言学转向。

推荐你读一下维特根斯坦和波普尔的著作。

中国人不是不聪明,而是心态不正,心术不正!!

还有一个很重要的问题,就是中国缺乏科学哲学。

中国人缺乏对科学哲学的了解和重视。

知乎用户 彼尔苍乎​ 发表

因为美国人更有钱

正儿八经的中国留学生到了欧美任何一个国家基本上都可以傲视任何一个专业

中国差的不是人,不是教育,而是时间和金钱

对于一个国家而言,教育长线投资是真正的十年树木,百年树人

你从 4 岁起开始接受的教育,大约会在 40 岁以后开始逐渐发挥作用。而中国真正开始注重现代化教育大约是在改革开放之后,也就是从大约 80 后那一批人开始。最早的 80 后,大概马上也就 40 岁了

随着,一批又一批接受现代化教育的中国人逐渐成长起来,随着中国的积累一步又一步的深厚,我们可以相信在不久的将来

知乎用户 lili 发表

美国中小学教育。

下限很低,但上限也不封顶。

知乎用户 yu liu 发表

你数学是有多差

知乎用户 极客晨星 发表

美国中小学的数学课并非想象中那样简单,而能做出超级牛的东西本身就说明很牛,如果有人觉得你以前不牛,那只是牛的不明显罢了!

作为所在的编程领域,计算机是在美国研制,大部分编程语言也是在美国开发出来…… 数学相关的超级牛的东西很多,作为基础科学的 “数学” 难道只是 “简单” 两个字可以形容的吗?如果你这样认为,那只是自欺欺人!

![](data:image/svg+xml;utf8,)

美国中小学数学教育

美国小学、初中一般只进行分层教育,数学课程没有太大的选择性。

而美国高中课程计划充分体现多样化、自主化的特色,不仅不同的州之间课程计划不同,即便是不同学校,课程计划也不相同。

每所高中都会提供丰富的课程计划供不同学生选择。各个学校会有很大的差异,但由于受高考、AP、双学分的影响,高中数学课程大都有共性的内容。这里以格林欧克高级中学(Glenoak High School)数学课程计划为例说明。

美国中小学数学教育主要有四方面的内容,值得我们学习。

问题解决
问题解决的含义,在不断地演变着。
杜威、波利亚等哲学家、数学家早已提出问题解决的思想,但作为数学教育核心目标是 1981 年美国数学教师联合会提出的。以后经过不断地讨论、实践,“问题解决” 的含义愈来愈完善,特别是 1989 年《学校数学课程标准与评估》把 “问题解决” 作为中小学数学教育的核心标准之一,并对其内容作了系统地说明。
认为:问题解决是一个过程,这个过程使学生体验在他们身边世界的数学力量和有用性。问题解决也是一种探究和应用的方法, 为学习和使用数学提供一种 consistentcontext。问题情境能建立一个 “需要知道” 和培养一种概念发展的动机。
“问题解决” 作为一种数学教育思想,渗透到美国数学教学的整个过程,而不仅仅是显性化地解决一些实际问题,而且也适度、适时地把数学与生活自然地联系在一起。
建构主义
建构主义无论是作为哲学还是心理学,对于数学教的影响都是很大的,加上数学认知心理研究的进展,美国数学教学在 20 世纪 90 年代产生了新的革命。
1989 年全美教师联合会《学校数学课程标准和评价》,不仅是内容变化上引起极大关注,而且对学习的认识,吸收了新的学习理论的研究成果。在 20 世纪 90 年代,强调学生的过程性活动,或许是许多教材编写追求的目。
影响比较大的《整合数学》就比较典型,将数学融合在一起,强调数学的生成过程,强调学生经历整个数学发现的过程,由学生发现数学、建构数学知识。课堂上不再是简单的学生显性活动,而是关注学生思维的真正发展。
批判性思维
批判性思维(Critical Thinking)在 20 世纪 90 年代以后逐步受到美国教育的重视,每个学科都非常重视批判性思维能力的培养。数学教学也毫不例外地考虑如何培养批判性思维能力
表现在以下几个方面:
首先是教学目标中使用的目标动词与批判性思维接轨,以便在进行数学知识与技能教学时,同时达到批判性思维的培养目标;
第二是设置一些批判性思维的训练栏目,以专门训练某种批判性思维的技能;
第三是在习题中选择一些问题,专门训练批判性思维解决问题的能力。
但所有这一切,与社会研究、科学、英语课程比起来,数学教学过程中关于批判性思维的培养,远远未能发挥起应有的作用。其它学科培养的批判性思维的基本技能使用的思维多是数学思维的方法,而数学教学也许本身已经习惯了,并没有将其单独析出。
加强基础
2001 年,美国数学教学重新重视基础,各种测试促使他们重视基础。
可以看到,那些基础训练比起中国学生来说,并不逊色。
然而,什么是基础?怎样打牢学生的基础?
美国对基础的认识,尽管多数教师会为一些成绩落后的学生补课,促进他们达到要求。对于数学来说,基础就是对数学本质的理解,数学思想方法的运用,而不仅仅停留在数学知识与技能的表面.这一点,在今天的美国教学中充分地体现着。
几点思考
从美国中小学数学教育的现状可以得到许多启发。
当我国大力宣传问题解决时,他们的教学却在进行学生活动的尝试;当我国极力推崇建构主义时,他们却在 “反璞归真”,追求数学基础;当我国极力寻找与数学与生活联系的情境时,他们却很随意地删去了教材每章、节前最精彩的 “情境”,而直接将数学端给学生.
今天,有些美国数学教师仍然使用 20 世纪 90 年代的数学教科书教学,因为他们更喜欢数学与实际的联系,认为数学源于生活,更易于激发学生的学习兴趣;有的教师反对将代数和几何分开,因为整合数学,几何中有代数,代数中有几何;有的教师为了让那些落后的学生获得好成绩,他们更倾向于直接教给学生数学基本技能,然后进行大量的练习,因为当与现实问题联系过多时,会增加学习数学的难度,学生还没有理解基本的数学概念、掌握基本的数学技能,问题解决只是幻想。
实际上,简单评价谁更合理,似乎没有准确的答案.因为数学教育受制于政治、社会、文化等太多因素。但这并不意味着没有统一的评价标准.这些评价标准关键是对于数学教育中核心概念的上位认识。

从学习方式上看,重视动手实践,重视自主探究,重视合作学习;

从学习内容上看,重视基础知识与基本技能的学习,重视基本方法的学习,重视问题解决.

那么,我们的中小学生学的数学能达到什么水平呢?

奥数!升学!考试!

这可能就是现在为什么推崇素质教育的原因,这也是我从事少儿编程教育的希望!

知乎用户 gyuyuytvugj 发表

哪有那么多乱七八糟的道理…… 其实道理非常简单…… 就是美国二战后从欧洲收割了一大批科学家,人家牛人教牛人自然就是良性循环啦。

事实上你初中高中那点基础数学对于真正搞数学研究的人来说都太简单了,你考 60 分和 150 分对于真正数学研究者来说就跟百分制的 6 分和 15 分没啥区别。真正关键的区别在于你研究生博士期间能不能跟上一流大牛,接触到一流的问题。

而美国收割了这一批欧洲科学家,再以世界最强的财力给予他们最好的科研条件,不出成果才是奇怪了。

你去 math genealogy(Mathematics Genealogy Project) 上随便找个牛人搜搜看,往上几代就是个欧洲的名字命名定理的大牛,这些人的徒子徒孙都在美国开枝散叶呢。

随便挑一个有名点的比如特仑苏陶,他的谱系是 Terence Tao-Elias Stein-Antoni Zygmund-Aleksander Michal Rajchman-Hugo Dyonizy Steinhaus-David Hilbert。你体会下…… 数数看里面有多少教科书里的名字。

中国就算满打满算从民国算起到现在自然科学也就发展了百来年,怎么跟别人几百年的底蕴来比?

知乎用户 瓜子可乐甜筒 发表

因为影响当今世界发展的中坚力量不是现在学校里小学生而是中年人,这批中年人他们小学的时候大概也就六七十年代的时候,当国内还在扫盲想方设法恢复高考,美国那边的小孩子已经在电视里看到阿波罗登月了,你要想知道现在数学简单会造成的影响,那得等个四五十年再看,建议每天少提点问题,少花心思活的长久点,好能几十年后去验证。

知乎用户 头像狗头保命 发表

在美国学简单数学的和做超级牛东西的是两拨人,做超级牛东西的人学的数学一点也不简单。

中国科学界绝大部分是中国人在做科研,引进科学家只占很小一部分。作出超级牛东西的不是 “美国人”,而是 “在其他地区学得差不多了再去美国搞研究的人”,别说美国中小学数学超级简单了,就算美国中小学不教数学,也不耽误他们出超级牛的成果,因为人家根本不是在美国学的,美国中小学教什么跟他们出不出成果有个蛋的关系。

你让美国像中国这样单用自己本国教育教出来的人搞科研,我包他什么东西都做不出来,五年之内肯定被中国追上。

看问题要看本质,不要只看现象,更不要强行联系因果。

而且中国教育并非全程领先美国,他们中小学数学是简单,大学开始发力,各种大学教材虽然卖得贵死个人,但真有干货,简单明快符合认知科学,极易入门,嫌贵可以买上一届的二手教材,同样牛逼。

我们中小学数学是不错,但大学之后的教育有个很明显的断层,上过大学的人就会发现,教授们基本都是全力科研无心教学,上课不管学生学不学得来,自己讲个痛快就完事儿,再加上大学教材写得是完全违反认知科学,用反科学的方式教科学,学得好不好全靠天赋。按部就班学完高中教育的普通学生是接不下大学那个风格的,高中是把饲料切碎放你槽里甚至填鸭,大学是放羊,有时草都没得给你吃,那个地都不是草地,是沙地,要吃草还得自己跑好几里地去找来吃。一进一出,中小学还领先的教育成果就这么被大学糟蹋没了。

知乎用户 白龍輝 发表

我覺得應該把這個問題改成「美國不像中日韓這樣嚴格執行槍支管制,導致美國經常出現大規模槍殺案悲劇,爲何全球一流人才都爭先恐後跑去美國?他們難道不怕聞名全球的美式校園槍擊案嗎?

我覺得中國不要去跟歐美比科技,跟韓國、日本、新加坡比一比就可以了。

只說一件事情給各位感受一下——2019 年初,中國的科技人員剛剛搞出來一個在月球背面平安落地的月球探測車 (玉兔二號),而美國於 1970 年代發射的「旅行者」系列探測器已經快飛出太陽系了,兩顆「旅行者」號探測器至今仍在向地球傳送可以解析的無線電信號,據科學家推算,它們已飛到太陽系的邊界「奧爾特雲」(太陽高能粒子輻射區的最外圍,被學術界視爲太陽勢力範圍的邊界),是人類有史以來飛行距離最遠的人造機器。1970 年代美國研發「旅行者」系列探測器的時候,中國的知識分子還在農村勞動呢,這就是中美在科技上的差距。

原回答發表一段時間之後,補充一下:

我在這問題的諸多回答中看到很多中國人以一種不屑的心態寫道:

「美國有什麼了不起?還不是靠外國技術移民,兩次世界大戰和蘇聯解體時,美國遠離動亂中心,保持了經濟和政治局勢穩定,吸收了太多的世界級優秀人才。如果沒有這些天才的移民,美國啥也不是……」

對於發表上述言論的人,我只想問他們一個問題——美國爲啥一直遠離動亂中心?爲什麼美國可以長久保持經濟和政治局勢穩定?爲啥其他國家的局勢就不能像美國這樣穩定?爲什麼?爲什麼?爲什麼?爲什麼?

我敢肯定,這幫看不起美國的傻逼答不出我上面提出的這幾個問題,他們最多只能猜測「可能是因爲美國在美洲,遠離戰火紛飛的歐洲和亞洲」,對於這種腦殘式的猜想,你只要再問一個關聯問題——「墨西哥也在美洲,而且就在美國旁邊,爲啥墨西哥不能吸引全球優秀科技人才?」他們就徹底傻逼了,因爲他們的大腦死機了(抱歉更正一下:不是大腦死機,應該是小腦死機,因爲這幫看不起美國的蠢貨沒有大腦,只有小腦和腦幹)。

知乎用户 汗浴 发表

好多好棒的回答。很详细。谢谢大家。其实这个问题我也有考究过。但亲身的经历更能扼要地道出当中的玄妙。其实很简单,一切的一切皆是社会以及文化造就、当然还有教学方式。

每个回答都很有道理、那么我也来阐述一下我的观点:

在美国生活十年整、我所能感受到的是一个清纯的环境。这里没有卡拉 OK、没有沐足、夜总会等等烦嚣的娱乐节目。(当然、在洛杉矶和纽约此等多华人聚集的城市还是会有几家普普通通做留学生生意的)。大家可以很专注地去围绕自己感兴趣的事物发展、可以静心观察社会所需。

再来就是健康的团队文化、大家有共同目标、互帮互助推动一个伟大的概念。看看 Uber, Facebook, Airbnb 这些公司、多么团结一致、多么自豪。暂且不谈苹果谷歌那等巨业。当然还是会有勾心斗角、但比较少、且大多在零售业和金融业。创新行业里实在少。

说到教育、美国讲求获得、中国鼓励通过。当然、美国也是按照分数来毕业、但、不代表毕不了业就死定。收获是自己的、facebook 的马克就是中途辍学而找到仕途。

聪明睿智者,中国不乏其人。可是往往社会风气、价值衡量标准、迂旧的交际方式等等陈规都局限了不少才俊。

天时人和,奈何失地利。

一定与地利有密切关连。地利在此包含本地的文化内涵、教育等等以上提及过的因素。

为什么说和地利有莫大关系呢?早前看过一篇文章是说日本为什么没有太多年青创业者、内容有很好的分析利弊缘由。而提到日本、相信大家都知道那是一个发达的、教育完善的、社会系统规则的国家。比中国进步。可是日本也没用能像美国般推动那么多世界性的新文化。只能留守多年前奠定的基础例如 Panasonic, SONY, Toyota 这些。因此、我觉得是因为那是美国。按照历史发展、近十年就是美国领先这些领域的时间段。有天时,有地利,有人和。

最后、中国近代文化中很强调快、因为人口多竞争大、慢就会输。而试问要研发一件传世的经典产品又怎能快呢?于是便东借西抄快快拼凑成一个小米王国。很成功、商业模式上很值得学习。但回到楼主所问及的主题、创造以及创新嘛、这点似乎小米的一切口号就变得苍白无力了。

纯属个人的模糊见解,绝非深度言辞。望抱聊天心态互相交流。

毕。

知乎用户 Jadedrip 发表

人家只是口算能力差,又不是傻。

知乎用户 匿名用户 发表

因为他们保护了思维的乐趣。

要说清楚这个问题,我们要从【万恶的考试】说起。

考试最直接的功能,是选拔人才竞技互动

在这个问题下,我们需要先确定好讨论的范围,就我自己的理解,如果观点是【美国人数学学的简单】,那么这里所指的应该不是顶尖级别的学生,而是通识阶段的中高级数学学习和考察。

美国中学阶段最流行的有两个数学竞赛,一个是 AMC(American Mathematics Competition),另外一个是 MathCounts

注:这两个考试的考察对象以高年级小学生与初中生为主,年龄段略大于国内小学奥数考生。

中国的中高难度数学竞赛有两种,一种是各大杯赛(华杯赛为代表),另一种是各大中学的小升初选拔考试。

在正常课内教育的范围,两个国家的难度其实差不多,大部分学生都能轻松完成课内知识,这点不做太多探讨。

而当我们把目光转向中高端竞赛的时候,事情开始有了变化,两国考试的差异变得明显起来。

现实里,大部分参加美国数学竞赛的学生,目的不外乎以下几种:

①喜欢数学,想挑战自己;

②自己有数学天赋,希望得到提升;

③为了以后可以方便升学。

而参加中国数学竞赛的大部分学生只有一个目的:升学

(这里我有预感有人会抬杠,所以请注意,我说的是大部分)。

以我自己目前正在辅导的一个学生为例,她在美国念书,刚升入中学,对数学有兴趣,也有天赋,现在是他们学校 MathCounts 校队的成员(这个竞赛有团队赛)。

她自己的数学水平大概是:国内小升初可以顺利考入区重点以上的中学,市重点也有机会,AMC8 无压力通关,AMC10 合格水平。

据她所说,她们校队成员水平都不错,大家都是冲着对数学的热爱加入进来,比较类似国内中小城市里的那些兴趣类数学竞赛,真的是因为喜欢数学,或是有天赋才参加,她学的很开心,会主动思考,也已经开始有了自己的逻辑体系,当然,偶尔也会偷懒(小朋友的通病……)。她的情况,可以代表很大一批美国中小学数学竞赛参与者的状况。

而对于国内,奥数则是另一回事,在目前国内的一二线城市里,对于大部分无权无势的中产以下家庭来说,小升初考试留给他们的机会并不多,最公平靠谱的,可能就是奥数测试了(对于大部分买不起高端学区房的家庭来说,各大名校的点招考试是他们的孩子进入这些中学的唯一机会,虽然政府已经对奥数考试有了一些限制政策,但教育资源紧缺这个刚需不解决,考察就不会停止,而且,就奥数考试本身来说,应该是相对公平的考察了)。

当然,这里用奥数这个名字有点奇怪,因为他们参加的并不是真正意义上的奥林匹克数学竞赛,而是明显高于课本水准的数学测试,所以为了方便称呼,我们把这些内容统称为奥数。

而正如我们所熟知的那样,一个考试,一旦涉及到全员的参与和竞争,在大入口与小出口的压力之下,训练的形式立马会转变到高强度的训练量上,出题风格也会因此发生相应的改变,而这种风气下最大的痛苦,都留给了那些处于中游的学生。

换言之,大部分数学天赋普通的小学生,被迫进行着大量重复的刷题训练,在其中找不到任何快乐,破坏了他们原本可能对数学以及思维产生的兴趣。

插个题外话,就我自己的观察,一般来说,重视思维的家长,孩子的数学成绩都不差,而重视技巧和刷题的家长,孩子的成绩会差别很大。

而作为一个家长,自己的中小学数学基础已经很扎实了,但如果用这种已成的事实去引导孩子快速学习的话,最后只能收获到一个被填满的小孩,而思维性则会得到破坏。

通过上面的内容,大家可能已经能够认识到一个事实,对于大部分中国学生来说,思维和数学已经变成了一种痛苦,这些本该是高速运转的年轻大脑,现在已经开始排斥思考,他们习惯性的把难题推给家长和老师,坐在一旁做一个被动的接收者

而后天的差异,也就从这个时候开始埋下了苗头。

比较完了外部环境对于个人成长的影响,接下来,我们开始研究数学学习的本体,看看同样作为考察的考试,两国考试的侧重点到底有哪些差异。

首先需要确定我们的研究对象,我选择的样本是这样的。

中国的部分:各大小学数学杯赛以及小升初名校选拔考试的真题。

美国的部分:AMC8 以及部分 AMC10 的真题(AMC8 要求八年级以下)。

分析开始:

知识体系:两国在考察内容的框架上没有什么区别,主要的知识点都集中在:数论、计算、几何、逻辑思维、排列组合这几块。

侧重点:这里的差异较明显,国内的部分以套路化的数论及应用题为主,考察学生对公式和技巧掌握的熟练程度,以及快速解题的能力。

美国的部分以基础计算和逻辑推理、几何为主,当然也有很多的操作事务类题。

侧重点这个部分,我认为是最值得展开说的一点。

国内的题,坦率的讲,有些难题过分追求技巧了,这也是为什么网上流传着很多大学生也没法解的小学奥数题的原因,因为他的解题技巧和公式是比较专业的,如果你不是专门研究过的人,很难了解。

而美国的题,我个人感觉,考察的是数学基础,也就是说,你对数学的理解深刻,你的逻辑清晰,一般都能找到方法去进行试探,而不是坐在那里一筹莫展。

说到了这里,我们的结论又回到了最开始谈到的那点,也就是说,中国的数学竞赛,追求的是专业竞技化,鼓励学生迅速通过知识体系中的解题工具进行解答;而美国,追求的是思维性的训练,培养学生对于问题的辨析和分解,抽象与建模。

问题讨论到这里,可能只算是开了个头,不过仍然有了些阶段性的结论,我们不妨在这个结论上做一些分析,回到题主的问题,为什么【美国学生能做出更牛的东西?】

我的结论是:

1. 美国各方面都发达,他们的学习以及实验条件更好,有利于创新活动的进行,经济基础决定上层建筑,这也是不争的事实。

  1. 在教育方面,美国早期教育里对思维乐趣的保护,对思维素质的培养,引导他们一大批的精英学子成为了具有开创能力的大师,而中国偏向于工具化和效率化的教育思路,更有利于培养出高级工程师。

其实站在整个人类的角度来看,这样的分工也是很有效率的,只是对于国人来说,这个结果不太令人满意。

小彩蛋:

AMC10/12 考题为 25 道选择题,答对一道给 6 分,答错不给分,总分 150.

有意思的是,如果你空着不答,也给 1.5 分。

妈妈再也不用担心我蒙不对选择题了!

知乎用户 波罗谛海 发表

因为各国的数学天才都到美国去深造了呀!天知道中国的本科以上高等教育就是在赶人走!

知乎用户 蹦得儿 发表

哎呦看到美国本科生感叹自己数学难 我尴尬症都犯了。我赞美你们并亲吻你的戒指,但是那些在国外读书的数学成绩差的本科生们,请承认这一点,不是美国数学和中国一样难,是你们出去以后没爹妈逼着你学习,你更废柴了一点。(废柴,但不耽误你们回国捞钱。)

让我用个非常真实非常现实的例子来解释为什么美国数学更强。中国大学生,学习十种 ODE 方程解的存在唯一和求解。美国大学生学了三种。剩下的时间,美国学生又学了 numerical ODE 的解法。 结果去研究生 或者去公司,你发现公司研究所就用那三种 ODE,但有的时候还需要你写一些代码 做一些分析,美国人脱颖而出。他们记得不如中国人牢靠,但忘了查查书就行了,但查书一时半刻解决不了的部分,比如 numerical Ode,他们学过。

美国大学学的东西更关键,更热门。

中美大学数学不是差在学生的勤奋,差在老师的水平。

你说中国学校多学的七种 ODE 是为了打基础?为了研究?你说中国学生更厉害?都不是。

**根本原因在于中国老师教了很多没有那么有用的东西。**不是说那 7 个 ODE 没用,但现在不是最热门的领域。**美国一般大学的教授研究不行 但跟工业界联系密切,美国教授知道公司用什么。他就教你这个。**中国除了顶尖学府,大部分跟工业界的联系并不在研究上(你懂的)。更重要的是,大部分教你 ODE 的老师压根就不是教授。

知乎用户 朱泓州 发表

无他。第一,美国比中国多了两百年的积累。第二,美国的大学数学比中国大学难

知乎用户 drawingpen 发表

讲道理,真正能回答你问题的答案都在那些答主的备份详情里躺着呢

知乎用户 AustonLee 发表

我一直都想说这个问题,感觉我们被带偏了。美国中小学数学简单吗,答案是公立学校简单,快乐教育,入学的都是穷人,这点很多学者都已经提出了,这些孩子未来从事的是社会螺丝钉式的工作,只有极少数有毅力有理想有抱负的穷人孩子才能从其中脱颖而出。与之相反的是美国的私立精英教育是非常残酷的,孩子要承担相当重的学业压力,这些孩子未来大多从事美国主流的精英工作,如从政,律师,医生,金融。这些私立学校教育资源丰富,教学条件好,但价格昂贵,穷人只能上免费的公立学校。这就是美国的现实困境,用钱制造了一个人为的教育屏障从而锁死穷人的上升通道。

上述论点无数专家学者都已经说明了,不需要过多赘述。

接下来我们看看美国的精英教育创造了一批什么样的人才。我刚才所说美国人其实不屑于玩理工科的,他们认为那些数学物理化学好的人是 nerd,书呆子。精英阶层应该是管理型人才,所以那些富豪子弟并不会让自己的孩子读理工科,政界人士大多都是法律出身就是一个例子。而我国相当一部分政府官员出身理工科,这点也有相关调查证实。既然美国的精英教育产出的都是管理型人才,那美国科技为何如此发达,这就是美国金融霸权提供了收割全世界财富的国家机器,而金融霸权来自于军事和科技霸权,通过高福利吸引全世界理工科人才到美国做科研。科研成果为美国所用,进一步确立科技霸权,讲科技转化为金融和军事,巩固霸权地位,行成正向循环。中国的科学家绝大多数是具有爱国情怀的,所以愿意以低薪留在本国做科研。但是那也是建立在我国具有优越的基础设施和便利的生活条件,并且具有极强的民族文化底蕴。这些能让我们的科技不断变强。换做印度如何,印度人已经用脚投票了,相当一部分印度人才流入美国,都是能走就走。所以以前你会发现很多重大发明发现是美国本土培育的人才产出的。但现在这个比例越来越少,美国很大一部分现在产出的高质量论文都是印度人或者华人再美工作产出的。

美国本土的精英都去华盛顿或者华尔街了。安心搞科研的太少了。

所以你问美国中小学数学简单,但美国论文产出世界第一,就这原因。

知乎用户 横山老尸​ 发表

美国做出超级牛的东西来的人……

不是学中小学超级简单数学的那群人。

美国的阶级是固定的。

所以,固然你能以很便宜的价格享受医疗服务,前提是你必须在狭小的过道中经历漫长的排队和在你发烧、难受得要死的时候得到一个 “回家休息” 的诊断。

所以,固然你能以很便宜甚至免费的价格送孩子去读书,前提是你的孩子必须学习那种长大后账都算不清的、简单的数学和其它知识。

所以,固然你能以比中国便宜至少 300% 的价格购买一辆汽车并愉快的使用它,前提是你必须忍受比中国最大城市还要变态的堵车和胡乱插道。

………………

不过,如果你非常有钱的话,那风景就完全不同了:

你可以将你的孩子送进干净、宽敞的学校,这些学校教什么呢?教孩子礼仪、教孩子社交、教孩子不差于中国水平的数学、物理、化学和其它知识,甚至还能根据你孩子的兴趣方向,为你的孩子制定一整套私有的、定制化的教学方案,最终,将你的孩子送进常青藤深造。

你还可以享受一对一私人医生的温馨服务,你需要付出的仅仅是一个电话预约,如果你很有钱,你甚至可以让你的私人医生随叫随到,如果你非常有钱,你甚至可以拥有一个由专家组成的私人医疗队,24 小时不间断为你服务。

你还能买一架私人直升飞机,经过美国相关部门贴心的服务后,用不到一周的时间领取适航证,高高在上的翱翔在蓝天之上,点上一根来自古巴的 Julietas,用怜悯的眼神看着下方的劳苦大众,心无波澜。

……………………

至于上面回答中有人说 “美国给予不热爱数学的学生最基础的数学教育,而给予热爱数学的天才最高水平的数学教育” 这种话,从某些角度来看其实也没错,可惜的是,他们只看到了高端的教育方式,忽略了美国低端的教育方式。

在美国,一切的社会服务都有高端和低端之分,而高端的社会公共服务,并不是谁都能享受的。

知乎用户 Michael 发表

很多人都被带跑偏了

不否认美国大学教育的优势

但是换个说法,如果有人对你说,清华北大的厉害之处在于培养了很多高考状元,你认同这个说法吗?肯定不能这么说呀,因为获得高考状元在前,去清华北大在后。那清华北大牛在哪?培养中国各行各业绝大部分的人才。问题是,究竟是清华北大的本身教育牛逼,还是清华北大聚集了全国最聪明最有资源的人群?到底怎么算牛逼的构成因子,用互相成就更合适。

说到底美国也是这样,一个是汇聚了一个国家人才,一个是汇聚了全世界的人才。美国依靠体制和待遇,吸引全世界最牛逼的人才,光靠土生土长的美国人,不见得比中国做的更好。

知乎用户 王彦岭 发表

先假定题目的 “美国中小学的数学比我们简单” 成立。

咱们的数学教育培养出来的大多数是人肉计算器。

知乎用户 星月无痕 发表

人口普遍创造力不足是根本原因之一,而创造力不足是基础教育的问题。

中国的基础教育存在很大的不足,尤其是软实力方面,比如创新思维能力、逻辑思维能力的培养。

举例:在知乎阅读答案及其下面的评论,越来越发现,能够清晰地表达出自己真实想法的人真不多,很多人都存在【逻辑思维能力不足】,导致他们表达时缺乏逻辑性,逻辑漏洞千疮百孔。

逻辑思维能力和创新思维能力,是应该从幼儿阶段就有计划、有意识地去培养的,从小养成良好的思维习惯,幼儿园阶段必不可少。但是现在我国的幼儿教育是什么水平,不用多说了吧?一没有系统的理论体系做指导,二没有充足的实践反馈进化的机会。甚至提前把小学的课拿到幼儿园里去教(国家强调不准在幼儿园阶段教小学的课程知识,政策推出多少年了,又有多少幼儿园落实到位了?)过早把思维方式禁锢住了,创造性自然不足。

幼儿教育阶段,我国从观念、到理论系统、再到课程体系,普遍存在缺失。输在起跑线上就不足为奇了。

当然不是说所有中国孩子将来长大了在思维能力上都不如别人,少部分人还是很出色的,但是要有足够的人数达到那个能力水平,才能使金字塔的塔基足够宽广,这个金字塔才足够稳、才能建得足够高啊。

中国为什么足球不行?参加足球运动的总人口基数就不够。

中国为什么乒乓球牛?参加乒乓球运动的总人口基数碾压全世界。

孩子再大一点,上了小学,就开始了灌输式教育的噩梦,无论简单的问题还是复杂的问题,都有一个【对】与【错】的评价跟着你。明明 A、B 两个答案都是正确的,选 A 的就会被判错,选 B 的才会被判对,这种愚蠢的教育方式,从根本上长久不懈地在扼杀孩子的主动思维意识。把启迪孩子思维的机会变成迫使孩子听话、顺从的木偶训练。

前两天网上看到一个真事,小学生写作业,数字【4】我们平常书写的时候,

可以写成不封口的:

![](data:image/svg+xml;utf8,)

也可以写成封口的【4】

一个小学生写成不封口的样子,老师画了一个大大的红叉叉,家长向老师提出质疑时,老师回答说课堂上讲了多次要写成封口的,你的孩子就是不听话,非要写成不封口的。

这就是以【听话】、【顺从】的评判标准代替了知识本身对错的评判标准,这样的教育方式培养出来的孩子,基本上都会或多或少变成【领导的话比真理更正确】的小人思想。(有权惩罚我的人说的话如果与真理相矛盾,那么一定是真理错了,真理是对的也要说成错的。这不是小人是什么?)

经过长期不懈的这种木偶训练,学生都变成【这个结论是权威说的,所以就是正确的】、【记住这个是正确答案就行了,不允许质疑、不允许思考】的听话的乖孩子。

借用李云龙说的一句话:不能什么都让你大旅长占了啊,又想让我搞枪、又想让我当乖孩子,这叫不讲理啊。既然基础教育把孩子都培养成听话的乖孩子,就别怪孩子长大了搞不来枪。

填鸭式、机械记忆式的教育,加上伤害力最大的【对错式教育】,将整个孩子的成长期人为扭曲为只有对和错的二元世界。一切创新都是在怀疑中产生的,一切创新都是在打破常规中产生的,但是怀疑和打破常规在人生大脑发育的最关键时期,是几乎彻底禁止的。等到大脑发育的关键时期都过去了,这时突然要求这个大脑产生创造性思维?不是笑话吗?

好比人的身高,有几个重要的【身高突增】的年龄段(俗称【蹿个】),那么在【蹿个儿】的重要年龄段科学锻炼、均衡营养当然有助于孩子长得更高一些,如果在【蹿个】期不允许锻炼,饭都不给吃饱,天天饿得半夜爬起来喝凉水去,等到人长到 28 了,嫌孩子长得矮,再锻炼、再补充营养也晚了。

比如各种思维的碰撞,极度缺乏,因为从小到大都是一个模板套出来的月饼,大家的想法都差不了多少,能碰撞出什么新鲜玩意?

中国的基础教育,有一个阶段是为培养【工匠】、【工具】服务的,这样的教育培养出来的【人才】的,是为当时国家发展需求提供【工匠】和【工具】,为了迅速弥补某一种类型的人才的不足,而制定的具有时代特色、国家各色的适用型教育方式。

于是教育优先投资高等教育,我急着用人才啊,你们高校赶紧给我输送有用的人才,我给你拨款。什么?小学、初中?我给你小学阶段投资(不仅是资金还包括资源)什么时候见效果?(6 岁到 22 岁需要 16 年)什么?要 16 年?您后边歇歇吧,我这高校 4 年就给我人才,我凭什么等你 16 年的?

还有一个方面,就是大环境的不同。随便举个例子:

比如创业的门槛,苹果、HP、Google、Amazon、Youtube 都是从车库创业开始的,中国创业门槛呢?首先要租房吧,租的还必须是【允许注册公司】的房,这种房都贵的一塌糊涂,中国的房地产经济支柱扼杀了大量创业者。美国 100 个想创业的,98 个在自家车库就开始了;中国 100 个想创业的,80 个被注册公司的门槛给挡住了。

房地产产业化提前预支了 70 年的社会创造力。高房价太高了整个社会的整体成本,包括创新成本、创业成本、甚至跳槽成本。

房租那么高,如果凭自己的存款,最多支持三个月,之后如果再没工资拿就要睡马路上了。在这样的条件下,有几个人敢于辞职创业?又几个人会出于个人的兴趣而去创业?

没有创业的行动,谁会像真的创业一样去进行思考和准备?脑子里想的东西不一样,得到的结果自然不同。本来创造性思维能力就不占优势,再加上愿意把这个思维能力开动起来的人少之又少,打工者和创业者思考的方式都不一样。

当一个一个人才都被生活压得喘不上气的时候,谁还有闲情逸致去追求自己的兴趣爱好呢?所以把自己的爱好当做事业的人,是幸运的,因为可以堂而皇之地利用工作之便把自己的爱好做到极致。如果你根本就不喜欢正在做的职业,很难想象你会做出多么超群的成绩,最起码的动力都不足。

实例:

别的地方不说,知乎里就有大量的实例,有多少人说话表现出来的都是【非黑即白】的思维?【不是 100 分就一定是 0 分】的思维?当然,有些是出于主观目的故意带节奏,但不排除小时候接受【对错式教育】养成的思维习惯。

还有近几年网上突然泛起的【科学教信徒】,把科学当做宗教般的去信仰,本质上是一种【教条式】的信仰。信的是表面上甚至是字面上带不带【科学】两个字,而不是本质上符不符合科学精神。这都是儿时接受【权威】教育、【对错】教育、【记忆】教育、【不准思考、不准质疑】教育导致的自然而然的习惯。

因为上学时锻炼【讲理】的机会不多,尤其是没有约束的、没有对错评判的【讲理】锻炼机会更少,导致突然上自媒体了给了自由【讲理】的机会了,反倒不知道怎么用这种【自由】了。

因为从小到大没见过多少正确的【讲理】方法,生物多样性见得少,只见过兔子式【讲理】,没见过(更没有体验过)犀牛斑马鲨鱼式【讲理】,于是漫无边际地去随意创造【非兔子式】【讲理】,当然会错误百出【混不讲理】了,只见过兔子的人,无论如何画不出犀牛和斑马的。于是在创造性【讲理】的过程中,一旦脱离了【兔子】式讲理,各种【混不讲理】的手段就全都招呼上了。

知乎用户 ttss 发表

马斯克在 24 岁进入斯坦福大学,攻读材料科学与应用物理的博士课程,但在入学的第二天就辍学,开始创办 Zip2。

扎克伯格 2002 年进入哈佛就读,2004 年辍学,全身心投入 Facebook 的创业。

比尔 · 盖茨 18 岁考入哈佛,一年后辍学,和好友保罗 · 艾伦创办微软公司。

史蒂夫 - 乔布斯 1972 年高中毕业后,在波兰的一所大学中只念了一学期的书。1974 年乔布斯在一家公司找到设计电脑游戏的工作。两年后,时年 21 岁的乔布斯和 26 岁的沃兹尼艾克在乔布斯家的车库里成立了苹果电脑公司。

国外大佬全在辍学创业,国内人士全在内卷考研。

究竟是什么原因造成的这一切呢?还不是社会现实根本就是没有出路。完全不靠能力,靠不要脸使下三滥手段。

国内唯二两个企业,一个华为,不仅在国外被打压,在国内还要被打成爱国营销。一个京东,老板太耿直直接被人仙人跳。

知乎用户 heng 发表

当过不少次 TA 了,根据个人经验,我觉得可能有如下几个原因

1. 中国考的难,但是大多数人还是学不会,或者死记硬背应付了事,很快就忘记了

2. 美国是纯精英教育,金字塔尖那部分人学得相当多,教育资源也好

3. 更重要的是科学素养和坚定的意志品质,这个考试不考

4. 美国吸引的是全世界最牛的人才,不用靠本国的教育系统

5. 做出很牛的东西本来也就不需要那么多人,大多数时候一两个牛人带着一群普通人就 OK 了

知乎用户 实话直说 发表

国外属于自由学习,没人强迫,老师也没想让所有人都变得成为数学家,所以课程简单,但自己喜欢数学的孩子,会自发主动学习,比如高中业余时间就会去大学旁听,学习大学的课程,这样的人就是当数学家的,他们可能会偏科很严重,但也没人在乎。当然很多老百姓,就笨的不行,100 以内加减法都要用计算器,高中连普通方程都解不明白。

国内是标准化学习,基本上只要你上完小学,上完初中,上完高中,你就必须要达到一定水平,说白了最差的差生,也希望他能达到最基本的及格水平。当然这样教育往往课业量重,孩子没有时间去主攻自己喜欢的科目。剩下压力大,最后变成只为对付应试教育,学习就是为了考试,考完了就完事大吉了,再也不想去看书本,恨不得都扔了,只要成绩过关,能上好学校就行。

知乎用户 想当教授的门卫 发表

美国人在小学和中学的外语是选修,还有几乎不学政治,他们有大把的时间做自己喜欢的事情。再来看看我国的中学,英语那是必须的主课,政治也必须的。再加上中文是世界上数得上的难学。人的精力是有限的……

中国考研除了专业课,必须考政治和英语。面试时,导师更加趋向于你的外语成绩,而不太在乎你的专业。美国考研只有专业课。

日本有很多获得诺贝尔奖的,没出过国,不会讲英语,对政治不敏感,只钻研学问。我国可能除了专业课,还需要花费大力气学一些其它的东西。如郑强教授所说的那样……

差距就这么拉开了,实属无奈。

最简单好用的 VPS,没有之一,注册立得 100 美金
comments powered by Disqus

See Also

谷爱凌的成功,到底和普通人有没有关系?

知乎用户 马达熊​ 发表 目前知乎大部分网友,奋斗一生,最高大概能达到谷爱凌姥姥的水平。 成为某一领域的专家,有一点知名度,写几本书,退休后偶尔还能被人请去讲讲课。 我高中是中国最好的几所高中之一。 我当年的同学里,最成功的,也就是谷爱凌妈 …

谷爱凌的两个祖国:在湾区滑雪,在海淀补课

2月8日,谷爱凌在自由式滑雪女子大跳台决赛中获得金牌。这名出生于美国的运动员于2019年归化中国,代表中国参加2022年冬奥会。赛前她就被寄予厚望,认为有机会替中国拿到三块金牌。2014年、2018年两届冬奥会,中国一共只拿到3块金牌。为了 …

奴隶社会|谷爱凌的成功,和普通人有什么关系

这几天铺天盖地的是谷爱凌奥运摘金的新闻。 对于她个人,她的家庭,这是了不起的成就,值得大声庆祝。 其实不止是谷爱凌,每一个运动员,不管出生在哪里,国籍是什么,能有机会参赛,并全力以赴取得自己的最好成绩,都值得被大大的庆祝! 无论这个赛事是不 …

怎么客观地评价我国的教育体制?

知乎用户 老猫仙​​ 发表 题主,中国的教育体制本质上就像你提出的问题一样,非常的 “不具体,大,空,高度总结,实则在忽略细节”。 客观讨论中国的教育体制——谁会客观?需要谁的客观?如何定义客观? 别强调 “中国” 的教育体制,本质上所有国 …